OMNC Practice

अब Quizwiz के साथ अपने होमवर्क और परीक्षाओं को एस करें!

What interventions would the nurse apply to support the breastfeeding mother? Note: Credit will be given only if all correct choices and no incorrect choices are selected. Select all that apply. 1. Assist the mother to begin breastfeeding within the first hour after birth. 2. Have the baby returned to the nursery after feeding so that the mother can get adequate rest. 3. Teach the mother to recognize and respond to early infant feeding cues. 4. Inform the mother about community resources that support breastfeeding. 5. Instruct the mother to avoid eating foods that might upset the newborn's stomach

1, 3, 4 Explanation: 1. Throughout the first 2 hours after birth, but especially during the first hour of life, most infants are usually alert and ready to breastfeed. 3. The new mother should be taught to recognize and respond to early infant feeding cues. The timing of newborn feedings is ideally determined by physiologic and behavioral cues rather than a set schedule. 4. It is important that parents receive verbal and written instructions and community resource information to which they can later refer.

During a home visit, a new mother who is breastfeeding complains that her nipples are sore and cracked. Which measures should the nurse suggest? Note: Credit will be given only if all correct choices and no incorrect choices are selected. Select all that apply. 1. Alternate the baby's nursing positions throughout the day. 2. Breastfeed the baby always in a seated position 3. Avoid placing the areola in the baby's mouth. 4. Insert a finger between the infant's gums to break the latch before removing from the breast. 5. Wash the breasts with warm water, and avoid drying soaps.

1, 4, 5 Explanation: 1. Changing positions alters the focus of greatest stress and promotes more complete emptying of the breasts. 4. To prevent trauma, the mother should also be taught to gently insert her finger between the infant's gums to break the latch before removing the baby from the breast. 5. The nurse can instruct the mother in a number of measures to promote comfort and healing and to prevent skin breakdown. Washing the breasts with warm water and avoiding drying soaps are recommended

Which of the following is a common barbiturate used in labor? 1. Seconal 2. Valium 3. Phenergan 4. Vistaril

1. Seconal

The charge nurse is looking at the charts of laboring clients. Which client is in greatest need of further intervention? 1. Woman at 7 cm, fetal heart tones auscultated every 90 minutes 2. Woman at 10 cm and pushing, external fetal monitor applied 3. Woman with meconium-stained fluid, internal fetal scalp electrode in use 4. Woman in preterm labor, external monitor in place

: 1 Explanation: 1. During active labor, the fetal heart tones should be auscultated every 30 minutes; every 90 minutes is not frequent enough.

The nurse is assessing a client who has severe preeclampsia. What assessment finding should be reported to the physician? 1. Excretion of less than 300 mg of protein in a 24-hour period 2. Platelet count of less than 100,000/mm3 3. Urine output of 50 mL per hour 4. 12 respirations

: 2 Explanation: 2. HELLP syndrome (hemolysis, elevated liver enzymes, and low platelet count) complicates 10% to 20% of severe preeclampsia cases and develops prior to 37 weeks' gestation 50% of the time. Vascular damage is associated with vasospasm, and platelets aggregate at sites of damage, resulting in low platelet count (less than 100,000/mm3).

Which client requires immediate intervention by the labor and delivery nurse? 1. Client at 8 cm, systolic blood pressure has increased 35 mm Hg 2. Client who delivered 1 hour ago with WBC of 50,000 3. Client at 5 cm with a respiratory rate of 22 between contractions 4. Client in active labor with polyuria

: 2 Explanation: 2. The white blood cell (WBC) count increases to 25,000/mm3 to 30,000/mm3 during labor and early postpartum. This count is abnormally high, and requires further assessment and provider notification.

A woman is hospitalized with severe preeclampsia. The nurse is meal-planning with the client and encourages a diet that is high in what? 1. Sodium 2. Carbohydrates 3. Protein 4. Fruits

: 3 Explanation: 3. The client who experiences preeclampsia is losing protein.

A standard ultrasound examination is performed during the second or third trimester and includes an evaluation of which of the following? Note: Credit will be given only if all correct choices and no incorrect choices are selected. Select all that apply. 1. Confirm fetal heart activity. 2. Evaluate the cervix. 3. Determine fetal presentation. 4. Amniotic fluid volume. 5. Fetal number.

: 3, 4, 5 Explanation: 3. A standard ultrasound examination is performed during the second or third trimester and includes an evaluation to determine fetal presentation. 4. A standard ultrasound examination is performed during the second or third trimester and includes an evaluation of amniotic fluid volume. 5. A standard ultrasound examination is performed during the second or third trimester and includes an evaluation of fetal number.

A pregnant client who was of normal prepregnancy weight is now 30 weeks pregnant. She asks the nurse what appropriate weight gain for her should be. What is the nurse's best response? 1. "25-35 pounds" 2. "30-40 pounds" 3. "17-18 pounds" 4. "Less than 15 pounds"

Answer: 1 Explanation: 1. An appropriate weight gain for a woman of normal weight before pregnancy would be 25-35 pounds.

The nurse understands that the classic symptom of endometritis in a postpartum client is which of the following? 1. Purulent, foul-smelling lochia 2. Decreased blood pressure 3. Flank pain 4. Breast is hot and swollen

Answer: 1 Explanation: 1. Assessment findings consistent with endometritis are foul-smelling lochia, fever, uterine tenderness on palpation, lower abdominal pain, tachycardia, and chills.

The nurse knows that a baby born to a mother who had oligohydramnios could show signs of which of the following? 1. Respiratory difficulty 2. Hypertension 3. Heart murmur 4. Decreased temperature

Answer: 1 Explanation: 1. Because there is less fluid available for the fetus to use during fetal breathing movements, pulmonary hypoplasia may develop.

A couple is requesting fertility counseling. The nurse practitioner has identified the factors listed below in the woman's health history, and knows which of them could be contributing to the couple's infertility? 1. The client is 38 years old. 2. The client was 13 years old when she started her menses. 3. The client works as a dental hygienist 3 days a week. 4. The client jogs 2 miles a day.

Answer: 1 Explanation: 1. As the eggs of older women age, their fertility is reduced.

To prevent sudden infant death syndrome (SIDS), the nurse encourages the parents of a term infant to place the infant in which position when the infant is sleeping? 1. On the parents' waterbed 2. Swaddled in the infant swing 3. On the back 4. On the sides

Answer: 3 Explanation: 3. Babies should sleep on their backs every time they are put down for sleep.

A pregnant client is admitted to the emergency department with bleeding. The nurse realizes that the client might have placenta previa. Which signs would be suggestive of placenta previa? 1. Bright red vaginal bleeding 2. Sudden onset of vaginal bleeding 3. Firm and hard uterus 4. Change in the size of abdomen

Answer: 1 Explanation: 1. As the lower uterine segment contracts and dilates, the placental villi are torn from the uterine wall, causing bright red bleeding.

If oligohydramnios occurs in the first part of pregnancy, the nurse knows that there is a danger of which of the following? 1. Major congenital anomalies 2. Fetal adhesions 3. Maternal diabetes 4. Rh sensitization

Answer: 2 Explanation: 2. If oligohydramnios occurs in the first part of pregnancy, there is a danger of fetal adhesions (one part of the fetus may adhere to another part).

A client was admitted to the labor area at 5 cm with ruptured membranes about 14 hours ago. What assessment data would be most beneficial for the nurse to collect? 1. Blood pressure 2. Temperature 3. Pulse 4. Respiration

Answer: 2 Explanation: 2. Rupture of membranes places the mother at risk for infection. The temperature is the primary and often the first indication of a problem.

The nurse assesses four newborns. Which of the following assessment findings would place a newborn at risk for developing physiologic jaundice? 1. Cephalohematoma 2. Mongolian spots 3. Telangiectatic nevi 4. Molding

Answer: 1 Explanation: 1. A cephalohematoma is a collection of blood resulting from ruptured blood vessels between the surface of a cranial bone and the periosteal membrane. They may be associated with physiologic jaundice, because there are extra red blood cells being destroyed within the cephalohematoma.

The nurse is caring for four newborns who have recently been admitted to the newborn nursery. Which labor event puts the newborn at risk for an alteration of health? 1. The infant's mother has group B streptococcal (GBS) disease. 2. The infant's mother had an IV of lactated Ringer's solution. 3. The infant's mother had a labor that lasted 12 hours. 4. The infant's mother had a cesarean birth with her last child.

Answer: 1 Explanation: 1. A common cause of neonatal distress is early-onset group B streptococcal (GBS) disease. Infected mothers transmit GBS infection to their infants during labor and birth. All infants of mothers identified as at risk should be assessed and observed for signs and symptoms of sepsis.

The postpartum homecare nurse has performed home visits to four breastfeeding mothers. Which mother is experiencing an expected outcome? 1. Breasts are engorged; placing fresh cabbage leaves inside her bra 2. Sore and cracked nipples; using hydrogel dressings to facilitate healing 3. Breast engorgement; accompanied by erythema 4. Concerns about milk supply; supplementing with formula

Answer: 1 Explanation: 1. A compress of fresh green cabbage leaves helps reduce engorgement.

The home health nurse is admitting a client at 18 weeks who is pregnant with twins. Which nursing action is most important? 1. Teach the client about foods that are good sources of protein. 2. Assess the client's blood pressure in her upper right arm. 3. Determine whether the pregnancy is the result of infertility treatment. 4. Collect a cervicovaginal fetal fibronectin (fFN) specimen.

Answer: 1 Explanation: 1. A daily intake of 4000 kcal (minimum) and 135 g protein is recommended for a woman with normal-weight twins.

) The prenatal clinic nurse is explaining test results to a client who has had an assessment for fetal well-being. Which statement indicates that the client understands the test result? 1. "The normal Doppler velocimetry wave result indicates my placenta is getting enough blood to the baby." 2. "The reactive non-stress test means that my baby is not growing because of a lack of oxygen." 3. "Because my contraction stress test was positive, we know that my baby will tolerate labor well." 4. "My biophysical profile score of 6 points to everything being normal and healthy for my baby."

Answer: 1 Explanation: 1. A decrease in fetal cardiac output or an increase in resistance of placental vessels will reduce umbilical artery blood flow. Doppler velocimetry is best used when intrauterine growth restriction is diagnosed; therefore, the baby is getting an adequate blood supply.

A woman is admitted to the birth setting in early labor. She is 3 cm dilated, -2 station, with intact membranes and FHR of 150 beats/min. Her membranes rupture spontaneously, and the FHR drops to 90 beats/min with variable decelerations. What would the initial response from the nurse be? 1. Perform a vaginal exam. 2. Notify the physician. 3. Place the client in a left lateral position. 4. Administer oxygen at 2 L per nasal cannula.

Answer: 1 Explanation: 1. A drop in fetal heart rate accompanied by variable decelerations is consistent with a prolapsed cord. The nurse would assess for prolapsed cord via vaginal examination.

The nurse notes the following findings in a client at 12-weeks' gestation. Which of the findings would enable the nurse to tell the client that she is diagnostically pregnant? 1. Fetal heart rate by Doppler 2. Positive pregnancy test 3. Positive Chadwicks sign 4. Montgomery gland enlargement

Answer: 1 Explanation: 1. A fetal heart rate by Doppler is a diagnostic (positive) change of pregnancy.

The nurse auscultates the FHR and determines a rate of 112 beats/min. Which action is appropriate? 1. Inform the maternal client that the rate is normal. 2. Reassess the FHR in 5 minutes because the rate is low. 3. Report the FHR to the doctor immediately. 4. Turn the maternal client on her side and administer oxygen.

Answer: 1 Explanation: 1. A fetal heart rate of 112 beats/min. falls within the normal range of 110-160 beats/min., so there is no need to inform the doctor.

A 27-year-old married woman is 16 weeks pregnant and has an abnormally low maternal serum alpha-fetoprotein test. Which statement indicates that the couple understands the implications of this test result? 1. "We have decided to have an abortion if this baby has Down syndrome." 2. "If we hadn't had this test, we wouldn't have to worry about this baby." 3. "I'll eat plenty of dark green leafy vegetables until I have the ultrasound." 4. "The ultrasound should be normal because I'm under the age of 35."

Answer: 1 Explanation: 1. A low maternal serum alpha-fetoprotein test can indicate trisomy 18 or trisomy 21 (Down syndrome). Many couples abort a fetus that has a genetic abnormality that significantly affects quality of life or has multiple medical problems. Down syndrome is more likely to occur in the fetuses of women over the age of 35 at delivery, but is not limited to this age group.

A cesarean section is ordered for a pregnant client. Because the client is to receive general anesthesia, what is the primary danger with which the nurse is concerned? 1. Fetal depression 2. Vomiting 3. Maternal depression 4. Uterine relaxation

Answer: 1 Explanation: 1. A primary danger of general anesthesia is fetal depression. The depression in the fetus is directly proportional to the depth and duration of the anesthesia.

A primary herpes simplex infection in the first trimester can increase the risk of which of the following? 1. Spontaneous abortion 2. Preterm labor 3. Intrauterine growth restriction 4. Neonatal infection

Answer: 1 Explanation: 1. A primary herpes simplex infection can increase the risk of spontaneous abortion when infection occurs in the first trimester.

The nurse is making client assignments for the next shift. Which client is most likely to experience a complicated labor pattern? 1. 34-year-old woman at 39 weeks' gestation with a large-for-gestational-age (LGA) fetus 2. 22-year-old woman at 23 weeks' gestation with ruptured membranes 3. 30-year-old woman at 41 weeks' gestation and estimated fetal weight 7 pounds 8 ounces 4. 43-year-old woman at 37 weeks' gestation with hypertension

Answer: 1 Explanation: 1. A risk factor for hypotonic uterine contraction patterns includes a large-for-gestational-age (LGA) fetus.

A client and her husband have contacted their physician about fertility problems. At the initial visit, the nurse instructs them about the infertility workup. Which statement by the client would indicate that the instructions have been successful? 1. "The first test that we need to schedule is a semen analysis." 2. "We need to schedule the Pap smear test first." 3. "We need to schedule an appointment with the social worker in order to adopt." 4. "We need to schedule an appointment with a marriage counselor."

Answer: 1 Explanation: 1. A semen analysis is one of the first diagnostic tests, prior to doing invasive procedures.

The student nurse notices that a newborn weighs less today compared with the newborn's birth weight three days ago. The nursing instructor explains that newborns lose weight following birth due to which of the following? 1. A shift of intracellular water to extracellular spaces. 2. Loss of meconium stool. 3. A shift of extracellular water to intracellular spaces. 4. The sleep-wake cycle.

Answer: 1 Explanation: 1. A shift of intracellular water to extracellular space and insensible water loss account for the 5% to 10% weight loss.

How would the nurse best analyze the results from a client's sonogram that shows the fetal shoulder as the presenting part? 1. Breech, transverse 2. Breech, longitudinal 3. Breech, frank 4. Vertex, transverse

Answer: 1 Explanation: 1. A shoulder presentation is one type of breech presentation, and is also called a transverse lie.

The nurse is performing discharge teaching for a newly delivered first-time mother and her infant on the 2nd postpartum day. Which statement by the mother indicates that teaching has been successful? 1. "Taking baths will help my perineum feel less sore each day." 2. "If I develop heavy bleeding, I should take my temperature." 3. "My bowel movements should resume in a week." 4. "I will go back to the doctor in 4 days for my RhoGAM shot."

Answer: 1 Explanation: 1. A sitz bath or tub bath promotes healing and provides relief from perineal discomfort during the initial weeks following birth.

A variety of drugs are used either alone or in combination to provide relief of postpartum pain. Which of the following would be an option for pain relief? 1. Nonsteroidal anti-inflammatory agents 2. Proquad 3. Methergine 4. Intravenous oxytocin

Answer: 1 Explanation: 1. A variety of drugs are used alone or in combination to provide relief of postpartum pain. An option would include nonsteroidal anti-inflammatory agents such as ibuprofen and ketorolac.

What condition is due to poor peripheral circulation? 1. Acrocyanosis 2. Mottling 3. Harlequin sign 4. Jaundice

Answer: 1 Explanation: 1. Acrocyanosis is a bluish discoloration of the hands and feet that may be present in the first 24 hours after birth and is due to poor peripheral circulation, which results in vasomotor instability and capillary stasis, especially when the baby is exposed to cold.

The nurse is planning an educational session for pregnant vegans. What information should the nurse include? 1. Eating beans and rice provides complete protein needs. 2. Soy is not a good source of protein for vegans. 3. Rice contains a high level of vitamin B12. 4. Vegan diets are excessively high in iron.

Answer: 1 Explanation: 1. Adequate dietary protein can be obtained by consuming a varied diet with adequate caloric intake and plant-based proteins. Consuming an assortment of plant proteins throughout the day such as beans and rice, peanut butter on whole-grain bread, and whole-grain cereal with soy milk ensures that the expectant mother obtains all essential amino acids.

Appropriate nursing interventions for the application of erythromycin ophthalmic ointment (Ilotycin) include which of the following? 1. Massaging eyelids gently following application 2. Irrigating eyes after instillation 3. Using a syringe to apply ointment 4. Instillation is in the upper conjunctival surface of each eye

Answer: 1 Explanation: 1. After administration, the nurse massages the eyelid gently to distribute the ointment.

The nurse is preparing a class on breastfeeding for pregnant women in their first trimester. The women are from a variety of cultural backgrounds, and all speak English well. Which statement should the nurse include in this presentation? 1. "Although some cultures believe colostrum is not good for the baby, it provides protection from infections and helps the digestive system to function." 2. "Some women are uncomfortable with exposing their breasts to nurse their infant, but it really isn't a big deal. You will get used to it." 3. "No religion prescribes a feeding method, so you all can choose whatever method makes the most sense to you." 4. "In most cultures, it is culturally acceptable to speak about intimate matters in front of their families."

Answer: 1 Explanation: 1. Although it is true that some cultures believe colostrum to be unhealthy, colostrum helps to protect the infant from disease and illness.

The nurse is preparing a client for amniocentesis. Which statement would indicate that the client clearly understands the risks of an amniocentesis? 1. "I might go into labor early." 2. "It could produce a congenital defect in my baby." 3. "Actually, there are no real risks to this procedure." 4. "The test could stunt my baby's growth."

Answer: 1 Explanation: 1. Amniocentesis has the potential for causing a spontaneous abortion.

The pregnant client and her partner are both 40 years old. The nurse is explaining the options of chorionic villus sampling (CVS) and amniocentesis for genetic testing. The nurse should correct the client if she makes which statement? 1. "Amniocentesis results are available sooner than CVS results are." 2. "CVS carries a higher risk of limb abnormalities." 3. "Amniocentesis cannot detect a neural tube defect." 4. "CVS is performed through my belly or my cervix."

Answer: 1 Explanation: 1. Amniocentesis results take longer to process than do CVS results.

The nurse is teaching the parents of an infant with an inborn error of metabolism how to care for the infant at home. What information does teaching include? 1. Specially prepared formulas 2. Cataract problems 3. Low glucose concentrations 4. Administration of thyroid medication

Answer: 1 Explanation: 1. An afflicted PKU infant can be treated by a special diet that limits ingestion of phenylalanine. Special formulas low in phenylalanine, such as Lofenalac, Minafen, and Albumaid XP, are available.

The nurse has completed a community presentation about the changes of pregnancy, and knows that the lesson was successful when a community member states that which of the following is one probable or objective change of pregnancy? 1. "Enlargement of the uterus" 2. "Hearing the baby's heart rate" 3. "Increased urinary frequency" 4. "Nausea and vomiting"

Answer: 1 Explanation: 1. An examiner can perceive the objective (probable) changes that occur in pregnancy. Enlargement of the uterus is a probable change.

Which statement is best to include when teaching a pregnant adolescent about nutritional needs of pregnancy? 1. "It is important to eat iron-rich foods like meat every day." 2. "Calcium and milk aren't needed until the third trimester." 3. "Folic acid intake is the key to having a healthy baby." 4. "You just need to pay attention to what you eat now."

Answer: 1 Explanation: 1. An inadequate iron intake is a major concern with the adolescent diet. Iron needs are high for the pregnant teen because of the requirement for iron by the enlarging maternal muscle mass and blood volume. Giving specific examples is helpful when giving nutritional information.

The postpartum unit nurse is caring for a client who delivered a term stillborn infant yesterday. The mother is heard screaming at the nutrition services worker, "This food is horrible! You people are incompetent and can't cook a simple edible meal!" The nurse understands this as which of the following? 1. An indication the mother is in the anger phase of grief. 2. An abnormal response to the loss of the child. 3. Reactive stress management techniques in use. 4. Denial of the death of the child she delivered yesterday.

Answer: 1 Explanation: 1. Anger, resulting from feelings of loss, loneliness, and, perhaps, guilt, is a common reaction. Anger may be projected at significant others and/or healthcare team members.

To reduce possible side effects from a cesarean section under general anesthesia, clients are routinely given which type of medication? 1. Antacids 2. Tranquilizers 3. Antihypertensives 4. Anticonvulsants

Answer: 1 Explanation: 1. Antacids are routinely administered before surgery for a cesarean section.

The nurse is scheduling a client for an external cephalic version (ECV). Which finding in the client's chart requires immediate intervention? 1. Previous birth by cesarean 2. Frank breech ballotable 3. 37 weeks, complete breech 4. Failed ECV last week

Answer: 1 Explanation: 1. Any previous uterine scar is a contraindication to ECV. Prior scarring of the uterus may increase the risk of uterine tearing or uterine rupture.

A nursing instructor is demonstrating an assessment on a newborn using the Ballard gestational assessment tool. The nurse explains that which of the following tests should be performed after the first hour of birth, when the newborn has had time to recover from the stress of birth? 1. Arm recoil 2. Square window sign 3. Scarf sign 4. Popliteal angle

Answer: 1 Explanation: 1. Arm recoil is slower in healthy but fatigued newborns after birth; therefore, arm recoil is best elicited after the first hour of birth, when the baby has had time to recover from the stress of birth.

The nurse has received end-of-shift reports in the high-risk maternity unit. Which client should the nurse see first? 1. The client at 26 weeks' gestation with placenta previa experiencing blood on toilet tissue after a bowel movement 2. The client at 30 weeks' gestation with placenta previa whose fetal monitor strip shows late decelerations 3. The client at 35 weeks' gestation with grade I abruptio placentae in labor who has a strong urge to push 4. The client at 37 weeks' gestation with pregnancy-induced hypertension whose membranes ruptured spontaneously

Answer: 1 Explanation: 1. Assessment of the woman with placenta previa must be ongoing to prevent or treat complications that are potentially lethal to the mother and fetus. Painless, bright red vaginal bleeding is the best diagnostic sign of placenta previa. This client is the highest priority.

) The pediatric clinic nurse is reviewing lab results with a 2-month-old infant's mother. The infant's hemoglobin has decreased since birth. Which statement by the mother indicates the need for additional teaching? 1. "My baby isn't getting enough iron from my breast milk." 2. "Babies undergo physiologic anemia of infancy." 3. "This results from dilution because of the increased plasma volume." 4. "Delaying the cord clamping did not cause this to happen."

Answer: 1 Explanation: 1. At 2 months of age, infants increase their plasma volume, which results in physiologic anemia. This condition is not related to iron in the breast milk.

Babies should sleep in what position every time they are put down for sleep? 1. On their backs 2. On their stomachs 3. On their left sides 4. On their right sides

Answer: 1 Explanation: 1. Babies should sleep on their backs every time they are put down for sleep.

A woman has a hydatidiform mole (molar pregnancy) evacuated, and is prepared for discharge. The nurse should make certain that the client understands that what is essential? 1. That she not become pregnant until after the follow-up program is completed 2. That she receive RhoGAM with her next pregnancy and birth 3. That she has her blood pressure checked weekly for the next 30 days 4. That she seek genetic counseling with her partner before the next pregnancy

Answer: 1 Explanation: 1. Because of the risk of choriocarcinoma, the woman treated for hydatidiform mole should receive extensive follow-up therapy. Follow-up care includes a baseline chest X-ray to detect lung metastasis and a physical examination including a pelvic examination. The woman should avoid pregnancy during this time because the elevated hCG levels associated with pregnancy would cause confusion as to whether cancer had developed.

The home health nurse is visiting a new mother whose baby was delivered by emergency cesarean after a car accident. The mother seems dazed, irritable, and unaware of her surroundings. She tells the nurse she has had trouble sleeping. What would the nurse suspect that the mother has? 1. Post-traumatic stress disorder 2. Postpartum blues 3. Postpartum psychosis 4. Disenfranchised grief

Answer: 1 Explanation: 1. Because of the traumatic nature of the birth and the client's symptoms, this condition is most likely post-traumatic stress disorder (PTSD). At particular risk for PTSD are women who have histories of prior trauma and/or prior psychiatric histories and women who undergo emergency cesarean sections.

The nurse wishes to demonstrate to a new family their infant's individuality. Which assessment tool would be most appropriate for the nurse to use? 1. Brazelton Neonatal Behavioral Assessment Scale 2. New Ballard Score 3. Dubowitz gestational age scale 4. Ortolani maneuver

Answer: 1 Explanation: 1. Brazelton Neonatal Behavioral Assessment Scale is an assessment tool that identifies the newborn's repertoire of behavioral responses to the environment and documents the newborn's neurologic adequacy and capabilities.

The nurse is explaining to a new prenatal client that the certified nurse-midwife will perform clinical pelvimetry as a part of the pelvic exam. The nurse knows that teaching has been successful when the client makes which statement about the reason for the exam? 1. "It will help us know how big a baby I can deliver vaginally." 2. "Doing this exam is a part of prenatal care at this clinic." 3. "My sister had both of her babies by cesarean." 4. "I am pregnant with my first child."

Answer: 1 Explanation: 1. By performing a series of assessments and measurements, the examiner assesses the pelvis vaginally to determine whether the size and shape are adequate for a vaginal birth; this procedure is called clinical pelvimetry.

Which of the following is a localized, easily identifiable soft area of the infant's scalp, generally resulting from a long and difficult labor or vacuum extraction? 1. Caput succedaneum 2. Cephalohematoma 3. Molding 4. Depressed fontanelles

Answer: 1 Explanation: 1. Caput succedaneum is a localized, easily identifiable soft area of the scalp, generally resulting from a long and difficult labor or vacuum extraction.

A nurse needs to evaluate the progress of a woman's labor. The nurse obtains the following data: cervical dilatation 6 cm; contractions mild in intensity, occurring every 5 minutes, with a duration of 30-40 seconds. Which clue in this data does not fit the pattern suggested by the rest of the clues? 1. Cervical dilatation 6 cm 2. Mild contraction intensity 3. Contraction frequency every 5 minutes 4. Contraction duration 30-40 seconds

Answer: 1 Explanation: 1. Cervical dilatation of 6 cm indicates the active phase of labor. During this phase the cervix dilates from about 4 to 7 cm and contractions and pain intensify.

The postpartum client who delivered 2 days ago has developed endometritis. Which entry would the nurse expect to find in this client's chart? 1. "Cesarean birth after extended labor with ruptured membranes." 2. "Unassisted childbirth and afterbirth." 3. "External fetal monitoring used throughout labor." 4. "The client has history of pregnancy-induced hypertension."

Answer: 1 Explanation: 1. Cesarean birth is the single most significant risk of postpartum endometritis as well as prolonged premature rupture of the amniotic membranes (PPROM).

What type of testing is an inexpensive way to predict the presence of tubal disease and may be more predictive of infertility than an abnormal HSG? 1. Chlamydia trachomatis IgG antibody testing 2. Preimplantation genetic testing 3. Noninvasive prenatal testing (NIPT) 4. DNA testing

Answer: 1 Explanation: 1. Chlamydia trachomatis IgG antibody testing is an inexpensive way to predict the presence of tubal disease and may be more predictive of infertility than an abnormal HSG.

During a prenatal exam, a client describes several psychosomatic symptoms and has several vague complaints. What could these behaviors indicate? 1. Abuse 2. Mental illness 3. Depression 4. Nothing, they are normal

Answer: 1 Explanation: 1. Chronic psychosomatic symptoms and vague complaints can be indicators of abuse.

Which of the following is a benefit of delayed umbilical cord clamping for the preterm infant? 1. Fewer infants require blood transfusion for anemia 2. Fewer infants require blood transfusion for high blood pressure 3. Increase in the incidence of intraventricular hemorrhage 4. Increase in incidence of infant breastfeeding

Answer: 1 Explanation: 1. Clinical trials in preterm infants found that delaying umbilical cord clamping was associated with fewer infants who required blood transfusion for anemia.

Which nonspecific immune mechanism has the ability of antibodies and phagocytic cells to clear pathogens from an organism? 1. Complement 2. Coagulation 3. Inflammatory response 4. Phagocytosis

Answer: 1 Explanation: 1. Complement helps or "complements" the ability of antibodies and phagocytic cells to clear pathogens from an organism.

A laboring client's obstetrician has suggested amniotomy as a method for creating stronger contractions and facilitating birth. The client asks, "What are the advantages of doing this?" What should the nurse cite in response? 1. Contractions elicited are similar to those of spontaneous labor. 2. Amniotomy decreases the chances of a prolapsed cord. 3. Amniotomy reduces the pain of labor and makes it easier to manage. 4. The client will not need an episiotomy.

Answer: 1 Explanation: 1. Contractions after amniotomy are similar to those of spontaneous labor.

A newborn delivered at term is being discharged. The parents ask the nurse how to keep their baby warm. The nurse knows additional teaching is necessary if a parent states which of the following? 1. "A quick cool bath will help wake up my son for feedings." 2. "I can check my son's temperature under his arm." 3. "My baby should be dressed warmly, with a hat." 4. "Cuddling my son will help to keep him warm."

Answer: 1 Explanation: 1. Cool baths will chill a newborn, and should not be given. Bathing under warm water is ideal.

A 43-year-old client has just had a positive pregnancy test. She cries, and states, "I just don't know what I'll do. I can't be pregnant." Which nursing diagnosis would be the most appropriate? 1. Decisional Conflict related to unexpected pregnancy 2. Knowledge, Deficient related to advanced maternal age 3. Depression related to unexpected pregnancy 4. Health Maintenance, Ineffective related to advanced maternal age

Answer: 1 Explanation: 1. Decisional Conflict related to unexpected pregnancy is the most appropriate nursing diagnosis.

The client at 37 weeks' gestation calls the clinic nurse to report that neither she nor her partner has felt fetal movement for the past 48 hours. The nurse anticipates that the physician will order which test to assess fetal viability? 1. Ultrasound 2. Serum progesterone levels 3. Computed tomography (CT) scan 4. Contraction stress test

Answer: 1 Explanation: 1. Diagnosis of intrauterine fetal death (IUFD) is confirmed by visualization of the fetal heart with absence of heart action on ultrasound.

The nurse is presenting a preconception counseling class. The nurse instructs the participants that niacin intake should increase during pregnancy to promote metabolic coenzyme activity. The nurse will know that teaching has been effective if a client suggests which food as a source of niacin? 1. Fish 2. Apples 3. Broccoli 4. Milk

Answer: 1 Explanation: 1. Dietary sources of niacin include meats, fish, and whole grains.

What would be a normal cervical dilatation rate in a first-time mother ("primip")? 1. 1.5 cm per hour 2. Less than 1 cm cervical dilatation per hour 3. 1 cm per hour 4. Less than 0.5 cm per hour

Answer: 1 Explanation: 1. Dilatation in a "multip" is about 1.5 cm per hour.

A client has delivered a stillborn child at 26 weeks' gestation. She tells the nurse that none of her friends have called or visited, and that her husband's parents seem unwilling to talk about the loss. The nurse recognizes the mother's grief as which of the following? 1. Disenfranchised grief 2. Bereavement 3. An intuitive style of coping 4. Denial

Answer: 1 Explanation: 1. Disenfranchised grief is not supported by the usual societal customs. People are uncomfortable discussing the loss with the parents and often pull away when their support is most needed.

Doppler flow studies (umbilical velocimetry) help to assess which of the following? 1. Placental function and sufficiency 2. Fetal heart rate 3. Fetal growth and fluid levels 4. Maturity of the fetal lungs

Answer: 1 Explanation: 1. Doppler flow studies (umbilical velocimetry) help to assess placental function and sufficiency. Uteroplacental insufficiency is a risk for a woman with preeclampsia. If fetal growth restriction is present, Doppler velocimetry of the umbilical artery is useful for fetal surveillance.

Before the nurse begins to dry off the newborn after birth, which assessment finding should the nurse document to ensure an accurate gestational rating on the Ballard gestational assessment tool? 1. Amount and area of vernix coverage 2. Creases on the sole 3. Size of the areola 4. Body surface temperature

Answer: 1 Explanation: 1. Drying the baby after birth will disturb the vernix and potentially alter the gestational age criterion. The nurse should document the amount and areas of vernix coverage before drying the newborn.

The nurse is assessing clients after delivery. For which client is early discharge at 24 hours after delivery appropriate? 1. Woman and baby who have had two successful breastfeedings 2. Woman who is bottle-feeding her infant and has not voided since delivery 3. Twins delivered at 35 weeks, bottle-feeding 4. Cesarean birth performed for fetal distress

Answer: 1 Explanation: 1. Early discharge may be advantageous if mother and baby are doing well, help is available for the mother at home, and the family and physician/CNM agree that both clients are healthy and ready for discharge. Feeding successfully is one of the physiologic needs of the infant and both mother and infant appear to be doing well.

A woman is being treated for preterm labor with magnesium sulfate. The nurse is concerned that the client is experiencing early drug toxicity. What assessment finding by the nurse indicates early magnesium sulfate toxicity? 1. Patellar reflexes weak or absent 2. Increased appetite 3. Respiratory rate of 16 4. Fetal heart rate of 120

Answer: 1 Explanation: 1. Early signs of magnesium sulfate toxicity are related to a decrease in deep tendon reflexes.

A postpartum client has inflamed hemorrhoids. Which nursing intervention would be appropriate? 1. Encourage sitz baths. 2. Position the client in the supine position. 3. Avoid stool softeners. 4. Decrease fluid intake.

Answer: 1 Explanation: 1. Encouraging sitz baths is the correct approach because moist heat decreases inflammation and provides for comfort.

During a postpartum home visit, which step should the nurse take to establish a caring relationship? 1. Ask family members how they want to be addressed. 2. Do a portion of what the nurse agrees to do for the family, to avoid overwhelming them. 3. Speak directly to the father when asking questions. 4. Present information to the family instead of asking questions.

Answer: 1 Explanation: 1. Establishing a caring relationship starts with introducing yourself to the family and by calling the family members by their surnames until you have been invited to use the given or a less formal name.

The client has been pushing for 3 hours, and the fetus is making a slow descent. The partner asks the nurse whether pushing for this long is normal. How should the nurse respond? 1. "Your baby is taking a little longer than average, but is making progress." 2. "First babies take a long time to be born. The next baby will be easier." 3. "The birth would go faster if you had taken prenatal classes and practiced." 4. "Every baby is different; there really are no norms for labor and birth."

Answer: 1 Explanation: 1. Establishing rapport and a trusting relationship and providing information that is true is best response.

The nurse is working with male teens whose partners are pregnant. What statement by the father-to-be requires that the nurse intervene? 1. He will be the only other person who will be present for the birth, although his girlfriend wants her mother to be with her. 2. He was very sexually active at an earlier age and he has had more sexual partners than his girlfriend. 3. The pregnancy does not seem real to him, and he is not sure what he should do to plan for the future. 4. He does not want to be married.

Answer: 1 Explanation: 1. Even if the adolescent father has been included in the health care of the client throughout the pregnancy, it is not unusual for her to want her mother as her primary support person during labor and birth. Overriding his girlfriend's expressed desire could be an indication that their relationship is abusive.

A 38-year-old client in her second trimester states a desire to begin an exercise program to decrease her fatigue. What is the most appropriate nursing response? 1. "Fatigue should resolve in the second trimester, but walking daily might help." 2. "Avoid a strenuous exercise regimen at your age. Drink coffee to combat fatigue." 3. "Avoid an exercise regimen due to your pregnancy. Try to nap daily." 4. "Fatigue will increase as pregnancy progresses, but running daily might help."

Answer: 1 Explanation: 1. Even mild to moderate exercise is beneficial during pregnancy. Regular exercise-at least 30 minutes of moderate exercise daily or at least most days of the week-is preferred.

The nurse is teaching a prenatal class about feeding methods. A father-to-be asks the nurse which method, breast or formula, leads to the fastest infant growth and weight gain. Which response by the nurse is best? 1. "In the first 3 to 4 months breastfed babies tend to gain weight faster." 2. "In the first 3 to 4 months there is no difference in weight gain." 3. "In the first 3 to 4 months bottle-fed babies grow faster." 4. "In the first 3 to 4 months growth isn't as important as your comfort with the method."

Answer: 1 Explanation: 1. Exclusively breastfed infants have the same or slightly higher weight gain than their formula-fed and combination-fed peers in the first 3 to 4 months.

A 3-month-old baby who was born at 25 weeks has been exposed to prolonged oxygen therapy. Due to oxygen therapy, the nurse explains to the parents, their infant is at a greater risk for which of the following? 1. Visual impairment 2. Hyperthermia 3. Central cyanosis 4. Sensitive gag reflex

Answer: 1 Explanation: 1. Extremely premature newborns are particularly susceptible to injury of the delicate capillaries of the retina causing characteristic retinal changes known as retinopathy of prematurity (ROP). Judicious use of supplemental oxygen therapy in the premature infant has become the norm.

In planning care for a new family immediately after birth, which procedure would the nurse most likely withhold for 1 hour to allow time for the family to bond with the newborn? 1. Eye prophylaxis medication 2. Drying the newborn 3. Vital signs 4. Vitamin K injection

Answer: 1 Explanation: 1. Eye prophylaxis medication instillation may be delayed up to 1 hour after birth to allow eye contact during parent-newborn bonding.

The nurse is supervising care by a new graduate nurse who is working with a couple who have experienced a stillbirth. Which statement made by the new nurse indicates that further instruction is necessary? 1. "I should stay out of their room as much as possible." 2. "The parents might express their grief differently from each other." 3. "My role is to help the family communicate and cope." 4. "Hopelessness might be expressed by this family."

Answer: 1 Explanation: 1. Families experiencing perinatal loss need support. The nurse should stay with the couple so they do not feel alone and isolated; however, cues that the couple wants to be alone should be assessed continuously.

A client who delivered 2 hours ago tells the nurse that she is exhausted and feels guilty because her friends told her how euphoric they felt after giving birth. How should the nurse respond? 1. "Everyone is different, and both responses are normal." 2. "Most mothers do feel euphoria; I don't know why you don't." 3. "It's good for me to know that because it might indicate a problem." 4. "Let me bring your baby to the nursery so that you can rest."

Answer: 1 Explanation: 1. Following birth, some women feel exhausted and in need of rest. Other women are euphoric and full of psychic energy, ready to retell their experience of birth repeatedly.

A nurse is checking the postpartum orders. The doctor has prescribed bed rest for 6-12 hours. The nurse knows this is an appropriate order if the client had which type of anesthesia? 1. Spinal 2. Pudendal 3. General 4. Epidural

Answer: 1 Explanation: 1. Following the birth, the woman may be kept flat. Although the effectiveness of the supine position to avoid headache following a spinal is controversial, the physician's orders may include lying flat for 6 to 12 hours.

The prenatal clinic nurse is caring for a 15-year-old client who is at 8 weeks' gestation. The client asks the nurse why she is supposed to gain so much weight. What is the best response by the nurse? 1. "Gaining 25-35 pounds is recommended for healthy fetal growth." 2. "It's what your certified nurse-midwife recommended for you." 3. "Inadequate weight gain delays lactation after delivery." 4. "Weight gain is important to ensure that you get enough vitamins."

Answer: 1 Explanation: 1. For an appropriate-weight woman, 25-35 pounds of weight gain is recommended for optimal fetal growth and development.

The nurse is caring for a laboring client with thrombocytopenia. During labor, it is determined that the client requires a cesarean delivery. The nurse is preparing the client for surgery, and should instruct the client that the recommended method of anesthesia is which of the following? 1. General anesthesia 2. Epidural anesthesia 3. Spinal anesthesia 4. Regional anesthesia

Answer: 1 Explanation: 1. General anesthesia will be recommended. Women with thrombocytopenia should avoid regional blocks.

The 19-year-old pregnant woman begins a job to "save money for the baby." What is the most significant developmental task the nurse understands this statement to demonstrate? 1. Striving for gaining autonomy and independence 2. Completed development of a sense of identity 3. Attainment of a sense of achievement 4. Having developed an intimate relationship

Answer: 1 Explanation: 1. Having a job is how most teens develop financial independence and autonomy.

The nurse at the prenatal clinic has four calls to return. Which phone call should the nurse return first? 1. Client at 32 weeks, reports headache and blurred vision. 2. Client at 18 weeks, reports no fetal movement in this pregnancy. 3. Client at 16 weeks, reports increased urinary frequency. 4. Client at 40 weeks, reports sudden gush of fluid and contractions.

Answer: 1 Explanation: 1. Headache and blurred vision are signs of preeclampsia, which is potentially life-threatening for both mother and fetus. This client has top priority.

A nurse is discussing diet with a pregnant woman. Which food should the nurse advise the client to avoid during her pregnancy? 1. Bologna 2. Cantaloupe 3. Spinach 4. Cornbread

Answer: 1 Explanation: 1. Hot dogs and other luncheon meats should not be eaten during pregnancy unless they are fully cooked.

The nurse is caring for a pregnant woman who admits to using cocaine and ecstasy on a regular basis. The client states, "Everybody knows that alcohol is bad during pregnancy, but what's the big deal about ecstasy?" What is the nurse's best response? 1. "Ecstasy can cause a high fever in you and therefore cause the baby harm." 2. "Ecstasy leads to deficiencies of thiamine and folic acid, which help the baby develop." 3. "Ecstasy produces babies with small heads and short bodies with brain function alterations." 4. "Ecstasy produces intrauterine growth restriction and meconium aspiration."

Answer: 1 Explanation: 1. Hyperthermia (elevated temperature) is a side effect of MDMA (ecstasy).

The nurse is caring for the newborn of a diabetic mother whose blood glucose level is 39 mg/dL. What should the nurse include in the plan of care for this newborn? 1. Offer early feedings with formula or breast milk. 2. Provide glucose water exclusively. 3. Evaluate blood glucose levels at 12 hours after birth. 4. Assess for hypothermia.

Answer: 1 Explanation: 1. IDMs whose serum glucose falls below 40 mg/dL should have early feedings with formula or breast milk (colostrum).

A woman's history and appearance suggest drug abuse. What is the nurse's best approach? 1. Ask the woman directly, "Do you use any street drugs?" 2. Ask the woman whether she would like to talk to a counselor. 3. Ask some questions about over-the-counter medications and avoid mention of illicit drugs. 4. Explain how harmful drugs can be for her baby.

Answer: 1 Explanation: 1. If drug abuse is suspected, the nurse should ask direct questions and be matter-of-fact and nonjudgmental to elicit honest responses.

An analgesic medication has been administered intramuscularly to a client in labor. How would the nurse evaluate if the medication was effective? 1. The client dozes between contractions. 2. The client is moaning during contractions. 3. The contractions decrease in intensity. 4. The contractions decrease in frequency.

Answer: 1 Explanation: 1. If the client dozes between contractions, the analgesic is effective. Analgesics decrease discomfort and increase relaxation.

The postpartum multipara is breastfeeding her new baby. The client states that she developed mastitis with her first child, and asks whether there is something she can do to prevent mastitis this time. What would the best response of the nurse be? 1. "Massage your breasts on a daily basis, and if you find a hardened area, massage it towards the nipple." 2. "Most first-time moms experience mastitis. It is really quite unusual for a woman having her second baby to get it again." 3. "Apply cabbage leaves to any areas that feel thickened or firm to relieve the swelling." 4. "Take your temperature once a day. This will help you to pick up the infection early, before it becomes severe."

Answer: 1 Explanation: 1. If the mother finds that one area of her breast feels distended or lumpy, she can massage the lumpy area toward the nipple as the infant nurses.

A client delivered 30 minutes ago. Which postpartal assessment finding would require close nursing attention? 1. A soaked perineal pad since the last 15-minute check 2. An edematous perineum 3. The client experiencing tremors 4. A fundus located at the umbilicus

Answer: 1 Explanation: 1. If the perineal pad becomes soaked in a 15-minute period or if blood pools under the buttocks, continuous observation is necessary. As long as the woman remains in bed during the first hour, bleeding should not exceed saturation of one pad.

The client gave birth to a 7 pound, 14 ounce female 30 minutes ago. The placenta has not yet delivered. Manual removal of the placenta is planned. What should the nurse prepare to do? 1. Start an IV of lactated Ringer's. 2. Apply anti-embolism stockings. 3. Bottle-feed the infant. 4. Send the placenta to pathology.

Answer: 1 Explanation: 1. In women who do not have an epidural in place, intravenous sedation may be required because of the discomfort caused by the procedure. An IV is necessary.

A 28-year-old woman has been an insulin-dependent diabetic for 10 years. At 36 weeks' gestation, she has an amniocentesis. A lecithin/sphingomyelin (L/S) ratio test is performed on the sample of her amniotic fluid. Because she is a diabetic, what would an obtained 2:1 ratio indicate for the fetus? 1. The fetus may or may not have immature lungs. 2. The amniotic fluid is contaminated. 3. The fetus has a neural tube defect. 4. There is blood in the amniotic fluid.

Answer: 1 Explanation: 1. Infants of diabetic mothers (IDMs) have a high incidence of false-positive results (i.e., the L/S ratio is thought to indicate lung maturity, but after birth the baby develops RDS).

Infants of women with preeclampsia during pregnancy tend to be small for gestational age (SGA) because of which condition? 1. Intrauterine growth restriction 2. Oliguria 3. Proteinuria 4. Hypertension

Answer: 1 Explanation: 1. Infants of women with preeclampsia during pregnancy tend to be small for gestational age (SGA) because of intrauterine growth restriction. The cause is related specifically to maternal vasospasm and hypovolemia, which result in fetal hypoxia and malnutrition.

The nurse is performing an assessment on an infant whose mother states that she feeds the infant in a supine position by propping the bottle. Based on this information, what would the nurse include in the assessment? 1. Otoscopic exam of the eardrum 2. Bowel sounds 3. Vital signs 4. Skin assessment

Answer: 1 Explanation: 1. Infants who bottle feed in a supine position have an increased risk of otitis media and dental caries in the older infant.

The client with insulin-dependent type 2 diabetes and an HbA1c of 5.0% is planning to become pregnant soon. What anticipatory guidance should the nurse provide this client? 1. Insulin needs decrease in the first trimester and usually begin to rise late in the first trimester as glucose use and glycogen storage by the woman and fetus increase. 2. The risk of ketoacidosis decreases during the length of the pregnancy. 3. Vascular disease that accompanies diabetes slows progression. 4. The baby is likely to have a congenital abnormality because of the diabetes.

Answer: 1 Explanation: 1. Insulin needs decrease in the first trimester and usually begin to rise late in the first trimester as glucose use and glycogen storage by the woman and fetus increase.

The homecare nurse is visiting a newborn-and-mother couplet. Which nursing action has the highest priority? 1. Establish rapport with the family members. 2. Review the hospital medical records. 3. Determine the newborn's sleeping arrangements. 4. Examine the umbilical cord stump.

Answer: 1 Explanation: 1. It is critical to establish rapport with family members prior to beginning any assessments. The nurse can rely on the same characteristics of a caring relationship that have been integral to hospital-based practice-regard for patients, genuineness, empathy, and establishment of trust and rapport. Page Ref: 916

The nurse is planning to teach couples factors that influence fertility. Which factor should not be included in the teaching plan? 1. Sexual intercourse should occur every day of the week. 2. Get up to urinate 1 hour after intercourse. 3. Do not douche. 4. Institute stress-reduction techniques.

Answer: 1 Explanation: 1. It is optimal if sexual intercourse occurs every other day during the fertile period.

Which assessment findings by the nurse would require obtaining a blood glucose level on the newborn? 1. Jitteriness 2. Sucking on fingers 3. Lusty cry 4. Axillary temperature of 98°F

Answer: 1 Explanation: 1. Jitteriness of the newborn is associated with hypoglycemia. Aggressive treatment is recommended after a single low blood glucose value if the infant shows this symptom.

The nurse is responding to phone calls. Whose call should the nurse return first? 1. A client at 37 weeks' gestation reports no fetal movement for 24 hours. 2. A client at 29 weeks' gestation reports increased fetal movement. 3. A client at 32 weeks' gestation reports decreased fetal movement X 2 days. 4. A client at 35 weeks' gestation reports decreased fetal movement X 4 hours.

Answer: 1 Explanation: 1. Lack of fetal movement can be an indication of nonreassuring fetal status or even fetal death. This client is the highest priority.

The nurse is preparing a prenatal class about infant feeding methods. The maternal nutritional requirements for breastfeeding and formula-feeding will be discussed. What statement should the nurse include? 1. "Breastfeeding requires a continued high intake of protein and calcium." 2. "Formula-feeding mothers should protect their health with a lot of calcium." 3. "Producing breast milk requires calories, but any source of food is fine." 4. "Formula-feeding mothers need a high protein intake to avoid fatigue."

Answer: 1 Explanation: 1. Lactation requires calories, along with increased protein and calcium intake.

The nurse is evaluating the effectiveness of phototherapy on a newborn. Which evaluation indicates a therapeutic response to phototherapy? 1. The newborn maintains a normal temperature 2. An increase of serum bilirubin levels 3. Weight loss 4. Skin blanching yellow

Answer: 1 Explanation: 1. Maintenance of temperature is an important aspect of phototherapy because the newborn is naked except for a diaper during phototherapy. The isolette helps the infant maintain his or her temperature while undressed.

The primiparous client has told the nurse that she is afraid she will develop hemorrhoids during pregnancy because her mother did. Which statement would be best for the nurse to make? 1. "It is not unusual for women to develop hemorrhoids during pregnancy." 2. "Most women don't have any problem until after they've delivered." 3. "If your mother had hemorrhoids, you will get them, too." 4. "If you get hemorrhoids, you probably will need surgery to get rid of them."

Answer: 1 Explanation: 1. Many pregnant women will develop hemorrhoids. Hemorrhoids are varicosities of the veins in the lower end of the rectum and anus. During pregnancy, the gravid uterus presses on the veins and interferes with venous circulation. As the pregnancy progresses, the straining that accompanies constipation can contribute to the development of hemorrhoids.

The nurse is working with a group of pregnant teens. Which statement indicates that teaching has been successful? 1. "Pregnant teens are more likely to quit school prior to graduation." 2. "Because I am young, I have a low risk for preeclampsia." 3. "My baby could come late because I am a teenager." 4. "I am more likely to use birth control after I have this baby."

Answer: 1 Explanation: 1. Many teenage mothers drop out of school during their pregnancy and then are less likely to complete their schooling.

The nurse seeks to involve the adolescent father in the prenatal care of his girlfriend. What is the rationale for this nursing strategy? 1. Having the father more involved with the birth 2. Avoiding conflict between the adolescent father and pregnant teenager 3. Including his name of the birth certificate 4. Avoiding legal action by the adolescent father's family

Answer: 1 Explanation: 1. Many young fathers genuinely want to be involved with their children and would have more contact and input if they could.

The nurse would expect a physician to prescribe which medication to a postpartum client with heavy bleeding and a boggy uterus? 1. Methylergonovine maleate (Methergine) 2. Rh immune globulin (RhoGAM) 3. Terbutaline (Brethine) 4. Docusate (Colace)

Answer: 1 Explanation: 1. Methylergonovine maleate is the drug used for the prevention and control of postpartum hemorrhage.

A new mother at 36 hours post-delivery has asked to be discharged to home. The nurse explains that criteria for discharge before the newborn is 48 hours old include which of the following? 1. The newborn's respiratory rate is less than 60/min. 2. Singleton birth at a minimum 35 weeks' gestation. 3. The newborn has passed at least three spontaneous stools. 4. The newborn has normal and stable vital signs for 24 hours before discharge.

Answer: 1 Explanation: 1. Minimal criteria include a respiratory rate in the newborn less than 60/min.

The nurse in a fertility clinic is working with a woman who has been undergoing infertility treatment with clomiphene citrate. Which statement would the nurse expect the woman to make? 1. "I feel moody so much of the time." 2. "If this doesn't work, I think my husband will leave me." 3. "This medication will guarantee a pregnancy." 4. "My risk of twins or triplets is the same as for the general population."

Answer: 1 Explanation: 1. Mood swings are a side effect of clomiphene citrate.

The nurse is working with a family who experienced the stillbirth of a son 2 months ago. Which statement by the mother would be expected? 1. "I seem to keep crying for no reason." 2. "The death of my son hasn't changed my life." 3. "I have not visited my son's gravesite." 4. "I feel happy all the time."

Answer: 1 Explanation: 1. Mourning may be manifested by certain behaviors and rituals, such as weeping, which help the person experience, accept, and adjust to the loss.

A woman in active labor is given nalbuphine hydrochloride (Nubain) 14 mg IV for pain relief. Half an hour later, her respirations are at 8 per minute. The physician would likely order which medication for this client? 1. Narcan 2. Reglan 3. Benadryl 4. Vistaril

Answer: 1 Explanation: 1. Narcan is useful for respiratory depression caused by nalbuphine (Nubain).

A Navajo client who is 36 weeks pregnant meets with a traditional healer as well as her physician. What does the nurse understand this to mean? 1. The client is seeking spiritual direction. 2. The client does not trust her physician. 3. The client will not adapt to mothering well. 4. The client is experiencing complications of pregnancy.

Answer: 1 Explanation: 1. Navajo clients are aware of the mind-soul connection, and might try to follow certain practices to have a healthy pregnancy and birth. Practices could include focus on peace and positive thoughts as well as certain types of prayers and ceremonies. A traditional healer may assist them.

The nurse is caring for the newborn of a diabetic mother. Which of the following should be included in the nurse's plan of care for this newborn? 1. Offer early feedings. 2. Administer an intravenous infusion of glucose. 3. Assess for hypercalcemia. 4. Assess for hyperbilirubinemia immediately after birth.

Answer: 1 Explanation: 1. Newborns of diabetic mothers may benefit from early feeding as they are extremely valuable in maintaining normal metabolism and lowering the possibility of such complications as hypoglycemia and hyperbilirubinemia.

The nurse has completed a community education session on growth patterns of infants. Which statement by a participant indicates that additional teaching is needed? 1. "Newborns should regain their birth weight by 1 week of age." 2. "Breastfed and formula-fed babies have different growth rates." 3. "Formula-fed infants regain their birth weight earlier than breastfed infant." 4. "Healthcare providers consider breastfeeding to be the 'gold standard' for neonatal nutrition."

Answer: 1 Explanation: 1. Newborns should gain at least 10 g/kg/day and be back to birth weight no later than day 14 of life.

In planning care for the fetal alcohol syndrome (FAS) newborn, which intervention would the nurse include? 1. Allow extra time with feedings. 2. Assign different personnel to the newborn each day. 3. Place the newborn in a well-lit room. 4. Monitor for hyperthermia.

Answer: 1 Explanation: 1. Newborns with fetal alcohol syndrome have feeding problems. Because of their feeding problems, these infants require extra time and patience during feedings.

The nurse is assessing a new mother 2 days after a normal vaginal delivery. The mother has chosen not to breastfeed. What would an abnormal finding be? 1. Weight loss of 3 pounds 2. Small amount of breast milk expressed 3. Pink striae on the abdomen 4. Lochia serosa

Answer: 1 Explanation: 1. Normal weight loss postpartum is in the range of 12 to 20-plus pounds.

The mother of a premature newborn questions why a gavage feeding catheter is placed in the mouth of the newborn and not in the nose. What is the nurse's best response? 1. "Most newborns are nose breathers." 2. "The tube will elicit the sucking reflex." 3. "A smaller catheter is preferred for feedings." 4. "Most newborns are mouth breathers."

Answer: 1 Explanation: 1. Orogastric insertion is preferable to nasogastric because most infants are obligatory nose breathers.

The nurse administered oxytocin 20 units at the time of placental delivery. Why was this primarily done? 1. To contract the uterus and minimize bleeding 2. To decrease breast milk production 3. To decrease maternal blood pressure 4. To increase maternal blood pressure

Answer: 1 Explanation: 1. Oxytocin is given to contract the uterus and minimize bleeding.

A woman has been having contractions since 4 a.m. At 8 a.m., her cervix is dilated to 5 cm. Contractions are frequent, and mild to moderate in intensity. Cephalopelvic disproportion (CPD) has been ruled out. After giving the mother some sedation so she can rest, what would the nurse anticipate preparing for? 1. Oxytocin induction of labor 2. Amnioinfusion 3. Increased intravenous infusion 4. Cesarean section

Answer: 1 Explanation: 1. Oxytocin is the drug of choice for labor augmentation or labor induction and may be administered as needed for hypotonic labor patterns.

What information should the nurse include when teaching a new mother how to successfully bottle-feed her newborn? 1. Proper dilution of powdered formula is essential to provide adequate nutrition. 2. Keep formula at room temperature for at least 4 hours to warm it, instead of microwaving it. 3. Use enough water to dilute the nutrient and calorie density so the infant will drink more formula. 4. Freeze newly prepared formula for up to 3 months.

Answer: 1 Explanation: 1. Parents should be instructed to follow the directions on the formula can label precisely as written.

A postpartum client calls the nursery to report that her newborn's umbilical cord stump is draining, and has a foul odor. What is the nurse's best response? 1. "Take your newborn to the pediatrician." 2. "Cover the cord stump with gauze." 3. "Apply Betadine around the cord stump." 4. "This is normal during healing."

Answer: 1 Explanation: 1. Parents should check cord each day for any odor, oozing of greenish yellow material, or reddened areas around the cord. They should report to healthcare provider any signs of infection.

A woman at 7 weeks' gestation is diagnosed with hyperemesis gravidarum. Which nursing diagnosis would receive priority? 1. Fluid Volume: Deficient 2. Cardiac Output, Decreased 3. Injury, Risk for 4. Nutrition, Imbalanced: Less than Body Requirements

Answer: 1 Explanation: 1. The newly admitted client with hyperemesis gravidarum has been experiencing excessive vomiting, and is in a fluid volume-deficit state.

The nurse is preparing to assess the fetus of a laboring client. Which assessment should the nurse perform first? 1. Perform Leopold maneuvers to determine fetal position. 2. Count the fetal heart rate between, during, and for 30 seconds following a uterine contraction (UC). 3. Dry the maternal abdomen before using the Doppler. 4. The diaphragm should be cooled before using the Doppler.

Answer: 1 Explanation: 1. Performing Leopold maneuvers is the first step.

The home care nurse is examining a 3-day-old infant. The child's skin on the sternum is yellow when blanched with a finger. The parents ask the nurse why jaundice occurs. What is the best response from the nurse? 1. "The liver of an infant is not fully mature, and doesn't conjugate the bilirubin for excretion." 2. "The infant received too many red blood cells after delivery because the cord was not clamped immediately." 3. "The yellow color of your baby's skin indicates that you are breastfeeding too often." 4. "This is an abnormal finding related to your baby's bowels not excreting bilirubin as they should."

Answer: 1 Explanation: 1. Physiologic jaundice is a common occurrence, and peaks at 3 to 5 days in term infants. The reduction in hepatic activity, along with a relatively large bilirubin load, decreases the liver's ability to conjugate bilirubin and increases susceptibility to jaundice.

The mother of a 3-day-old infant calls the clinic and reports that her baby's skin is turning slightly yellow. What should the nurse explain to the mother? 1. Physiologic jaundice is normal, and peaks at this age. 2. The newborn's liver is not working as well as it should. 3. The baby is yellow because the bowels are not excreting bilirubin. 4. The yellow color indicates that brain damage might be occurring.

Answer: 1 Explanation: 1. Physiologic jaundice occurs soon after birth. Bilirubin levels peak at 3 to 5 days in term infants.

The nurse knows that a contraindication to the induction of labor is which of the following? 1. Placenta previa 2. Isoimmunization 3. Diabetes mellitus 4. Premature rupture of membranes

Answer: 1 Explanation: 1. Placenta previa is a contraindication to the induction of labor.

A breastfeeding postpartum client reports sore nipples to the nurse during a home visit. What intervention would be the highest priority? 1. Infant positioning 2. Use of the breast shield 3. Use of breast pads 4. Type of soap used

Answer: 1 Explanation: 1. Poor latch and/or suck are the primary causes of nipple soreness and the baby's position at the breast is a critical factor in nipple soreness. Encouraging the mother to rotate positions when feeding the infant may decrease nipple soreness. Changing positions alters the focus of greatest stress and promotes more complete breast emptying.

The labor and delivery nurse is caring for a client whose labor is being induced due to fetal death in utero at 35 weeks' gestation. In planning intrapartum care for this client, which nursing diagnosis is most likely to be applied? 1. Powerlessness 2. Urinary Elimination, Impaired 3. Coping: Family, Readiness for Enhanced 4. Skin Integrity, Impaired

Answer: 1 Explanation: 1. Powerlessness is commonly experienced by families who face fetal loss. Powerlessness is related to lack of control in current situational crisis.

The nurse is inducing the labor of a client with severe preeclampsia. As labor progresses, fetal intolerance of labor develops. The induction medication is turned off, and the client is prepared for cesarean birth. Which statement should the nurse include in her preoperative teaching? 1. "Because of your preeclampsia you are at higher risk for hypotension after an epidural anesthesia." 2. "Because of your preeclampsia you might develop hypertension after a spinal anesthesia." 3. "Because of your preeclampsia your baby might have decreased blood pressure after birth." 4. "Because of your preeclampsia

Answer: 1 Explanation: 1. Pregnancies complicated by preeclampsia are high-risk situations. The woman with mild preeclampsia usually may have the analgesia or anesthesia of choice, although the incidence of hypotension with epidural anesthesia is increased. If hypotension occurs with the epidural block, it provides further stress on an already compromised cardiovascular system.

A client is admitted to the birth setting in early labor. She is 3 cm dilated, -2 station, with intact membranes, and FHR of 150 bpm. Her membranes rupture spontaneously, and the FHR drops to 90 bpm with variable decelerations. What would the nurse's initial response be? 1. Perform a vaginal exam 2. Notify the physician 3. Place the client in a left lateral position 4. Administer oxygen at 2 L per nasal cannula

Answer: 1 Explanation: 1. Prolapsed umbilical cord can occur when the membranes rupture. The fetus is more likely to experience variable decelerations because the amniotic fluid is insufficient to keep pressure off the umbilical cord. A vaginal exam is the best way to confirm.

The nurse is completing the discharge teaching of a young first-time mother. Which statement by the mother requires immediate intervention? 1. "I will put my baby to bed with his bottle so he doesn't get hungry during the night." 2. "My baby will probably have a bowel movement each breastfeeding, and will wet often." 3. "Nursing every 2 to 3 hours is normal, for a total of 8 to 12 feedings every day." 4. "I will drink fenugreek tea from my grandmother to prevent my milk from coming in."

Answer: 1 Explanation: 1. Putting a baby to bed with a propped bottle is a choking hazard, and should never be done.

The nurse is reviewing charts of clients who underwent cesarean births by request in the last two years. The hospital is attempting to decrease costs of maternity care. What findings contribute to increased health care costs in clients undergoing cesarean birth by request? 1. Increased abnormal placenta implantation in subsequent pregnancies 2. Decreased use of general anesthesia with greater use of epidural anesthesia 3. Prolonged anemia, requiring blood transfusions every few months 4. Coordination of career projects of both partners leading to increased income

Answer: 1 Explanation: 1. Repeat cesarean births are associated with greater risks including increased incidence of abnormal placentation in subsequent pregnancies and the increased risk of mortality secondary to surgery, which would contribute to increased health care costs.

The community nurse is working with a client from Southeast Asia who has delivered her first child. Her mother has come to live with the family for several months. The nurse understands that the main role of the grandmother while visiting is to do which of the following? 1. Help the new mother by allowing her to focus on resting and caring for the baby. 2. Teach her son-in-law the right way to be a father because this is his first child. 3. Make sure that her daughter does not become abusive towards the infant. 4. Pass on the cultural values and beliefs to the newborn grandchild.

Answer: 1 Explanation: 1. Rest, seclusion, and dietary restraint practices in many traditional non-Western cultures (South Asian groups) are designed to assist the woman and her baby during postpartum vulnerable periods.

The client with blood type A, Rh-negative, delivered yesterday. Her infant is blood type AB, Rh-positive. Which statement indicates that teaching has been effective? 1. "I need to get RhoGAM so I don't have problems with my next pregnancy." 2. "Because my baby is Rh-positive, I don't need RhoGAM." 3. "If my baby had the same blood type I do, it might cause complications." 4. "Before my next pregnancy, I will need to have a RhoGAM shot."

Answer: 1 Explanation: 1. Rh-negative mothers who give birth to Rh-positive infants should receive Rh immune globulin (RhoGAM) to prevent alloimmunization.

A pregnant client asks the nurse, "What is this "knuckle test" that is supposed to tell whether my baby has a genetic problem?" What does the nurse correctly explain? 1. "In the first trimester, the nuchal translucency measurement is added to improve the detection rate for Down syndrome and trisomy 18." 2. "You will need to ask the physician for an explanation." 3. "It tests for hemophilia A or B." 4. "It tests for Duchenne muscular dystrophy."

Answer: 1 Explanation: 1. Screening tests, such as nuchal translucency ultrasound are designed to gather information about the risk that the pregnancy could have chromosome abnormalities or open spina bifida.

Which of the following conditions would predispose a client for thrombophlebitis? 1. Severe anemia 2. Cesarean delivery 3. Anorexia 4. Hypocoagulability

Answer: 1 Explanation: 1. Severe anemia would predispose a client for thrombophlebitis.

The introduction of a new baby into the family is often the beginning of which of the following? 1. Sibling rivalry 2. Inconsistent childrearing 3. Toilet training 4. Weaning

Answer: 1 Explanation: 1. Sibling rivalry results from children's fear of change in the security of their relationships with their parents.

The parents of a newborn are receiving discharge teaching. The nurse explains that the infant should have several wet diapers per day. Which statement by the parents indicates that further education is necessary? 1. "Our baby was born with kidneys that are too small." 2. "A baby's kidneys don't concentrate urine well for several months." 3. "Feeding our baby frequently will help the kidneys function." 4. "Kidney function in an infant is very different from that in an adult."

Answer: 1 Explanation: 1. Size of the kidneys is rarely an issue.

The nurse is teaching a class on vaginal birth after cesarean (VBAC). Which statement by a participant indicates that additional information is needed? 1. "Because the scar on my belly goes down from my navel, I am not a candidate for a VBAC." 2. "My first baby was in a breech position, so for this pregnancy, I can try a VBAC if the baby is head-down." 3. "Because my hospital is so small and in a rural area, they won't let me attempt a VBAC." 4. "The rate of complications from VBAC is lower than the rate of complications from a cesarean."

Answer: 1 Explanation: 1. Skin incision is not indicative of uterine incision. Only the uterine incision is a factor in deciding whether VBAC is advisable. The classic vertical incision was commonly done in the past and is associated with increased risk of uterine rupture in subsequent pregnancies and labor.

The nurse is instructing parents of a newborn about voiding and stool characteristics. Which of the following would be considered an abnormal pattern? 1. Large amounts of uric acid crystals in the first days of life 2. At least 6 to 10 wet diapers a day after the first few days of life 3. 1 to 2 stools a day for formula-fed baby 4. Urine that is straw to amber color without foul smell

Answer: 1 Explanation: 1. Small, not large, amounts of uric acid crystals are normal in the first days of life.

After teaching a pregnant client about the effects of smoking on pregnancy, the nurse knows that the client needs further education when she makes which statement? 1. "I am at increased risk for preeclampsia." 2. "I am at increased risk for preterm birth." 3. "I am at increased risk for placenta previa." 4. "I am at increased risk for abruptio placentae."

Answer: 1 Explanation: 1. Smoking is not associated with increased risk for preeclampsia.

The parents of a newborn have just been told their infant has tetralogy of Fallot. The parents do not seem to understand the explanation given by the physician. What statement by the nurse is best? 1. "With this defect, not enough of the blood circulates through the lungs, leading to a lack of oxygen in the baby's body." 2. "The baby's aorta has a narrowing in a section near the heart that makes the left side of the heart work harder." 3. "The blood vessels that attach to the ventricles of the heart are positioned on the wrong sides of the heart." 4. "Your baby's heart doesn't circulate blood well because the left ventricle is smaller and thinner than normal."

Answer: 1 Explanation: 1. Tetralogy of Fallot is a cyanotic heart defect that comprises four abnormalities: pulmonary stenosis, ventricular septal defect, overriding aorta, and right ventricle hypertrophy. The severity of symptoms depends on the degree of pulmonary stenosis, the size of the ventricular septal defect, and the degree to which the aorta overrides the septal defect.

The nurse discussing different pregnancy programs explains that which of the following is critical to the success of an adolescent pregnancy-prevention program? 1. Including role models from the same cultural and racial backgrounds 2. Focusing on the adolescent female 3. Having short-term, informal programs available twice per year 4. Focusing on the expectations of the adolescents' parents

Answer: 1 Explanation: 1. The National Campaign to Prevent Teen and Unplanned Pregnancy's task forces found that the programs most effective at preventing teen pregnancy include models from the same cultural and racial backgrounds as the participants.

The nurse assesses the postpartum client to have moderate lochia rubra with clots. Which nursing intervention would be appropriate? 1. Assess fundus and bladder status. 2. Catheterize the client. 3. Administer Methergine IM per order. 4. Contact the physician immediately.

Answer: 1 Explanation: 1. The amount, consistency, color, and odor of the lochia are monitored on an ongoing basis. Increased bleeding is most often related to uterine atony and responds to fundal massage, expression of any clots, and emptying the bladder.

The nurse is making an initial assessment of the newborn. Which of the following data would be considered normal? 1. Chest circumference 31.5 cm, head circumference 33.5 cm 2. Chest circumference 30 cm, head circumference 29 cm 3. Chest circumference 38 cm, head circumference 31.5 cm 4. Chest circumference 32.5 cm, head circumference 36 cm

Answer: 1 Explanation: 1. The average circumference of the head at birth is 32 to 37 cm. Average chest circumference ranges from 30 to 35 cm at birth. The circumference of the head is approximately 2 cm greater than the circumference of the chest at birth. Answer 1 is the only choice in which both the chest and head circumferences fall within the norm in terms of actual size and comparable size.

Two hours ago, a client at 39 weeks' gestation was 3 cm dilated, 40% effaced, and +1 station. Frequency of contractions was every 5 minutes with duration 40 seconds and intensity 50 mmHg. The current assessment is 4 cm dilated, 40% effaced, and +1 station. Frequency of contractions is now every 3 minutes with 40-50 seconds' duration and intensity of 40 mmHg. What would the priority intervention be? 1. Begin oxytocin after assessing for CPD. 2. Give terbutaline to stop the preterm labor. 3. Start oxygen at 8 L/min. 4. Have the anesthesiologist give the client an epidural.

Answer: 1 Explanation: 1. The client is having hypertonic contractions. Cephalopelvic disproportion (CPD) must be excluded. If CPD exists, oxytocin (Pitocin) augmentation should not be used. Oxytocin is the drug of choice for labor augmentation or labor induction.

The client delivered by cesarean birth 3 days ago and is being discharged. Which statement should the nurse include in the discharge teaching? 1. "If your incision becomes increasingly painful, call the doctor." 2. "It is normal for the incision to ooze greenish discharge in a few days." 3. "Increasing redness around the incision is a part of the healing process." 4. "A fever is to be expected because you had a surgical delivery."

Answer: 1 Explanation: 1. The client should call the doctor if the incision becomes increasingly painful. After cesarean delivery, wound infection is most often associated with concurrent endometritis. The wound is typically red, indurated, tender at the margins, and draining purulent exudate. Some women have cellulitis without actual purulent drainage.

While completing the medical and surgical history during the initial prenatal visit, the 16-year-old primigravida interrupts with "Why are you asking me all these questions? What difference does it make?" Which statement would best answer the client's questions?" 1. "We ask these questions to detect anything that happened in your past that might affect the pregnancy." 2. "We ask these questions to see whether you can have prenatal visits less often than most clients do." 3. "We ask these questions to make sure that our paperwork and records are complete and up to date." 4. "We ask these questions to look for any health problems in the past that might affect your parenting."

Answer: 1 Explanation: 1. The course of a pregnancy depends on a number of factors, including the past pregnancy history (if this is not a first pregnancy), prepregnancy health of the woman, presence of disease/illness states, family history, emotional status, and past health care.

The laboring client's fetal heart rate baseline is 120 beats per minute. Accelerations are present to 135 beats/min. During contractions, the fetal heart rate gradually slows to 110, and is at 120 by the end of the contraction. What nursing action is best? 1. Document the fetal heart rate. 2. Apply oxygen via mask at 10 liters. 3. Prepare for imminent delivery. 4. Assist the client into Fowler's position.

Answer: 1 Explanation: 1. The described fetal heart rate has a normal baseline; the presence of accelerations indicates adequate fetal oxygenation, and early decelerations are normal. No intervention is necessary.

The school nurse is planning a presentation on pregnancy for 13- and 14-year-olds who are currently pregnant. When planning the content of this presentation, what should the nurse keep in mind about these teens? 1. They are working on independence and autonomy. 2. They are no longer developing a sense of achievement. 3. They are confident in their own identity. 4. They are in control of their impulses.

Answer: 1 Explanation: 1. The developmental tasks of adolescence include developing an identity, gaining autonomy and independence, developing intimacy in a relationship, developing comfort with one's own sexuality, and developing a sense of achievement. Teens in early adolescence will not have achieved all of these tasks yet.

To identify the duration of a contraction, the nurse would do which of the following? 1. Start timing from the beginning of one contraction to the completion of the same contraction. 2. Time between the beginning of one contraction and the beginning of the next contraction. 3. Palpate for the strength of the contraction at its peak. 4. Time from the beginning of the contraction to the peak of the same contraction.

Answer: 1 Explanation: 1. The duration of each contraction is measured from the beginning of the contraction to the completion of the contraction.

The nurse is planning care for three newly delivered adolescents and their babies. What should the nurse keep in mind when planning their care? 1. The baby's father should be encouraged to participate when the nurse is providing instruction. 2. A class for all the adolescents would decrease teaching effectiveness. 3. The schools that the adolescents attend will provide teaching on bathing. 4. Adolescents understand the danger signals in newborns.

Answer: 1 Explanation: 1. The father, if he is involved, should be included as much as possible. If classes are offered in the hospital during the postpartum stay, the adolescent mother and father should be strongly encouraged to attend and participate.

The fetal heart rate baseline is 140 beats/min. When contractions begin, the fetal heart rate drops suddenly to 120, and rapidly returns to 140 before the end of the contraction. Which nursing intervention is best? 1. Assist the client to change position. 2. Apply oxygen to the client at 2 liters per nasal cannula. 3. Notify the operating room of the need for a cesarean birth. 4. Determine the color of the leaking amniotic fluid.

Answer: 1 Explanation: 1. The fetus is exhibiting variable decelerations, which are caused by cord compression. Sometimes late or variable decelerations are due to the supine position of the laboring woman. In this case, the decrease in uterine blood flow to the fetus may be alleviated by raising the woman's upper trunk or turning her to the side to displace pressure of the gravid uterus on the inferior vena cava.

The prenatal clinic nurse has received four phone calls. Which client should the nurse call back first? 1. Pregnant woman at 28 weeks with history of asthma who is reporting difficulty breathing and shortness of breath 2. Pregnant woman at 6 weeks with a seizure disorder who is inquiring which foods are good folic acid sources for her 3. Pregnant woman at 35 weeks with a positive HBsAG who is wondering what treatment her baby will receive after birth 4. Pregnant woman at 11 weeks with untreated hyperthyroidism who is describing the onset of vaginal bleeding

Answer: 1 Explanation: 1. The goal of therapy is to prevent maternal exacerbations because even a mild exacerbation can cause severe hypoxia-related complications in the fetus.

The nurse is presenting a class to women who are currently pregnant or are planning pregnancy in the near future. Which client statement indicates that additional teaching is required? 1. "The older a woman is when she conceives, the safer the pregnancy is." 2. "Pregnant teens can have additional nutritional needs." 3. "A woman whose sisters all had hypertension will be watched carefully." 4. "Pregnancy may be more difficult to achieve in my 40s."

Answer: 1 Explanation: 1. The health risks associated with pregnancy vary by age. The risk for maternal death is significantly higher for women over age 35 and even higher for women age 40 and older. The incidence of low-birth-weight infants, preterm births, miscarriage, stillbirth, and perinatal morbidity and mortality is higher among women age 35 or older.

The mother of a 2-day-old male has been informed that her child has sepsis. The mother is distraught and says, "I should have known that something was wrong. Why didn't I see that he was so sick?" What is the nurse's best reply? 1. "Newborns have immature immune function at birth, and illness is very hard to detect." 2. "Your mothering skills will improve with time. You should take the newborn class." 3. "Your baby didn't get enough active acquired immunity from you during the pregnancy." 4. "The immunity your baby gets in utero doesn't start to function until he is 4 to 8 weeks old."

Answer: 1 Explanation: 1. The immune responses in neonates are usually functionally impaired when compared with adults.

The kosher diet followed by many Jewish people forbids the eating of what foods? 1. Pig products and shellfish 2. Dairy products 3. All animal products 4. Dairy products and eggs

Answer: 1 Explanation: 1. The kosher diet followed by many Jewish people forbids the eating of pig products and shellfish. Certain cuts of meat from sheep and cattle are allowed, as are fish with fins and scales. In addition, many Jews believe that meat and dairy products should not be mixed or eaten at the same meal.

What is the major adverse side effect of epidural anesthesia? 1. Maternal hypotension 2. Decrease in variability of the FHR 3. Vertigo 4. Decreased or absent respiratory movements

Answer: 1 Explanation: 1. The major adverse effect of epidural anesthesia is maternal hypotension caused by a spinal blockade, which lowers peripheral resistance, decreases venous return to the heart, and subsequently lessens cardiac output and lowers blood pressure.

After administration of an epidural anesthetic to a client in active labor, it is most important to assess the mother immediately for which of the following? 1. Hypotension 2. Headache 3. Urinary retention 4. Bradycardia

Answer: 1 Explanation: 1. The most common complication of an epidural is maternal hypotension.

What is one of the most common initial signs of nonreassuring fetal status? 1. Meconium-stained amniotic fluid 2. Cyanosis 3. Dehydration 4. Arrest of descent

Answer: 1 Explanation: 1. The most common initial signs of nonreassuring fetal status are meconium-stained amniotic fluid and changes in the fetal heart rate (FHR).

What is the most significant maternal risk factor for preterm birth? 1. Previous preterm birth 2. Smoking 3. Stress 4. Substance abuse

Answer: 1 Explanation: 1. The most significant maternal risk factor for preterm birth is a previous preterm birth.

Which of the following behaviors noted in the postpartum client would require the nurse to assess further? 1. Responds hesitantly to infant cries. 2. Expresses satisfaction about the sex of the baby. 3. Friends and family visit the client and give advice. 4. Talks to and cuddles with the infant frequently.

Answer: 1 Explanation: 1. The mother tends to respond verbally to any sounds emitted by the newborn, such as cries, coughs, sneezes, and grunts. Responding hesitantly to infant cries might need further assessment to determine what the mother is feeling.

The nurse is conducting several home visits over the course of a week. Which action is appropriate to maintain safety? 1. Provide a daily schedule of visits to supervisors, including client addresses and phone numbers. 2. Maintain distance from threatening pets but do not insist that they be kept out of the room. 3. If an unsafe situation arises, discuss safety concerns with the client before continuing with the visit. 4. Lock personal belongings in the car trunk prior to entering the client's home.

Answer: 1 Explanation: 1. The nurse should notify the supervisor when leaving for a visit, and should check in as soon as the visit is completed.

The nurse is teaching a group of new parents about their infants. The infants are all 4 weeks of age or younger. Which statement should the nurse include? 1. "Your baby will respond to you the most if you look directly into his eyes and talk to him." 2. "Each baby is different. Don't try to compare your infant's behavior with any other child's behavior." 3. "If the sound level around your baby is high, the baby will wake up and be fussy or cry." 4. "If your baby is a cuddler, it is because you rocked and talked to her during your pregnancy."

Answer: 1 Explanation: 1. The parents' visual (en face) and auditory (soft, continuous voice) presence stimulates their infant to orient to them.

The nurse prepares to admit to the nursery a newborn whose mother had meconium-stained amniotic fluid. The nurse knows this newborn might require which of the following? 1. Initial resuscitation 2. Vigorous stimulation at birth 3. Phototherapy immediately 4. An initial feeding of iron-enriched formula

Answer: 1 Explanation: 1. The presence of meconium in the amniotic fluid indicates that the fetus may be suffering from asphyxia. Meconium-stained newborns or newborns who have aspirated particulate meconium often have respiratory depression at birth and require resuscitation to establish adequate respiratory effort.

It is 1 week before a pregnant client's due date. The nurse notes on the chart that the client's pulse rate was 74-80 before pregnancy. Today, the client's pulse rate at rest is 90. What action should the nurse should take? 1. Chart the findings. 2. Notify the physician of tachycardia. 3. Prepare the client for an electrocardiogram (EKG). 4. Prepare the client for transport to the hospital.

Answer: 1 Explanation: 1. The pulse rate frequently increases during pregnancy, although the amount varies from almost no increase to an increase of 10 to 15 beats per minute. This is a normal response, and does not indicate a need for emergency measures or treatment.

At 32 weeks' gestation, a woman is scheduled for a second non-stress test (following one she had at 28 weeks' gestation). Which statement by the client would indicate an adequate understanding of this procedure? 1. "I can't get up and walk around during the test." 2. "I'll have an IV started before the test." 3. "I can still smoke before the test." 4. "I need to have a full bladder for this test."

Answer: 1 Explanation: 1. The purpose of the non-stress test is to determine the results of movement on fetal heart rate. The NST is typically performed with the woman in the semi-Fowler's position with a small pillow or blanket under the right hip to displace the uterus to the left.

A nursing instructor is demonstrating how to perform a heel stick on a newborn. To obtain an accurate capillary hematocrit reading, what does the nursing instructor tell the student do? 1. Rub the heel vigorously with an isopropyl alcohol swab prior to obtaining blood. 2. Use a previous puncture site. 3. Cool the heel prior to obtaining blood. 4. Use a sterile needle and aspirate.

Answer: 1 Explanation: 1. The site should be cleaned by rubbing vigorously with 70% isopropyl alcohol swab. The friction produces local heat, which aids vasodilation.

A woman gave birth last week to a fetus at 18 weeks' gestation after her first pregnancy. She is in the clinic for follow-up, and notices that her chart states she has had one abortion. The client is upset over the use of this word. How can the nurse best explain this terminology to the client? 1. "Abortion is the obstetric term for all pregnancies that end before 20 weeks." 2. "Abortion is the word we use when someone has miscarried." 3. "Abortion is how we label babies born in the second trimester." 4. "Abortion is what we call all babies who are born dead."

Answer: 1 Explanation: 1. The term abortion means a birth that occurs before 20 weeks' gestation or the birth of a fetus-newborn who weighs less than 500 g. An abortion may occur spontaneously or it may be induced by medical or surgical means.

The pregnant client at 41 weeks is scheduled for labor induction. She asks the nurse whether induction is really necessary. What response by the nurse is best? 1. "Babies can develop postmaturity syndrome, which increases their chances of having complications after birth." 2. "When infants are born 2 or more weeks after their due date, they have meconium in the amniotic fluid." 3. "Sometimes the placenta ages excessively, and we want to take care of that problem before it happens." 4. "The doctor wants to be proactive in preventing any problems with your baby if he gets any bigger."

Answer: 1 Explanation: 1. The term postmaturity applies to the infant who is born after 42 completed weeks of gestation and demonstrates characteristics of postmaturity syndrome.

The nurse assesses the newborn's ears to be parallel to the outer and inner canthus of the eye. The nurse documents this finding to be which of the following? 1. A normal position 2. A possible chromosomal abnormality 3. Facial paralysis 4. Prematurity

Answer: 1 Explanation: 1. The top of the ear (pinna) is parallel to the outer and inner canthus of the eye in the normal newborn.

The nurse is present when a mother and her partner are told that their 35-week fetus has died. Which nursing intervention should the nurse perform first? 1. Encourage open communication with the family and the healthcare team. 2. Ask the family to withhold questions until the next day. 3. Request that another nurse come and care for this family. 4. Contact a local funeral home to help the family with funeral plans.

Answer: 1 Explanation: 1. The top priority for the nurse is to encourage open communications. The nurse functions as an advocate for the family in organizing interdisciplinary involvement, maintaining continuity of care, offering the opportunity for open communication, and ensuring that the family's wishes regarding their loss experience are honored.

Carbohydrates provide the body's primary source of energy as well as fiber necessary for proper bowel functioning. If the carbohydrate intake is not adequate, the body will use which of the following for energy? 1. Iron 2. Protein 3. Vitamin C 4. Vitamin D

Answer: 2 Explanation: 2. If the carbohydrate intake is not adequate, the body uses protein for energy. Protein then becomes unavailable for growth needs.

After being in labor for several hours with no progress, a client is diagnosed with CPD (cephalopelvic disproportion), and must have a cesarean section. The client is worried that she will not be able to have any future children vaginally. After sharing this information with her care provider, the nurse would anticipate that the client would receive what type of incision? 1. Transverse 2. Infraumbilical midline 3. Classic 4. Vertical

Answer: 1 Explanation: 1. The transverse incision is made across the lowest and narrowest part of the abdomen and is the most common lower uterine segment incision.

The laboring client is complaining of tingling and numbness in her fingers and toes, dizziness, and spots before her eyes. The nurse recognizes that these are clinical manifestations of which of the following? 1. Hyperventilation 2. Seizure auras 3. Imminent birth 4. Anxiety

Answer: 1 Explanation: 1. These symptoms all are consistent with hyperventilation.

The nurse is observing a student nurse care for a neonate undergoing intensive phototherapy. Which action by the student nurse indicates an understanding of how to provide this care? 1. Urine specific gravity is assessed each voiding. 2. Eye coverings are left off to help keep the baby calm. 3. Temperature is checked every 6 hours. 4. The infant is taken out of the isolette for diaper changes.

Answer: 1 Explanation: 1. This action is correct. Specific gravity provides one measure of urine concentration. Highly concentrated urine is associated with a dehydrated state. Weight loss is also a sign of developing dehydration in the newborn.

The nurse is caring for several pregnant clients. Which client should the nurse anticipate is most likely to have a newborn at risk for mortality or morbidity? 1. 37-year-old, with a history of multiple births and preterm deliveries who works in a chemical factory 2. 23-year-old of low socioeconomic status, unmarried 3. 16-year-old who began prenatal care at 30 weeks 4. 28-year-old with a history of gestational diabetes

Answer: 1 Explanation: 1. This client is at greatest risk because she has multiple risk factors: age over 35, high parity, history of preterm birth, and exposure to chemicals that might be toxic.

The client delivered vaginally 2 hours ago after receiving an epidural analgesia. She has a slight tingling sensation in both lower extremities, but normal movement. She sustained a second-degree perineal laceration. Her perineum is edematous and ecchymotic. What should the nurse include in the plan of care for this client? 1. Assist the client to the bathroom in 2 hours to void. 2. Place a Foley catheter now. 3. Apply warm packs to the perineum three times a day. 4. Allow the client to rest for the next 8 hours.

Answer: 1 Explanation: 1. This client is at risk for urinary retention and bladder overdistention. Overdistention occurs postpartum when the woman is unable to empty her bladder, usually because of trauma or the effects of anesthesia. After the effects of anesthesia have worn off, if the woman cannot void, postpartum urinary retention is highly indicative of a urinary tract infection (UTI). Assisting the client to the bathroom is the most likely intervention that will prevent urinary retention.

The mother of a 16-week-old infant calls the clinic concerned because she cannot feel the posterior fontanelle on her infant. Which response by the nurse would be most appropriate? 1. "It is normal for the posterior fontanelle to close by 8 to 12 weeks after birth." 2. "Bring your infant to the clinic immediately." 3. "This is due to overriding of the cranial bones during labor." 4. "Your baby must be dehydrated."

Answer: 1 Explanation: 1. This is a normal finding at 16 weeks. The posterior fontanelle closes within 8 to 12 weeks.

The nurse is answering phone calls at the pediatric clinic. Which call should the nurse return first? 1. Mother of a 2-week-old infant who doesn't make eye contact when talked to 2. Father of a 1-week-old infant who sleeps through the noise of an older sibling 3. Father of a 6-day-old infant who responds more to mother's voice than to father's voice 4. Mother of a 3-week-old infant who has begun to suck on the fingers of the right hand

Answer: 1 Explanation: 1. This is an abnormal finding. Orientation to the environment is determined by an ability to respond to cues given by others and by a natural ability to fix on and to follow a visual object horizontally and vertically. Inability or lack of response may indicate visual or auditory problems.

The nurse is seeing prenatal clients in the clinic. Which client is exhibiting expected findings? 1. 12 weeks' gestation, with fetal heart tones heard by Doppler fetoscope 2. 22 weeks' gestation, client reports no fetal movement felt yet 3. 16 weeks' gestation, fundus three finger breadths above umbilicus 4. Marked edema

Answer: 1 Explanation: 1. This is an expected finding because fetal heart tones should be heard by 12 weeks using a Doppler fetoscope.

The nurse asks a woman how her husband is dealing with the pregnancy. The nurse concludes that counseling is needed when the woman makes which statement? 1. "My husband is ready for the pregnancy to end so that we can have sex again." 2. "My husband is much more attentive to me now that I am pregnant." 3. "My husband seems more worried about our finances now than he was before the pregnancy." 4. "My husband plays his favorite music for my belly so the baby will learn to like it."

Answer: 1 Explanation: 1. This is implying that the woman and her husband are not having sex, which indicates the need for counseling. Sex is fine with a normal pregnancy.

The nurse is admitting a client for a cerclage procedure. The client asks for information about the procedure. What is the nurse's most accurate response? 1. "A stitch is placed in the cervix to prevent a spontaneous abortion or premature birth." 2. "The procedure is done during the third trimester." 3. "Cerclage is always placed after the cervix has dilated and effaced." 4. "An uncomplicated elective cerclage may is done on inpatient basis."

Answer: 1 Explanation: 1. This is the correct description of cerclage.

The nurse is preparing for a postpartum home visit. The client has been home for a week, is breastfeeding, and experienced a third-degree perineal tear after vaginal delivery. The nurse should assess the client for which of the following? 1. Dietary intake of fiber and fluids 2. Dietary intake of folic acid and prenatal vitamins 3. Return of hemoglobin and hematocrit levels to baseline 4. Return of protein and albumin to predelivery levels

Answer: 1 Explanation: 1. This mother needs to avoid the risk of constipation. She might be hesitant to have a bowel movement due to anticipated pain from the perineal tear, and constipation will decrease the healing of the laceration.

At birth, an infant weighed 6 pounds 12 ounces. Three days later, he weighs 5 pounds 2 ounces. What conclusion should the nurse draw regarding this newborn's weight? 1. This weight loss is excessive. 2. This weight loss is within normal limits. 3. This weight gain is excessive. 4. This weight gain is within normal limits.

Answer: 1 Explanation: 1. This newborn has lost more than 10% of the birth weight; this weight loss is excessive. Following birth, caloric intake is often insufficient for weight gain until the newborn is 5 to 10 days old. During this time there may be a weight loss of 5% to 10% in term newborns.

The labor nurse is caring for a client at 38 weeks' gestation who has been diagnosed with symptomatic placenta previa. Which physician order should the nurse question? 1. Begin oxytocin drip rate at 0.5 milliunits/min. 2. Assess fetal heart rate every 10 minutes. 3. Weigh all vaginal pads. 4. Assess hematocrit and hemoglobin.

Answer: 1 Explanation: 1. This order should be questioned, as this client is not a good candidate for labor induction.

) The nurse is preparing new parents for discharge with their newborn. The father asks the nurse why the baby's head is so pointed and puffy-looking. What is the best response by the nurse? 1. "His head is molded from fitting through the birth canal. It will become more round." 2. "We refer to that as 'cone head,' which is a temporary condition that goes away." 3. "It might mean that your baby sustained brain damage during birth, and could have delays." 4. "I think he looks just like you. Your head is much the same shape as your baby's."

Answer: 1 Explanation: 1. This statement is accurate and directly answers the father's question.

The client at 14 weeks' gestation has undergone a transvaginal ultrasound to assess cervical length. The ultrasound revealed cervical funneling. How should the nurse explain these findings to the client? 1. "Your cervix has become cone-shaped and more open at the end near the baby." 2. "Your cervix is lengthened, and you will deliver your baby prematurely." 3. "Your cervix is short, and has become wider at the end that extends into the vagina." 4. "Your cervix was beginning to open but now is starting to close up again."

Answer: 1 Explanation: 1. Transvaginal ultrasound can most accurately identify shortened cervical length and cervical funneling, which is a cone-shaped indentation in the cervical os indicating cervical insufficiency or risk of preterm labor.

The nurse is caring for a 2-hour-old newborn whose mother is diabetic. The nurse assesses that the newborn is experiencing tremors. Which nursing action has the highest priority? 1. Obtain a blood calcium level. 2. Take the newborn's temperature. 3. Obtain a bilirubin level. 4. Place a pulse oximeter on the newborn.

Answer: 1 Explanation: 1. Tremors are a sign of hypocalcemia. Diabetic mothers tend to have decreased serum magnesium levels at term. This could cause secondary hypoparathyroidism in the infant.

A pregnant client is lactose intolerant. Which alternative food could this client eat to get sufficient calcium? 1. Turnip greens 2. Green beans 3. Cantaloupe 4. Nectarines

Answer: 1 Explanation: 1. Turnip greens are rich in calcium.

By inquiring about the expectations and plans that a laboring woman and her partner have for the labor and birth, the nurse is primarily doing which of the following? 1. Recognizing the client as an active participant in her own care. 2. Attempting to correct any misinformation the client might have received. 3. Acting as an advocate for the client. 4. Establishing rapport with the client.

Answer: 1 Explanation: 1. Understanding the couple's expectations and plans helps the nurse provide optimal nursing care and facilitate the best possible birth experience.

The client has experienced a postpartum hemorrhage at 6 hours postpartum. After controlling the hemorrhage, the client's partner asks what would cause a hemorrhage. How should the nurse respond? 1. "Sometimes the uterus relaxes and excessive bleeding occurs." 2. "The blood collected in the vagina and poured out when your partner stood up." 3. "Bottle-feeding prevents the uterus from getting enough stimulation to contract." 4. "The placenta had embedded in the uterine tissue abnormally."

Answer: 1 Explanation: 1. Uterine atony (relaxation of the uterus) is the leading cause of early postpartum hemorrhage, accounting for over 50% of postpartum hemorrhage cases.

The client tells the nurse that she has come to the hospital so that her baby's position can be changed. The nurse would begin to organize the supplies needed to perform which procedure? 1. A version 2. An amniotomy 3. Leopold maneuvers 4. A ballottement

Answer: 1 Explanation: 1. Version, or turning the fetus, is a procedure used to change the fetal presentation by abdominal or intrauterine manipulation.

The student nurse attempts to take a newborn's vital signs, but the newborn is crying. What nursing action would be appropriate? 1. Place a gloved finger in the newborn's mouth. 2. Take the vital signs. 3. Wait until the newborn stops crying. 4. Place a hot water bottle in the isolette.

Answer: 1 Explanation: 1. Vital sign assessments are most accurate if the newborn is at rest, so measure pulse and respirations first if the baby is quiet. To soothe a crying baby, the nurse should place a moistened, unpowdered, gloved finger in the baby's mouth, and then complete the assessment while the baby suckles.

After telling a mother that her 13-year-old daughter is pregnant, the nurse would expect the mother to respond with which statement? 1. "We had such high hopes for you." 2. "But you have always been a happy child." 3. "I've always liked that boy." 4. "This is just one of those things that happen."

Answer: 1 Explanation: 1. When an adolescent pregnancy is first revealed to the teen's mother, the result is often anger, shame, or sorrow. The degree of negative response will be determined by the age of the teen, the family expectations for the teen, and the presence or absence of other teen pregnancies in the family or support network.

) A client is concerned because she has been told her blood type and her baby's are incompatible. What is the nurse's best response? 1. "This is called ABO incompatibility. It is somewhat common but rarely causes significant hemolysis." 2. "This is a serious condition, and additional blood studies are currently in process to determine whether you need a medication to prevent it from occurring with a future pregnancy." 3. "This is a condition caused by a blood incompatibility between you and your husband, but does not affect the baby." 4. "This type of condition is very common, and the baby can receive a medication to prevent jaundice from occurring."

Answer: 1 Explanation: 1. When blood types, not Rh, are incompatible, it is called ABO incompatibility. The incompatibility occurs as a result of the maternal antibodies present in her serum and interaction between the antigen sites on the fetal RBCs.

The nurse is making an initial visit to a postpartum family's home. The mother states that she is having difficulty with breastfeeding. Which resource should the nurse tell the family about? 1. The lactation consultant at the hospital 2. Free immunizations through the county public health department clinics 3. Sources of free formula at a local food pantry 4. A support group for mothers who are experiencing postpartum depression

Answer: 1 Explanation: 1. When the client specifies a problem with breastfeeding, the best resource the nurse should inform the family about is the lactation consultant.

The laboring client presses the call light and reports that her water has just broken. What would the nurse's first action be? 1. Check fetal heart tones. 2. Encourage the mother to go for a walk. 3. Change bed linens. 4. Call the physician.

Answer: 1 Explanation: 1. When the membranes rupture, the nurse notes the color and odor of the amniotic fluid and the time of rupture and immediately auscultates the FHR.

Two hours after delivery, a client's fundus is boggy and has risen to above the umbilicus. What is the first action the nurse would take? 1. Massage the fundus until firm 2. Express retained clots 3. Increase the intravenous solution 4. Call the physician

Answer: 1 Explanation: 1. When the uterus becomes boggy, pooling of blood occurs within it, resulting in the formation of clots. Anything left in the uterus prevents it from contracting effectively. Thus if it becomes boggy or appears to rise in the abdomen, the fundus should be massaged until firm.

The client has asked the nurse why her cervix has only changed from 1 to 2 cm in 3 hours of contractions occurring every 5 minutes. What is the nurse's best response to the client? 1. "Your cervix has also effaced, or thinned out, and that change in the cervix is also labor progress." 2. "When your perineal body thins out, your cervix will begin to dilate much faster than it is now." 3. "What did you expect? You've only had contractions for a few hours. Labor takes time." 4. "The hormones that cause labor to begin are just getting to be at levels that will change your cervix."

Answer: 1 Explanation: 1. With each contraction, the muscles of the upper uterine segment shorten and exert a longitudinal traction on the cervix, causing effacement. Effacement is the taking up (or drawing up) of the internal os and the cervical canal into the uterine side walls.

During newborn resuscitation, how does the nurse evaluate the effectiveness of bag-and-mask ventilations? 1. The rise and fall of the chest 2. Sudden wakefulness 3. Urinary output 4. Adequate thermoregulation

Answer: 1 Explanation: 1. With proper resuscitation, chest movement is observed for proper ventilation. Pressure should be adequate to move the chest wall.

A 26-year-old client is having her initial prenatal appointment. The client reports to the nurse that she suffered a pelvic fracture in a car accident 3 years ago. The client asks whether her pelvic fracture might affect her ability to have a vaginal delivery. What response by the nurse is best? 1. "It depends on how your pelvis healed." 2. "You will need to have a cesarean birth." 3. "Please talk to your doctor about that." 4. "You will be able to delivery vaginally."

Answer: 1 Explanation: 1. Women with a history of pelvic fractures may also be at risk for cephalopelvic disproportion (CPD).

A client with diabetes is receiving preconception counseling. The nurse will emphasize that during the first trimester, the woman should be prepared for which of the following? 1. The need for less insulin than she normally uses 2. Blood testing for anemia 3. Assessment for respiratory complications 4. Assessment for contagious conditions

Answer: 1 Explanation: 1. Women with diabetes often require less insulin during the first trimester.

The nurse evaluates the diet of a pregnant client and finds that it is low in zinc. The nurse knows that zinc intake should increase during pregnancy to promote protein metabolism. Which food should the nurse suggest in order to increase intake of zinc? 1. Shellfish 2. Bananas 3. Yogurt 4. Cabbage

Answer: 1 Explanation: 1. Zinc is found in greatest concentration in meats, shellfish, and poultry. Other good sources include whole grains and legumes.

The labor nurse would not encourage a mother to bear down until the cervix is completely dilated, to prevent which of the following? Note: Credit will be given only for all correct choices and no incorrect choices. Select all that apply. 1. Maternal exhaustion 2. Cervical edema 3. Tearing and bruising of the cervix 4. Enhanced perineal thinning 5. Having to perform an episiotomy

Answer: 1, 2, 3 Explanation: 1. If the cervix is not completely dilated, maternal exhaustion can occur. 2. If the cervix is not completely dilated, cervical edema can occur. 3. If the cervix is not completely dilated, tearing and bruising of the cervix can occur.

The nurse is cross-training maternal-child health unit nurses to provide home-based care for parents after discharge. Which statements indicate that additional teaching is required? Note: Credit will be given only if all correct choices and no incorrect choices are selected. Select all that apply. 1. "The behavioral assessment should be done as soon after birth as possible." 2. "The behavioral assessment can be performed without input from parents." 3. "The behavioral assessment might be incomplete in a 1-hour home visit." 4. "The behavioral assessment includes orientation and motor activity." 5. "The behavioral assessment can detect neurological impairments."

Answer: 1, 2 Explanation: 1. Because the first few days after birth are a period of behavioral disorganization, the complete assessment should be done on the third day after birth. 2. Parental input is required. It provides a way for the healthcare provider, in conjunction with the parents (primary caregivers), to identify and understand the individual newborn's states, temperament, capabilities, and individual behavior patterns.

A couple in their late 30s are pregnant for the first time. In evaluating the care delivered, the nurse assesses the mother for which of the following? Note: Credit will be given only if all correct and no incorrect choices are selected. Select all that apply. 1. Makes appropriate healthcare choices 2. Receives effective healthcare through the pregnancy 3. Has a partner who is not interested in child care 4. Cannot cope with her life change 5. Wishes to have amniocentesis done

Answer: 1, 2 Explanation: 1. Expected outcomes of nursing care for a pregnant couple over 35 include that the client and her partner are knowledgeable about the pregnancy and make appropriate healthcare choices. 2. Expected outcomes of nursing care for a pregnant couple over 35 include the client and her partner receive effective health care throughout the pregnancy, birth, and postpartum period.

In caring for the premature newborn, the nurse must assess hydration status continually. Assessment parameters should include which of the following? Note: Credit will be given only if all correct choices and no incorrect choices are selected. Select all that apply. 1. Volume of urine output 2. Weight 3. Blood pH 4. Head circumference 5. Bowel sounds

Answer: 1, 2 Explanation: 1. In order to assess hydration status, volume of urine output must be evaluated. 2. In order to assess hydration status, the infant's weight must be evaluated.

A client at 32 weeks' gestation comes to the clinic with urinary burning and frequency. The nurse explains that urinary tract infections are common in pregnancy due to which of the following? Note: Credit will be given only if all correct and no incorrect choices are selected. Select all that apply. 1. Ureteral atonia 2. Stasis of urine 3. Increased glomerular filtration rate 4. Increased renal plasma flow 5. Increased clearance of urea

Answer: 1, 2 Explanation: 1. The presence of amino acids and glucose in the urine in conjunction with the tendency toward ureteral atonia and stasis of urine in the ureters may increase the risk of urinary tract infection. 2. The presence of amino acids and glucose in the urine in conjunction with the tendency toward ureteral atonia and stasis of urine in the ureters may increase the risk of urinary tract infection.

A client attending a prenatal class asks why episiotomies are performed. The nurse explains that risk factors that predispose women to episiotomies include which of the following? Note: Credit will be given only if all correct choices and no incorrect choices are selected. Select all that apply. 1. Large or macrosomic fetus 2. Use of forceps 3. Shoulder dystocia 4. Maternal health 5. Shorter second stage

Answer: 1, 2, 3 Explanation: 1. A large fetus places a woman at risk for an episiotomy to prevent lacerations. 2. Use of forceps or vacuum extractor is a risk factor that predisposes women to episiotomies. 3. Shoulder dystocia is a risk factor that predisposes women to episiotomies.

A cesarean section is ordered for the laboring client with whom the nurse has worked all shift. The client will receive general anesthesia. The nurse knows that potential complications of general anesthesia include which of the following? Note: Credit will be given only if all correct choices and no incorrect choices are selected. Select all that apply. 1. Fetal depression that is directly proportional to the depth and duration of the anesthesia 2. Poor fetal metabolism of anesthesia, which inhibits use with preterm infants 3. Uterine relaxation 4. Increased gastric motility 5. Itching of the face and neck

Answer: 1, 2, 3 Explanation: 1. A primary danger of general anesthesia is fetal depression, because the medication reaches the fetus in about 2 minutes. The depression is directly proportional to the depth and duration of anesthesia. 2. The poor fetal metabolism of general anesthetic agents is similar to that of analgesic agents administered during labor. General anesthesia is not advocated when the fetus is considered to be at high risk, particularly in preterm birth. 3. Most general anesthetic agents cause some degree of uterine relaxation.

The nurse is caring for a client who has just been informed of the demise of her unborn fetus. Which common cognitive responses to loss would the nurse anticipate? Note: Credit will be given only if all correct choices and no incorrect choices are selected. Select all that apply. 1. Denial and disbelief 2. Sense of unreality 3. Poor concentration 4. Palpitations 5. Loss of appetite

Answer: 1, 2, 3 Explanation: 1. Denial and disbelief are common cognitive responses to fetal loss. 2. A sense of unreality is a common cognitive response to fetal loss. 3. Poor concentration is a common cognitive response to loss.

During the initial prenatal visit, the nurse obtains a weight of 42 kg (92.4 lb). The nurse must further assess the client for information about which of the following? Note: Credit will be given only if all correct and no incorrect choices are selected. Select all that apply. 1. Eating habits 2. Foods regularly eaten 3. Income limitations 4. Blood pressure and pulse rate 5. Weight loss during pregnancy

Answer: 1, 2, 3 Explanation: 1. For a client whose weight is less than 100 lb, the nurse would obtain information on eating habits. 2. For a client whose weight is less than 100 lb, the nurse would obtain information on foods regularly eaten. 3. For a client whose weight is less than 100 lb, the nurse would obtain information on income limitations.

What are some of the advantages and disadvantages of formula-feeding that a nurse should discuss with new parents? Note: Credit will be given only if all correct choices and no incorrect choices are selected. Select all that apply. 1. The nutritional value of formula depends on the proper preparation/dilution. 2. There is a potential for bacterial contamination during preparation and storage. 3. Both parents can participate in positive parent-infant interaction during feeding. 4. Refrigeration is not necessary if preparing more than one bottle at a time. 5. Formula has higher levels of essential fatty acids, lactose, cystine, and cholesterol than does breast milk.

Answer: 1, 2, 3 Explanation: 1. Formula composition does not vary unless the instructions for preparation and dilution are not followed. 2. Bacteria can be introduced if bottles and nipples are not properly cleaned. 3. If a mother has chosen not to breastfeed, then the father can be involved with bottle feedings from the start.

A client received epidural anesthesia during the first stage of labor. The epidural is discontinued immediately after delivery. This client is at increased risk for which problem during the fourth stage of labor? 1. Nausea 2. Bladder distention 3. Uterine atony 4. Hypertension

Answer: 2 Explanation: 2. Nursing care following an epidural block includes frequent assessment of the bladder to avoid bladder distention.

The nurse is caring for a client who could be at risk for uterine rupture. The nurse is monitoring the fetus closely for which of the following? Note: Credit will be given only if all correct choices and no incorrect choices are selected. Select all that apply. 1. Late decelerations 2. Bradycardia 3. Loss of ability to determine fetal station 4. Tachycardia 5. Early decelerations

Answer: 1, 2, 3 Explanation: 1. Late decelerations could be seen with uterine rupture. 2. Bradycardia is seen if there is uterine rupture. 3. The uterus is not holding the fetus in place anymore if the uterus ruptures.

A nurse suspects that a postpartum client has mastitis. Which data support this assessment? Note: Credit will be given only if all correct choices and no incorrect choices are selected. Select all that apply. 1. Shooting pain between breastfeedings 2. Late onset of nipple pain 3. Pink, flaking, pruritic skin of the affected nipple 4. Nipple soreness when the infant latches on 5. Pain radiating to the underarm area from the breast

Answer: 1, 2, 3 Explanation: 1. Mastitis is characterized by shooting pain between feedings, often radiating to the chest wall. 2. Mastitis is characterized by late-onset nipple pain. 3. The skin of the affected breast becomes pink, flaking, and pruritic. Page Ref: 983

A client is admitted to the labor and delivery unit in active labor. What nursing diagnoses might apply to the client with suspected abruptio placentae? Note: Credit will be given only if all correct choices and no incorrect choices are selected. Select all that apply. 1. Fluid Volume, Deficient, Risk for, related to hypovolemia secondary to excessive blood loss 2. Tissue Perfusion: Peripheral, Ineffective, related to blood loss secondary to uterine atony following birth 3. Anxiety related to concern for own personal status and the baby's safety 4. Knowledge, Deficient related to lack of information about inherited genetic defects 5. Alteration in Respiratory Function related to blood loss

Answer: 1, 2, 3 Explanation: 1. Maternal and perinatal fetal mortality are concerns due to hypoxia. 2. Maternal and perinatal fetal mortality are concerns due to blood loss. 3. This mother would be anxious for herself and her baby.

A pregnant asthmatic client is being seen for her initial prenatal visit. The nurse knows that the fetal implications of maternal asthma include which of the following? Note: Credit will be given only if all correct and no incorrect choices are selected. Select all that apply. 1. Prematurity 2. Low birth weight 3. Hypoxia with maternal exacerbation 4. Congenital anomalies from the medications 5. Perinatal transfer of the asthma

Answer: 1, 2, 3 Explanation: 1. One implication of maternal asthma is that the infant is at risk for prematurity. 2. One implication of maternal asthma is that the infant is at risk for low birth weight. 3. One implication of maternal asthma is that the infant is at risk for hypoxia if the mother has an exacerbation of her asthma.

The OB-GYN nurse works in a clinic with a culturally diverse group of clients whose specific actions during pregnancy are often determined by cultural beliefs. The nurse recognizes that these beliefs about pregnancy and childbirth fall into which categories? Note: Credit will be given only if all correct and no incorrect choices are selected. Select all that apply. 1. Prescriptive beliefs 2. Restrictive beliefs 3. Taboos 4. Cultural humility 5. Folk treatment beliefs

Answer: 1, 2, 3 Explanation: 1. Prescriptive beliefs or requirements describe expected behaviors. 2. Restrictive beliefs are stated negatively and limit behaviors. 3. Taboo beliefs refer to specific supernatural consequences.

Which findings would the nurse expect when assessing a newborn infected with syphilis? Note: Credit will be given only if all correct choices and no incorrect choices are selected. Select all that apply. 1. Rhinitis 2. Fissures on mouth corners 3. Red rash around anus 4. Lethargy 5. Large for gestational age

Answer: 1, 2, 3 Explanation: 1. Rhinitis is evident in the newborn exposed to syphilis. 2. Fissures on mouth corners and an excoriated upper lip indicate exposure to syphilis. 3. A red rash around the mouth and anus is observed.

The clinic nurse is culturally sensitive when, while assessing the pregnant client, he asks about which of the following? Note: Credit will be given only if all correct and no incorrect choices are selected. Select all that apply. 1. The family's expectations of the healthcare system 2. Which cultural practices should be incorporated into care 3. Any alternative healer who should be consulted 4. Positive consequences of the client's healthcare beliefs 5. The client's giving up her practices and adopting the practices of the dominant culture

Answer: 1, 2, 3 Explanation: 1. The nurse needs to ask about the client's expectations of the healthcare system. 2. The nurse should ask about any cultural or spiritual practices that should be incorporated into care. 3. The culturally sensitive nurse will ask whether any alternative healer should be consulted about care.

What self-care measures would a nurse recommend for a client in her first trimester to reduce the discomfort of nausea and vomiting? Note: Credit will be given only if all correct and no incorrect choices are selected. Select all that apply. 1. Avoid odors or causative factors. 2. Have small but frequent meals. 3. Drink carbonated beverages. 4. Drink milk before arising in the morning. 5. Eat highly seasoned food.

Answer: 1, 2, 3 Explanation: 1. The nurse would recommend for a client in her first trimester to avoid odors and caustic factors to reduce the discomfort of nausea and vomiting. 2. The nurse would recommend for a client in her first trimester to have small but frequent meals to reduce the discomfort of nausea and vomiting. 3. The nurse would recommend for a client in her first trimester to drink carbonated beverages to reduce the discomfort of nausea and vomiting.

The prenatal period should be used to expose the prospective parents to up-to-date, evidence-based information about which of the following topics? Note: Credit will be given only if all correct and no incorrect choices are selected. Select all that apply. 1. Breastfeeding 2. Pain relief 3. Obstetric complications and procedures 4. Toddler care 5. Antepartum adjustment

Answer: 1, 2, 3 Explanation: 1. The prenatal period should expose prospective parents to up-to-date, evidence-based information about breastfeeding. 2. The prenatal period should expose prospective parents to up-to-date, evidence-based information about pain relief. 3. The prenatal period should expose prospective parents to up-to-date, evidence-based information about obstetric complications and procedures.

In planning a program to reduce teen pregnancy rates, the nurse uses an evidence-based approach. The nurse learns that more research is needed for which of the following? Note: Credit will be given only if all correct and no incorrect choices are selected. Select all that apply. 1. Teen pregnancy rate in the United States 2. Use of birth control by adolescents 3. Prevention of sexually transmitted infections 4. Long-active, reversible contraception is highly effective for teens 5. Intrauterine devices are safe and effective when used by adolescents

Answer: 1, 2, 3 Explanation: 1. The teen pregnancy rate in the United States requires more research. 2. The use of birth control by adolescents requires more research. 3. Preventing sexually transmitted infections in teenagers requires more research.

The nurse is working with a pregnant woman who has systemic lupus erythematosus (SLE). What does the nurse anticipate the infant might be born with? Note: Credit will be given only if all correct and no incorrect choices are selected. Select all that apply. 1. A tendency to bleed excessively 2. An increased chance of developing infections 3. A hemoglobin less than optimal for good health 4. Problems with vision 5. Hearing loss

Answer: 1, 2, 3 Explanation: 1. This is true, as the infant might be born with thrombocytopenia. 2. This is true, as the infant might be born with neutropenia. 3. This is true, as the infant might be born with anemia.

A postpartal client recovering from deep vein thrombosis is being discharged. What areas of teaching on self-care and anticipatory guidance should the nurse discuss with the client? Note: Credit will be given only if all correct choices and no incorrect choices are selected. Select all that apply. 1. Avoid crossing the legs. 2. Avoid prolonged standing or sitting. 3. Take frequent walks. 4. Take a daily aspirin dose of 650 mg. 5. Avoid long car trips.

Answer: 1, 2, 3 Explanation: 1. Women should be taught to avoid prolonged standing or sitting in one position or sitting with legs crossed. 2. Women should be taught to avoid prolonged standing or sitting in one position or sitting with legs crossed. 3. Women should be advised to avoid a sedentary lifestyle and to exercise as much as possible (walking is ideal).

The nurse caring for a client in labor anticipates fetal macrosomia and shoulder dystocia. Appropriate management of shoulder dystocia is essential in order to prevent which fetal complications? Note: Credit will be given only if all correct choices and no incorrect choices are selected. Select all that apply. 1. Brachial plexus injury 2. Fractured clavicle 3. Asphyxia 4. Neurological damage 5. Puerperal infection

Answer: 1, 2, 3, 4 Explanation: 1. Brachial plexus injury occurs due to improper or excessive traction applied to the fetal head. 2. Complications in macrosomia include fractured clavicles. 3. Complications in macrosomia include asphyxia of the fetus. 4. Neurological damage is a complication of macrosomia.

The nurse is planning a home visit for a new mother and newborn who were discharged prior to 48 hours after delivery. In preparation for the home visit, what should the nurse do? Note: Credit will be given only if all correct choices and no incorrect choices are selected. Select all that apply. 1. Review the client's records. 2. Gather materials and equipment that might be needed. 3. Make a pre-visit telephone call to determine time and day of visit. 4. Contact the healthcare provider about any special concerns. 5. Schedule additional home visits or follow-up contacts with community agencies.

Answer: 1, 2, 3, 4 Explanation: 1. Communication with the primary healthcare provider(s) and a thorough review of inpatient records give the nurse an understanding of current needs and any special concerns for each individual mother-baby couplet. 2. Before the home visit, the nurse prepares by identifying the purpose of the visit and gathering anticipated materials and equipment. 3. The nurse should make a previsit telephone call to arrange the appointment with the woman and her family. 4. Communication with the primary healthcare provider(s) and a thorough review of inpatient records give the nurse an understanding of current needs and any special concerns for each individual mother-baby couplet.

The partner of a pregnant client comes to the clinic with her. He complains to the nurse that he is experiencing different physical changes. The nurse determines he is experiencing couvade when he describes which symptoms? Note: Credit will be given only if all correct and no incorrect choices are selected. Select all that apply. 1. Fatigue 2. Increased appetite 3. Headache 4. Backache 5. High anxiety level

Answer: 1, 2, 3, 4 Explanation: 1. Couvade is demonstrated by increased fatigue in the partner. 2. Couvade is demonstrated by an increased appetite in the partner. 3. Couvade is demonstrated by the partner's having headaches. 4. Couvade is demonstrated by the partner's experiencing backache.

What fetal factors require a cesarean birth? 1. Severe intrauterine growth restriction (IUGR) 2. Fetal anomalies 3. Unfavorable fetal position or presentation 4. Preterm birth 5. Lack of maternal attachment

Answer: 1, 2, 3, 4 Explanation: 1. Fetal factors such as severe intrauterine growth restriction (IUGR), preterm birth, fetal anomalies, nonreassuring fetal status, or unfavorable fetal position or presentation require cesarean birth. 2. Fetal factors such as severe intrauterine growth restriction (IUGR), preterm birth, fetal anomalies, nonreassuring fetal status, or unfavorable fetal position or presentation require cesarean birth. 3. Fetal factors such as severe intrauterine growth restriction (IUGR), preterm birth, fetal anomalies, nonreassuring fetal status, or unfavorable fetal position or presentation require cesarean birth. 4. Fetal factors such as severe intrauterine growth restriction (IUGR), preterm birth, fetal anomalies, nonreassuring fetal status, or unfavorable fetal position or presentation require cesarean birth.

The nurse is explaining to a new mother that the newborn behavioral assessment includes which of the following? Note: Credit will be given only for all correct choices and for no incorrect choices. Select all that apply. 1. Habituation 2. Motor activity 3. Self-quieting activity 4. Cuddliness 5. Reflexes

Answer: 1, 2, 3, 4 Explanation: 1. Habituation is the newborn's ability to diminish or shut down innate responses to specific stimuli. 2. The newborn's motor tone is assessed in the most characteristic state of responsiveness. 3. Assessment is based on how often, how quickly, and how effectively newborns can use their resources to quiet and console themselves when upset or distressed. 4. Cuddliness encompasses the infant's need for and response to being held.

The nurse is presenting a class on preterm labor, its causes, and treatments to a group of newly pregnant couples. Which statements regarding preterm labor are true? Note: Credit will be given only of all correct choices and no incorrect choices are selected. Select all that apply. 1. Antepartum hemorrhage can cause preterm labor. 2. Trauma can cause preterm labor. 3. Infection can cause preterm labor. 4. Magnesium sulfate is a drug used to stop contractions. 5. Sedatives and narcotics may be given to stop labor.

Answer: 1, 2, 3, 4 Explanation: 1. Hemorrhage from placenta previa or abruption can cause preterm labor. 2. Trauma to the abdomen or uterus can cause preterm labor. 3. Infections such as urinary tract infections can cause preterm labor. 4. Magnesium sulfate acts as a CNS depressant by decreasing the quantity of acetylcholine released by motor nerve impulses and thereby blocking neuromuscular transmission.

The nurse is working with a woman who abuses stimulants. The nurse is aware that the fetus is at risk for which of the following? Note: Credit will be given only if all correct and no incorrect choices are selected. Select all that apply. 1. Withdrawal symptoms 2. Cardiac anomalies 3. Sudden infant death syndrome 4. Being small for gestational age 5. Fetal alcohol syndrome

Answer: 1, 2, 3, 4 Explanation: 1. Infants born to mothers who abuse stimulants such as amphetamines can have withdrawal symptoms. 2. Infants born to mothers who abuse stimulants such as cocaine can be born with cardiac anomalies. 3. Infants born to mothers who abuse stimulants such as cocaine can have sudden infant death syndrome. 4. Infants born to mothers who abuse stimulants such as nicotine can be small for gestational age.

The postpartum nurse provides anticipatory guidance for the new mother as well as teaching on self-care and infant care before discharge. Which topics should be included? Note: Credit will be given only if all correct choices and no incorrect choices are selected. Select all that apply. 1. Role changes brought on by the addition to the family unit 2. The realities of having a new baby, and how it affects previous lifestyle 3. Potential complications such as infant colic and postpartum issues 4. Sexuality and contraception 5. Toilet-training and preschool options

Answer: 1, 2, 3, 4 Explanation: 1. It is helpful for the nurse to advise parents that they may experience feelings of uncertainty as they grow into the parental role and alter their family processes to accommodate the new family member. 2. Guidance is essential in assisting the family to cope with role changes and the realities of a new baby. 3. Guidance is essential in assisting the family to cope with potential complications such as infant colic and postpartum health issues. 4. It is important for the nurse to present information about changes that may affect sexual activity and to discuss the importance of contraception during the early postpartum period.

The nurse is caring for a client who is having fetal tachycardia. The nurse knows that possible causes include which of the following? Note: Credit will be given only if all correct choices and no incorrect choices are selected. Select all that apply. 1. Maternal dehydration 2. Maternal hyperthyroidism 3. Fetal hypoxia 4. Prematurity 5. Anesthesia or regional analgesia

Answer: 1, 2, 3, 4 Explanation: 1. Maternal dehydration can cause fetal tachycardia. 2. Maternal hyperthyroidism can cause fetal tachycardia. 3. Fetal tachycardia can indicate fetal hypoxia. 4. Prematurity can cause fetal tachycardia.

When teaching the new mother about the composition of breast milk, the nurse explains that the fat content can range from 30 to 50 grams/liter. Which factors affect the fat content of breast milk? Note: Credit will be given only if all correct choices and no incorrect choices are selected. Select all that apply. 1. Maternal parity 2. Duration of pregnancy 3. Stage of lactation 4. Time of day 5. Vitamin C intake

Answer: 1, 2, 3, 4 Explanation: 1. Multiparous mothers produce milk with a lower content of fatty acids than primigravidas. 2. The milk of a mother who delivers a preterm infant has a greater concentration of DHA and ARA than does the milk of a mother who gives birth to a full-term infant. 3. Phospholipids and cholesterol levels are higher in colostrum than in mature milk, although overall fat content is higher in mature breast milk than in colostrum. 4. Fat content is generally higher in the evening and lower in the early morning.

The nurse knows that a mother who has been treated for Beta streptococcus passes this risk on to her newborn. Risk factors for neonatal sepsis caused by Beta streptococcus include which of the following? Note: Credit will be given only if all correct choices and no incorrect choices are selected. Select all that apply. 1. Prematurity 2. Maternal intrapartum fever 3. Membranes ruptured for longer than 18 hours 4. A previously infected infant with GBS disease 5. An older mother having her first baby

Answer: 1, 2, 3, 4 Explanation: 1. Prematurity is a risk factor. 2. Maternal intrapartum fever is a risk factor. 3. Prolonged rupture of membranes is a risk factor. 4. A previously infected infant increases the risk.

Characteristics of a caring relationship that the nurse cultivates when interacting with the client in the home include which of the following? Note: Credit will be given only if all correct choices and no incorrect choices are selected. Select all that apply. 1. Regard for the client 2. Genuineness 3. Empathy 4. Rapport 5. Attachment

Answer: 1, 2, 3, 4 Explanation: 1. Showing regard to the client by introducing oneself and calling the client by name helps demonstrate a positive regard for the client. 2. The nurse displays genuineness by meaning what he says, conveying verbal and nonverbal messages that are congruent, and being nonjudgmental. 3. The nurse needs to listen to the client without judgment and to try to view events from the client's point of view. 4. Trust and rapport is established by doing what you say will you do, being prepared for the visit and being on time, and following up on any areas that are needed.

What assessments of the newborn should be completed during the initial home visit? Note: Credit will be given only if all correct choices and no incorrect choices are selected. Select all that apply. 1. Sleep-wake cycles 2. Parent-infant interaction 3. Fontanelles 4. Umbilical cord status 5. Breast engorgement

Answer: 1, 2, 3, 4 Explanation: 1. The infant's sleep-wake cycles need to be assessed. 2. It is important to assess whether the parent is beginning to attach and bond to the infant. 3. The infant's fontanelles should be assessed. 4. The umbilical cord stump should show no signs of infection.

The nurse caring for a newborn with anemia would expect which initial laboratory data to be included in the initial assessment? Note: Credit will be given only if all correct choices and no incorrect choices are selected. Select all that apply. 1. Hemoglobin 2. Hematocrit 3. Reticulocyte count 4. Direct Coombs' test 5. Cord serum OgM

Answer: 1, 2, 3, 4 Explanation: 1. The initial laboratory workup for anemia should include hemoglobin measurements. 2. The initial laboratory workup for anemia should include hematocrit measurements. 3. The initial laboratory workup for anemia should include a reticulocyte count. 4. The direct Coombs' test reveals the presence of antibody-coated (sensitized) Rh-positive red blood cells in the newborn and should be included in the initial laboratory workup for anemia.

A 16-year-old is making her first prenatal visit to the clinic in her fourth month of pregnancy. What is the nurse's first responsibility? 1. Contact the social worker. 2. Develop a trusting relationship. 3. Schedule the client for prenatal classes. 4. Teach the client about proper nutrition.

Answer: 2 Explanation: 2. Developing a trusting relationship with the pregnant adolescent is essential. Honesty and respect for the individual and a caring attitude promote the client's self-esteem.

What are the nurse's responsibilities when teaching the new mother about infant feeding? Note: Credit will be given only if all correct choices and no incorrect choices are selected. Select all that apply. 1. The nurse should be well informed about infant nutrition and feeding methods. 2. The nurse should provide accurate and consistent information. 3. The nurse should use each interaction to support the parents and promote the family's sense of confidence. 4. The nurse should familiarize the mother with information about community resources that might be helpful after discharge. 5. The nurse should aggressively promote breastfeeding, even if the parents have decided to bottle-feed their infant.

Answer: 1, 2, 3, 4 Explanation: 1. The nurse should provide information on maternal and infant nutrition, fluid requirements, and feeding methods. 2. The nurse should provide information that is accurate and consistent. Nurses have a responsibility to educate parents. 3. Every interaction is a chance to educate and support the new parents in learning how to care for the newborn. Nurses have a responsibility to educate parents. 4. It is important that parents receive verbal and written instructions and community resource information to which they can later refer.

The nurse educator is presenting a class on the different kinds of miscarriages. Miscarriages, or spontaneous abortions, are classified clinically into which of the following different categories? Note: Credit will be given only if all correct and no incorrect choices are selected. Select all that apply. 1. Threatened abortion 2. Incomplete abortion 3. Complete abortion 4. Missed abortion 5. Acute abortion

Answer: 1, 2, 3, 4 Explanation: 1. Unexplained cramping, bleeding, or backache indicates the fetus might be in jeopardy. This is a threatened abortion. 2. In an incomplete abortion, parts of the products of conception are retained, most often the placenta. 3. In a complete abortion, all the products of conception are expelled. The uterus is contracted and the cervical os may be closed. 4. In a missed abortion, the fetus dies in utero but is not expelled.

The nurse is teaching a new mother about ways to manage fatigue after she returns home. Which instructions should the nurse include? Note: Credit will be given only if all correct choices and no incorrect choices are selected. Select all that apply. 1. Take frequent rest periods. 2. Nap when the newborn is sleeping. 3. Avoid overdoing housework and unnecessary chores. Do not clean when infant is sleeping. 4. Avoid having others come to the house to do housework and interfere with rest. 5. Utilize friends and family to provide help and support, such as cooking a meal.

Answer: 1, 2, 3, 5 Explanation: 1. Adequate rest is essential to a smooth postpartum transition. The nurse can encourage rest by organizing activities to avoid frequent interruptions for the woman. 2. Mothers should be counseled to sleep when the baby sleeps. 3. Mothers should be counseled to delegate or postpone unnecessary chores and activities and to sleep when the baby sleeps. 5. Mothers should be counseled to utilize family and friends for support.

During a visit to the obstetrician, a pregnant client questions the nurse about the potential need for an amniotomy. The nurse explains that an amniotomy is performed to do which of the following? Note: Credit will be given only if all correct choices and no incorrect choices are selected. Select all that apply. 1. Stimulate the beginning of labor 2. Augment labor progression 3. Allow application of an internal fetal electrode 4. Allow application of an external fetal monitor 5. Allow insertion of an intrauterine pressure catheter

Answer: 1, 2, 3, 5 Explanation: 1. Amniotomy is the artificial rupture of the amniotic membranes and can be used to induce labor. 2. Amniotomy can be done to augment labor. 3. Amniotomy allows access to the fetus in order to apply an internal fetal electrode to the fetal scalp. 5. Amniotomy may be performed during labor to allow an intrauterine pressure catheter to be inserted.

During home care of a low-risk pregnant client, the nurse provides care by assessing which of the following? Note: Credit will be given only if all correct and no incorrect choices are selected. Select all that apply. 1. Urine 2. Weight 3. Diet 4. Pelvic measurements 5. Physical activity

Answer: 1, 2, 3, 5 Explanation: 1. Assessment of the client's urine can be done in the home setting. 2. Obtaining the client's weight can be done in the home setting. 3. Assessing the client's dietary intake can be done in the home setting. 5. The client's physical activity can be assessed in the home setting.

Childbirth preparation offers several advantages including which of the following? Note: Credit will be given only for all correct choices and no incorrect choices. Select all that apply. 1. It helps a pregnant woman and her support person understand the choices in the birth setting. 2. It promotes awareness of available options. 3. It provides tools for a pregnant woman and her support person to use during labor and birth. 4. Women who receive continuous support during labor require more analgesia, and have more cesarean and instrument births. 5. Each method has been shown to shorten labor.

Answer: 1, 2, 3, 5 Explanation: 1. Childbirth preparation offers several advantages. It helps a pregnant woman and her support person understand the choices in the birth setting, promotes awareness of available options, and provides tools for them to use during labor and birth. 2. Childbirth preparation offers several advantages. It helps a pregnant woman and her support person understand the choices in the birth setting, promotes awareness of available options, and provides tools for them to use during labor and birth. 3. Childbirth preparation offers several advantages. It helps a pregnant woman and her support person understand the choices in the birth setting, promotes awareness of available options, and provides tools for them to use during labor and birth. 5. Childbirth preparation offers several advantages. Each method has been shown to shorten labor.

The nurse is assessing the gestational age of a 1-hour-old newborn. Which physical characteristics does the nurse assess? Note: Credit will be given only if all correct choices and no incorrect choices are selected. Select all that apply. 1. Sole creases 2. Amount of breast tissue 3. Amount of lanugo 4. Reflexes 5. Testicular descent

Answer: 1, 2, 3, 5 Explanation: 1. Clinical gestational age assessment tools have two components: external physical characteristics and neurologic or neuromuscular development evaluations. Physical characteristics generally include sole creases, amount of breast tissue, amount of lanugo, cartilaginous development of the ear, testicular descent, and scrotal rugae or labial development. 2. Clinical gestational age assessment tools have two components: external physical characteristics and neurologic or neuromuscular development evaluations. Physical characteristics generally include sole creases, amount of breast tissue, amount of lanugo, cartilaginous development of the ear, testicular descent, and scrotal rugae or labial development. 3. Clinical gestational age assessment tools have two components: external physical characteristics and neurologic or neuromuscular development evaluations. Physical characteristics generally include sole creases, amount of breast tissue, amount of lanugo, cartilaginous development of the ear, testicular descent, and scrotal rugae or labial development. 5. Clinical gestational age assessment tools have two components: external physical characteristics and neurologic or neuromuscular development evaluations. Physical characteristics generally include sole creases, amount of breast tissue, amount of lanugo, cartilaginous development of the ear, testicular descent, and scrotal rugae or labial development.

When blood pressure and other signs indicate that the preeclampsia is worsening, hospitalization is necessary to monitor the woman's condition closely. At that time, which of the following should be assessed? Note: Credit will be given only if all correct choices and no incorrect choices are selected. Select all that apply. 1. Fetal heart rate 2. Blood pressure 3. Temperature 4. Urine color 5. Pulse and respirations

Answer: 1, 2, 3, 5 Explanation: 1. Determine the fetal heart rate along with blood pressure, or monitor continuously with the electronic fetal monitor if the situation indicates. 2. Determine blood pressure every 1 to 4 hours, or more frequently if indicated by medication or other changes in the woman's status. 3. Determine temperature every 4 hours, or every 2 hours if elevated or if premature rupture of the membranes (PROM) has occurred. 5. Determine pulse rate and respirations along with blood pressure.

When doing a neurologic assessment of a newborn, what would the nurse recognize? Note: Credit will be given only if all correct choices and no incorrect choices are selected. Select all that apply. 1. Muscle tone is assessed by moving various parts of the newborn's body while the newborn's head remains in a neutral position. 2. The newborn is somewhat hypertonic. 3. Muscle tone should be symmetrical. 4. Shortly after birth, the infant is flaccid at rest. 5. Diminished muscle tone requires further evaluation.

Answer: 1, 2, 3, 5 Explanation: 1. Moving various parts of the newborn's body while the newborn's head remains in a neutral position is the correct way to assess muscle tone. 2. The newborn will resist the examiner's attempts to extend the elbow and knee joints. 3. Muscle tone should be symmetrical. 5. If decreased muscle tone is noted, further evaluation is necessary.

The nurse is counseling a group of first-trimester clients on diet increases that are necessary during pregnancy. Which information would be necessary to tell the pregnant women? Note: Credit will be given only if all correct choices and no incorrect choices are selected. Select all that apply. 1. An increase of protein is necessary to provide amino acids necessary for fetal development. 2. Protein contributes to the body's overall energy metabolism. 3. The recommended protein during pregnancy is 70 grams each day. 4. The increased amount of protein that a pregnant woman needs is 15 grams a day. 5. The quality of protein is as important as the amount.

Answer: 1, 2, 3, 5 Explanation: 1. During pregnancy, the woman needs increased amounts of protein to provide amino acids for fetal development, blood volume expansion, and growth of other maternal tissues. 2. Protein contributes to overall energy metabolism. 3. The amount of protein recommended each day during pregnancy is 70 grams. 5. Plant proteins can meet a woman's protein needs; however, more of a given protein may be needed because animal proteins are of higher quality.

The nurse is teaching a prenatal class about postpartum changes. The nurse explains that factors that might interfere with uterine involution include which of the following? Note: Credit will be given only if all correct choices and no incorrect choices are selected. Select all that apply. 1. Prolonged labor 2. Difficult birth 3. Full bladder 4. Breastfeeding 5. Infection

Answer: 1, 2, 3, 5 Explanation: 1. During prolonged labor, the muscles relax because of prolonged time of contraction during labor. 2. During a difficult birth, the uterus is manipulated excessively, causing fatigue. 3. As the uterus is pushed up and usually to the right, pressure on a full bladder interferes with effective uterine contraction. 5. Inflammation and infection interfere with uterine muscle's ability to contract effectively.

Upon assessing the FHR tracing, the nurse determines that there is fetal tachycardia. The fetal tachycardia would be caused by which of the following? Note: Credit will be given only if all correct choices and no incorrect choices are selected. Select all that apply. 1. Early fetalhypoxia 2. Prolonged fetal stimulation 3. Fetal anemia 4. Fetal sleep cycle 5. Infection

Answer: 1, 2, 3, 5 Explanation: 1. Early fetal hypoxia can cause fetal tachycardia. 2. Prolonged fetal stimulation can cause fetal tachycardia. 3. Fetal anemia can cause fetal tachycardia. 5. Infection can cause fetal tachycardia.

For what common side effects of epidural anesthesia should the nurse watch? Note: Credit will be given only if all correct choices and no incorrect choices are selected. Select all that apply. 1. Elevated maternal temperature 2. Urinary retention 3. Nausea 4. Long-term back pain 5. Local itching

Answer: 1, 2, 3, 5 Explanation: 1. Elevated maternal temperature is a potential side effect of epidural anesthesia. 2. Urinary retention is a potential side effect of epidural anesthesia. 3. Nausea is a potential side effect of epidural anesthesia. 5. Pruritus may occur at any time during the epidural infusion. It usually appears first on the face, neck, or torso and is generally the result of the agent used in the epidural infusion. Benadryl, an antihistamine, can be administered to manage pruritus.

) In caring for a client with a uterine rupture, the nurse determines which nursing diagnoses to be appropriate? Note: Credit will be given only if all correct choices and no incorrect choices are selected. Select all that apply. 1. Gas Exchange, Impaired 2. Fear related to unknown outcome 3. Coping, Ineffective 4. Mobility: Physical, Impaired 5. Anxiety

Answer: 1, 2, 3, 5 Explanation: 1. Gas Exchange, Impaired diagnosis could apply to both mother and fetus. 2. The client would experience fear related to an unknown outcome. 3. Ineffective coping would be due to emergent situation secondary to uterine rupture. 5. There will be anxiety related to emergency procedures and unknown fetal outcome.

Which of the following symptoms, if progressive, are indicative of CHF, the heart's signal of its decreased ability to meet the demands of pregnancy? Note: Credit will be given only if all correct and no incorrect choices are selected. Select all that apply. 1. Palpitations 2. Heart murmurs 3. Dyspnea 4. Frequent urination 5. Rales

Answer: 1, 2, 3, 5 Explanation: 1. Palpitations are indicative of CHF. 2. Heart murmurs are indicative of CHF. 3. Dyspnea is indicative of CHF. 5. Rales are indicative of CHF.

Which interventions can the nurse utilize to provide continuity of care for the postpartal client who experienced a complication and is now ready to return home? Note: Credit will be given only if all correct choices and no incorrect choices are selected. Select all that apply. 1. Encourage the client to take advantage of home visits. 2. Make telephone calls as a follow-up to check on the client and newborn. 3. Provide information about postpartal support groups. 4. Refer to mental health professionals to help screen the client for any mental health problems as a result of the complications experienced in the hospital. 5. Supply information about postpartum expectations designed to meet the specific needs of a variety of families.

Answer: 1, 2, 3, 5 Explanation: 1. Home visits, especially for early discharge families, are invaluable in fostering positive adjustments for the new family. 2. Telephone follow-up at 2 to 3 weeks postpartum to ask whether the mother is experiencing difficulties is also helpful. 3. Support groups in which child care is available can be an invaluable community service for the postpartum client. 5. Social support teaching guides are available to assist in helping postpartum women explore their needs for postpartum support.

The nurse is presenting a class for nursing students on multiple-gestation pregnancy. Which statements about multiple-gestation pregnancies are accurate? Note: Credit will be given only if all correct choices and no incorrect choices are selected. Select all that apply. 1. Hypertension is a major maternal complication. 2. Gestational diabetes occurs more often. 3. Maternal anemia occurs. 4. Pulmonary embolism is 12 times more likely to develop during pregnancy with multiple gestations. 5. Multiple gestations are more likely to acquire HELLP.

Answer: 1, 2, 3, 5 Explanation: 1. Hypertension is a complication in multiple-gestation pregnancies. 2. Gestational diabetes occurs more often in multiple gestations. 3. Maternal anemia occurs because of demands of the multiple gestations. 5. Multiple gestations are more likely to acquire HELLP (hemolytic anemia, elevated liver enzymes, and low platelet count) syndrome, a complication resulting from eclampsia or preeclampsia.

The nurse is teaching a class on reading a fetal monitor to nursing students. The nurse explains that bradycardia is a fetal heart rate baseline below 110 and can be caused by which of the following? Note: Credit will be given only if all correct choices and no incorrect choices are selected. Select all that apply. 1. Maternal hypotension 2. Prolonged umbilical cord compression 3. Fetal dysrhythmia 4. Central nervous system malformation 5. Late fetal asphyxia

Answer: 1, 2, 3, 5 Explanation: 1. Maternal hypotension results in decreased blood flow to the fetus. 2. Cord compression can cause fetal bradycardia. 3. This will cause fetal bradycardia if there is a fetal heart block. 5. This is a depression of myocardial activity.

At her 6-week postpartum checkup, a new mother voices concerns to the nurse. She states that she is finding it hard to have time alone to even talk on the phone without interruption. Her family lives in another state, and she has contact with them only by phone. She is still having difficulty getting enough sleep and worries that she will not be a good mother. Appropriate nursing interventions would include providing which of the following? Note: Credit will be given only if all correct choices and no incorrect choices are selected. Select all that apply. 1. Anticipatory guidance about the realities of being a parent. 2. Parenting literature and reference manuals. 3. Phone numbers and locations of local parenting groups. 4. Referral for specialized interventions related to postpartum blues. 5. Phone numbers and names of postpartum doulas.

Answer: 1, 2, 3, 5 Explanation: 1. Postpartum nurses need to be aware of the long-term adjustments and stresses that the childbearing family faces as its members adjust to new and different roles. 2. Postpartum nurses need to be aware of the long-term adjustments and stresses that the childbearing family faces as its members adjust to new and different roles. 3. New mother support groups are helpful for women who lack a social support system. 5. Postpartum doulas are professionals trained to help the new mother after the birth of the baby.

) The nurse has completed the physical assessment of a client in early labor, and proceeds with the social assessment. A social history of the client would include which of the following? Note: Credit will be given only if all correct choices and no incorrect choices are selected. Select all that apply. 1. Use of drugs and alcohol 2. Family violence or sexual assault 3. Current living situation 4. Type of insurance 5. Availability of resources

Answer: 1, 2, 3, 5 Explanation: 1. Risk factors such as the use of drugs or alcohol during the pregnancy can influence the labor and birth. 2. It is imperative to ask the woman about domestic violence and to assess any degree of psychologic or physical harm, either potential or real. 3. A social assessment includes asking about the woman's current living situation. This dialog provides an opportunity for the nurse to continue to build support, to provide information when requested, and to be direct yet supportive. 5. A social assessment includes asking about resources available to the family.

A prenatal client asks the nurse about conditions that would necessitate a cesarean delivery. The nurse explains that cesarean delivery generally is performed in the presence of which of the following? Note: Credit will be given only if all correct choices and no incorrect choices are selected. Select all that apply. 1. Complete placenta previa 2. Placental abruption 3. Umbilical cord prolapse 4. Precipitous labor 5. Failure to progress

Answer: 1, 2, 3, 5 Explanation: 1. When the placenta completely covers the uterine opening, a cesarean is performed. 2. Premature separation of the placenta from the uterine wall requires an immediate cesarean. 3. A prolapsed cord is an emergency requiring an immediate cesarean. 5. Failure to progress in labor can necessitate a cesarean birth.

A newborn who has not voided by 48 hours after birth should be assessed for which of the following? Note: Credit will be given only if all correct and no incorrect choices are selected. Select all that apply. 1. Restlessness 2. Pain 3. Kidney distention 4. Adequacy of fluid intake 5. Lethargy

Answer: 1, 2, 4 Explanation: 1. A newborn who has not voided by 48 hours after birth should be assessed for restlessness. 2. A newborn who has not voided by 48 hours after birth should be assessed for pain. 4. A newborn who has not voided by 48 hours after birth should be assessed for adequacy of fluid intake.

Premonitory signs of labor include which of the following? Note: Credit will be given only for all correct choices and no incorrect choices. Select all that apply. 1. Braxton Hicks contractions 2. Cervical softening and effacement 3. Weight gain 4. Rupture of membranes 5. Sudden loss of energy

Answer: 1, 2, 4 Explanation: 1. A premonitory sign of labor includes Braxton Hicks contractions. 2. A premonitory sign of labor includes cervical softening and effacement. 4. A premonitory sign of labor includes rupture of membranes.

The nurse is caring for a client in labor. Which signs and symptoms would indicate the client is progressing into the second stage of labor? Note: Credit will be given only for all correct choices and no incorrect choices. Select all that apply. 1. Bulging perineum 2. Increased bloody show 3. Spontaneous rupture of the membranes 4. Uncontrollable urge to push 5. Inability to breathe through contractions

Answer: 1, 2, 4 Explanation: 1. As the fetal head continues its descent, the perineum begins to bulge, flatten, and move anteriorly. 2. Bloody show increases as a woman enters the second stage of labor. 4. As the fetal head descends, the woman has the urge to push because of pressure of the fetal head on the sacral and obturator nerves.

Marked changes occur in the cardiopulmonary system at birth include which of the following? Note: Credit will be given only if all correct choices and no incorrect choices are selected. Select all that apply. 1. Closure of the foramen ovale 2. Closure of the ductus venosus 3. Mean blood pressure of 31 to 61 mmHg in full-term resting newborns 4. Increased systemic vascular resistance and decreased pulmonary vascular resistance 5. Opening of the ductus arteriosus

Answer: 1, 2, 4 Explanation: 1. Closure of the foramen ovale is a function of changing arterial pressures. 2. Closure of the ductus venosus is related to mechanical pressure changes that result from severing the cord, redistribution of blood, and cardiac output. 4. Increased systemic vascular resistance and decreased pulmonary vascular resistance; with the loss of the low-resistance placenta, systemic vascular resistance increases, resulting in greater systemic pressure. The combination of vasodilation and increased pulmonary blood flow decreases pulmonary vascular resistance.

Risk factors for tachysystole include which of the following? Note: Credit will be given only if all correct choices and no incorrect choices are selected. Select all that apply. 1. Cocaine use 2. Placental abruption 3. Low-dose oxytocin titration regimens 4. Uterine rupture 5. Smoking

Answer: 1, 2, 4 Explanation: 1. Cocaine use is a risk factor for tachysystole. 2. Placental abruption is a risk factor for tachysystole. 4. Uterine rupture is a risk factor for tachysystole.

A newborn is determined to have physiological jaundice. The nurse explains the steps involved in conjugation and excretion of bilirubin to the parents. Which factors would the nurse include in the explanation? Note: Credit will be given only if all correct choices and no incorrect choices are selected. Select all that apply. 1. At birth, the newborn's liver begins to conjugate bilirubin or convert it from a yellow lipid-soluble pigment to a water-soluble pigment. 2. Unconjugated bilirubin can leave the bloodstream and enter the tissues, causing a yellow hue to the skin and sclera. 3. Unconjugated bilirubin results from the destruction of white blood cells. 4. The infant is able to excrete conjugated bilirubin, but not unconjugated bilirubin. 5. The newborn's liver has greater metabolic and enzymatic activity at birth than does an adult liver, increasing the newborn's susceptibility to jaundice.

Answer: 1, 2, 4 Explanation: 1. Conjugation, or the changing of bilirubin into an excretable form, is the conversion of the yellow lipid-soluble pigment (unconjugated, indirect) into water-soluble pigment (excretable, direct). 2. Jaundice (icterus) is the yellowish coloration of the skin and sclera caused by the presence of bilirubin in elevated concentrations. 4. Unconjugated bilirubin is fat soluble, has a propensity for fatty tissues, is not in an excretable form, and is a potential toxin.

The client is undergoing an emergency cesarean birth for fetal bradycardia. The client's partner has not been allowed into the operating room. What can the nurse do to alleviate the partner's emotional distress? Note: Credit will be given only if all correct choices and no incorrect choices are selected. Select all that apply. 1. Allow the partner to wheel the baby's crib to the newborn nursery. 2. Allow the partner to be near the operating room where the newborn's first cry can be heard. 3. Have the partner wait in the client's postpartum room. 4. Encourage the partner to be in the nursery for the initial assessment. 5. Teach the partner how to take the client's blood pressure.

Answer: 1, 2, 4 Explanation: 1. Effective measures include allowing the partner to take the baby to the nursery. 2. Effective measures include allowing the partner to be in a place near the operating room, where the newborn's first cry can be heard. 4. Effective measures include involving the partner in postpartum care in the recovery room.

A full-term infant has just been born. Which interventions should the nurse perform first? Note: Credit will be given only if all correct choices and no incorrect choices are selected. Select all that apply. 1. Placing the infant in a radiant-heated unit 2. Suctioning the infant with a bulb syringe 3. Wrapping the infant in a blanket 4. Evaluating the newborn using the Apgar system 5. Offering a feeding of 5% glucose water

Answer: 1, 2, 4 Explanation: 1. If the newborn is placed in a radiant-heated unit, he or she is dried, laid on a dry blanket, and left uncovered under the radiant heat. 2. Newborns are suctioned with a bulb syringe to clear mucus from the newborn's mouth. 4. The purpose of the Apgar score is to evaluate the physical condition of the newborn at birth.

The nurse is teaching an infertile couple about the causes of infertility. The nurse tells them that infertility can be caused by which of the following? Note: Credit will be given only if all correct and no incorrect choices are selected. Select all that apply. 1. Immunological responses 2. Congenital anomalies 3. Patent fallopian tubes 4. Hypothyroidism 5. Favorable cervical mucus

Answer: 1, 2, 4 Explanation: 1. Immunological responses, such as antisperm antibodies, can cause infertility. 2. Congenital anomalies, such as a septate uterus, can cause infertility. 4. Hypothyroidism is a cause of infertility.

Which strategies would the nurse utilize to promote culturally competent care for the postpartum client? Note: Credit will be given only if all correct choices and no incorrect choices are selected. Select all that apply. 1. Examine one's own cultural beliefs, biases, stereotypes, and prejudices. 2. Respect the values and beliefs of others. 3. Limit the alternative food choices offered clients to minimize conflicts. 4. Incorporate the family's cultural practices into the care. 5. Evaluate whether the family's cultural practices fit into Western norms.

Answer: 1, 2, 4 Explanation: 1. It is important for nurses to recognize that they are approaching their patient's care from their own perspective and that, to individualize care for each mother, they need to assess the woman's preferences, her level of acculturation and assimilation to Western culture, her linguistic abilities, and her educational level. 2. It is important for nurses to recognize that they are approaching their patient's care from their own perspective and that, to individualize care for each mother, they need to assess the woman's preferences, her level of acculturation and assimilation to Western culture, her linguistic abilities, and her educational level. 4. The nurse should have the mother exercise her choices when possible and support those choices, with the help of cultural awareness and a sound knowledge base.

What of the following nursing interventions are appropriate when caring for the family experiencing a stillbirth? Note: Credit will be given only if all correct choices and no incorrect choices are selected. Select all that apply. 1. Use active listening techniques. 2. Avoid the use of clichés. 3. Avoid periods of silence. 4. Wrap the infant in a blanket before the parents see the infant. 5. Do not permit the parents of an infant with birth defects to hold the infant.

Answer: 1, 2, 4 Explanation: 1. It is important to allow the parents to verbalize their concerns. 2. The nurse can facilitate a healthy mourning process for the family by using active listening techniques and avoiding the use of clichés and platitudes. 4. The infant should be wrapped in a blanket to allow parents to see the infant before viewing any deformities.

What maternity unit policies promote postpartal family wellness and shared parenting? Note: Credit will be given only if all correct choices and no incorrect choices are selected. Select all that apply. 1. Mother-baby care or couplet care on the postpartum unit 2. Skin-to-skin contact between the mother and baby and the father and baby 3. Newborn kept in the nursery to allow mother to rest between feedings 4. On-demand feeding schedule for both breastfed and bottle-fed infants 5. Limited visiting hours for the father so that the mother can sleep as needed

Answer: 1, 2, 4 Explanation: 1. Mother-baby care is an important part of the family-centered care approach. 2. The nurse should encourage skin-to-skin contact between mother (or father) and baby to promote breastfeeding and bonding. 4. The mother-baby unit is conducive to an on-demand feeding schedule for both breastfeeding and formula-feeding infants.

A nurse working with adolescents recognizes risk factors for pregnancy include which of the following? Note: Credit will be given only if all correct and no incorrect choices are selected. Select all that apply. 1. Low socioeconomic status 2. A belief in invulnerability 3. A highly functional family 4. Physical, emotional, or sexual abuse 5. Accurate information about contraception

Answer: 1, 2, 4 Explanation: 1. Poverty is a major risk factor for adolescent pregnancy. 2. The sense of invulnerability and "it won't happen to me" thinking can lead adolescents to participate in sexual activity with an overly optimistic view of the outcomes of the risks associated with their actions. 4. Physical, emotional, or sexual abuse increases the risk of teenage pregnancy.

The nurse tells a mother that the doctor is preparing to circumcise her newborn. The mother expresses concern that the infant will be uncomfortable during the procedure. The nurse explains that the physician will numb the area before the procedure. Additional methods of comfort often used during the procedure include which of the following? Note: Credit will be given only if all correct choices and no incorrect choices are selected. Select all that apply. 1. Providing a pacifier 2. Stroking the head 3. Restraining both arms and legs 4. Talking to the infant 5. Giving the infant a sedative before the procedure

Answer: 1, 2, 4 Explanation: 1. Providing a pacifier is an accepted method of soothing during the circumcision. 2. Stroking the head is an accepted method of soothing during the circumcision. 4. Talking to the infant is an accepted method of soothing during the circumcision.

The nurse has written the nursing diagnosis Injury, Risk for for a diabetic pregnant client. Interventions for this diagnosis include which of the following? Note: Credit will be given only if all correct and no incorrect choices are selected. Select all that apply. 1. Assessment of fetal heart tones 2. Perform oxytocin challenge test, if ordered 3. Refer the client to a diabetes support group 4. Assist with the biophysical profile assessment 5. Develop an appropriate teaching plan

Answer: 1, 2, 4 Explanation: 1. Reassuring fetal heart rate variability and accelerations are interpreted as adequate placental oxygenation. 2. The nurse would perform oxytocin challenge test (OCT)/contraction stress test (CST) and non-stress tests as determined by physician. 4. The nurse assists the physician in performing a biophysical profile assessment.

Which of the following are specific culturally sensitive nursing considerations the nurse integrates into care of the pregnant client? Note: Credit will be given only if all correct and no incorrect choices are selected. Select all that apply. 1. Counseling about home remedies 2. Discussing components of a balanced diet 3. Refusing to discuss birthing choices 4. Encouraging use of support systems 5. Instructing the client to use no home remedies

Answer: 1, 2, 4 Explanation: 1. The culturally sensitive nurse should find out what medications and home remedies the client is using, and counsel the client regarding overall effects. 2. The culturally sensitive nurse will discuss the importance of a well-balanced diet during pregnancy with consideration of the client's cultural beliefs and practices. 4. Encouraging the use of support systems and spiritual aids that provide comfort for the mother is important in culturally sensitive care.

The nurse is presenting a class on the pathophysiology of the different abortions. Some of the causes are which of the following? Note: Credit will be given only if all correct choices and no incorrect choices are selected. Select all that apply. 1. Chromosomal abnormalities 2. Insufficient or excessive hormonal levels 3. Sexual intercourse in the first trimester 4. Infections in the first trimester 5. Cervical insufficiency

Answer: 1, 2, 4, 5 Explanation: 1. Chromosomal defects are generally seen as spontaneous abortions during weeks 4 to 8. 2. Insufficient or excessive hormonal levels usually will result in spontaneous abortion by 10 weeks' gestation. 4. Infectious and environmental factors may also be seen in first trimester pregnancy loss. 5. In late spontaneous abortion, the cause is usually a maternal factor, for example, cervical insufficiency or maternal disease, and fetal death may not precede the onset of abortion.

The nurse is planning an early-pregnancy class session on nutrition. Which information should the nurse include? Note: Credit will be given only if all correct choices and no incorrect choices are selected. Select all that apply. 1. Protein is important for fetal development. 2. Iron helps both mother and baby maintain the oxygen-carrying capacity of the blood. 3. Calcium prevents constipation at the end of pregnancy. 4. Zinc facilitates synthesis of RNA and DNA. 5. Vitamin A promotes development of the baby's eyes.

Answer: 1, 2, 4, 5 Explanation: 1. During pregnancy, the woman needs increased amounts of protein to provide amino acids for fetal development. 2. Iron deficiency anemia is associated with an increased incidence of preterm birth, low-birth-weight infants, and maternal and infant mortality. 4. Zinc is involved in RNA and DNA synthesis, and milk production during lactation. 5. Vitamin A promotes healthy formation and development of the fetal eyes.

When caring for a laboring client with oligohydramnios, what should the nurse be aware of? Note: Credit will be given only if all correct choices and no incorrect choices are selected. Select all that apply. 1. Increased risk of cord compression 2. Decreased variability 3. Labor progress is often more rapid than average 4. Presence of periodic decelerations 5. During gestation, fetal skin and skeletal abnormalities can occur

Answer: 1, 2, 4, 5 Explanation: 1. During the labor and birth, the lessened amounts of fluid reduce the cushioning effect for the umbilical cord, and cord compression is more likely to occur. 2. The nurse should evaluate the EFM tracing for the presence of nonperiodic decelerations or other nonreassuring signs (such as increasing or decreasing baseline, decreased variability, or presence of periodic decelerations). 4. The nurse should evaluate the EFM tracing for the presence of nonperiodic decelerations or other nonreassuring signs (such as increasing or decreasing baseline, decreased variability, or presence of periodic decelerations). 5. During the gestational period, fetal skin and skeletal abnormalities may occur because fetal movement is impaired as a result of inadequate amniotic fluid volume.

The nurse is caring for a client who is showing a sinusoidal fetal heart rate pattern on the monitor. The nurse knows that possible causes for this pattern include which of the following? Note: Credit will be given only if all correct choices and no incorrect choices are selected. Select all that apply. 1. Fetal anemia 2. Chronic fetal bleeding 3. Maternal hypotension 4. Twin-to-twin transfusion 5. Umbilical cord occlusion

Answer: 1, 2, 4, 5 Explanation: 1. Fetal anemia can cause a sinusoidal heart rate. 2. Chronic fetal bleeding can cause a sinusoidal heart rate. 4. Twin-to-twin transfusion will cause a sinusoidal heart rate. 5. Umbilical cord occlusion can cause a sinusoidal fetal heart rate.

The nurse is evaluating the plan of care for a pregnant client with a heart disorder. The nurse concludes that the plan was successful when data indicate which of the following? Note: Credit will be given only if all correct and no incorrect choices are selected. Select all that apply. 1. The client gave birth to a healthy baby. 2. The client did not develop congestive heart failure. 3. The client developed thromboembolism. 4. The client identified manifestations of potential complications. 5. The client can identify her condition and its impact on her pregnancy, labor and birth, and postpartum period.

Answer: 1, 2, 4, 5 Explanation: 1. Giving birth to a healthy baby is an expected outcome of the pregnancy. 2. An expected outcome is that the woman does not develop congestive heart failure, thromboembolism, or infection. 4. An expected outcome is that the woman is able to identify potential complications and notify the healthcare provider. 5. The woman must be able to discuss her condition and its possible impact on her pregnancy, labor and birth, and the postpartum period.

The nurse is assessing a client before administering an analgesic. What are some of the factors the nurse should consider? Note: Credit will be given only if all correct choices and no incorrect choices are selected. Select all that apply. 1. The client is willing to receive medication after being advised about it. 2. The client's vital signs are stable. 3. The partner agrees to use of the medication. 4. The client has no known allergies to the medication. 5. The client is aware of the contraindications of the medication.

Answer: 1, 2, 4, 5 Explanation: 1. Medication should be explained to the client before it is administered. 2. Vital signs need to be stable before any analgesic medication is administered. 4. Ask the client about allergies before administering any medications. 5. Clients should always be aware of the contraindications of the medication.

When caring for a 13-year-old client in labor, how would the nurse provide sensitive care? Note: Credit will be given only if all correct choices and no incorrect choices are selected. Select all that apply. 1. Using simple and concrete instructions 2. Providing soothing encouragement and comfort measures 3. Making all decisions for the client when she expresses a feeling of helplessness 4. Deciding whom the client should allow in the room 5. Providing encouragement and support of the client's decisions

Answer: 1, 2, 5 Explanation: 1. A client at this developmental stage will need concrete and simplified instructions. 2. Touch, soothing encouragement, and measures to promote her comfort help her maintain control and meet her needs for dependence. 5. Establishing rapport without recrimination will provide emotional support and encouragement.

When providing anticipatory guidance to a new mother, what information does the nurse convey about the newborn's neurologic and sensory/perceptual functioning? Note: Credit will be given only if all correct and no incorrect choices are selected. Select all that apply. 1. Newborns respond to and interact with the environment in a predictable pattern of behavior, reacting differently to a variety of stresses. 2. The usual position of the newborn is with extremities partially flexed, legs near the abdomen. 3. Newborns do not react to bright light, and their eye movements do not permit them to fixate on faces or objects until they are 3 months of age. 4. Newborns have the capacity to utilize self-quieting behaviors to quiet and comfort themselves. 5. The newborn is very sensitive to being touched, cuddled, and held.

Answer: 1, 2, 4, 5 Explanation: 1. Newborns respond to and interact with the environment in a predictable pattern of behavior that is shaped somewhat by their intrauterine experience. 2. Normal newborns are usually in a position of partially flexed extremities with the legs near the abdomen. 4. Self-quieting ability is the ability of newborns to use their own resources to quiet and comfort themselves. 5. The newborn is very sensitive to being touched, cuddled, and held; thus touch may be the most important of all of the senses for the newborn infant.

When preparing for and performing an assessment of the postpartum client, which of the following would the nurse do? Note: Credit will be given only if all correct choices and no incorrect choices are selected. Select all that apply. 1. Ask the client to void before assessing the uterus. 2. Inform the client of the need for regular assessments. 3. Defer client teaching to another time. 4. Perform the procedures as gently as possible. 5. Take precautions to prevent exposure to body fluids.

Answer: 1, 2, 4, 5 Explanation: 1. Palpating the fundus when the woman has a full bladder may give false information about the progress of involution. Ask the woman to void before assessment. 2. The nurse should provide an explanation of the purposes of regular assessment to the woman. 4. The woman should be relaxed before starting, and procedures should be performed as gently as possible, to avoid unnecessary discomfort. 5. Gloves should be worn before starting the assessment.

Which findings would indicate the presence of a perineal wound infection? Note: Credit will be given only if all correct choices and no incorrect choices are selected. Select all that apply. 1. Redness 2. Tender at the margins 3. Vaginal bleeding 4. Hardened tissue 5. Purulent drainage

Answer: 1, 2, 4, 5 Explanation: 1. Redness is a classic sign of a perineal wound infection. 2. The wound is typically red, indurated, tender at the margins, and draining purulent exudate. 4. The wound is typically red, indurated, tender at the margins, and draining purulent exudate. 5. Purulent drainage is a classic signs of a perineal wound infection.

The nurse is assisting a mother to bottle-feed her newborn, who has been crying. The nurse suggests that prior to feeding, the mother should do which of the following? 1. Offer a pacifier 2. Burp the newborn 3. Unwrap the newborn 4. Stroke the newborn's spine and feet

Answer: 2 Explanation: 2. Crying results in increased ingestion of air even before the infant has started feeding. Infants who are very hungry also gulp more air. For these situations, instruct the parents to burp their infant frequently.

The nurse is talking with a couple who have been trying to get pregnant for 5 years. They are now at the fertility clinic seeking help. The nurse assesses their emotional responses as part of the workup. Which responses would the nurse expect to hear? Note: Credit will be given only if all correct choices and no incorrect choices are selected. Select all that apply. 1. Experiencing a sense of loss of status 2. Feelings of failure because they cannot make a baby 3. Healthy relationship with healthcare partners 4. Stress on the marital and sexual relationship 5. Feelings of frustration

Answer: 1, 2, 4, 5 Explanation: 1. The couple may experience feelings of loss of status and ambiguity as a couple. 2. Feelings of failure are common. 4. The couple may experience stress on the marital and sexual relationship. 5. Tests and treatments may heighten feelings of frustration or anger between partners.

The nurse is caring for a newborn with full fontanelles and "setting sun" eyes. Which nursing interventions should be included in the care plan? Note: Credit will be given only if all correct choices and no incorrect choices are selected. Select all that apply. 1. Measure head circumference daily. 2. Assess for bulging fontanelles. 3. Avoid position changes. 4. Watch for signs of infection. 5. Use a gel pillow under the head.

Answer: 1, 2, 4, 5 Explanation: 1. The infant has congenital hydrocephalus. The nurse should measure and plot occipital-frontal baseline measurements, then measure head circumference once a day. 2. The infant has congenital hydrocephalus. Fontanelles should be checked for bulging and sutures for widening. 4. Infants with hydrocephalus are prone to infection. 5. The infant has congenital hydrocephalus. The enlarged head should be supported with a gel pillow.

A mother and her newborn have been discharged after a hospital stay of less than 48 hours. What are essential components the nurse must include in the first postpartum home visit? Note: Credit will be given only if all correct choices and no incorrect choices are selected. Select all that apply. 1. Assessment of color 2. Measurement of weight 3. Measurement of height 4. Assessment of mother-newborn interaction 5. Reinforcement of information about feeding and sleep patterns

Answer: 1, 2, 4, 5 Explanation: 1. The nurse should assess the newborn's general health, hydration, and degree of jaundice. 2. The nurse should weigh the infant. 4. This is an opportune time to assess the quality of mother-newborn interaction and details of newborn behavior. 5. Parents frequently need further clarification and reinforcement of maternal or family education in neonatal care, particularly feeding and sleep position.

Remedies for back pain in pregnancy that are supported by research evidence and may safely be taught to any pregnant woman by the nurse include which of the following? Note: Credit will be given only if all correct choices and no incorrect choices are selected. Select all that apply. 1. Pelvic tilt 2. Water aerobics 3. Sit-ups 4. Proper body mechanics 5. Maintaining good posture

Answer: 1, 2, 4, 5 Explanation: 1. The pelvic tilt can help restore proper body alignment and relieve back pain. 2. Exercise is an effective treatment for lower back pain. Exercise in water seems to provide benefits while being physically comfortable for expectant mothers. 4. The use of proper posture and good body mechanics throughout pregnancy is important. 5. Good posture is important because it allows more room for the stomach to function.

The nurse is providing postpartum care to an obese client. As part of care for this client, the nurse should do which of the following? Note: Credit will be given only if all correct choices and no incorrect choices are selected. Select all that apply. 1. Apply sequential compression devices 2. Have the mother ambulate as early as possible 3. Encourage bottle-feeding over breastfeeding 4. Supervise breastfeeding 5. Instruct the client on signs of infection

Answer: 1, 2, 4, 5 Explanation: 1. The use of sequential compression devices (SCDs) and early ambulation are essential to the prevention of deep vein thrombosis, especially if the client had a cesarean birth. 2. Ambulation should be encouraged as soon as possible to prevent pneumonia. 4. The new mother may need extra supervision and assistance when breastfeeding her baby to ensure newborn safety. 5. The obese client has needs similar to all postpartum client, but she needs special attention to prevent injury, respiratory complications, thromboembolic disease, and infection, for which she is at high risk.

A pregnant client calls the clinic nurse to say she is worried about symptoms she is experiencing. The nurse advises the client to come immediately to the clinic because of which reported symptoms? Note: Credit will be given only if all correct and no incorrect choices are selected. Select all that apply. 1. Vaginal bleeding 2. Abdominal pain 3. Constipation 4. Epigastric pain 5. Blurring of vision

Answer: 1, 2, 4, 5 Explanation: 1. Vaginal bleeding can indicate abruptio placentae, placenta previa, or lesions of cervix or vagina, or it can be "bloody show," and requires that the client be seen. 2. Abdominal pain can signal premature labor or abruptio placentae, and requires that the client be seen. 4. Epigastric pain must be evaluated, as it can indicate preeclampsia or ischemia in a major abdominal vessel. 5. Dizziness, blurring of vision, double vision, or spots before the eyes can indicate either hypertension and/or preeclampsia and requires the client be seen.

The nurse is caring for a newborn with jaundice. The parents question why the newborn is not under phototherapy lights. The nurse explains that the fiber-optic blanket is beneficial because of which of the following? Note: Credit will be given only if all correct choices and no incorrect choices are selected. Select all that apply. 1. Lights can stay on all the time. 2. The eyes do not need to be covered. 3. The lights will need to be removed for feedings. 4. Newborns do not get overheated. 5. Weight loss is not a complication of this system.

Answer: 1, 2, 4, 5 Explanation: 1. With the fiber-optic blanket, the light stays on at all times. 2. The eyes do not have to be covered with a fiber optic blanket. 4. With the fiber-optic blanket, greater surface area is exposed and there are no thermoregulation issues. 5. Fluid and weight loss are not complications of fiber-optic blankets.

During labor, the client at 4 cm suddenly becomes short of breath, cyanotic, and hypoxic. The nurse must prepare or arrange immediately for which of the following? Note: Credit will be given only if all correct choices and no incorrect choices are selected. Select all that apply. 1. Intravenous access 2. Cesarean delivery 3. Immediate vaginal delivery 4. McRoberts maneuver 5. A crash cart

Answer: 1, 2, 5 Explanation: 1. When an amniotic fluid embolism is suspected, intravenous access is obtained as quickly as possible. 2. Shortness of breath, cyanosis, and hypoxia are symptoms of an amniotic fluid embolus, which necessitates immediate cesarean delivery. 5. The chances of a code are high, so the crash cart needs to be available.

) The client and her partner are carriers of sickle-cell disease. They are considering prenatal diagnosis with either amniocentesis or chorionic villus sampling (CVS). Which statements indicate that further teaching is needed on these two diagnostic procedures? Note: Credit will be given only if all correct choices and no incorrect choices are selected. Select all that apply. 1. "Chorionic villus sampling carries a lower risk of miscarriage." 2. "Amniocentesis can be done earlier in my pregnancy than CVS." 3. "Neither test will conclusively diagnose sickle-cell disease in our baby." 4. "The diagnosis comes sooner if we have CVS, not amniocentesis." 5. "Amniocentesis is more accurate in diagnosis than the CVS."

Answer: 1, 2, 5 Explanation: 1. CVS has a risk of spontaneous abortion of 0.3% in cases. This rate is higher than second trimester amniocentesis. 2. CVS is performed in some medical centers for first trimester diagnosis after 9 completed weeks. Amniocentesis is performed between 15 and 20 weeks' gestation. 5. Much like amniocentesis, chorionic villus sampling (CVS) is a procedure that is used to detect genetic, metabolic, and DNA abnormalities. CVS permits earlier diagnosis than can be obtained by amniocentesis.

What information should the nurse include when teaching the postpartal client and partner about resumption of sexual activity? Note: Credit will be given only if all correct choices and no incorrect choices are selected. Select all that apply. 1. Couples should be encouraged to abstain from intercourse until the episiotomy is healed and the lochial flow has stopped. 2. Postpartum women often experience vaginal dryness, and should be encouraged to use some kind of lubrication initially during intercourse. 3. Breastfeeding the newborn after intercourse can reduce the chance of milk spouting from the nipples. 4. Maternal changes in libido are usually indicative of psychological depression. 5. Maternal fatigue is often a significant factor limiting the resumption of sexual intercourse.

Answer: 1, 2, 5 Explanation: 1. Currently, the couple is advised to abstain from intercourse until the episiotomy is healed and the lochia has stopped. 2. Because postpartum women often experience vaginal dryness due to hormonal changes, the use of a water-based lubrication, such as K-Y jelly or Astroglide, may initially be necessary during intercourse. 5. Maternal fatigue is often a significant factor limiting the resumption of sexual intercourse. While interest and desire vary, most couples resume sexual activity within 3 months.

Risk factors associated with increased risk of thromboembolic disease include which of the following? Note: Credit will be given only if all correct choices and no incorrect choices are selected. Select all that apply. 1. Diabetes mellitus 2. Varicose veins 3. Hypertension 4. Adolescent pregnancy 5. Malignancy

Answer: 1, 2, 5 Explanation: 1. Diabetes mellitus is a risk factor for thromboembolic disease. 2. Varicose veins are a risk factor for thromboembolic disease. 5. Malignancy is a risk factor for thromboembolic disease.

The nurse is caring for a client who has just experienced a stillbirth. Which factors does the nurse recognize as potentially complicating the parents' response to this loss? Note: Credit will be given only if all correct choices and no incorrect choices are selected. Select all that apply. 1. Unsupportive family 2. Adolescent mother 3. Strong religious faith 4. Open communication between the parents 5. Persistent denial of the situation

Answer: 1, 2, 5 Explanation: 1. Features of bereaved individuals' circumstances that will put them at risk include an unsupportive or unavailable family. 2. With regard to age, adolescent parents probably pose the greatest challenge to nursing interventions. 5. Persistent denial hampers the grieving and healing processes.

The client presents to the labor and delivery unit stating that her water broke 2 hours ago. Indicators of normal labor include which of the following? Note: Credit will be given only if all correct choices and no incorrect choices are selected. Select all that apply. 1. Fetal heart rate of 130 with average variability 2. Blood pressure of 130/80 3. Maternal pulse of 160 4. Protein of +1 in urine 5. Odorless, clear fluid on underwear

Answer: 1, 2, 5 Explanation: 1. Fetal heart rate (FHR) of 110-160 with average variability is a normal indication. 2. Maternal vital sign of blood pressure below 140/90 is a normal indication. 5. Fluid clear and without odor if membranes ruptured is a normal indication.

The nurse is planning a prenatal class for a group of pregnant adolescents. The nurse will establish which goals for the class? Note: Credit will be given only if all correct and no incorrect choices are selected. Select all that apply. 1. Provide anticipatory guidance 2. Prepare the participants for labor and birth 3. Avoid discussion of the client's conflicts 4. Provide community resources if asked 5. Help the participants develop coping skills

Answer: 1, 2, 5 Explanation: 1. Goals for a prenatal class designed for adolescents must include anticipatory guidance. 2. Goals for a prenatal class designed for adolescents must include preparing the participants for labor and the birth of their baby. 5. Goals for a prenatal class designed for adolescents must include helping the participants develop adequate coping skills.

Fetal factors that possibly indicate electronic fetal monitoring include which of the following? 1. Meconium passage 2. Multiple gestation 3. Preeclampsia 4. Grand multiparity 5. Decreased fetal movement

Answer: 1, 2, 5 Explanation: 1. Meconium passage is an indicator for electronic fetal monitoring. 2. Multiple gestation is an indicator for electronic fetal monitoring. 5. Decreased fetal movement is an indicator for electronic fetal monitoring.

Which of the following are potential disadvantages to breastfeeding? Note: Credit will be given only if all correct choices and no incorrect choices are selected. Select all that apply. 1. Pain with breastfeeding 2. Leaking milk 3. Equal feeding responsibilities with fathers 4. Vaginal wetness 5. Embarrassment

Answer: 1, 2, 5 Explanation: 1. Nipple tenderness is the most common source of discomfort and is usually related to improper positioning and/or not obtaining a proper attachment of the infant on the breast. Pain can also be related to engorgement or infection. 2. Some women will leak milk when their breasts are full and it is nearly time to breastfeed again or whenever they experience let-down. Mothers should be given reassurance that this problem diminishes over time. 5. Some mothers feel uncomfortable about breastfeeding because they are modest or may feel embarrassed because our society views breasts as sexual objects and/or an unfriendly social environment makes it difficult to breastfeed in public. This is not an easy issue to overcome.

A new mother is questioning the nurse about the advantages of breastfeeding her newborn. Which information should the nurse include in the teaching session? Note: Credit will be given only if all correct choices and no incorrect choices are selected. Select all that apply. 1. Breast milk has immunological advantages, including varying degrees of protection from bacterial and viral infections. 2. Breastfeeding has been shown to increase maternal-infant attachment. 3. Breastfeeding can be initially supplemented with bottle feedings so that the father does not feel left out of the infant's care. 4. Breastfeeding often causes nipple tenderness, and may be discouraged until healing occurs. 5. Breastfeeding provides a psychologic advantage to the mother, who derives satisfaction knowing that she is providing her infant with the optimal nutritional start in life.

Answer: 1, 2, 5 Explanation: 1. Secretory IgA, an immunoglobulin present in colostrum and mature breast milk, has antiviral, antibacterial, and antigenic-inhibiting properties. 2. Early breastfeeding can enhance maternal-infant bonding. 5. The mother's sense of accomplishment in being able to satisfy her baby's needs for nourishment and comfort can be a tremendous source of personal satisfaction.

Which statements by a breastfeeding class participant indicate that teaching by the nurse was effective? Note: Credit will be given only if all correct choices and no incorrect choices are selected. Select all that apply. 1. "Breastfed infants get more skin-to-skin contact and sleep better." 2. "Breastfeeding raises the level of a hormone that makes me feel good." 3. "Breastfeeding is complex and difficult, and I probably won't succeed." 4. "Breastfeeding is worthwhile, even if it costs more overall." 5. "Breastfed infants have fewer digestive and respiratory illnesses."

Answer: 1, 2, 5 Explanation: 1. Skin-to-skin contact after birth helps the baby maintain his or her body temperature, helps with self-regulation, increases maternal oxytocin levels, helps the mother to notice subtle feeding cues, and promotes bonding. 2. Hormones of lactation promote maternal feelings and sense of well-being. 5. This is a true statement. The immunologic advantages of human milk include varying degrees of protection from respiratory tract and gastrointestinal tract infections.

The nurse is administering erythromycin (Ilotycin) ointment to a newborn. What factors are associated with administration of this medication? Note: Credit will be given only if all correct choices and no incorrect choices are selected. Select all that apply. 1. The medication should be instilled in the lower conjunctival sac of each eye. 2. The eyelids should be massaged gently to distribute the ointment. 3. The medication must be given immediately after delivery. 4. The medication does not cause any discomfort to the infant. 5. The medication can interfere with the baby's ability to focus.

Answer: 1, 2, 5 Explanation: 1. Successful eye prophylaxis requires that the medication be instilled in the lower conjunctival sac of each eye. 2. After administration, the nurse massages the eyelid gently to distribute the ointment. 5. Eye prophylaxis medication can cause chemical conjunctivitis, which gives the newborn some discomfort and can interfere with the baby's ability to focus on the parents' faces.

3) A client in labor is requesting pain medication. The nurse assesses her labor status first, focusing on which of the following? Note: Credit will be given only if all correct choices and no incorrect choices are selected. Select all that apply. 1. Contraction pattern 2. Amount of cervical dilatation 3. When the labor began 4. Whether the membranes are intact or ruptured 5. Fetal presenting part

Answer: 1, 2, 5 Explanation: 1. The client should have a good contraction pattern before receiving an analgesic. 2. The nurse should evaluate the amount of cervical dilatation before analgesic medication is administered. 5. If normal parameters are absent or if nonreassuring maternal or fetal factors are present, the nurse may need to complete further assessments with the physician/CNM.

Which of the following activities allows the nurse to provide individualized parent teaching on the maternal-infant unit? Note: Credit will be given only if all correct choices and no incorrect choices are selected. Select all that apply. 1. Teach by example and role modeling when caring for the newborn in the client's room. 2. Teach at every opportunity, even during the night shift, if the occasion arises. 3. Teach using newborn care videos and group classes. 4. Teach using the 24-hour educational television channels in the client's room. 5. Teach using one-to-one instruction while in the client's room.

Answer: 1, 2, 5 Explanation: 1. The nurse can be an excellent role model for families. Teaching by example is a very effective way to teach infant care. 2. One-to-one teaching while the nurse is in the client's room is shown to be the most effective educational model. 5. One-to-one teaching while the nurse is in the client's room is the most effective educational model.

To maintain a healthy temperature in the newborn, which of the following actions should be taken? Note: Credit will be given only if all correct choices and no incorrect choices are selected. Select all that apply. 1. Keep the newborn's clothing and bedding dry. 2. Reduce the newborn's exposure to drafts. 3. Do not use the radiant warmer during procedures. 4. Do not wrap the newborn. 5. Encourage the mother to snuggle with the newborn under blankets.

Answer: 1, 2, 5 Explanation: 1. To maintain a healthy temperature in the newborn, keep the newborn's clothing and bedding dry. 2. To maintain a healthy temperature in the newborn, reduce the newborn's exposure to drafts. 5. To maintain a healthy temperature in the newborn, encourage the mother to snuggle with the newborn under blankets.

During the history, the client admits to being HIV-positive and says she knows that she is about 16 weeks pregnant. Which statements made by the client indicate an understanding of the plan of care both during the pregnancy and postpartally? Note: Credit will be given only if all correct choices and no incorrect choices are selected. Select all that apply. 1. "During labor and delivery, I can expect the zidovudine (ZDV) to be given in my IV." 2. "After delivery, the dose of zidovudine (ZDV) will be doubled to prevent further infection." 3. "My baby will be started on zidovudine (ZDV) for six weeks following the birth." 4. "My baby's zidovudine (ZDV) will be given in a cream form." 5. "My baby will not need zidovudine (ZDV) if I take it during my pregnancy."

Answer: 1, 3 Explanation: 1. ART therapy generally it includes oral Zidovudine (ZDV) daily, IV ZDV during labor and until birth, and ZDV therapy for the infant for 6 weeks following birth. 3. ART therapy generally it includes oral Zidovudine (ZDV) daily, IV ZDV during labor and until birth, and ZDV therapy for the infant for 6 weeks following birth.

A couple request to see their stillborn infant. How should the nurse prepare the infant? Note: Credit will be given only if all correct choices and no incorrect choices are selected. Select all that apply. 1. Wrapping the infant in a blanket 2. Removing all blankets from the infant 3. Placing a hat on the infant 4. Removing any identification from the infant 5. Placing a diaper on the infant

Answer: 1, 3 Explanation: 1. After bathing, the infant should be placed in a suitable-sized gown and then wrapped in a blanket. Many parents will eventually remove the covering to inspect the infant; however, applying a covering allows them time to adjust to the appearance at their own pace. 3. A hat can be applied to cover birth defects. This allows the parents an opportunity to view the infant before seeing the birth defect.

The pregnant client states she does not want "to take all these supplements." What recommendations could the nurse make for the client? Note: Credit will be given only if all correct choices and no incorrect choices are selected. Select all that apply. 1. "Folic acid has been found to be essential for minimizing the risk of neural tube defects." 2. "You do not have to take these supplements if you think you are healthy enough." 3. "Most women do not have adequate intake of iron pre-pregnancy, and iron needs increase with pregnancy." 4. "These medications do the same thing. I will call your physician to cancel one of your medications." 5. "You should take the folic acid, but the vitamins are not that important."

Answer: 1, 3 Explanation: 1. An inadequate intake of folic acid has been associated with neural tube defects (NTDs) (e.g., spina bifida, anencephaly). 3. Iron is essential because many pregnant women do not have adequate intake of iron before pregnancy.

A client had an epidural inserted 2 hours ago. It is functioning well, the client is stable, and labor is progressing. Which parts of the nurse's assessment have the highest priority? Note: Credit will be given only if all correct choices and no incorrect choices are selected. Select all that apply. 1. Assess blood pressure every hour. 2. Assess the pulse rate every hour. 3. Palpate the bladder. 4. Auscultate the lungs. 5. Assess the reflexes.

Answer: 1, 3 Explanation: 1. Blood pressure should be monitored every 1 to 2 minutes for the first 10 minutes and then every 5 to 15 minutes until the block wears off because hypotension is the most common side effect of epidural anesthesia. 3. Nursing care following an epidural block includes frequent assessment of the bladder to avoid bladder distention. Catheterization may be necessary, because most women are unable to void.

A pregnant client at 30 weeks' gestation has had a steady rise in blood pressure. She is now 20 mmHg above her systolic baseline. The nurse advises her to immediately report which symptoms? Note: Credit will be given only if all correct and no incorrect choices are selected. Select all that apply. 1. Dizziness 2. Even a small amount of dependent edema 3. Spots before her eyes 4. Persistent nausea and vomiting 5. Vaginal spotting

Answer: 1, 3 Explanation: 1. Dizziness can be a sign of hypertension or preeclampsia, and should be reported immediately. 3. Spots before the eyes can be a sign of hypertension or preeclampsia, and should be reported immediately.

A client is experiencing excessive bleeding immediately after the birth of her newborn. After speeding up the IV fluids containing oxytocin, with no noticeable decrease in the bleeding, the nurse should anticipate the physician requesting which medications? Note: Credit will be given only if all correct choices and no incorrect choices are selected. Select all that apply. 1. Methergine 2. Coumadin 3. Misoprostol 4. Serotonin reuptake inhibitors (SSRIs) 5. Nonsteroidal anti-inflammatory drugs

Answer: 1, 3 Explanation: 1. Methergine is commonly used orally for postpartum hemorrhage. 3. Misoprostol is commonly used rectally for postpartum hemorrhage.

The client at 34 weeks' gestation has been stabbed in the low abdomen by her boyfriend. She is brought to the emergency department for treatment. Which statements indicate that the client understands the treatment being administered? Note: Credit will be given only if all correct choices and no incorrect choices are selected. Select all that apply. 1. "The baby needs to be monitored to check the heart rate." 2. "My bowel has probably been lacerated by the knife." 3. "I might need an ultrasound to look at the baby." 4. "The catheter in my bladder will prevent urinary complications." 5. "The IV in my arm will replace the amniotic fluid if it is leaking."

Answer: 1, 3 Explanation: 1. Ongoing assessments of trauma include evaluation of uterine tone, contractions and tenderness, fundal height, fetal heart rate, intake and output and other indicators of shock, normal postoperative evaluation in those women requiring surgery, determination of neurologic status, and assessment of mental outlook and anxiety level. 3. In cases of noncatastrophic trauma, where the mother's life is not directly threatened, fetal monitoring for 4 hours should be sufficient if there is no vaginal bleeding, uterine tenderness, contractions, or leaking amniotic fluid.

The nurse is at the home of a postpartum client for an initial assessment. The client gave birth by cesarean section 1 week earlier. Which statements should the nurse include? Note: Credit will be given only if all correct choices and no incorrect choices are selected. Select all that apply. 1. "Because you had a cesarean, I'd like to assess your incision." 2. "You aren't having any problems nursing, right?" 3. "How rested do you feel since you came home?" 4. "Because you are bottle-feeding, I won't assess your breasts." 5. "You should remain at home for the first 3 weeks after delivery."

Answer: 1, 3 Explanation: 1. The nurse should assess the cesarean incision. 3. The nurse should talk with the mother about her fatigue level and ability to rest and sleep. Page Ref: 929, 931

What are the primary complications of placenta accrete? Note: Credit will be given only if all correct choices and no incorrect choices are selected. Select all that apply. 1. Maternal hemorrhage 2. Insomnia 3. Failure of the placenta to separate following birth of the infant 4. Autonomic dysreflexia 5. Shoulder dystocia

Answer: 1, 3 Explanation: 1. The primary complications of placenta accreta are maternal hemorrhage and failure of the placenta to separate following birth of the infant. 3. The primary complications of placenta accreta are maternal hemorrhage and failure of the placenta to separate following birth of the infant.

Before performing Leopold maneuvers, what would the nurse do? Note: Credit will be given only if all correct choices and no incorrect choices are selected. Select all that apply. 1. Have the client empty her bladder. 2. Place the client in Trendelenburg position. 3. Have the client lie on her back with her feet on the bed and knees bent. 4. Turn the client to her left side. 5. Have the client lie flat with her ankles crossed.

Answer: 1, 3 Explanation: 1. The woman should have recently emptied her bladder before performing Leopold maneuvers. 3. The woman should lie on her back with her abdomen uncovered. To aid in relaxation of the abdominal wall, the shoulders should be raised slightly on a pillow and the knees drawn up a little.

The nurse is explaining clinical pelvimetry to a client. The nurse explains that the anteroposterior diameters consist of which of the following? Note: Credit will be given only if all correct and no incorrect choices are selected. Select all that apply. 1. Diagonal conjugate 2. Transverse diameter 3. Conjugata vera 4. Obstetric conjugate 5. Oblique diameter

Answer: 1, 3, 4 Explanation: 1. The diagonal conjugate is a part of the anteroposterior diameter measurement. 3. The conjugata vera is a part of the anteroposterior diameter measurement. 4. The obstetric conjugate is a part of the anteroposterior diameter measurement.

The nurse encourages a new mother to feed the newborn as soon as the newborn shows interest. The nurse bases this recommendation on which benefits of early feedings? Note: Credit will be given only if all correct choices and no incorrect choices are selected. Select all that apply. 1. Early feedings stimulate peristalsis. 2. Colostrum is thinner than mature milk. 3. Early feedings enhance maternal-infant bonding. 4. Early feedings promote the passage of meconium. 5. Colostrum contains a high number of calories.

Answer: 1, 3, 4 Explanation: 1. Early breastfeeding stimulates the newborn's peristalsis. 3. Early breastfeeding enhances maternal-infant bonding. 4. Early breastfeeding promotes the passage of meconium.

The nurse is instructing a pregnant client on her nutritional needs. The nurse tells the client that nutrition is needed for fetal development because fetal growth occurs in overlapping stages from increases of which of the following? Note: Credit will be given only if all correct and no incorrect choices are selected. Select all that apply. 1. Cell numbers 2. Cell membranes 3. Cell size and number 4. Cell size alone 5. Vitamin and mineral intake

Answer: 1, 3, 4 Explanation: 1. Fetal growth occurs by an increase in cell numbers in one stage. 3. Fetal growth occurs by an increase in cell size and number during one stage. 4. Fetal growth occurs by an increase in cell size alone during one stage.

A postpartum mother questions whether the environmental temperature should be warmer in the baby's room at home. The nurse responds that the environmental temperature should be warmer for the newborn. This response is based on which newborn characteristics that affect the establishment of thermal stability? Note: Credit will be given only if all correct choices and no incorrect choices are selected. Select all that apply. 1. Newborns have less subcutaneous fat than do adults. 2. Infants have a thick epidermis layer. 3. Newborns have a large body surface to weight ratio. 4. Infants have increased total body water. 5. Newborns have more subcutaneous fat than do adults.

Answer: 1, 3, 4 Explanation: 1. Heat transfer from neonatal organs to skin surface is increased compared to adults due to the neonate's decreased subcutaneous fat. 3. Heat transfer from neonatal organs to skin surface is increased compared to adults due to the neonate's large body surface to weight ratio. 4. Preterm infants have increased heat loss via evaporation due to increased total body water.

Which assessment findings would lead the nurse to suspect that a newborn might have a congenital heart defect? Note: Credit will be given only if all correct choices and no incorrect choices are selected. Select all that apply. 1. Cyanosis 2. Heart murmur 3. Bradycardia 4. Low urinary outputs 5. Tachypnea

Answer: 1, 3, 4, 5 Explanation: 1. Central cyanosis is defined as a visible, blue discoloration of the skin caused by decreased oxygen saturation levels and is a common manifestation of a cardiac defect. 3. The signs of congestive heart failure include tachycardia, not bradycardia. 4. The signs of congestive heart failure include low urinary output. 5. The signs of congestive heart failure include tachypnea.

Which statements might a pregnant teenager be likely to make at her initial prenatal examination? Note: Credit will be given only if all correct choices and no incorrect choices are selected. Select all that apply. 1. "I didn't know I could get pregnant the first time I had sex." 2. "Several of my friends go to clinics to get contraception." 3. "It's no big deal; two of my best friends have babies, too." 4. "I was 13 years old when I had sex the first time." 5. "My family and my boyfriend are really happy and supportive."

Answer: 1, 3, 4 Explanation: 1. Many factors contribute to the high teenage pregnancy rate, and one of them is lack of knowledge about conception. 3. Many factors contribute to the high teenage pregnancy rate, and one of them is a decreased social stigma of being a young and single mother. 4. Many factors contribute to the high teenage pregnancy rate, and one of them is a younger age at onset of sexual activity.

Postpartum nutritional status is determined primarily by assessing which of the following? Note: Credit will be given only if all correct choices and no incorrect choices are selected. Select all that apply. 1. Dietary history 2. Menstrual history 3. Mother's weight 4. Hemoglobin levels 5. Mother's height

Answer: 1, 3, 4 Explanation: 1. Postpartum nutritional status is determined by assessing the new mother's dietary history. 3. Postpartum nutritional status is determined by assessing the new mother's weight. 4. Postpartum nutritional status is determined by assessing the new mother's hemoglobin levels.

The nurse in the OB-GYN clinic is working with a client who is seeking her initial prenatal visit. The nurse will use the acronym TPAL to document the client's number of which of the following? Note: Credit will be given only if all correct and no incorrect choices are selected. Select all that apply. 1. Term infants born 2. Children living in the home 3. Pregnancies ending in abortion 4. Preterm infants born 5. Pregnancies that occurred

Answer: 1, 3, 4 Explanation: 1. T: number of term births the woman has experienced 3. A: number of pregnancies ending in either spontaneous or therapeutic abortion 4. P: number of preterm births

During a client's initial prenatal visit, the nurse must assess and document the client's current medical history, including which information? Note: Credit will be given only if all correct and no incorrect choices are selected. Select all that apply. 1. Body mass index 2. Infections before the last menstrual period 3. Homeopathic or herbal medication use 4. Blood type and Rh factor 5. History of previous pregnancies

Answer: 1, 3, 4 Explanation: 1. The body mass index is an important part of the current medical history to be assessed and documented. 3. Homeopathic and herbal medication use is important for the nurse to assess and document in the current medical history. 4. The blood type must be assessed and documented in the current medical history, as must the Rh factor.

The nurse is assessing a client in the third trimester of pregnancy. What physiologic changes in the client are expected? Note: Credit will be given only if all correct choices and no incorrect choices are selected. Select all that apply. 1. The client's chest circumference has increased by 6 cm during the pregnancy. 2. The client has a narrowed subcostal angle. 3. The client is using thoracic breathing. 4. The client may have epistaxis. 5. The client has a productive cough.

Answer: 1, 3, 4 Explanation: 1. The chest increase compensates for the elevated diaphragm. 3. Breathing changes from abdominal to thoracic as pregnancy progresses. 4. Epistaxis (nosebleeds) may occur and are primarily the result of estrogen-induced edema and vascular congestion of the nasal mucosa.

The nurse in the OB-GYN clinic counsels a couple that in autosomal dominant inheritance, which of the following occur? Note: Credit will be given only if all correct and no incorrect choices are selected. Select all that apply. 1. An affected individual might have an affected parent. 2. The affected individual has a 75% chance of passing on the abnormality. 3. Males and females are equally affected by the gene. 4. A father can pass the defective gene to a son. 5. There are no variances in the genetic pattern for autosomal dominant disorders.

Answer: 1, 3, 4 Explanation: 1. The family pedigree usually shows multiple generations having the disorder. 3. Males and females are equally affected in autosomal dominant disorders. 4. In autosomal dominant disorders, the father can pass the defective gene to a son.

The nurse initiates newborn admission procedures and evaluates the newborn's need to remain under observation by assessing which of the following? Note: Credit will be given only if all correct choices and no incorrect choices are selected. Select all that apply. 1. Respiratory rate 2. Skin texture 3. Airway clearance 4. Ability to feed 5. Head weight

Answer: 1, 3, 4 Explanation: 1. The nurse initiates newborn admission procedures and evaluates the newborn's need to remain under observation by assessing vital signs (body temperature, heart rate, respiratory rate). 3. The nurse initiates newborn admission procedures and evaluates the newborn's need to remain under observation by assessing airway clearance. 4. The nurse initiates newborn admission procedures and evaluates the newborn's need to remain under observation by assessing ability to feed.

The nurse is teaching the parents of a newborn who has been exposed to HIV how to care for the newborn at home. Which instructions should the nurse emphasize? Note: Credit will be given only if all correct choices and no incorrect choices are selected. Select all that apply. 1. Use proper hand-washing technique. 2. Provide three feedings per day. 3. Place soiled diapers in a sealed plastic bag. 4. Cleanse the diaper changing area with a 1:10 bleach solution after each diaper change. 5. Take the temperature rectally.

Answer: 1, 3, 4 Explanation: 1. The nurse should instruct the parents on proper hand-washing technique. 3. The nurse should instruct parents to that soiled diapers are to be placed in plastic bags, sealed, and disposed of daily. 4. The nurse should instruct parents that the diaper-changing areas should be cleaned with a 1:10 dilution of household bleach after each diaper change.

A nurse is instructing nursing students about the procedure for vitamin K administration. What information should be included? Note: Credit will be given only if all correct choices and no incorrect choices are selected. Select all that apply. 1. Gently massage the site after injection. 2. Use a 22-gauge, 1-inch needle. 3. Inject in the vastus lateralis muscle. 4. Cleanse the site with alcohol prior to injection. 5. Inject at a 45-degree angle.

Answer: 1, 3, 4 Explanation: 1. The nurse would remove the needle and massage the site with an alcohol swab. 3. Vitamin K is given intramuscularly in the vastus lateralis muscle. 4. Before injecting, the nurse must clean the newborn's skin site for the injection thoroughly with a small alcohol swab.

The nurse is preparing to give an injection of vitamin K to a newborn. Which considerations would be appropriate? Note: Credit will be given only if all correct choices and no incorrect choices are selected. Select all that apply. 1. Administer a dose of 0.5 to 1 mg within 1 hour of birth. 2. Administer the injection subcutaneously. 3. Use a 25-gauge, 5/8-inch needle for the injection. 4. Protect the medication bottle from light. 5. Give vitamin K prior to a circumcision procedure.

Answer: 1, 3, 4, 5 Explanation: 1. 0.5 to 1 mg is the correct dosage for vitamin K. 3. 25-gauge, 5/8-inch needle is the right size needle to use. 4. Vitamin K must be kept away from light. 5. A prophylactic injection of vitamin K1 is given to prevent hemorrhage, which can occur because of low prothrombin levels in the first few days of life.

The nurse is planning care for a client with hydramnios. For which interventions might the nurse need to prepare the client? Note: Credit will be given only if all correct choices and no incorrect choices are selected. Select all that apply. 1. Artificial rupture of the membranes 2. Amnioinfusion 3. Amniocentesis 4. Administration of prostaglandin synthesis inhibitor 5. Administration of indomethacin

Answer: 1, 3, 4, 5 Explanation: 1. Artificial rupture may be performed to remove the excessive fluid. 3. Amniocentesis may be performed to remove some excess fluid. 4. A prostaglandin synthesis inhibitor is used to treat hydramnios. 5. Indomethacin can decrease amniotic fluid by decreasing fetal urine output.

The nurse has admitted a woman with cervical insufficiency. The nurse is aware that causes of this condition include which of the following? Note: Credit will be given only if all correct choices and no incorrect choices are selected. Select all that apply. 1. Congenital factors 2. Intercourse during pregnancy 3. Infection 4. Increased uterine volume 5. Past cervical surgeries

Answer: 1, 3, 4, 5 Explanation: 1. Congenitally incompetent cervix may be found in women exposed to diethylstilbestrol (DES) or those with a bicornuate uterus. 3. Infection or trauma can cause acquired cervical incompetence. 4. Cervical insufficiency can occur in multiple-gestation pregnancies. 5. Previous elective abortion or cervical manipulation can lead to cervical insufficiency.

The client in the first trimester of pregnancy questions the nurse about the causes of fetal death. The nurse explains that factors associated with perinatal loss include which of the following? Note: Credit will be given only if all correct choices and no incorrect choices are selected. Select all that apply. 1. Maternal diabetes 2. Paternal hypertension 3. Fetal chromosomal disorders 4. Maternal infections 5. Placental abnormalities

Answer: 1, 3, 4, 5 Explanation: 1. Fetal loss can be a result of a number of physiologic maladaptations, including maternal diabetes. 3. Chromosomal abnormalities can be associated with fetal loss. 4. Infections such as human parvovirus B19, syphilis, streptococcal infection, and Listeria can lead to fetal loss. 5. Placental abnormalities such as abruptio placentae and placenta previa can result in fetal death.

Which of the following are important behaviors to assess in the neurologic assessment? Note: Credit will be given only if all correct choices and no incorrect choices are selected. Select all that apply. 1. State of alertness 2. Active posture 3. Quality of muscle tone 4. Cry 5. Motor activity

Answer: 1, 3, 4, 5 Explanation: 1. Important behaviors to assess are the state of alertness, resting posture, cry, and quality of muscle tone and motor activity. 3. Important behaviors to assess are the state of alertness, resting posture, cry, and quality of muscle tone and motor activity. 4. Important behaviors to assess are the state of alertness, resting posture, cry, and quality of muscle tone and motor activity. 5. Important behaviors to assess are the state of alertness, resting posture, cry, and quality of muscle tone and motor activity.

) The nurse is caring for a prenatal client. Reviewing the client's pregnancy history, the nurse identifies risk factors for an at-risk newborn, including which of the following? Note: Credit will be given only if all correct choices and no incorrect choices are selected. Select all that apply. 1. The mother's low socioeconomic status 2. Maternal age of 26 3. Mother's exposure to toxic chemicals 4. More than three previous deliveries 5. Maternal hypertension

Answer: 1, 3, 4, 5 Explanation: 1. Low socioeconomic status is associated with at-risk newborns. 3. Exposure to environmental dangers, such as toxic chemicals is associated with at-risk newborns. 4. Maternal factors such as multiparity are associated with at-risk newborns. 5. Preexisting maternal conditions, such as heart disease, diabetes, hypertension, hyperthyroidism, and renal disease are associated with at-risk newborns.

The nurse is providing discharge teaching to the parents of a newborn. The nurse should instruct the parents to notify the healthcare provider in case of which of the following? Note: Credit will be given only if all correct choices and no incorrect choices are selected. Select all that apply. 1. More than one episode of forceful vomiting. 2. More than 6 to 10 wet diapers per day. 3. A bluish discoloration of the skin with or without a feeding. 4. Refusal of two feedings in a row. 5. Development of eye drainage.

Answer: 1, 3, 4, 5 Explanation: 1. More than one episode of forceful vomiting or frequent vomiting over a 6-hour period should be reported to the healthcare provider. 3. Cyanosis (bluish discoloration of skin) with or without a feeding is a cause for concern, and should be reported to the healthcare provider immediately. 4. Refusal of two feedings in a row should be reported to the healthcare provider. 5. The infant should not have eye drainage after discharge and this condition should be reported to the healthcare provider.

Which fetal/neonatal risk factors would lead the nurse to anticipate a potential need to resuscitate a newborn? Note: Credit will be given only if all correct choices and no incorrect choices are selected. Select all that apply. 1. Nonreassuring fetal heart rate pattern/sustained bradycardia 2. Fetal scalp/capillary blood sample pH greater than 7.25 3. History of meconium in amniotic fluid 4. Prematurity 5. Significant intrapartum bleeding

Answer: 1, 3, 4, 5 Explanation: 1. Nonreassuring fetal heart rate pattern/sustained bradycardia would be considered a potential need to resuscitate a newborn. 3. History of meconium in amniotic fluid would be considered a potential need to resuscitate a newborn. 4. Prematurity would be considered a potential need to resuscitate a newborn. 5. Significant intrapartum bleeding would be considered a potential need to resuscitate a newborn.

) The incidence of complications and discomforts in the first year postpartum is common and women may experience which of the following? Note: Credit will be given only if all correct choices and no incorrect choices are selected. Select all that apply. 1. Pain 2. Excess energy 3. Urinary incontinence 4. Changes in mental health status 5. Sleep deprivation

Answer: 1, 3, 4, 5 Explanation: 1. Pain can be a discomfort in the first year postpartum. 3. Urinary incontinence can be a complication in the first year postpartum. 4. Changes in mental health status can be a complication in the first year postpartum. 5. Sleep deprivation can be a complication in the first year postpartum.

The nurse should inform the parents of a newborn that they should call their healthcare provider when which of the following occurs? Note: Credit will be given only if all correct choices and no incorrect choices are selected. Select all that apply. 1. Continual rise in temperature 2. Decreased frequency of stools 3. Absence of breathing longer than 20 seconds 4. Lethargy 5. Refusal of two feedings in a row

Answer: 1, 3, 4, 5 Explanation: 1. Parents should call their healthcare provider due to a continual rise in temperature. 3. Parents should call their healthcare provider in the absence of breathing longer than 20 seconds. 4. Parents should call their healthcare provider if the newborn exhibits lethargy and listlessness. 5. Parents should call their healthcare provider if the newborn has refused of two feedings in a row.

What would the nurse do to accurately assess a pregnant client's food intake? 1. Assess her most recent laboratory values. 2. Ask her to complete a nutritional questionnaire. 3. Observe for signs of hunger. 4. Ask about her cooking facilities.

Answer: 2 Explanation: 2. Diet may be evaluated using a food frequency questionnaire, which lists common categories of foods and asks the woman how frequently in a day (or week) she consumes foods from the list.

When general anesthesia is necessary for a cesarean delivery, what should the nurse be prepared to do? Note: Credit will be given only if all correct choices and no incorrect choices are selected. Select all that apply. 1. Administer an antacid to the client. 2. Place a wedge under her thigh. 3. Apply cricoid pressure during anesthesia intubation. 4. Preoxygenate for 3-5 minutes before anesthesia. 5. Place a Foley catheter in the client's bladder.

Answer: 1, 3, 4, 5 Explanation: 1. Prophylactic antacid therapy is given to reduce the acidic content of the stomach before general anesthesia. 3. During the process of rapid induction of anesthesia, the nurse applies cricoid pressure. 4. The woman should be preoxygenated with 3 to 5 minutes of 100% oxygen. 5. Urinary retention can be treated with the placement of an indwelling Foley catheter.

A new mother inquires about postpartum resources. What resources can the nurse suggest to provide assistance to the new mother? Note: Credit will be given only if all correct choices and no incorrect choices are selected. Select all that apply. 1. Breastfeeding support groups 2. Meals on Wheels 3. Lactation consultants 4. Postpartum classes 5. Internet sites

Answer: 1, 3, 4, 5 Explanation: 1. Support groups provide an opportunity for parents to interact with one another and share information. 3. Lactation consultants are helpful for women who are having breastfeeding problems or concerns. 4. Postpartum classes offer chances for the new mother to socialize, share concerns, and receive encouragement. 5. The nurse's role is to direct the new mother to reliable web sites.

Usually, the family is advised to arrive at the birth setting at the beginning of the active phase of labor or when which of the following occur? Note: Credit will be given only if all correct and no incorrect choices are selected. Select all that apply. 1. Rupture of membranes (ROM) 2. Increased fetal movement 3. Decreased fetal movement 4. Any vaginal bleeding 5. Regular, frequent uterine contractions (UCs)

Answer: 1, 3, 4, 5 Explanation: 1. The family is advised to arrive at the birth setting at the beginning of the active phase of labor or when the membranes rupture. 3. The family is advised to arrive at the birth setting at the beginning of the active phase of labor or when there is decreased fetal movement. 4. The family is advised to arrive at the birth setting at the beginning of the active phase of labor or when there is any vaginal bleeding. 5. The family is advised to arrive at the birth setting at the beginning of the active phase of labor or when there are regular, frequent uterine contractions.

The nurse is caring for a client who had a cesarean birth 4 hours ago. Which interventions would the nurse implement at this time? Note: Credit will be given only if all correct and no incorrect choices are selected. Select all that apply. 1. Administer analgesics as needed. 2. Encourage the client to ambulate to the bathroom to void. 3. Encourage leg exercises every 2 hours. 4. Encourage the client to cough and deep-breathe every 2 to 4 hours. 5. Encourage the use of breathing, relaxation, and distraction.

Answer: 1, 3, 4, 5 Explanation: 1. The nurse continues to assess the woman's pain level and provide relief measures as needed. 3. Within the first 12 hours postoperatively, unless medically contraindicated, the woman should be assisted to dangle her legs on the side of the bed. 4. The woman is encouraged to cough and breathe deeply and to use incentive spirometry every 2 to 4 hours while awake for the first few days following cesarean birth. 5. The nurse should encourage the use of breathing, relaxation, and distraction techniques.

A nurse is performing a postpartum assessment during the first home visit to a client 3 days postdelivery. Which actions will the nurse perform? Note: Credit will be given only if all correct choices and no incorrect choices are selected. Select all that apply. 1. Palpate the breasts. 2. Auscultate the carotid. 3. Check vaginal discharge. 4. Assess the extremities. 5. Inspect the perineum.

Answer: 1, 3, 4, 5 Explanation: 1. The nurse needs to assess for fullness and engorgement. 3. The nurse should check vaginal discharge for amount and color. 4. The extremities should be checked for any redness, edema, and tenderness. 5. The perineum needs to be checked for healing.

Antibiotics have been ordered for a newborn with an infection. Which interventions would the nurse prepare to implement? Note: Credit will be given only if all correct choices and no incorrect choices are selected. Select all that apply. 1. Obtain skin cultures. 2. Restrict parental visits. 3. Evaluate bilirubin levels. 4. Administer oxygen as ordered. 5. Observe for signs of hypoglycemia.

Answer: 1, 3, 4, 5 Explanation: 1. The nurse will assist in obtaining skin cultures. Skin cultures are taken of any lesions or drainage from lesions or reddened areas. 3. The nurse will observe for hyperbilirubinemia, anemia, and hemorrhagic symptoms. 4. The nurse will administer oxygen as ordered. 5. The nurse will observe for signs of hypoglycemia.

When planning care for the premature newborn diagnosed with respiratory distress syndrome, which potential complications would the nurse anticipate? Note: Credit will be given only if all correct choices and no incorrect choices are selected. Select all that apply. 1. Hypoxia 2. Respiratory alkalosis 3. Metabolic acidosis 4. Massive atelectasis 5. Pulmonary edema

Answer: 1, 3, 4, 5 Explanation: 1. The physiologic alterations of RDS can produce hypoxia as a complication. As a result of hypoxia, the pulmonary vasculature constricts, pulmonary vascular resistance increases, and pulmonary blood flow is reduced. 3. The physiologic alterations of RDS can produce metabolic acidosis as a complication. Because cells lack oxygen, the newborn begins an anaerobic pathway of metabolism, with an increase in lactate levels and a resulting base deficit. 4. The physiologic alterations of RDS can produce massive atelectasis as a complication. Upon expiration, the instability increases the atelectasis, which causes hypoxia and acidosis because of the lack of gas exchange. 5. The physiologic alterations of RDS can produce pulmonary edema as a complication. Opacification of the lungs on X-ray image may be due to massive atelectasis, diffuse alveolar infiltrate, or pulmonary edema.

A first-time 22-year-old single labor client, accompanied by her boyfriend, is admitted to the labor unit with ruptured membranes and mild to moderate contractions. She is determined to be 2 centimeters dilated. Which nursing diagnoses might apply during the current stage of labor? Note: Credit will be given only if all correct choices and no incorrect choices are selected. Select all that apply. 1. Fear/Anxiety related to discomfort of labor and unknown labor outcome 2. Knowledge, Deficient, related to lack of information about pushing methods 3. Pain, Acute, related to uterine contractions, cervical dilatation, and fetal descent 4. Pain, Acute, related to perineal trauma 5. Coping: Family, Compromised, related to labor process

Answer: 1, 3, 5 Explanation: 1. A Fear/Anxiety diagnosis would apply to the first stage of labor for a first-time labor client. 3. Contractions become more regular in frequency and duration, increasing discomfort and pain. 5. The woman and her boyfriend are about to undergo one of the most meaningful and stressful events in life together. Physical and psychologic resources, coping mechanisms, and support systems will all be challenged.

The nurse understands that a client's pregnancy is progressing normally when what physiologic changes are documented on the prenatal record of a woman at 36 weeks' gestation? Note: Credit will be given only if all correct choices and no incorrect choices are selected. Select all that apply. 1. The joints of the pelvis have relaxed, causing a waddling gait. 2. The cervix is firm and blue-purple in color. 3. The uterus vasculature contains one sixth of the total maternal blood volume. 4. Gastric emptying time is delayed, and the client complains of constipation and bloating. 5. Supine hypotension occurs when the client lies on her back.

Answer: 1, 3, 4, 5 Explanation: 1. The sacroiliac, sacrococcygeal, and pubic joints of the pelvis relax in the later part of the pregnancy, presumably as a result of hormonal changes. This often causes a waddling gait. 3. By the end of pregnancy, one sixth of the total maternal blood volume is contained within the vascular system of the uterus. 4. Gastric emptying time and intestinal motility are delayed, leading to frequent complaints of bloating and constipation, which can be aggravated by the smooth muscle relaxation and increased electrolyte and water reabsorption in the large intestine. 5. The enlarging uterus may exert pressure on the vena cava when the woman lies supine, causing a drop in blood pressure. This is called the vena caval syndrome, or supine hypotension.

The nurse is caring for an infant of a diabetic mother. Which potential complications would the nurse consider in planning care for this newborn? Note: Credit will be given if all correct choices and no incorrect choices are selected. Select all that apply. 1. Tremors 2. Hyperglycemia 3. Hyperbilirubinemia 4. Respiratory distress syndrome 5. Birth trauma

Answer: 1, 3, 4, 5 Explanation: 1. Tremors are a clinical sign of hypocalcemia. 3. Hyperbilirubinemia is caused by slightly decreased extracellular fluid volume, which increases the hematocrit level. 4. Respiratory distress syndrome (RDS) is a complication that occurs more frequently in newborns of diabetic mothers whose diabetes is not well controlled. 5. Because most IDMs are macrosomic, trauma may occur during labor and vaginal birth resulting in shoulder dystocia, brachial plexus injuries, subdural hemorrhage, cephalohematoma, and asphyxia.

Clinical features of posttraumatic stress disorder (PTSD) include which of the following? 1. Difficulty sleeping 2. Acute awareness 3. Flashbacks 4. The need to be constantly around others 5. Irritability

Answer: 1, 3, 5 Explanation: 1. A clinical feature of PTSD is difficulty thinking. 3. A clinical feature of PTSD is intrusive thoughts and flashbacks to the threatening event. 5. A clinical feature of PTSD is irritability.

The nurse is caring for a client who finally conceived after several unsuccessful attempts at in vitro fertilization. The client has just been diagnosed with a perinatal loss. What should the nurse's plan of care include? Note: Credit will be given only if all correct choices and no incorrect choices are selected. Select all that apply. 1. Giving accurate and honest information 2. Encouraging the couple to try right away to get pregnant again 3. Validating the many losses the client has experienced 4. Providing possible explanations for the fetal demise 5. Assessing where the client is in the grieving process, and communicating with compassion

Answer: 1, 3, 5 Explanation: 1. Families can cope with extreme situations when they are properly informed in an honest and forthright manner. 3. The nurse should be compassionate, give accurate and honest information, and validate the many losses incurred. 5. The nurse caring for a couple who has had a previous loss needs to be kind, compassionate, and patient.

A new mother is concerned because the anterior fontanelle swells when the newborn cries. Explaining normal findings concerning the fontanelles, the nurse states which of the following? Note: Credit will be given only if all correct choices and no incorrect choices are selected. Select all that apply. 1. The fontanelles can swell with crying. 2. The fontanelles might be depressed. 3. The fontanelles can pulsate with the heartbeat. 4. The fontanelles might bulge. 5. The fontanelles can swell when stool is passed.

Answer: 1, 3, 5 Explanation: 1. Newborn fontanelles can swell when the newborn cries. 3. Newborn fontanelles can pulsate with the heartbeat. 5. Newborn fontanelles can swell when the newborn passes a stool.

The nurse is caring for the newborn of a drug-addicted mother. Which assessment findings would be typical for this newborn? Note: Credit will be given only if all correct choices and no incorrect choices are selected. Select all that apply. 1. Hyperirritability 2. Decreased muscle tone 3. Exaggerated reflexes 4. Low pitched cry 5. Transient tachypnea

Answer: 1, 3, 5 Explanation: 1. Newborns born to drug-addicted mothers exhibit hyperirritability. 3. Newborns born to drug-addicted mothers exhibit exaggerated reflexes. 5. Newborns born to drug-addicted mothers exhibit transient tachypnea.

A client at 34 weeks' gestation complains about pyrosis. The nurse teaches the patient that approaches to relieve the pyrosis include which of the following? Note: Credit will be given only if all correct and no incorrect choices are selected. Select all that apply. 1. Eat small, frequent meals 2. Use high-sodium antacids 3. Avoid fried, fatty foods 4. Take sodium bicarbonate after meals 5. Do not lie down after eating

Answer: 1, 3, 5 Explanation: 1. Pyrosis (heartburn) can be relieved by eating small, more frequent meals. 3. Avoiding fatty, fried foods can relieve pyrosis. 5. Sitting up after meals will help decrease the pyrosis.

Women with eating disorders who become pregnant are at risk for a variety of complications including which of the following? Note: Credit will be given only if all correct choices and no incorrect choices are selected. Select all that apply. 1. Premature birth 2. Too many nutrients available for the fetus 3. Miscarriage 4. High birth weight 5. Perinatal mortality

Answer: 1, 3, 5 Explanation: 1. Risks to the mother and baby include premature birth. 3. Risks to the mother and baby include miscarriage. 5. Risks to the mother and baby include perinatal mortality.

A new mother is concerned about a mass on the newborn's head. The nurse assesses this to be a cephalohematoma based on which characteristics? Note: Credit will be given only if all correct choices and no incorrect choices are selected. Select all that apply. 1. The mass appeared on the second day after birth. 2. The mass appears larger when the newborn cries. 3. The head appears asymmetrical. 4. The mass appears on only one side of the head. 5. The mass overrides the suture line.

Answer: 1, 4 Explanation: 1. A cephalohematoma is a collection of blood resulting from ruptured blood vessels between the surface of a cranial bone and the periosteal membrane. These areas emerge as defined hematomas between the first and second days. 4. Cephalohematomas can be unilateral or bilateral, but do not cross the suture lines.

Which nursing interventions would be included in the plan of care for a family that has just been informed of a perinatal loss? Note: Credit will be given only if all correct choices and no incorrect choices are selected. Select all that apply. 1. Provide the parents with a private place and time to express their grief. 2. Offer reassurance that parents can have a subsequent successful pregnancy. 3. Allow the parents to participate in personal grief rituals. 4. Encourage interaction with other families. 5. Offer to give the family mementos of the infant such as footprints, crib card, and lock of hair.

Answer: 1, 3, 5 Explanation: 1. The couple with a known or suspected fetal demise should immediately be placed in a private room. When possible, the woman should be in a room that is farthest away from other laboring women. 3. The nurse should assist the couple in exploring their feelings and help them to make decisions about who will be present and what rituals will occur during and following the birth. 5. In a fetal demise, mementos are some of the few memories the parents have to provide them comfort after the death of their baby. Every effort should be made to offer as many quality mementos as possible, such as pictures and hand- or footprint molds and cards.

The newborn's cry should have which of the following characteristics? Note: Credit will be given only if all correct choices and no incorrect choices are selected. Select all that apply. 1. Medium pitch 2. Shrillness 3. Strength 4. High pitch 5. Lusty

Answer: 1, 3, 5 Explanation: 1. The newborn's cry should be strong, lusty, and of medium pitch. 3. The newborn's cry should be strong, lusty, and of medium pitch. 5. The newborn's cry should be strong, lusty, and of medium pitch.

The clinic nurse is assessing how the prenatal client is meeting developmental tasks using Rubin's tasks, including which of the following? Note: Credit will be given only if all correct and no incorrect choices are selected. Select all that apply. 1. Ensuring safe passage through pregnancy, labor, and birth. 2. Turning in on oneself to focus on the child. 3. Seeking commitment and acceptance of self as mother to the infant. 4. Completing the tasks of nesting at the appropriate time. 5. Seeking acceptance of the child by others.

Answer: 1, 3, 5 Explanation: 1. The tasks Rubin identified form the basis for a mutually gratifying relationship with the baby, and include ensuring safe passage through pregnancy, labor, and birth. 3. The tasks Rubin identified form the basis for a mutually gratifying relationship with the baby, and include seeking commitment and acceptance of self as mother. 5. The tasks Rubin identified form the basis for a mutually gratifying relationship with the baby, and include seeking acceptance of the child by others.

A client at 18 weeks' gestation has been diagnosed with a hydatidiform mole. In addition to vaginal bleeding, which signs or symptoms would the nurse expect to see? Note: Credit will be given only if all correct choices and no incorrect choices are selected. Select all that apply. 1. Hyperemesis gravidarum 2. Diarrhea and hyperthermia 3. Uterine enlargement greater than expected 4. Polydipsia 5. Vaginal bleeding

Answer: 1, 3, 5 Explanation: 1. This is often seen in clients with hydatidiform mole. 3. This is a classic sign of hydatidiform mole. 5. This is a classic symptom of hydatidiform mole.

The nurse is admitting a client who was diagnosed with hydramnios. The client asks why she has developed this condition. The nurse should explain that hydramnios is sometimes associated with which of the following? Note: Credit will be given only if all correct choices and no incorrect choices are selected. Select all that apply. 1. Rh sensitization 2. Postmaturity syndrome 3. Renal malformation or dysfunction 4. Maternal diabetes 5. Large-for-gestational-age infants

Answer: 1, 4 Explanation: 1. Hydramnios is associated with Rh sensitization. 4. Hydramnios is associated with maternal diabetes.

A clinic nurse is preparing diagrams of pelvic shapes. Which pelvic shapes are considered least adequate for vaginal childbirth? Note: Credit will be given only for all correct choices and no incorrect choices. Select all that apply. 1. Android 2. Anthropoid 3. Gynecoid 4. Platypelloid 5. Lambdoidal suture

Answer: 1, 4 Explanation: 1. In the android and platypelloid types, the pelvic diameters are diminished. Labor is more likely to be difficult (longer) and a cesarean birth is more likely. 4. In the android and platypelloid types, the pelvic diameters are diminished. Labor is more likely to be difficult (longer) and a cesarean birth is more likely.

The nurse is teaching a prenatal class about false labor. The nurse should teach clients that false labor most likely will include which of the following? Note: Credit will be given only for all correct choices and no incorrect choices. Select all that apply. 1. Contractions that do not intensify while walking 2. An increase in the intensity and frequency of contractions 3. Progressive cervical effacement and dilatation 4. Pain in the abdomen that does not radiate 5. Contractions are at regular intervals

Answer: 1, 4 Explanation: 1. True labor contractions intensify while walking. 4. The discomfort of true labor contractions usually starts in the back and radiates around to the abdomen.

) The nurse is caring for a postpartum client who is at risk for developing early postpartum hemorrhage. What interventions would be included in the plan of care to detect this complication? Note: Credit will be given only if all correct choices and no incorrect choices are selected. Select all that apply. 1. Weigh perineal pads if the client has a slow, steady, free flow of blood from the vagina. 2. Massage the uterus every 2 hours. 3. Maintain vascular access. 4. Obtain blood specimens for hemoglobin and hematocrit. 5. Encourage the client to void if the fundus is displaced upward or to one side.

Answer: 1, 4 Explanation: 1. Weighing the perineal pads will indicate whether the client is bleeding more than anticipated. 4. The nurse reviews hemoglobin and hematocrit levels when available, and compares them to the admission baseline.

To answer a client's question about home pregnancy tests and their accuracy, the nurse must know that accuracy is affected by which of the following? Note: Credit will be given only if all correct and no incorrect choices are selected. Select all that apply. 1. Unclear directions 2. Unable to comprehend the directions 3. Blood in the specimen giving a false reading 4. Completing the test too late 5. Tagged antibodies becoming outdated

Answer: 1, 4 Explanation: 1. Women may not comprehend the HPT instructions, which can affect the accuracy results. 4. False-negative results typically occur when the test is completed too early or too late.

The nurse is assessing a client who has been diagnosed with an early postpartum hemorrhage. Which findings would the nurse expect? Note: Credit will be given only if all correct choices and no incorrect choices are selected. Select all that apply. 1. A boggy fundus that does not respond to massage 2. Small clots and a moderate amount of lochia rubra on the pad 3. Decreased pulse and increased blood pressure 4. Hematoma formation or bulging/shiny skin in the perineal area 5. Rise in the level of the fundus of the uterus

Answer: 1, 4, 5 Explanation: 1. A boggy fundus indicates that the uterus is not contracted and will continue to bleed. 4. Shiny or bulging skin could indicate the presence of a hematoma. 5. The uterine cavity can distend with up to 1000 mL or more of blood causing the fundus to rise.

The nurse is evaluating the expected outcomes of care for a pregnant adolescent. For the evaluation, the nurse will ask which of the following questions? Note: Credit will be given only if all correct and no incorrect choices are selected. Select all that apply. 1. Was a trusting relationship developed? 2. Did the adolescent explore financial options open to teen parents? 3. Did the adolescent make plans for continuing her education? 4. Can the adolescent problem-solve? 5. Did the adolescent follow recommendations?

Answer: 1, 4, 5 Explanation: 1. A trusting relationship must be developed with the adolescent client. 4. The adolescent must be able to appropriately problem-solve. 5. The adolescent must follow the recommendations of the healthcare team to deliver a healthy baby.

The nurse is caring for a postpartum client who is experiencing afterpains following the birth of her third child. Which comfort measure should the nurse implement to decrease her pain? Note: Credit will be given only if all correct choices and no incorrect choices are selected. Select all that apply. 1. Offer a warm water bottle for her abdomen. 2. Call the physician to report this finding. 3. Inform her that this is not normal, and she will need an oxytocic agent. 4. Administer a mild analgesic to help with breastfeeding. 5. Administer a mild analgesic at bedtime to ensure rest.

Answer: 1, 4, 5 Explanation: 1. A warm water bottle placed against the low abdomen may reduce the discomfort of afterpains. 4. The breastfeeding mother may find it helpful to take a mild analgesic agent approximately 1 hour before feeding her infant. 5. An analgesic agent such as ibuprofen is also helpful at bedtime if the afterpains interfere with the mother's rest.

A client at 32 weeks' gestation is admitted with painless vaginal bleeding. Placenta previa has been confirmed by ultrasound. What should be included in the nursing plan? Note: Credit will be given only if all correct choices and no incorrect choices are selected. Select all that apply. 1. No vaginal exams 2. Encouraging activity 3. No intravenous access until labor begins 4. Evaluating fetal heart rate with an external monitor 5. Monitoring blood loss, pain, and uterine contractility

Answer: 1, 4, 5 Explanation: 1. Expectant management of placenta previa is made by localizing the placenta via tests that require no vaginal examination. 4. Expectant management of placenta previa, when the client is at less than 37 weeks' gestation, includes evaluating FHR with an external monitor. 5. Expectant management of placenta previa, when the client is at less than 37 weeks' gestation, includes monitoring blood loss, pain, and uterine contractility.

Which factors would the nurse observe that would indicate a new mother's early attachment to the newborn? Note: Credit will be given only if all correct choices and no incorrect choices are selected. Select all that apply. 1. Face-to-face contact and eye contact 2. Failure to choose a name for the baby 3. Decreased interest in the infant's cues 4. Pointing out familial traits of the newborn 5. Displaying satisfaction with the infant's sex

Answer: 1, 4, 5 Explanation: 1. Face-to-face contact and eye contact indicates that the mother is attracted to the infant and is attending to the infant's behavior. 4. The ability to point out family traits shows that she is pleased with the baby's appearance and recognizes the infant as belonging to the family unit. 5. Showing pleasure with the infant's appearance and sex indicates bonding is occurring.

The parents are asking the nurse about their newborn's behavior. The nurse begins to teach the parents about their newborn and involve them in their baby's care. What are these interventions directed at promoting to the parents? Note: Credit will be given only if all correct and no incorrect choices are selected. Select all that apply. 1. Identification of responses or activities that best meet the special needs of their newborn. 2. Ability to evaluate the neurologic capacity of their newborn. 3. Understanding that the baby's temperament will be the same as their own. 4. Positive attachment experiences. 5. Understanding of the newborn's various behaviors.

Answer: 1, 4, 5 Explanation: 1. Families learn which responses, interventions, or activities best meet the special needs of their newborn, and this understanding fosters positive attachment experiences. 4. Families learn which responses, interventions, or activities best meet the special needs of their newborn, and this understanding fosters positive attachment experiences. 5. Parents usually need help in understanding the behaviors of their baby.

The nurse knows that the Bishop scoring system for cervical readiness includes which of the following? Note: Credit will be given only if all correct choices and no incorrect choices are selected. Select all that apply. 1. Fetal station 2. Fetal lie 3. Fetal presenting part 4. Cervical effacement 5. Cervical softness

Answer: 1, 4, 5 Explanation: 1. Fetal station is one of the components evaluated by the Bishop scoring system. 4. Cervical effacement is one of the components evaluated by the Bishop scoring system. 5. Cervical consistency is one of the components evaluated by the Bishop scoring system.

The nurse's goals for prenatal classes may include which of the following? Note: Credit will be given only if all correct and no incorrect choices are selected. Select all that apply. 1. Increasing self-esteem 2. Preparing the participants for childrearing 3. Offering information on the risks of breastfeeding 4. Providing anticipatory guidance about pregnancy 5. Helping participants develop more adaptive coping skills

Answer: 1, 4, 5 Explanation: 1. Goals for prenatal classes would include increasing self-esteem. 4. Goals for prenatal class would include providing anticipatory guidance about pregnancy. 5. Goals for prenatal class would include helping participants develop more adaptive coping skills.

Slowly removing some amniotic fluid is a treatment for hydramnios. What consequence can occur with the withdrawal of fluid? 1. Preterm labor 2. Prolapsed cord 3. Preeclampsia 4. Placenta previa

Answer: 2 Explanation: 2. A needle or a fetal scalp electrode is used to make a small puncture in the amniotic sac. There is a risk that the force of the fluid could make a larger hole in the amniotic sac, thus increasing the risk of a prolapsed cord.

The clinic nurse assesses a newborn that is not progressing as expected. Genetic tests are ordered. The nurse explains to the parents that the laboratory tests to be done include which of the following? Note: Credit will be given only if all correct and no incorrect choices are selected. Select all that apply. 1. Chromosome analysis 2. Complete blood count 3. Phenylketonuria 4. Enzyme assay 5. Antibody titers

Answer: 1, 4, 5 Explanation: 1. Laboratory analysis includes chromosome analysis. 4. Laboratory analysis includes enzyme assay for inborn errors of metabolism. 5. Laboratory analysis includes antibody titers for infectious teratogens.

The nurse is planning a group session for clients who are beginning infertility evaluation. Which statements should be included in this session? Note: Credit will be given only if all correct choices and no incorrect choices are selected. Select all that apply. 1. "Infertility can be stressful for a marriage." 2. "The doctor will be able to tell why you have not conceived." 3. "Your insurance will pay for the infertility treatments." 4. "Keep communicating with one another through this process." 5. "Support organizations can be helpful to deal with the emotional issues associated with infertility."

Answer: 1, 4, 5 Explanation: 1. Often an intact marriage will become stressed by the intrusive but necessary infertility procedures and treatments. 4. Communication is important; clients should communicate verbally and share feelings and support. 5. Referral to mental health professionals is helpful when the emotional issues become too disruptive in the couple's relationship or life. Couples should be made aware of infertility support and education organizations, which may help meet some of these needs and validate their feelings.

Many newborns exposed to HIV/AIDS show signs and symptoms of disease within days of birth that include which of the following? Note: Credit will be given only if all correct choices and no incorrect choices are selected. Select all that apply. 1. Swollen glands 2. Hard stools 3. Smaller than average spleen and liver 4. Rhinorrhea 5. Interstitial pneumonia

Answer: 1, 4, 5 Explanation: 1. Signs that may be seen in the early infancy period include swollen glands. 4. Signs that may be seen in the early infancy period include rhinorrhea. 5. Signs that may be seen in the early infancy period include interstitial pneumonia.

A client is admitted to the labor suite. It is essential that the nurse assess the woman's status in relation to which infectious diseases? Note: Credit will be given only if all correct choices and no incorrect choices are selected. Select all that apply. 1. Chlamydia trachomatis 2. Rubeola 3. Varicella 4. Group B streptococcus 5. Acute pyelonephritis

Answer: 1, 4, 5 Explanation: 1. The infant may develop chlamydial pneumonia and Chlamydia trachomatis may be responsible for premature labor and fetal death. Chlamydial infection should be assessed. 4. Women may transmit GBS to their fetus in utero or during childbirth. GBS is a leading infectious cause of neonatal sepsis and mortality and should be assessed. 5. Acute pyelonephritis should be assessed as there is an increased risk of premature birth and intrauterine growth restriction (IUGR).

Which nursing diagnoses can apply to the couple experiencing a perinatal loss? Note: Credit will be given only if all correct choices and no incorrect choices are selected. Select all that apply. 1. Grieving related to the imminent loss of a child 2. Fear related to discomfort of labor and unknown outcome 3. Knowledge, Deficient related to lack of information about involution 4. Powerlessness related to lack of control in current situational crisis 5. Spiritual Distress, Risk for related to intense suffering secondary to unexpected fetal loss

Answer: 1, 4, 5 Explanation: 1. The nurse should anticipate that the family will experience the grieving process for the lost fetus. 4. The parents are faced with the sudden and unanticipated death of the unborn child, which occurred without any input or control on their part. 5. Spiritual distress is a common reaction of parents who experience an unanticipated loss.

A client who is having false labor most likely would have which of the following? Note: Credit will be given only for all correct choices and no incorrect choices. Select all that apply. 1. Contractions that do not intensify while walking 2. An increase in the intensity and frequency of contractions 3. Progressive cervical effacement and dilatation 4. Pain in the abdomen that does not radiate 5. Contractions that lessen with rest and warm tub baths

Answer: 1, 4, 5 Explanation: 1. True labor contractions intensify while walking. 4. True labor results in progressive dilation, increased intensity and frequency of contractions, and pain in the back that radiates to the abdomen. 5. In true labor, contractions do not lessen with rest and warm tub baths.

Methods to increase fertility awareness include which of the following? Note: Credit will be given only if all correct and no incorrect choices are selected. Select all that apply. 1. Home assessment of cervical mucus 2. Pharmacologic agents 3. Therapeutic insemination 4. IVF 5. Basal body temperature (BBT) recordings

Answer: 1, 5 Explanation: 1. Methods to increase fertility awareness include home assessment of cervical mucus and basal body temperature (BBT) recordings. 5. Methods to increase fertility awareness include home assessment of cervical mucus and basal body temperature (BBT) recordings.

The nurse is preparing an educational in-service presentation about jaundice in the newborn. What content should the nurse include in this presentation? Note: Credit will be given only if all correct choices and no incorrect choices are selected. Select all that apply. 1. Physiologic jaundice occurs after 24 hours of age. 2. Pathologic jaundice occurs after 24 hours of age. 3. Phototherapy increases serum bilirubin levels. 4. The need for phototherapy depends on the bilirubin level and age of the infant. 5. Kernicterus causes irreversible neurological damage.

Answer: 1, 5 Explanation: 1. Physiologic or neonatal jaundice is a normal process that occurs during transition from intrauterine to extrauterine life and appears after 24 hours of life. 5. Kernicterus refers to the deposition of unconjugated bilirubin in the basal ganglia of the brain and to permanent neurologic sequelae of untreated hyperbilirubinemia.

The pregnant teen who was prescribed prenatal vitamins at her initial prenatal visit states that she does not like to take them. How should the nurse respond? Note: Credit will be given only if all correct choices and no incorrect choices are selected. Select all that apply. 1. "Folic acid has been found to be essential for minimizing the risk of neural tube defects." 2. "You do not have to take these supplements if you think you are healthy enough." 3. "These medications do the same thing. I will call your doctor to cancel one of your medications." 4. "You can trust your doctor to know what you need." 5. "You need the supplements because your dietary intake may not be adequate for fetal development."

Answer: 1, 5 Explanation: 1. The CDC estimates that most neural tube defects could be prevented if women followed folic acid supplementation recommendations before they know they are pregnant. 5. One role of the nurse is educator, and this client needs additional information on why she needs the supplements. This response answers the client's concerns.

A mother and her newborn are being discharged 2 days after delivery. The general discharge instructions provided by the nurse include which of the following? Note: Credit will be given only if all correct choices and no incorrect choices are selected. Select all that apply. 1. Always place the infant in a supine position in the crib. 2. Support the infant's head when carrying for the first week or two. 3. Do not allow the baby to fall asleep in someone's arms. 4. Cover the cord stump with a bandage. 5. Use a bulb syringe to suction mucus from the infant's nostrils as necessary

Answer: 1, 5 Explanation: 1. The newborn should be placed on his or her back (supine) for sleeping. 5. During the first few days of life, the newborn has increased mucus, and gentle suctioning with a bulb syringe may be indicated.

The nurse is caring for a client in the transition phase of labor and notes that the fetal monitor tracing shows average short-term and long-term variability with a baseline of 142 beats per minute. What actions should the nurse take in this situation? Note: Credit will be given only if all correct choices and no incorrect choices are selected. Select all that apply. 1. Provide caring labor support. 2. Administer oxygen via face mask. 3. Change the client's position. 4. Speed up the client's intravenous. 5. Reassure the client and her partner that she is doing fine.

Answer: 1, 5 Explanation: 1. The tracing is normal, so the nurse can continue support of the labor. 5. The nurse can reassure the client at this time, as the tracing is normal.

The nurse is reviewing amniocentesis results. Which of the following would indicate that client care was appropriate? 1. The client who is Rh-positive received Rh immune globulin after the amniocentesis. 2. The client was monitored for 30 minutes after completion of the test. 3. The client began vaginal spotting before leaving for home after the test. 4. The client identified that she takes insulin before each meal and at bedtime.

Answer: 2 Explanation: 2. 20 to 30 minutes of fetal monitoring is performed after the amniocentesis.

A nurse examining a prenatal client recognizes that a lag in progression of measurements of fundal height from week to week and month to month could signal what condition? 1. Twin pregnancy 2. Intrauterine growth restriction 3. Hydramnios 4. Breech position

Answer: 2 Explanation: 2. A lag in progression of measurements of fundal height from month to month could signal intrauterine growth restriction (IUGR).

A client at 37 weeks' gestation has a mildly elevated blood pressure. Her antenatal testing demonstrates three contractions in 10 minutes, no decelerations, and accelerations four times in 1 hour. What would this test be considered? 1. Positive non-stress test 2. Negative contraction stress test 3. Positive contraction stress test 4. Negative non-stress test

Answer: 2 Explanation: 2. A negative CST shows three contractions of good quality lasting 40 or more seconds in 10 minutes without evidence of late decelerations. This is the desired result.

A postpartum woman is at increased risk for developing urinary tract problems because of which of the following? 1. Decreased bladder capacity 2. Inhibited neural control of the bladder following the use of anesthetic agents 3. Increased bladder sensitivity 4. Abnormal postpartum diuresis

Answer: 2 Explanation: 2. A postpartum woman is at increased risk for developing urinary tract problems because of inhibited neural control of the bladder following the use of anesthetic agents.

The special care nursery nurse is working with parents of a 3-day-old infant who was born with myelomeningocele and has developed an infection. Which statement from the mother is unexpected? 1. "If I had taken better care of myself, this wouldn't have happened." 2. "I've been sleeping very well since I had the baby." 3. "This is probably the doctor's fault." 4. "If I hadn't seen our baby's birth, I wouldn't believe she is ours."

Answer: 2 Explanation: 2. A sick infant is a source of great anxiety for parents. This response is from the mother would be unexpected.

Which of the following tests provides information about the fetal number? 1. Amniocentesis 2. Standard second-trimester sonogram 3. Beta hCG 4. Maternal serum alpha-fetoprotein

Answer: 2 Explanation: 2. A standard (comprehensive) second trimester sonogram provides the information about the fetus, placenta, and uterine conditions including fetal number.

One day after giving birth vaginally, a client develops painful vesicular lesions on her perineum and vulva. She is diagnosed with a primary herpes simplex 2 infection. What is the expected care for her neonate? 1. Meticulous hand washing and antibiotic eye ointment administration. 2. Intravenous acyclovir (Zovirax) and contact precautions. 3. Cultures of blood and CSF and serial chest x-rays every 12 hours. 4. Parental rooming-in and four intramuscular injections of penicillin.

Answer: 2 Explanation: 2. Administering intravenous acyclovir (Zovirax) and contact precautions are appropriate measures for an infant at risk for developing herpes simplex 2 infection.

A woman has just delivered a stillborn child at 26 weeks' gestation. Which nursing action is appropriate at this time? 1. Remind the mother that she will be able to have another baby in the future. 2. Dress the infant in a gown and swaddle it in a receiving blanket. 3. Ask the woman whether she would like the doctor to prescribe a sedative for her. 4. Remove the baby from the delivery room as soon as possible.

Answer: 2 Explanation: 2. After bathing, the infant should be placed in a suitable-sized gown and then wrapped in a blanket.

The pregnant client at 14 weeks' gestation is in the clinic for a regular prenatal visit. Her mother also is present. The grandmother-to-be states that she is quite uncertain about how she can be a good grandmother to this baby because she works full-time. Her own grandmother was retired, and was always available when needed by a grandchild. What is the nurse's best response to this concern? 1. "Don't worry. You'll be a wonderful grandmother. It will all work out fine." 2. "What are your thoughts on what your role as grandmother will include?" 3. "As long as there is another grandmother available, you don't have to worry." 4. "Grandmothers are supposed to be available. You should retire from your job."

Answer: 2 Explanation: 2. Although relationships with parents can be very complex, the expectant grandparents often become increasingly supportive of the expectant couple, even if conflicts previously existed. But it can be difficult for even sensitive grandparents to know how deeply to become involved in the childrearing process. In some areas, classes for grandparents provide information about changes in birthing and parenting practices.

The breastfeeding mother is concerned that her milk production has decreased. The nurse knows that further client teaching is needed based on which statement? 1. "I am drinking a minimum of 8 to 10 glasses of liquid a day." 2. "I have started cutting back on my protein intake." 3. "At least three times a day, I drink a glass of milk." 4. "My calorie intake is higher than during the pregnancy."

Answer: 2 Explanation: 2. An adequate protein intake is essential while breastfeeding because protein is an important component of breast milk.

The nurse expects an initial weight loss for the average postpartum client to be which of the following? 1. 5 to 8 pounds 2. 10 to 12 pounds 3. 12 to 15 pounds 4. 15 to 20 pounds

Answer: 2 Explanation: 2. An initial weight loss of 10 to 12 lbs. occurs as a result of the birth of infant, placenta, and amniotic fluid.

Narcotic analgesia is administered to a laboring client at 10:00 a.m. The infant is delivered at 12:30 p.m. What would the nurse anticipate that the narcotic analgesia could do? 1. Be used in place of preoperative sedation 2. Result in neonatal respiratory depression 3. Prevent the need for anesthesia with an episiotomy 4. Enhance uterine contractions

Answer: 2 Explanation: 2. Analgesia given too late is of no value to the woman and may cause neonatal respiratory depression.

A 25-year-old primigravida is at 20 weeks' gestation. The nurse takes her vital signs and notifies the healthcare provider immediately because of which finding? 1. Pulse 88/minute 2. Rhonchi in both bases 3. Temperature 37.4° C (99.3° F) 4. Blood pressure 122/78

Answer: 2 Explanation: 2. Any abnormal breath sounds should be reported to the healthcare provider.

During labor, the fetus was in a brow presentation, but after a prolonged labor, the fetus converted to face presentation and was delivered vaginally with forceps assist. What should the nurse explain to the parents? 1. The infant will need to be observed for meconium aspiration. 2. Facial edema and head molding will subside in a few days. 3. The infant will be given prophylactic antibiotics. 4. Breastfeeding will need to be delayed for a day or two.

Answer: 2 Explanation: 2. Any facial edema and head molding that result from the use of forceps at birth will subside in a few days.

Approximately 80% of anovulatory women have which condition? 1. Turner syndrome 2. Polycystic ovary syndrome (PCOS) 3. Klinefelter syndrome 4. Fragile X syndrome

Answer: 2 Explanation: 2. Approximately 80% of anovulatory women have polycystic ovary syndrome (PCOS), causing insulin resistance and hyperinsulinemia.

Upon delivery of the newborn, what nursing intervention most promotes parental attachment? 1. Placing the newborn under the radiant warmer. 2. Placing the newborn on the mother's abdomen. 3. Allowing the mother a chance to rest immediately after delivery. 4. Taking the newborn to the nursery for the initial assessment.

Answer: 2 Explanation: 2. As the baby is placed on the mother's abdomen or chest, she frequently reaches out to touch and stroke her baby. When the newborn is placed in this position, the father or partner also has a very clear, close view and can also reach out to touch the baby.

The student nurse has performed a gestational age assessment of an infant, and finds the infant to be at 32 weeks. On which set of characteristics is the nurse basing this assessment? 1. Lanugo mainly gone, little vernix across the body 2. Prominent clitoris, enlarging minora, anus patent 3. Full areola, 5 to 10 mm bud, pinkish-brown in color 4. Skin opaque, cracking at wrists and ankles, no vessels visible

Answer: 2 Explanation: 2. At 30 to 32 weeks' gestation, the clitoris is prominent, and the labia majora are small and widely separated. As gestational age increases, the labia majora increase in size. At 36 to 40 weeks, they nearly cover the clitoris. At 40 weeks and beyond, the labia majora cover the labia minora and clitoris.

The labor and delivery nurse is reviewing charts. The nurse should inform the supervisor about which client? 1. Client at 5 cm requesting labor epidural analgesia 2. Client whose cervix remains at 6 cm for 4 hours 3. Client who has developed nausea and vomiting 4. Client requesting her partner to stay with her

Answer: 2 Explanation: 2. Average cervical change in the active phase of the first stage of labor is 1.2 cm/hour; thus, this client's lack of cervical change is unexpected, and should be reported to the supervisor.

A woman in labor asks the nurse to explain the electronic fetal heart rate monitor strip. The fetal heart rate baseline is 150 with accelerations to 165, variable decelerations to 140, and moderate long-term variability. Which statement indicates that the client understands the nurse's teaching? 1. "The most important part of fetal heart monitoring is the absence of variable decelerations." 2. "The most important part of fetal heart monitoring is the presence of variability." 3. "The most important part of fetal heart monitoring is the fetal heart rate baseline." 4. "The most important part of fetal heart monitoring is the depth of decelerations."

Answer: 2 Explanation: 2. Baseline variability is a reliable indicator of fetal cardiac and neurologic function and well-being. The opposing "push-pull" balancing between the sympathetic nervous system and the parasympathetic nervous system directly affects the FHR.

The laboring client is having moderately strong contractions lasting 60 seconds every 3 minutes. The fetal head is presenting at a -2 station. The cervix is 6 cm and 100% effaced. The membranes spontaneously ruptured prior to admission, and clear fluid is leaking. Fetal heart tones are in the 140s with accelerations to 150. Which nursing action has the highest priority? 1. Encourage the husband to remain in the room. 2. Keep the client on bed rest at this time. 3. Apply an internal fetal scalp electrode. 4. Obtain a clean-catch urine specimen.

Answer: 2 Explanation: 2. Because the membranes are ruptured and the head is high in the pelvis at a -2 station, the client should be maintained on bed rest to prevent cord prolapse.

A new mother who is breastfeeding tells the nurse that her infant is spitting up frequently, has very loose stools and copious gas, and feeds for only short periods of time. The nurse suspects a feeding intolerance and, after questioning the mother about her diet, suggests that she do which of the following? 1. Stop breastfeeding and switch to formula. 2. Eliminate dairy products from her diet. 3. Supplement breastfeeding with a soy-based formula. 4. Offer the baby water between feedings.

Answer: 2 Explanation: 2. Breastfeeding babies may not be allergic to the mother's milk but rather to the cow's milk protein (an antigen) in the mother's milk. By eliminating the culprit (e.g., the bovine protein) from the mother's diet and therefore from the breast milk, the mother can continue to breastfeed, providing optimal nutrition and immune factors to her infant.

Placing the baby at mother's breast facilitates early latch and promotes successful breastfeeding. When should breastfeeding be initiated? 1. 6 to 12 hours after birth 2. Within 1 hour of birth 3. 24 hours after birth 4. 48 hours after birth

Answer: 2 Explanation: 2. Breastfeeding should be initiated within the first hour of life unless medically contraindicated.

The homecare nurse is seeing a client at 6 weeks postpartum. Which statement by the client indicates the need for immediate intervention? 1. "The baby sleeps 7 hours each night now." 2. "My flow is red, and I need to wear a pad." 3. "My breasts no longer leak between feedings." 4. "I started back on the pill 2 weeks ago."

Answer: 2 Explanation: 2. By 6 weeks postpartum, lochia should be absent or minimal in amount, requiring only a pantiliner. Red, heavy flow is not an expected finding, and requires intervention.

To prevent the spread of infection, the nurse teaches the postpartum client to do which of the following? 1. Address pain early 2. Change peri-pads frequently 3. Avoid overhydration 4. Report symptoms of uterine cramping

Answer: 2 Explanation: 2. Changing peri-pads frequently decreases skin contact with a moist medium that favors bacteria growth.

The nurse suspects clubfoot in the newborn and assesses for the condition by doing which of the following? 1. Adducting the foot and listening for a click. 2. Moving the foot to midline and determining resistance. 3. Extending the foot and observing for pain. 4. Stimulating the sole of the foot.

Answer: 2 Explanation: 2. Clubfoot is suspected when the foot does not turn to a midline position or align readily.

A newly diagnosed insulin-dependent type 1 diabetic with good blood sugar control is at 20 weeks' gestation. She asks the nurse how her diabetes will affect her baby. What would the best explanation include? 1. "Your baby could be smaller than average at birth." 2. "Your baby will probably be larger than average at birth." 3. "As long as you control your blood sugar, your baby will not be affected at all." 4. "Your baby might have high blood sugar for several days."

Answer: 2 Explanation: 2. Characteristically, infants of mothers with diabetes are large for gestational age (LGA) as a result of high levels of fetal insulin production stimulated by the high levels of glucose crossing the placenta from the mother. Sustained fetal hyperinsulinism and hyperglycemia ultimately lead to excessive growth, called macrosomia, and deposition of fat.

The nurse is assessing a woman at 10 weeks' gestation who is addicted to alcohol. The woman asks the nurse, "What is the point of stopping drinking now if my baby probably has been hurt by it already?" What is the best response by the nurse? 1. "It won't help your baby, but you will feel better during your pregnancy if you stop now." 2. "If you stop now, you and your baby have less chance of serious complications." 3. "If you limit your drinking to once a week, your baby will be okay." 4. "You might as well stop it now, because once your baby is born, you'll have to give up alcohol if you plan on breastfeeding."

Answer: 2 Explanation: 2. Chronic abuse of alcohol can undermine maternal health by causing malnutrition, bone marrow suppression, increased incidence of infections, and liver disease. The effects of alcohol on the fetus may result in fetal alcohol spectrum disorders (FASD).

The laboring client participated in childbirth preparation classes that strongly discouraged the use of medications and intervention during labor. The client has been pushing for two hours, and is exhausted. The physician requests that a vacuum extractor be used to facilitate the birth. The client first states that she wants the birth to be normal, then allows the vacuum extraction. Following this, what should the nurse assess the client for after the birth? 1. Elation, euphoria, and talkativeness 2. A sense of failure and loss 3. Questions about whether or not to circumcise 4. Uncertainty surrounding the baby's name

Answer: 2 Explanation: 2. Clients who participate in childbirth classes that stress the normalcy of birth may feel a sense of loss or failure if an intervention is used during their labor or birth.

The nurse is completing the gestational age assessment on a newborn while in the mother's postpartum room. During the assessment, the mother asks what aspects of the baby are being checked. What is the nurse's best response? 1. "I'm checking to make sure the baby has all of its parts." 2. "This assessment looks at both physical aspects and the nervous system." 3. "This assessment checks the baby's brain and nerve function." 4. "Don't worry. We perform this check on all the babies."

Answer: 2 Explanation: 2. Clinical gestational age assessment tools have two components: external physical characteristics and neurologic or neuromuscular development evaluations.

What is required for any women receiving oxytocin (Pitocin)? 1. CPR 2. Continuous electronic fetal monitoring 3. Administering oxygen by mask 4. Nonstress test

Answer: 2 Explanation: 2. Continuous electronic fetal monitoring (EFM) is required for any women receiving oxytocin (Pitocin).

The nurse is training a nurse new to the labor and delivery unit. They are caring for a laboring client who will have a forceps delivery. Which action or assessment finding requires intervention? 1. Regional anesthesia is administered via pudendal block. 2. The client is instructed to push between contractions. 3. Fetal heart tones are consistently between 110 and 115. 4. The client's bladder is emptied using a straight catheter.

Answer: 2 Explanation: 2. During the contraction, as the forceps are applied, the woman should avoid pushing.

A client is admitted to the labor unit with contractions 1-2 minutes apart lasting 60-90 seconds. The client is apprehensive and irritable. This client is most likely in what phase of labor? 1. Active 2. Transition 3. Latent 4. Second

Answer: 2 Explanation: 2. During transition, contractions have a frequency of 1 1/2 to 2 minutes, a duration of 60 to 90 seconds, and are strong in intensity. When the woman enters the transition phase, she may demonstrate significant anxiety.

Which of the following is important for the development of the central nervous system of the fetus? 1. Calcium and phosphorus 2. Essential fatty acids 3. Iron 4. Vitamin D

Answer: 2 Explanation: 2. Essential fatty acids are important for the development of the central nervous system of the fetus. Of particular interest are the omega-3 fatty acids and their derivatives.

What can be determined based on ultrasound visualization or the lack of visualization of an intertwin membrane? 1. Toxicity 2. Amnionicity 3. Variability 4. Prematurity

Answer: 2 Explanation: 2. Evidence supports the use of ultrasound for accurately determining chorionicity and amnionicity in multiple pregnancies. Determination of amnionicity is based on ultrasound visualization or the lack of visualization of an intertwin membrane.

The postpartum homecare client asks the nurse why the visit is taking place. Which response is best? 1. "We make homecare visits to reinforce any teaching that you didn't quite grasp in the hospital." 2. "We make homecare visits to verify that both you and the baby are safe and doing well." 3. "We make homecare visits to ensure you are breastfeeding correctly." 4. "We make homecare visits to thoroughly assess your baby to make sure he is growing."

Answer: 2 Explanation: 2. Family well-being should be determined through a comprehensive assessment that includes physical, emotional, and social functioning.

The client at 39 weeks' gestation is undergoing a cesarean birth due to breech presentation. General anesthesia is being used. Which situation requires immediate intervention? 1. The baby's hands and feet are blue at 1 minute after birth. 2. The fetal heart rate is 70 prior to making the skin incision. 3. Clear fluid is obtained from the baby's oropharynx. 4. The neonate cries prior to delivery of the body.

Answer: 2 Explanation: 2. Fetal bradycardia occurs when the fetal heart rate falls below 110 beats/minute during a 10-minute period of continuous monitoring. When fetal bradycardia is accompanied by decreased variability, it is considered ominous and could be a sign of fetal compromise.

The physician has determined the need for forceps. The nurse should explain to the client that the use of forceps is indicated because of which of the following? 1. Her support person is exhausted 2. Premature placental separation 3. To shorten the first stage of labor 4. To prevent fetal distress

Answer: 2 Explanation: 2. Fetal conditions indicating the need for forceps include premature placental separation, prolapsed umbilical cord, and nonreassuring fetal status.

A client who is experiencing her first pregnancy has just completed the initial prenatal examination with a certified nurse-midwife. Which statement indicates that the client needs additional information? 1. "Because we heard the baby's heartbeat, I am undoubtedly pregnant." 2. "Because I haven't felt the baby move yet, we don't know whether I'm pregnant." 3. "My last period was 2 months ago, which means I'm 2 months along." 4. "The increased size of my uterus means that I am finally pregnant."

Answer: 2 Explanation: 2. Fetal movement is a subjective, or presumptive, change of pregnancy, and is not a reliable indicator in the early months of pregnancy.

The client is in the second stage of labor. The fetal heart rate baseline is 170, with minimal variability present. The nurse performs fetal scalp stimulation. The client's partner asks why the nurse did that. What is the best response by the nurse? 1. "I stimulated the top of the fetus's head to wake him up a little." 2. "I stimulated the top of the fetus's head to try to get his heart rate to accelerate." 3. "I stimulated the top of the fetus's head to calm the fetus down before birth." 4. "I stimulated the top of the fetus's head to find out whether he is in distress."

Answer: 2 Explanation: 2. Fetal scalp stimulation is done when there is a question regarding fetal status. An acceleration indicates fetal well-being.

A mother who is HIV-positive has given birth to a term female. What plan of care is most appropriate for this infant? 1. Test with a HIV serologic test at 8 months. 2. Begin prophylactic AZT (Zidovudine) administration. 3. Provide 4 to 5 large feedings throughout the day. 4. Encourage the mother to breastfeed the child.

Answer: 2 Explanation: 2. For infants, AZT is started prophylactically 2 mg/kg/dose PO every 6 hours beginning as soon after birth as possible and continuing for 6 weeks.

The client at 9 weeks' gestation has been told that her HIV test was positive. The client is very upset, and tells the nurse, "I didn't know I had HIV! What will this do to my baby?" The nurse knows teaching has been effective when the client makes which statement? 1. "I cannot take the medications that control HIV during my pregnancy, because they will harm the baby." 2. "My baby can get HIV during the pregnancy and through my breast milk." 3. "The pregnancy will increase the progression of my disease and will reduce my CD4 counts." 4. "The HIV won't affect my baby, and I will have a low-risk pregnancy without additional testing."

Answer: 2 Explanation: 2. HIV transmission can occur during pregnancy and through breast milk; however, it is believed that the majority of all infections occur during labor and birth.

The nurse is working in a teen pregnancy clinic. In order to give the pregnant adolescent a role in her prenatal care, the nurse should allow the teen to do which of the following? 1. Choose the type of prenatal vitamin she takes 2. Measure and record her weight at each visit 3. Choose the schedule of her prenatal visits 4. Decide whether she wants her labor to be induced

Answer: 2 Explanation: 2. Having the client weigh herself and record her weight provides her the opportunity to participate in her own care plan.

The nurse is calling clients at 4 weeks postpartum. Which of the following clients should be seen immediately? 1. The client who describes feeling sad all the time 2. The client who reports hearing voices talking about the baby 3. The client who states she has no appetite and wants to sleep all day 4. The client who says she needs a refill on her sertraline (Zoloft) next week

Answer: 2 Explanation: 2. Hearing voices is an indication the client is experiencing postpartum psychosis, and is the highest priority because the voices might tell her to harm her baby.

A client at 16 weeks' gestation has a hematocrit of 35%. Her prepregnancy hematocrit was 40%. Which statement by the nurse best explains this change? 1. "Because of your pregnancy, you're not making enough red blood cells." 2. "Because your blood volume has increased, your hematocrit count is lower." 3. "This change could indicate a serious problem that might harm your baby." 4. "You're not eating enough iron-rich foods like meat."

Answer: 2 Explanation: 2. Hemoglobin and hematocrit levels drop in early to mid-pregnancy as a result of pregnancy-associated hemodilution. Because the plasma volume increase (50%) is greater than the erythrocyte increase (25%), the hematocrit decreases slightly.

A client is at 12 weeks' gestation with her first baby. She has cardiac disease, class III. She states that she had been taking sodium warfarin (Coumadin), but her physician changed her to heparin. She asks the nurse why this was done. What should the nurse's response be? 1. "Heparin is used when coagulation problems are resolved." 2. "Heparin is safer because it does not cross the placenta." 3. "They are the same drug, but heparin is less expensive." 4. "Coumadin interferes with iron absorption in the intestines."

Answer: 2 Explanation: 2. Heparin is safest for the client to take because it does not cross the placental barrier.

The nurse is admitting a client with possible hydramnios. When is hydramnios most likely suspected? 1. Hydramnios is most likely suspected when there is less amniotic fluid than normal for gestation. 2. Hydramnios is most likely suspected when the fundal height increases disproportionately to the gestation. 3. Hydramnios is most likely suspected when the woman has a twin gestation. 4. Hydramnios is most likely suspected when the quadruple screen comes back positive.

Answer: 2 Explanation: 2. Hydramnios should be suspected when the fundal height increases out of proportion to the gestational age.

The client has undergone an ultrasound, which estimated fetal weight at 4500 g (9 pounds 14 ounces). Which statement indicates that additional teaching is needed? 1. "Because my baby is big, I am at risk for excessive bleeding after delivery." 2. "Because my baby is big, his blood sugars could be high after he is born." 3. "Because my baby is big, my perineum could experience trauma during the birth." 4. "Because my baby is big, his shoulders could get stuck and a collarbone broken."

Answer: 2 Explanation: 2. Hypoglycemia, not hyperglycemia, is a potential complication experienced by a macrosomic fetus.

The nursing instructor is conducting a class about attachment behaviors. Which statement by a student indicates the need for further instruction? 1. "The en face position promotes bonding and attachment." 2. "Ideally, initial skin-to-skin contact occurs after the baby has been assessed and bathed." 3. "In reciprocity, the interaction of mother and infant is mutually satisfying and synchronous." 4. "The needs of the mother and of her infant are balanced during the phase of mutual regulation."

Answer: 2 Explanation: 2. Ideally, initial skin-to-skin contact is immediate. The benefits of this practice are supported by a preponderance of evidence.

A woman is 16 weeks pregnant. She has had cramping, backache, and mild bleeding for the past 3 days. Her physician determines that her cervix is dilated to 2 centimeters, with 10% effacement, but membranes are still intact. She is crying, and says to the nurse, "Is my baby going to be okay?" In addition to acknowledging the client's fear, what should the nurse also say? 1. "Your baby will be fine. We'll start IV, and get this stopped in no time at all." 2. "Your cervix is beginning to dilate. That is a serious sign. We will continue to monitor you and the baby for now." 3. "You are going to miscarry. But you should be relieved because most miscarriages are the result of abnormalities in the fetus." 4. "I really can't say. However, when your physician comes, I'll ask her to talk to you about it."

Answer: 2 Explanation: 2. If bleeding persists and abortion is imminent or incomplete, the woman may be hospitalized, IV therapy or blood transfusions may be started to replace fluid, and dilation and curettage (D&C) or suction evacuation is performed to remove the remainder of the products of conception.

The physicians/CNM opts to use a vacuum extractor for a delivery. What does the nurse understand? 1. There is little risk with vacuum extraction devices. 2. There should be further fetal descent with the first two "pop-offs." 3. Traction is applied between contractions. 4. The woman often feels increased discomfort during the procedure.

Answer: 2 Explanation: 2. If more than three "pop-offs" occur (the suction cup pops off the fetal head), the procedure should be discontinued. Page Ref: 641

A client in labor is found to have meconium-stained amniotic fluid upon rupture of membranes. At delivery, the nurse finds the infant to have depressed respirations and a heart rate of 80. What does the nurse anticipate? 1. Delivery of the neonate on its side with head up, to facilitate drainage of secretions. 2. Direct tracheal suctioning by specially trained personnel. 3. Preparation for the immediate use of positive pressure to expand the lungs. 4. Suctioning of the oropharynx when the newborn's head is delivered.

Answer: 2 Explanation: 2. If the infant has absent or depressed respirations, heart rate less than 100 beats/min, or poor muscle tone, direct tracheal suctioning by specially trained personnel is recommended.

A client is to receive fertility drugs prior to in vitro fertilization. What is the expected action of this medication? 1. Prolonging of the luteal phase 2. Stimulation of ovulation 3. Suppression of menstruation 4. Promotion of cervical mucus production

Answer: 2 Explanation: 2. In IVF, a woman's ovaries are stimulated by a combination of medications, one or more oocytes are aspirated from her ovaries and fertilized in the laboratory, and then they are placed into her uterus after normal embryo development has begun.

The nurse is preparing a class for expectant fathers. Which information should the nurse include? 1. Siblings adjust readily to the new baby. 2. Sexual activity is safe for normal pregnancy. 3. The expectant mother decides the feeding method. 4. Fathers are expected to be involved in labor and birth.

Answer: 2 Explanation: 2. In a healthy pregnancy, there is no medical reason to limit sexual activity.

A new grandmother comments that when her children were born, they stayed in the nursery. The grandmother asks the nurse why her daughter's baby stays mostly in the room instead of the nursery. How should the nurse respond? 1. "Babies like to be with their mothers more than they like to be in the nursery." 2. "Contact between parents and babies increases attachment." 3. "Budget cuts have decreased the number of nurses in the nursery." 4. "Why do you ask? Do you have concerns about your daughter's parenting?"

Answer: 2 Explanation: 2. In a mother-baby unit, the newborn's crib is placed near the mother's bed, where she can see her baby easily; this is conducive to an on-demand feeding schedule for both breastfeeding and formula-feeding infants.

A Chinese woman who is 12 weeks pregnant reports to the nurse that ginseng and bamboo leaves help reduce her anxiety. How should the nurse respond to this client? 1. Advise the client to give up the bamboo leaves but to continue taking ginseng. 2. Advise the client to give up all herbal remedies. 3. Tell the client that her remedies have no scientific foundation. 4. Assess where the client obtains her remedy, and investigate the source.

Answer: 2 Explanation: 2. In some cases, the nurse might want to suggest remedies that may be more effective than herbal remedies. However, if the home remedy is not harmful, there is no reason for the nurse to ask a client to discontinue this practice.

A client arrives in the labor and delivery unit and describes her contractions as occurring every 10-12 minutes, lasting 30 seconds. She is smiling and very excited about the possibility of being in labor. On exam, her cervix is dilated 2 cm, 100% effaced, and -2 station. What best describes this labor? 1. Second phase 2. Latent phase 3. Active phase 4. Transition phase

Answer: 2 Explanation: 2. In the early or latent phase of the first stage of labor, contractions are usually mild. The woman feels able to cope with the discomfort. The woman is often talkative and smiling and is eager to talk about herself and answer questions.

In utero, what is the organ responsible for gas exchange? 1. Umbilical vein 2. Placenta 3. Inferior vena cava 4. Right atrium

Answer: 2 Explanation: 2. In utero, the placenta is the organ of gas exchange.

Which of the following would be considered a clinical sign of hemorrhage? 1. Increased blood pressure 2. Increasing pulse 3. Increased urinary output 4. Hunger

Answer: 2 Explanation: 2. Increasing pulse, widening pulse pressure would be considered a clinical sign of hemorrhage.

The nurse is planning care for four infants who were born on this shift. The infant who will require the most detailed assessment is the one whose mother has which of the following? 1. A history of obsessive-compulsive disorder (OCD) 2. Chlamydia 3. Delivered six other children by cesarean section 4. A urinary tract infection (UTI)

Answer: 2 Explanation: 2. Infants born to mothers with chlamydia infections are at risk for neonatal pneumonia and conjunctivitis, and require close observation of the respiratory status and eyes. Page

The community nurse has identified that the mother who gave birth to a stillborn baby last week is an intuitive griever. Which behavior has the nurse encountered that would lead to this assessment? 1. The mother verbalized that her problem-solving skills have been helpful during this process. 2. The mother repeatedly talks about her thoughts, feelings, and emotions about losing her child. 3. The mother talks little about her experience, and appears detached and unaffected by the loss of her child. 4. The mother has asked close friends, co-workers, and relatives not to call or visit.

Answer: 2 Explanation: 2. Intuitive grievers tend to feel their way through the loss and seek emotional and psychosocial support.

The laboring client and her partner have arrived at the birthing unit. Which step of the admission process should be undertaken first? 1. The sterile vaginal exam 2. Welcoming the couple 3. Auscultation of the fetal heart rate 4. Checking for ruptured membranes

Answer: 2 Explanation: 2. It is important to establish rapport and to create an environment in which the family feels free to ask questions. The support and encouragement of the nurse in maintaining a caring environment begin with the initial admission.

The nurse is caring for a newborn who was recently circumcised. Which nursing intervention is appropriate following the procedure? 1. Keep the infant NPO for 4 hours following the procedure. 2. Observe for urine output. 3. Wrap dry gauze tightly around the penis. 4. Clean with cool water with each diaper change.

Answer: 2 Explanation: 2. It is important to observe for the first voiding after a circumcision to evaluate for urinary obstruction related to penile injury and/or edema.

5) What is the primary carbohydrate in mammalian milk that plays a crucial role in the nourishment of the newborn? 1. Colostrum 2. Lactose 3. Lactoferrin 4. Secretory IgA

Answer: 2 Explanation: 2. Lactose is the primary carbohydrate in mammalian milk.

Late preterm infants have higher infant morbidity and mortality rates than term infants. Which of the following complications can they experience? 1. Hyperglycemia 2. Jaundice 3. Motor difficulties 4. Sensory complications

Answer: 2 Explanation: 2. Late preterm infants can experience jaundice.

A client calls the labor and delivery unit and tells the nurse that she is 39 weeks pregnant and that over the last 4 or 5 days, she has noticed that although her breathing has become easier, she is having leg cramps, a slight amount of edema in her lower legs, and an increased amount of vaginal secretions. The nurse tells the client that she has experienced which of the following? 1. Engagement 2. Lightening 3. Molding 4. Braxton Hicks contractions

Answer: 2 Explanation: 2. Lightening describes the effect occurring when the fetus begins to settle into the pelvic inlet.

The need for forceps has been determined. The client's cervix is dilated to 10 cm, and the fetus is at +2 station. What category of forceps application would the nurse anticipate? 1. Input 2. Low 3. Mid 4. Outlet

Answer: 2 Explanation: 2. Low forceps are applied when the leading edge of the fetal head is at +2 station.

The community nurse is planning care for a family that experienced the loss of twins at 20 weeks. Which of these steps should be part of the nurse's care of this family? 1. Base care on the reactions of previous clients who experienced stillbirth. 2. Express the belief that the family will be able to get through this experience. 3. Encourage the couple to keep their feelings to themselves. 4. Honor the birth by reminding the couple that their babies are happy in heaven.

Answer: 2 Explanation: 2. Maintaining belief is defined as believing in the parents' capacity to get through the event and face a future with meaning and it is one of the attributes of caring theory.

A couple who have sought fertility counseling have been told that the man's sperm count is very low. The nurse advises the couple that spermatogenesis is impaired when which condition occurs? 1. The vas deferens is ligated. 2. Male obesity is present. 3. The prostate gland is enlarged. 4. The flagella are segmented.

Answer: 2 Explanation: 2. Male obesity is associated with poor spermatogenesis and increased amount of time to conception.

The nurse is assisting a multiparous woman to the bathroom for the first time since her delivery 3 hours ago. When the client stands up, blood runs down her legs and pools on the floor. The client turns pale and feels weak. What would be the first action of the nurse? 1. Assist the client to empty her bladder 2. Help the client back to bed to check the fundus 3. Assess her blood pressure and pulse 4. Begin an IV of lactated Ringer's solution

Answer: 2 Explanation: 2. Massaging the fundus is the top priority because of the excessive blood loss. If the fundus is not firm, gentle fundal massage is performed until the uterus contracts.

The laboring client brought a written birth plan indicating that she wanted to avoid pain medications and an epidural. She is now at 6 cm and states, "I can't stand this anymore! I need something for pain! How will an epidural affect my baby?" What is the nurse's best response? 1. "The narcotic in the epidural will make both you and the baby sleepy." 2. "It is unlikely that an epidural will decrease your baby's heart rate." 3. "Epidurals tend to cause low blood pressure in babies after birth." 4. "I can't get you an epidural, because of your birth plan."

Answer: 2 Explanation: 2. Maternal hypotension results in uteroplacental insufficiency in the fetus, which is manifested as late decelerations on the fetal monitoring strip. The risk of hypotension can be minimized by hydrating the vascular system with 500 to 1000 mL of IV solution before the procedure and changing the woman's position and/or increasing the IV rate afterward.

A client presents to the antepartum clinic with a history of a 20-pound weight loss. Her pregnancy test is positive. She is concerned about gaining the weight back, and asks the nurse if she can remain on her diet. What is the nurse's best response? 1. "As long as you supplement your diet with the prenatal vitamin, the amount of weight you gain in pregnancy is not significant." 2. "I understand that gaining weight after such an accomplishment might not look attractive, but weight gain during pregnancy is important for proper fetal growth." 3. "Dieting during pregnancy is considered child neglect." 4. "Excessive weight gain in pregnancy is due to water retention, so weight loss following birth will not be an issue."

Answer: 2 Explanation: 2. Maternal weight gain is an important factor in fetal growth and in infant birth weight. An adequate weight gain over time indicates an adequate caloric intake.

Mild or chronic anemia in an infant may be treated adequately which of the following? 1. Transfusions with O-negative or typed and cross-matched packed red cells 2. Iron supplements or iron-fortified formulas 3. Steroid therapy 4. Antibiotics or antivirals

Answer: 2 Explanation: 2. Mild or chronic anemia in an infant may be treated adequately with iron supplements or iron-fortified formulas.

A pregnant client who swims 3-5 times per week asks the nurse whether she should stop this activity. What is the appropriate nursing response? 1. "You should decrease the number of times you swim per week." 2. "Continuing your exercise program would be beneficial." 3. "You should discontinue your exercise program immediately." 4. "You should consider a less strenuous type of exercise."

Answer: 2 Explanation: 2. Mild to moderate exercise is beneficial during pregnancy. Regular exercise-at least 30 minutes of moderate exercise daily or at least most days of the week-is preferred.

The nurse is caring for a client who experienced the birth of a stillborn son earlier in the day. The client is from a culture where a woman's status is dominated by themes of motherhood and childrearing. What behavior would the nurse expect in this client? 1. Crying inconsolably 2. Expressing feelings of failure as a woman 3. Requesting family members to be present 4. Showing little emotion

Answer: 2 Explanation: 2. Mothers will often blame themselves, whether by commission or omission, particularly in cultures where a woman's status is dominated by themes of motherhood and childrearing.

The clinic nurse is assisting with an initial prenatal assessment. The following findings are present: spider nevi present on lower legs; dark pink, edematous nasal mucosa; mild enlargement of the thyroid gland; mottled skin and pallor on palms and nail beds; heart rate 88 with murmur present. What is the best action for the nurse to take based on these findings? 1. Document the findings on the prenatal chart. 2. Have the physician see the client today. 3. Instruct the client to avoid direct sunlight. 4. Analyze previous thyroid hormone lab results.

Answer: 2 Explanation: 2. Mottling of the skin is indicative of possible anemia. These abnormalities must be reported to the healthcare provider immediately.

The nurse notes that a 36-hour-old newborn's serum bilirubin level has increased from 14 mg/dL to 16.6 mg/dL in an 8-hour period. What nursing intervention would be included in the plan of care for this newborn? 1. Continue to observe 2. Begin phototherapy 3. Begin blood exchange transfusion 4. Stop breastfeeding

Answer: 2 Explanation: 2. Neonatal hyperbilirubinemia must be considered pathologic if the serum bilirubin concentration is rising by more than 0.2 mg/dL per hour. If the newborn is over 24 hours old, which is past the time where an increase in bilirubin would result from pathologic causes, phototherapy may be the treatment of choice to prevent the possible complications of kernicterus.

The nurse is caring for a client with hydramnios. What will the nurse watch for? 1. Possible intrauterine growth restriction 2. Newborn congenital anomalies 3. Newborn postmaturity and renal malformations 4. Fetal adhesions

Answer: 2 Explanation: 2. Newborn congenital anomalies occur with hydramnios

The nurse is assessing a primiparous client who indicates that her religion is Judaism. Why is this information is pertinent for the nurse to assess? 1. Religious and cultural background can impact what a client eats during pregnancy. 2. It provides a baseline from which to ask questions about the client's religious and cultural background. 3. Knowing the client's beliefs and behaviors regarding pregnancy is not important. 4. Clients sometimes encounter problems in their pregnancies based on what religion they practice.

Answer: 2 Explanation: 2. Nurses have an obligation to be aware of other cultures and develop a culturally sensitive plan of care to meet the needs of the childbearing woman and her family.

The nurse should anticipate the labor pattern for a fetal occiput posterior position to be which of the following? 1. Shorter than average during the latent phase 2. Prolonged as regards the overall length of labor 3. Rapid during transition 4. Precipitous

Answer: 2 Explanation: 2. Occiput posterior (OP) position of the fetus is the most common fetal malposition and occurs when the head remains in the direct OP position throughout labor. This can prolong the overall length of labor.

A nurse is evaluating the diet plan of a breastfeeding mother. Which beverage is most likely to cause intolerance in the infant? 1. Orange juice 2. Milk 3. Decaffeinated tea 4. Water

Answer: 2 Explanation: 2. Often fussy breastfeeding or cow's milk-based formula-fed infants are switched to a lactose-free formula because of concerns about lactose intolerance.

The student nurse notices that the newborn seems to focus on the mother's eyes. The nursing instructor explains that this newborn behavior is which of the following? 1. Habituation 2. Orientation 3. Self-quieting 4. Reactivity

Answer: 2 Explanation: 2. Orientation is the newborn's ability to be alert to, to follow, and to fixate on complex visual stimuli that have a particular appeal and attraction. The newborn prefers the human face and eyes, and bright shiny objects.

The nurse is preparing an educational session on phenylketonuria for a family whose neonate has been diagnosed with the condition. Which statement by a parent indicates that teaching was effective? 1. "This condition occurs more frequently among Japanese people." 2. "We must be very careful to avoid most proteins to prevent brain damage." 3. "Carbohydrates can cause our baby to develop cataracts and liver damage." 4. "Our baby's thyroid gland isn't functioning properly."

Answer: 2 Explanation: 2. PKU is the inability to metabolize phenylalanine, an amino acid found in most dietary protein sources. Excessive accumulation of phenylalanine and its abnormal metabolites in the brain tissue leads to progressive, irreversible intellectual disability.

To promote infant security in the hospital, the nurse instructs the parents of a newborn to do which of the following? 1. Keep the baby in the room at all times. 2. Check the identification of all personnel who transport the newborn. 3. Place a "No Visitors" sign on the door. 4. Keep the baby in the nursery at all times.

Answer: 2 Explanation: 2. Parent should be instructed to allow only people with proper birthing unit identification to remove the baby from the room. If parents do not know the staff person, they should call the nurse for assistance.

The nurse is caring for a client at 35 weeks' gestation who has been critically injured in a shooting. Which statement by the paramedics bringing the woman to the hospital would cause the greatest concern? 1. "Blood pressure 110/68, pulse 90." 2. "Entrance wound present below the umbilicus." 3. "Client is positioned in a left lateral tilt." 4. "Clear fluid is leaking from the vagina."

Answer: 2 Explanation: 2. Penetrating trauma includes gunshot wounds and stab wounds. The mother generally fares better than the fetus if the penetrating trauma involves the abdomen as the enlarged uterus is likely to protect the mother's bowel from injury.

The visiting nurse evaluates a 2-day-old breastfed newborn at home and notes that the baby appears jaundiced. When explaining jaundice to the parents, what would the nurse tell them? 1. "Jaundice is uncommon in newborns." 2. "Some newborns require phototherapy." 3. "Jaundice is a medical emergency." 4. "Jaundice is always a sign of liver disease."

Answer: 2 Explanation: 2. Physiologic jaundice is a normal process that can occur after 24 hours of life in about half of healthy newborns. It is not a sign of liver disease. Physiologic jaundice might require phototherapy.

A new mother is concerned about spoiling her newborn. The home care nurse teaches the mother which of the following? 1. Newborns can be manipulative, so caution is advised. 2. Meeting the infant's needs develops a trusting relationship. 3. An infant who is rocked to sleep every night is being spoiled. 4. Crying is good for babies, and letting them cry it out is advised.

Answer: 2 Explanation: 2. Picking babies up when they cry teaches them that adults are responsive to their needs. This helps build a sense of trust and security.

What type of forceps are designed to be used with a breech presentation? 1. Midforceps 2. Piper 3. Low 4. High

Answer: 2 Explanation: 2. Piper forceps are designed to be used with a breech presentation. They are applied after the birth of the body, when the fetal head is still in the birth canal and assistance is needed.

The community nurse is working with poor women who are formula-feeding their infants. Which statement indicates that the nurse's education session was effective? 1. "I should use only soy-based formula for the first year." 2. "I follow the instructions for mixing the powdered formula exactly." 3. "It is okay to add more water to the formula to make it last longer." 4. "The mixed formula can be left on the counter for a day."

Answer: 2 Explanation: 2. Powdered formula is the least expensive type of formula. Parents will need to be briefed on safety precautions during formula preparation and they should be instructed to follow the directions on the formula package label precisely as written.

Which of the following is the most prevalent medical complication of pregnant adolescents? 1. Constipation 2. Preeclampsia-eclampsia 3. Heartburn 4. Rapid enlargement and sensitivity of breasts

Answer: 2 Explanation: 2. Preeclampsia-eclampsia is the most prevalent medical complication of pregnant adolescents and is typically characterized by high blood pressure, proteinuria, and edema.

Couples at risk for having a detectable single gene or chromosomal anomaly may wish to undergo which procedure? 1. Preimplantation genetic screening (PGS) 2. Preimplantation genetic diagnosis (PGD) 3. Intracytoplasmic sperm injection (ICSI) 4. Gamete intrafallopian transfer (GIFT)

Answer: 2 Explanation: 2. Preimplantation genetic diagnosis (PGD) is a term used when one or both genetic parents carry a gene mutation and testing is performed to determine whether that mutation or unbalanced chromosomal compliment has been passed to the oocyte or embryo.

The nurse is reviewing assessment data from several different male clients. Which one should receive information about causes of infertility? 1. Circumcised client 2. Client with a history of premature ejaculation 3. Client with a history of measles at age 12 4. Client employed as an engineer

Answer: 2 Explanation: 2. Premature ejaculation is a possible cause of infertility.

The nurse is working with a mother who has just delivered her third child at 33 weeks' gestation. The mother says to the nurse, "This baby doesn't turn his head and suck like the older two children did. Why?" What is the best response by the nurse? 1. "Every baby is different. This is just one variation of normal that we see on a regular basis." 2. "This baby might not have a rooting or sucking reflex because she is premature." 3. "When she is wide awake and alert, she will probably root and suck even if she is early." 4. "She might be too tired from the birthing process and need a couple of days to recover."

Answer: 2 Explanation: 2. Preterm babies may have suppressed or absent root and suck reflexes.

The nurse is teaching a new mother how to encourage a sleepy baby to breastfeed. Which of the following instructions would not be included in that teaching? 1. Providing skin-to-skin contact 2. Swaddling the newborn in a blanket 3. Unwrapping the newborn 4. Allowing the newborn to feel and smell the mother's breast

Answer: 2 Explanation: 2. Remove the baby's blanket and clothing so that the infant is wearing only a diaper and T-shirt. Babies feed better when they are not bundled, and they can achieve better attachment without the bulk of extra clothing and blankets. Swaddling the newborn has the opposite effect.

The client delivered 30 minutes ago. Her blood pressure and pulse are stable. Vaginal bleeding is scant. The nurse should prepare for which procedure? 1. Abdominal hysterectomy 2. Manual removal of the placenta 3. Repair of perineal lacerations 4. Foley catheterization

Answer: 2 Explanation: 2. Retention of the placenta beyond 30 minutes after birth is termed retained placenta. Manual removal of the placenta is then performed.

Four minutes after the birth of a baby, there is a sudden gush of blood from the mother's vagina, and about 8 inches of umbilical cord slides out. What action should the nurse take first? 1. Place the client in McRoberts position. 2. Watch for the emergence of the placenta. 3. Prepare for the delivery of an undiagnosed twin. 4. Place the client in a supine position.

Answer: 2 Explanation: 2. Signs of placental separation usually appear around 5 minutes after birth of the infant, but can take up to 30 minutes to manifest. These signs are (1) a globular-shaped uterus, (2) a rise of the fundus in the abdomen, (3) a sudden gush or trickle of blood, and (4) further protrusion of the umbilical cord out of the vagina.

A pregnant couple have been notified that their 32-week fetus is dead. The father is yelling at the staff, and his wife is crying uncontrollably. Their 5-year-old daughter is banging the head of her doll on the floor. Which nursing action would be most helpful at this time? 1. Tell the father that his behavior is inappropriate. 2. Sit with the family and quietly communicate sorrow at their loss. 3. Help the couple to understand that their daughter is acting inappropriately. 4. Encourage the couple to send their daughter to her grandparents.

Answer: 2 Explanation: 2. Sitting down for a moment with the woman and her partner and acknowledging the loss in the event of a known demise or impending death will go a long way toward establishing a relationship of trust between the nurse and the parents.

The nurse is working with a new mother who delivered yesterday. The mother has chosen to breastfeed her infant. Which demonstration of skill is the best indicator that the client understands breastfeeding? 1. She puts the infant to breast when he is asleep to help wake him up. 2. She takes off her gown to achieve skin-to-skin contact. 3. She leans toward the infant so that he turns his head to access the nipple. 4. The infant is crying when he is brought to the breast.

Answer: 2 Explanation: 2. Skin-to-skin contact after birth helps the baby maintain his or her body temperature, helps with self-regulation, increases maternal oxytocin levels, helps the mother to notice subtle feeding cues, and promotes bonding.

A woman confides to the nurse that she has pica. What alternative could the nurse suggest to the client? 1. Replace laundry starch with salt. 2. Replace ice with frozen fruit pops. 3. Replace soap with cream cheese. 4. Replace soil with nuts.

Answer: 2 Explanation: 2. Some women are able to switch to frozen fruit pops instead of ice.

A client who is in the second trimester of pregnancy tells the nurse that she has developed a darkening of the line in the midline of her abdomen from the symphysis pubis to the umbilicus. What other expected changes during pregnancy might she also notice? 1. Lightening of the nipples and areolas 2. Reddish streaks called striae on her abdomen 3. A decrease in hair thickness 4. Small purplish dots on her face and arms

Answer: 2 Explanation: 2. Striae, or stretch marks, are reddish, wavy, depressed streaks that may occur over the abdomen, breasts, and thighs as pregnancy progresses.

The nurse is teaching a newborn care class to parents who are about to give birth to their first babies. Which statement by a parent indicates that teaching was effective? 1. "My baby will be able to focus on my face when she is about a month old." 2. "My baby might startle a little if a loud noise happens near him." 3. "Newborns prefer sour tastes." 4. "Our baby won't have a sense of smell until she is older."

Answer: 2 Explanation: 2. Swaddling, placing a hand on the abdomen, or holding the arms to prevent a startle reflex are other ways to soothe the newborn. The settled newborn is then able to attend to and interact with the environment.

The nurse begins a prenatal assessment on a 25-year-old primigravida at 20 weeks' gestation and immediately contacts the healthcare provider because of which finding? 1. Pulse 88/minute 2. Respirations 30/minute 3. Temperature 37.4° C (99.3° F) 4. Blood pressure 118/82

Answer: 2 Explanation: 2. Tachypnea is not a normal finding and requires medical care.

The homecare nurse is examining a newborn who is sleeping on a pillow in a basket, covered with a fluffy blanket. There is also a stuffed animal in the basket. The most important nursing action is to do which of the following? 1. Remove the stuffed animal from the basket and place it on the floor. 2. Teach the parents the risk of SIDS from soft items in the infant's bed. 3. Make certain that the blanket is firmly tucked under the baby. 4. Ask whether the color of the blanket has cultural significance.

Answer: 2 Explanation: 2. Teaching the parents about the risk of sudden infant death syndrome (SIDS) is the highest priority.

The nurse is caring for a jaundiced infant receiving bank light phototherapy in an isolette. Which finding requires an immediate intervention? 1. Eyes are covered, no clothing on, diaper in place 2. Axillary temperature 99.7°F 3. Infant removed from the isolette for breastfeeding 4. Loose bowel movement

Answer: 2 Explanation: 2. Temperature assessment is indicated to detect hypothermia or hyperthermia. Normal temperature ranges are 97.7°F-98.6°F. Vital signs should be monitored every 4 hours with axillary temperatures.

What indications would lead the nurse to suspect sepsis in a newborn? 1. Respiratory distress syndrome developing 48 hours after birth 2. Temperature of 97.0°F 2 hours after warming the infant from 97.4°F 3. Irritability and flushing of the skin at 8 hours of age 4. Bradycardia and tachypnea developing when the infant is 36 hours old

Answer: 2 Explanation: 2. Temperature instability is often seen with sepsis. Fever is rare in a newborn.

A woman has been admitted for an external version. She has completed an ultrasound exam and is attached to the fetal monitor. Prior to the procedure, why will terbutaline be administered? 1. To provide analgesia 2. To relax the uterus 3. To induce labor 4. To prevent hemorrhage

Answer: 2 Explanation: 2. Terbutaline is administered to achieve uterine relaxation.

Which of the following functions primarily to provide low-income women and children who are at risk for medical or nutritional problems with nutritious foods to supplement their diets, nutrition education and counseling, and screening and referrals to other health, welfare, and social programs? 1. ABM 2. WIC 3. ILCA 4. LLLI

Answer: 2 Explanation: 2. The Supplemental Nutrition Program for Women, Infants, and Children (WIC) functions primarily to provide low-income women and children who are at risk for medical or nutritional problems with nutritious foods to supplement their diets, nutrition education and counseling, and screening and referrals to other health, welfare, and social programs.

A laboring client's obstetrician has suggested amniotomy as a method for inducing labor. Which assessment(s) must be made just before the amniotomy is performed? 1. Maternal temperature, BP, and pulse 2. Estimation of fetal birth weight 3. Fetal presentation, position, and station 4. Biparietal diameter

Answer: 3 Explanation: 3. Before an amniotomy is performed, the fetus is assessed for presentation, position, station, and FHR.

The nurse anticipates that the physician will most likely order a cervicovaginal fetal fibronectin test for which client? 1. The client at 34 weeks' gestation with gestational diabetes 2. The client at 32 weeks' gestation with regular uterine contractions 3. The client at 37 weeks' multi-fetal gestation 4. The client at 20 weeks' gestation with ruptured amniotic membranes

Answer: 2 Explanation: 2. The absence of cervicovaginal fFN between 20 and 34 weeks' gestation has been shown to be a strong predictor of a woman not experiencing preterm birth due to spontaneous preterm labor or premature rupture of membranes. Positive findings indicate a 99% probability of birth within the next 2 weeks.

A nurse working in an adolescent clinic that deals with birth control, pregnancy, and referrals for adoption has a number of clients in early adolescence. Clients in that age group are how old? 1. 11-16 years old 2. 11-14 years old 3. 14 and 15 years old 4. 12-15 years old

Answer: 2 Explanation: 2. The age range for early adolescence is age 14 and under.

Of all the clients who have been scheduled to have a biophysical profile, the nurse should check with the physician and clarify the order for which client? 1. A gravida with intrauterine growth restriction 2. A gravida with mild hypotension of pregnancy 3. A gravida who is postterm 4. A gravida who complains of decreased fetal movement for 2 days

Answer: 2 Explanation: 2. The biophysical profile is used when there is a risk of placental and/or fetal compromise. The gravida with mild hypotension will need to be monitored more closely throughout the pregnancy, but is not a candidate at present for a biophysical profile.

On examination of the prenatal client, the nurse is aware that she will assess for a bluish pigmentation of the vagina. What is this objective (probable) sign of pregnancy also known as? 1. Hegar sign 2. Chadwick sign 3. Nightingale sign 4. Goodell sign

Answer: 2 Explanation: 2. The blue-purple discoloration of the cervix is Chadwick sign.

The laboring client is at 7 cm, with the vertex at a +1 station. Her birth plan indicates that she and her partner took Lamaze prenatal classes, and they have planned on a natural, unmedicated birth. Her contractions are every 3 minutes and last 60 seconds. She has used relaxation and breathing techniques very successfully in her labor until the last 15 minutes. Now, during contractions, she is writhing on the bed and screaming. Her labor partner is rubbing the client's back and speaking to her quietly. Which nursing diagnosis should the nurse incorporate into the plan of care for this client? 1. Fear/Anxiety related to discomfort of labor and unknown labor outcome 2. Pain, Acute, related to uterine contractions, cervical dilatation, and fetal descent 3. Coping: Family, Compromised, related to labor process 4. Knowledge, Deficient, related to lack of information about normal labor process and comfort measures

Answer: 2 Explanation: 2. The client is exhibiting signs of acute pain, which is both common and expected in the transitional phase of labor.

Every time the nurse enters the room of a postpartum client who gave birth 3 hours ago, the client asks something else about her birth experience. What action should the nurse take? 1. Answer questions quickly and try to divert her attention to other subjects. 2. Review the documentation of the birth experience and discuss it with her. 3. Contact the physician to warn him the client might want to file a lawsuit, based on her preoccupation with the birth experience. 4. Submit a referral to Social Services because of possible obsessive behavior.

Answer: 2 Explanation: 2. The client may talk about her labor and birth experience. The nurse should provide opportunities to discuss the birth experience in a nonjudgmental atmosphere if the woman desires to do so.

A postpartum client has just received a rubella vaccination. The client demonstrates understanding of the teaching associated with administration of this vaccine when she states which of the following? 1. "I will need another vaccination in 3 months." 2. "I must avoid getting pregnant for 1 month." 3. "This will prevent me from getting chickenpox." 4. "This will protect my newborn from getting the measles."

Answer: 2 Explanation: 2. The client must avoid pregnancy for at least 1 month after receiving the rubella vaccine.

The client presents for cervical ripening in anticipation of labor induction tomorrow. What should the nurse include in her plan of care for this client? 1. Apply an internal fetal monitor. 2. Monitor the client using electronic fetal monitoring. 3. Withhold oral intake and start intravenous fluids. 4. Place the client in a upright, sitting position.

Answer: 2 Explanation: 2. The client should be monitored using electronic fetal monitoring for at least 30 minutes and up to 2 hours after placement to assess the contraction pattern and the fetal status.

The nurse receives a phone call from a client who claims she is pregnant. The client reports that she has regular menses that occur every 28 days and last 5 days. The first day of her last menses was April 10. What would the client's estimated date of delivery (EDD) be if she is pregnant? 1. Nov. 13 2. Jan. 17 3. Jan. 10 4. Dec. 3

Answer: 2 Explanation: 2. The due date is Jan. 17. Nagele's rule is to add 7 days to the last menstrual period and subtract 3 months. The last menstrual period is April 10, therefore Jan. 17 is the EDD.

A client at 36 weeks' gestation is complaining of dyspnea when lying flat. What is the clinical reason for this complaint? 1. Maternal hypertension 2. Fundal height 3. Hydramnios 4. Congestive heart failure

Answer: 2 Explanation: 2. The dyspnea is resulting from the pressure of the enlarging uterus on the diaphragm.

A client at 10 weeks' gestation has developed cholecystitis. If surgery is required, what is the safest time during pregnancy? 1. Immediately, before the fetus gets any bigger 2. Early in the second trimester 3. As close to term as possible 4. The risks are too high to do it anytime in pregnancy

Answer: 2 Explanation: 2. The early second trimester is the best time to operate because there is less risk of spontaneous abortion or early labor, and the uterus is not so large as to impinge on the abdominal field.

The nurse is admitting a client to the birthing unit. What question should the nurse ask to gain a better understanding of the client's psychosocial status? 1. "How did you decide to have your baby at this hospital?" 2. "Who will be your labor support person?" 3. "Have you chosen names for your baby yet?" 4. "What feeding method will you use for your baby?"

Answer: 2 Explanation: 2. The expectant mother's partner or support person is an important member of the birthing team, and assessments of the couple's coping, interactions, and teamwork are integral to the nurse's knowledge base. The nurse's physical presence with the laboring woman provides the best opportunity for ongoing assessment.

A new mother is holding her 2-hour-old son. The delivery occurred on the due date. His Apgar score was 9 at both 1 and 5 minutes. The mother asks the nurse why her son was so wide awake right after birth, and now is sleeping so soundly. What is the nurse's best response? 1. "Don't worry. Babies go through a lot of these little phases." 2. "Your son is in the sleep phase. He'll wake up soon." 3. "Your son is exhausted from being born, and will sleep 6 more hours." 4. "Your breastfeeding efforts have caused excessive fatigue in your son."

Answer: 2 Explanation: 2. The first period of reactivity lasts approximately 30 minutes after birth. During this period the newborn is awake and active and may appear hungry and have a strong sucking reflex. After approximately half an hour, the newborn's activity gradually diminishes, and the heart rate and respirations decrease as the newborn enters the sleep phase. The sleep phase may last from a few minutes to 2 to 4 hours.

A postpartum mother is concerned that her newborn has not had a stool since birth. The newborn is 18 hours old. What is the nurse's best response? 1. "I will call your pediatrician immediately." 2. "Passage of the first stool within 48 hours is normal." 3. "Your newborn might not have a stool until the third day." 4. "Your newborn must be dehydrated."

Answer: 2 Explanation: 2. The first voiding should occur within 24 hours and first passage of stool within 48 hours.

The nurse working in an outpatient obstetric clinic assesses four primigravida clients. Which client findings would the nurse tell the physician about? 1. 17 weeks' gestation and client denies feeling fetal movement 2. 24 weeks' gestation and fundal height is at the umbilicus 3. 4-6 weeks' gestation and softening of the cervix 4. 34 weeks' gestation and complains of hemorrhoidal pain

Answer: 2 Explanation: 2. The fundal height at 24 weeks should be 24 cm. The fundal height is usually at the umbilicus at 20-22 weeks.

In succenturiate placenta, one or more accessory lobes of fetal villi have developed on the placenta, with vascular connections of fetal origin. What is the gravest maternal danger? 1. Cord prolapse 2. Postpartum hemorrhage 3. Paroxysmal hypertension 4. Brachial plexus injury

Answer: 2 Explanation: 2. The gravest maternal danger is postpartum hemorrhage if this minor lobe is severed from the placenta and remains in the uterus.

The client at 38 weeks' gestation has been diagnosed with oligohydramnios. Which statement indicates that teaching about the condition has been effective? 1. "My gestational diabetes might have caused this problem to develop." 2. "When I go into labor, I should come to the hospital right away." 3. "This problem was diagnosed with blood and urine tests." 4. "Women with this condition usually do not have a cesarean birth."

Answer: 2 Explanation: 2. The incidence of cord compression and resulting fetal distress is high when there is an inadequate amount of amniotic fluid. The client with oligohydramnios should come to the hospital in early labor.

The nurse is assessing a 2-hour-old newborn delivered by cesarean at 38 weeks. The amniotic fluid was clear. The mother had preeclampsia. The newborn has a respiratory rate of 80, is grunting, and has nasal flaring. What is the most likely cause of this infant's condition? 1. Meconium aspiration syndrome 2. Transient tachypnea of the newborn 3. Respiratory distress syndrome 4. Prematurity of the neonate

Answer: 2 Explanation: 2. The infant is term and was born by cesarean, and is most likely experiencing transient tachypnea of the newborn.

A fetal weight is estimated at 4490 grams in a client at 38 weeks' gestation. Counseling should occur before labor regarding which of the following? 1. Mother's undiagnosed diabetes 2. Likelihood of a cesarean delivery 3. Effectiveness of epidural anesthesia with a large fetus 4. Need for early delivery

Answer: 2 Explanation: 2. The likelihood of a cesarean delivery with a fetus over 4000 grams is high. This should be discussed with the client before labor.

Dystocia encompasses many problems in labor. What is the most common? 1. Meconium-stained amniotic fluid 2. Dysfunctional uterine contractions 3. Cessation of contractions 4. Changes in the fetal heart rate

Answer: 2 Explanation: 2. The most common problem is dysfunctional (or uncoordinated) uterine contractions that result in a prolongation of labor.

A nurse is evaluating the diet plan of a breastfeeding mother, and determines that her intake of fruits and vegetables is inadequate. The nurse explains that the nutritional composition of the mother's breast milk can be adversely affected by this nutritional inadequacy. Which strategy should the nurse recommend to the mother? 1. Stop breastfeeding 2. Provide newborn supplements to the newborn 3. Offer whole milk 4. Supplement with skim milk

Answer: 2 Explanation: 2. The mother may continue to breastfeed, but the caregiver may choose to prescribe additional vitamins for the newborn. Vitamins in human milk are influenced by the mother's vitamin intake, general nutritional status, and genetic differences.

The nurse assesses a sleeping 1-hour-old, 39-weeks'-gestation newborn. The assessment data that would be of greatest concern would be which of the following? 1. Temperature 97.9°F 2. Respirations 68 breaths/minute 3. Vital signs stable for only 2 hours 4. Heart rate 156 beats/min

Answer: 2 Explanation: 2. The normal respiratory rate is 30-60 breaths/min; 68 breaths/min could represent a less-than-ideal transition.

How does the nurse consider the spiritual needs of a couple experiencing a fetal loss? 1. Explaining the fetal loss in terms of the nurse's own religious beliefs 2. Providing an atmosphere of acceptance regarding the couple's spiritual rites 3. Referring the couple to the hospital chaplain at discharge 4. Informing the couple of religious rituals that have helped other couples to cope with fetal loss

Answer: 2 Explanation: 2. The nurse can facilitate the spiritual needs of the couple by providing an atmosphere of acceptance regarding spiritual rites and encouraging the couple's use of spiritual writings, prayers, and observances.

An infertile couple confides in the nurse at the infertility clinic that they feel overwhelmed with the decisions facing them. Which nursing strategy would be most appropriate? 1. Refer them to a marriage counselor. 2. Provide them with information and instructions throughout the diagnostic and therapeutic process. 3. Express concern and caring. 4. Inquire about the names they have chosen for their baby.

Answer: 2 Explanation: 2. The nurse can provide comfort to couples by offering a sympathetic ear, a nonjudgmental approach, and appropriate information and instruction throughout the diagnostic and therapeutic processes.

The nurse has just assisted the father in bathing the newborn 2 hours after birth. The nurse explains that the newborn must remain in the radiant warmer. This is based on which assessment data? 1. Heart rate 120 2. Temperature 96.8°F 3. Respiratory rate 50 4. Temperature 99.6°F

Answer: 2 Explanation: 2. The nurse rechecks the temperature after the bath and, if it is stable, dresses the newborn in a shirt, diaper, and cap; wraps the baby; and places the baby in an open crib at room temperature. If the baby's axillary temperature is below 36.5°C (97.7°F), the nurse returns the baby to the radiant warmer. The rewarming process should be gradual to prevent the possibility of hyperthermia.

A 26-year-old client is 28 weeks pregnant. She has developed gestational diabetes. She is following a program of regular exercise, which includes walking, bicycling, and swimming. What instructions should be included in a teaching plan for this client? 1. "Exercise either just before meals or wait until 2 hours after a meal." 2. "Carry hard candy (or other simple sugar) when exercising." 3. "If your blood sugar is 120 mg/dL, eat 20 g of carbohydrate." 4. "If your blood sugar is more than 120 mg/dL, drink a glass of whole milk."

Answer: 2 Explanation: 2. The nurse should advise her to carry a simple sugar such as hard candy because of the possibility of exercise-induced hypoglycemia.

A 16-year-old pregnant client is seen at her 10-weeks'-gestation visit. She tells the nurse that she felt the baby move that morning. What response by the nurse is appropriate? 1. "That is very exciting. The baby must be very healthy." 2. "Would you please describe what you felt for me?" 3. "That is impossible. The baby is not big enough yet." 4. "Would you please let me see whether I can feel the baby?"

Answer: 2 Explanation: 2. The nurse should ask the client to describe what she felt, as 10 weeks' gestation is too early to feel fetal movement.

Which statement by a nursing student preparing to care for a postpartum lesbian mother would indicate the need for further teaching? 1. "I can't let the client know I've never worked with lesbian mothers." 2. "I will have to adjust some of my discharge instruction for this mother." 3. "I don't need to include the partner when I provide care and instruction." 4. "Discharge teaching is exactly the same for lesbian mothers as for all others."

Answer: 2 Explanation: 2. The nurse should be aware that standardized postpartum instructions, particularly those related to intercourse and contraception might need to be individualized and amended.

A client is preparing to take a sitz bath for the first time. What will the nurse do? 1. Allow the client privacy during the sitz bath. 2. Place a call bell well within reach and check on the client frequently. 3. Discourage the client from taking a sitz bath. 4. Check on the client after the sitz bath.

Answer: 2 Explanation: 2. The nurse should explain the purpose and use of the sitz bath, anticipated effects, benefits, possible problems, and safety measures to prevent slipping or an injury from hot water. A call bell would be a safety measure.

An HIV-positive mother delivered 2 days ago. The infant will be placed in foster care. The nurse is planning discharge teaching for the foster parents on how to care for the newborn at home. Which instructions should the nurse include? 1. Do not add food supplements to the baby's diet. 2. Place soiled diapers in a sealed plastic bag. 3. Wash soiled linens in cool water with bleach. 4. Shield the baby's eyes from bright lights.

Answer: 2 Explanation: 2. The nurse should instruct the parents about proper hand-washing techniques, about proper disposal of soiled diapers, and to wear gloves when diapering.

The nurse is working with a laboring woman who has a known intrauterine fetal demise. To facilitate the family's acceptance of the fetal loss, after delivery the nurse should do which of the following? 1. Encourage the parents to look at the infant from across the room. 2. Offer the parents the choice of holding the infant in their arms. 3. Take the infant to the morgue immediately. 4. Call family members and inform them of the birth.

Answer: 2 Explanation: 2. The nurse should offer the couple the opportunity to see and hold the infant and reassure the couple that any decision they make for themselves is the right one.

Nurses who are interacting with expectant families from a different culture or ethnic group can provide more effective, culturally sensitive nursing care by doing what? 1. Recognizing that ultimately it is the family's right to make a woman's healthcare choices. 2. Obtaining a medical interpreter of the language the client speaks. 3. Evaluating whether the client's healthcare beliefs have any positive consequences for her health. 4. Accepting personal biases, attitudes, stereotypes, and prejudices.

Answer: 2 Explanation: 2. The nurse should provide for the services of an interpreter if language barriers exist

The pregnant client has asked the nurse what kinds of medications cause birth defects. Which statement would best answer this question? 1. "Birth defects are very rare. Don't worry; your doctor will watch for problems." 2. "To be safe, don't take any medication without talking to your doctor." 3. "Too much vitamin C is one of the most common issues." 4. "Almost all medications will cause birth defects in the first trimester."

Answer: 2 Explanation: 2. The nurse should remind the client of the need to check with her caregiver about medications. If a woman has taken a drug in category D or X, she should be informed of the risks associated with that drug and of her alternatives.

The community nurse is meeting a new mother for the first time. The client delivered her first child 5 days ago after a 12-hour labor. Neither the mother nor the infant had any complications during the birth or postpartum period. Which statement by the client would indicate to the nurse that the client is experiencing postpartum blues? 1. "I am so happy and blessed to have my new baby." 2. "One minute I'm laughing and the next I'm crying." 3. "My husband is helping out by changing the baby at night." 4. "Breastfeeding is going quite well now that the engorgement is gone."

Answer: 2 Explanation: 2. The postpartum blues consist of a transient period of depression that occurs during the first few days of puerperium. Symptoms may include mood swings, anger, weepiness, anorexia, difficulty sleeping, and a feeling of letdown.

A laboring client has received an order for epidural anesthesia. In order to prevent the most common complication associated with this procedure, what would the nurse expect to do? 1. Observe fetal heart rate variability 2. Hydrate the vascular system with 500-1000 mL of intravenous fluids 3. Place the client in the semi-Fowler's position 4. Teach the client appropriate breathing techniques

Answer: 2 Explanation: 2. The risk of hypotension can be minimized by hydrating the vascular system with 500 to 1000 mL of IV solution before the procedure and changing the woman's position and/or increasing the IV rate afterward.

A prenatal educator is asking a partner about normal psychological adjustment of an expectant mother during the second trimester of pregnancy. Which answer by the partner would indicate a typical expectant mother's response to pregnancy? 1. "She is very body-conscious, and hates every little change." 2. "She daydreams about what kind of parent she is going to be." 3. "I haven't noticed anything. I just found out she was pregnant." 4. "She has been having dreams at night about misplacing the baby."

Answer: 2 Explanation: 2. The second trimester brings increased introspection and consideration of how she will parent. She might begin to get furniture and clothing as concrete preparation, and feels movement and is aware of the fetus and incorporates it into herself.

The neonate was born 5 minutes ago. The body is bluish. The heart rate is 150. The infant is crying strongly. The infant cries when the sole of the foot is stimulated. The arms and legs are flexed, and resist straightening. What should the nurse record as this infant's Apgar score? 1. 7 2. 8 3. 9 4. 10

Answer: 2 Explanation: 2. The strong cry earns 2 points. The crying with foot sole stimulation earns 2 points. The limb flexion and resistance earn 2 points each. Bluish color earns 0 points. The Apgar score is 8.

During the nursing assessment of a woman with ruptured membranes, the nurse suspects a prolapsed umbilical cord. What would the nurse's priority action be? 1. To help the fetal head descend faster 2. To use gravity and manipulation to relieve compression on the cord 3. To facilitate dilation of the cervix with prostaglandin gel 4. To prevent head compression

Answer: 2 Explanation: 2. The top priority is to relieve compression on the umbilical cord to allow blood flow to reach the fetus. It is because some obstetric maneuvers to relieve cord compression are complicated that cesarean birth is sometimes necessary.

The client having her second child is scheduled for a cesarean birth because the baby is in a breech presentation. The client states, "I'm wondering what will be different this time compared with my first birth, which was vaginal." What response is best? 1. "We'll take good care of you and your baby. You'll be home before you know it." 2. "You'll be wearing a sequential compression device until you start walking." 3. "You will have a lot of pain, but there are medications that we give when it gets really bad." 4. "You won't be able to nurse until the baby is 12 hours old, because of your epidural."

Answer: 2 Explanation: 2. The use of sequential compression devices (SCDs) and early ambulation are essential to the prevention of deep vein thrombosis, especially if the client had a cesarean birth.

The prenatal clinic nurse is caring for a client with hyperemesis gravidarum at 14 weeks' gestation. The vital signs are: blood pressure 95/48, pulse 114, respirations 24. Which order should the nurse implement first? 1. Weigh the client. 2. Give 1 liter of lactated Ringer's solution IV. 3. Administer 30 mL Maalox (magnesium hydroxide) orally. 4. Encourage clear liquids orally.

Answer: 2 Explanation: 2. The vital signs indicate hypovolemia from dehydration, which leads to hypotension and increased pulse rate. Giving this client a liter of lactated Ringer's solution intravenously will reestablish vascular volume and bring the blood pressure up, and the pulse and respiratory rate down.

The nurse has received the end-of-shift report on the postpartum unit. Which client should the nurse see first? 1. Woman who is 2nd day post-cesarean, moderate lochia serosa 2. Woman day of delivery, fundus firm 2 cm above umbilicus 3. Woman who had a cesarean section, 1st postpartum day, 4 cm diastasis recti abdominis 4. Woman who had a cesarean section, 1st postpartum day, hypoactive bowel sounds all quadrants

Answer: 2 Explanation: 2. This client is the top priority. The fundus should not be positioned above the umbilicus after delivery. If the fundus is in the midline but higher than expected, it is usually associated with clots within the uterus.

The client with blood type O Rh-negative has given birth to an infant with blood type O Rh-positive. The infant has become visibly jaundiced at 12 hours of age. The mother asks why this is happening. What is the best response by the nurse? 1. "The RhoGAM you received at 28 weeks' gestation did not prevent alloimmunization." 2. "Your body has made antibodies against the baby's blood that are destroying her red blood cells." 3. "The red blood cells of your baby are breaking down because you both have type O blood." 4. "Your baby's liver is too immature to eliminate the red blood cells that are no longer needed."

Answer: 2 Explanation: 2. This explanation is accurate and easy for the client to understand. Newborns of Rh-negative and O blood type mothers are carefully assessed for blood type status, appearance of jaundice, and levels of serum bilirubin.

The adolescent client reports to the clinic nurse that her period is late, but that her home pregnancy test is negative. What should the nurse explain that these findings most likely indicate? 1. "This means you are not pregnant." 2. "You might be pregnant, but it might be too early for your home test to be accurate." 3. "We don't trust home tests. Come to the clinic for a blood test." 4. "Most people don't use the tests correctly. Did you read the instructions?"

Answer: 2 Explanation: 2. This is a true statement. Most home pregnancy tests have low false-positive rates, but the false-negative rate is slightly higher. Repeating the test in a week is recommended.

The client at 40 weeks' gestation reports to the nurse that she has had increased pelvic pressure and increased urinary frequency. Which response by the nurse is best? 1. "Unless you have pain with urination, we don't need to worry about it." 2. "These symptoms usually mean the baby's head has descended further." 3. "Come in for an appointment today and we'll check everything out." 4. "This might indicate that the baby is no longer in a head-down position."

Answer: 2 Explanation: 2. This is the best response because it most directly addresses what the client has reported.

At birth, an infant weighed 8 pounds 4 ounces. Three days later, the newborn is being discharged. The parents note that the baby now weighs 7 pounds 15 ounces. The nurse explains that the change in the newborn's weight is which of the following? 1. Excessive 2. Within normal limits 3. Less than expected 4. Unusual

Answer: 2 Explanation: 2. This newborn's weight loss is within normal limits. A weight loss of up to 10% for term newborns is considered within normal limits during the first week of life.

The prenatal clinic nurse is designing a new prenatal intake information form for pregnant clients. Which question is best to include on this form? 1. Where was the father of the baby born? 2. Do genetic diseases run in the family of the baby's father? 3. What is the name of the baby's father? 4. Are you married to the father of the baby?

Answer: 2 Explanation: 2. This question has the highest priority because it gets at the physiologic issue of inheritable genetic diseases that might directly impact the baby.

The nurse is working with a pregnant adolescent. The client asks the nurse how the baby's condition is determined during labor. The nurse's best response is that during labor, the nurse will do which of the following? 1. Check the client's cervix by doing a pelvic exam every 2 hours. 2. Assess the fetus's heart rate with an electronic fetal monitor. 3. Look at the color and amount of bloody show that the client has. 4. Verify that the client's contractions are strong but not too close together.

Answer: 2 Explanation: 2. This statement best answers the question the client has asked.

The nurse is caring for a 15-year-old who just delivered a 32-weeks'-gestation stillborn infant with numerous defects. In caring for this client, the nurse knows which of the following? 1. The client will likely do no grieving, as she is so young and the pregnancy was probably a mistake in any case. 2. Adolescents have a sense of invulnerability, an "It can't happen to me" mentality. 3. The client's mother will handle her daughter's grief, so the nurse doesn't need to be concerned. 4. The nurse will remove the baby before the client sees it.

Answer: 2 Explanation: 2. Though adolescents have a mature concept of death, it is often clouded by their sense of invulnerability, an "It can't happen to me" mentality.

The nurse is making a postpartum home visit in the summer. The new father asks about taking the baby to a family outing this weekend. The nurse should encourage the father to do which of the following? 1. Cover the infant with dark blankets to block the sun. 2. Keep the infant in the shade. 3. Uncover the infant's head to prevent hyperthermia. 4. Avoid taking the infant outdoors for 6 months.

Answer: 2 Explanation: 2. To prevent sunburn, the newborn should remain shaded, wear a light layer of clothing, or be protected with sunscreen specifically formulated for infants.

Before applying a cord clamp, the nurse assesses the umbilical cord. The mother asks why the nurse is doing this. What should the nurse reply? 1. "I'm checking the blood vessels in the cord to see whether it has one artery and one vein." 2. "I'm checking the blood vessels in the cord to see whether it has two arteries and one vein." 3. "I'm checking the blood vessels in the cord to see whether it has two veins and one artery." 4. "I'm checking the blood vessels in the cord to see whether it has two arteries and two veins."

Answer: 2 Explanation: 2. Two arteries and one vein are present in a normal umbilical cord.

The nurse is anticipating the arrival of a couple in the labor unit. It has been determined that the 37-week fetus has died in utero from unknown causes. What should the nurse include in the plan of care for this couple? 1. Allow the couple to adjust to the labor unit in the waiting area. 2. Place the couple in a labor room at the end of the hall with an empty room next door. 3. Encourage the father to go home and rest for a few hours. 4. Contact the mother's emergency contact person and explain the situation.

Answer: 2 Explanation: 2. Upon arrival to the facility, the couple with a known or suspected fetal demise should immediately be placed in a private room. When possible, the woman should be in a room that is farthest away from other laboring women.

The client has been pushing for two hours, and is exhausted. The fetal head is visible between contractions. The physician informs the client that a vacuum extractor could be used to facilitate the delivery. Which statement indicates that the client needs additional information about vacuum extraction assistance? 1. "A small cup will be put onto the baby's head, and a gentle suction will be applied." 2. "I can stop pushing and just rest if the vacuum extractor is used." 3. "The baby's head might have some swelling from the vacuum cup." 4. "The vacuum will be applied for a total of ten minutes or less."

Answer: 2 Explanation: 2. Vacuum extraction is an assistive delivery. The physician/CNM applies traction in coordination with uterine contractions.

The school nurse is planning a class about nutrition for pregnant teens, several of whom have been diagnosed with iron-deficiency anemia. In order to increase iron absorption, the nurse would encourage the teens to consume more of what beverage? 1. Gatorade 2. Orange juice 3. Milk 4. Green tea

Answer: 2 Explanation: 2. Vitamin C is found in citrus fruits and juices, and is known to enhance the absorption of iron from meat and non-meat sources.

A pregnant teenage client is diagnosed with iron-deficiency anemia. Which nutrient should the nurse encourage her to take to increase iron absorption? 1. Vitamin A 2. Vitamin C 3. Vitamin D 4. Vitamin E

Answer: 2 Explanation: 2. Vitamin C is known to enhance the absorption of iron from meat and nonmeat sources.

A woman is 32 weeks pregnant. She is HIV-positive but asymptomatic. The nurse knows what would be important in managing her pregnancy and delivery? 1. An amniocentesis at 30 and 36 weeks 2. Weekly non-stress testing beginning at 32 weeks' gestation 3. Application of a fetal scalp electrode as soon as her membranes rupture in labor 4. Administration of intravenous antibiotics during labor and delivery

Answer: 2 Explanation: 2. Weekly non-stress testing (NST) is begun at 32 weeks' gestation and serial ultrasounds are done to detect IUGR.

The postpartum client expresses concern about getting back to her prepregnant shape, and asks the nurse when she will be able to run again. Which statement by the client indicates that teaching was effective? 1. "I can start running in 2 weeks, and can breastfeed as soon as I am done." 2. "I should see how my energy level is at home, and increase my activity slowly." 3. "Running is not recommended for breastfeeding women." 4. "If I am getting 8 hours of sleep per day, I can start running."

Answer: 2 Explanation: 2. Women should be encouraged to limit the number of activities to prevent excessive fatigue, increase in lochia, and negative psychologic reactions, such as feeling overwhelmed. A regular exercise program including vigorous activities such as running, weight lifting, or competitive sports can usually be initiated after the 6-week postpartum examination or when approved by the client's physician/CNM.

The nurse is doing preconception counseling with a 28-year-old woman with no prior pregnancies. Which statement made by the client indicates to the nurse that the client has understood the teaching? 1. "I can continue to drink alcohol until I am diagnosed as pregnant." 2. "I need to stop drinking alcohol completely when I start trying to get pregnant." 3. "A beer once a week will not damage the fetus." 4. "I can drink alcohol while breastfeeding because it doesn't pass into breast milk."

Answer: 2 Explanation: 2. Women should discontinue drinking alcohol when they start to attempt to become pregnant due to possible effects of alcohol on the fetus.

Women with HIV should be evaluated and treated for other sexually transmitted infections and for what condition occurring more commonly in women with HIV? 1. Syphilis 2. Toxoplasmosis 3. Gonorrhea 4. Herpes

Answer: 2 Explanation: 2. Women with HIV should be evaluated and treated for other sexually transmitted infections and for conditions occurring more commonly in women with HIV, such as tuberculosis, cytomegalovirus, toxoplasmosis, and cervical dysplasia.

A multiparous client delivered her first child vaginally 2 years ago, and delivered an infant by cesarean yesterday due to breech presentation. Which statement would the nurse expect the client to make? 1. "I can't believe how much more tired I was with the first baby." 2. "I'm having significantly more pain this time than with my last birth." 3. "It is disappointing that I can't breastfeed because of the cesarean." 4. "Getting in and out of bed feels more comfortable than last time."

Answer: 2 Explanation: 2. Women with cesarean births have special needs: increased need for rest and sleep; incisional care; self-care; and relief of pain and discomfort.

The nurse is planning an in-service educational program to talk about disseminated intravascular coagulation (DIC). The nurse should identify which conditions as risk factors for developing DIC? Note: Credit will be given only if all correct choices and no incorrect choices are selected. Select all that apply. 1. Diabetes mellitus 2. Abruptio placentae 3. Fetal demise 4. Multiparity 5. Preterm labor

Answer: 2, 3 Explanation: 2. As a result of the damage to the uterine wall and the retroplacental clotting with covert abruption, large amounts of thromboplastin are released into the maternal blood supply, which in turn triggers the development of disseminated intravascular coagulation (DIC) and the resultant hypofibrinogenemia. 3. Perinatal mortality associated with abruptio placentae is approximately 25%. If fetal hypoxia progresses unchecked, irreversible brain damage or fetal demise may result.

In planning a conference for adolescents, the nurse researches the most common socioeconomic and cultural factors that contribute to adolescent pregnancy. Which true statements should be included in this conference? Note: Credit will be given only if all correct choices and no incorrect choices are selected. Select all that apply. 1. Economic status has a relatively low impact on an adolescent's sexual activity. 2. Nearly three quarters of all teens ages 15-19 have had sex at least once. 3. Poor self-esteem contributes to adolescent pregnancy. 4. 38% percent of sexually active students use a condom. 5. Sexual abuse is not an issue with teen mothers.

Answer: 2, 3 Explanation: 2. By 19 years of age, 70% of all teens have had intercourse. 3. This is a true statement. Poor self-esteem is a major contributing factor in adolescent pregnancy.

A client has just arrived in the birthing unit. What steps would be most important for the nurse to perform to gain an understanding of the physical status of the client and her fetus? Note: Credit will be given only if all correct choices and no incorrect choices are selected. Select all that apply. 1. Check for ruptured membranes and apply a fetal scalp electrode. 2. Auscultate the fetal heart rate between and during contractions. 3. Palpate contractions and resting uterine tone. 4. Assess the blood pressure, temperature, respiratory rate, and pulse rate. 5. Perform a vaginal exam for cervical dilation, and perform Leopold maneuvers.

Answer: 2, 3 Explanation: 2. Fetal heart rate auscultation gives information about the physical status of the fetus. 3. Contraction palpation provides information about the frequency, duration, and intensity of the contractions.

The postpartum client is suspected of having acute cystitis. Which symptoms would the nurse expect to see in this client? Note: Credit will be given only if all correct choices and no incorrect choices are selected. Select all that apply. 1. High fever 2. Frequency 3. Suprapubic pain 4. Chills 5. Nausea and vomiting

Answer: 2, 3 Explanation: 2. Frequency is characteristic of acute cystitis. 3. Suprapubic pain is characteristic of acute cystitis.

The nurse is working with a new mother who follows Muslim traditions. Which expectations and actions are appropriate for this client? Note: Credit will be given only if all correct choices and no incorrect choices are selected. Select all that apply. 1. To be sure she gets a kosher diet. 2. Expect that most visitors will be women. 3. Uncover only the necessary skin when assessing. 4. The father will take an active role in infant care. 5. She will prefer a male physician.

Answer: 2, 3 Explanation: 2. In Muslim cultures, emphasis on childrearing and infant care activities is on the mother and female relatives. 3. Women of the Islamic faith may have specific modesty requirements; the woman must be completely covered, with only her feet and hands exposed.

The nurse is assessing a newborn diagnosed with physiologic jaundice. Which findings would the nurse expect? Note: Credit will be given only if all correct choices and no incorrect choices are selected. Select all that apply. 1. Jaundice present within the first 24 hours of life 2. Appearance of jaundice symptoms after 24 hours of life 3. Yellowish coloration of the sclera of the eyes 4. Cephalohematoma or excessive bruising 5. Cyanosis

Answer: 2, 3 Explanation: 2. Physiologic or neonatal jaundice is a normal process that occurs during transition from intrauterine to extrauterine life and appears after 24 hours of life. 3. Jaundice is a yellowish coloration of the skin and sclera of the eyes that develops from the deposit of yellow pigment bilirubin in lipid/fat-containing tissues.

The nurse is planning an in-service presentation about perinatal loss. Which statements should the nurse include in this presentation? Note: Credit will be given only if all correct choices and no incorrect choices are selected. Select all that apply. 1. "Perinatal loss refers to third-trimester fetal death in utero." 2. "Perinatal loss occurs more frequently in assisted reproduction." 3. "Perinatal loss rates have declined in the United States over the past few years." 4. "Perinatal loss includes 25% of stillbirths occurring before the onset of labor." 5. "Perinatal loss rarely causes an emotional problem for the family."

Answer: 2, 3 Explanation: 2. Pregnancies conceived by in vitro fertilization have higher rates of pregnancy loss and pregnancy complications. 3. Perinatal loss in industrialized countries has declined in recent years as early diagnosis of congenital anomalies and advances in genetic testing techniques have increased the use of elective termination.

) The client presents to the clinic for an initial prenatal examination. She asks the nurse whether there might be a problem for her baby because she has type B Rh-positive blood and her husband has type O Rh-negative blood, or because her sister's baby had ABO incompatibility. What is the nurse's best answer? Note: Credit will be given only if all correct choices and no incorrect choices are selected. Select all that apply. 1. "Your baby would be at risk for Rh problems if your husband were Rh-negative." 2. "Rh problems only occur when the mother is Rh-negative and the father is not." 3. "ABO incompatibility occurs only after the baby is born." 4. "We don't know for sure, but we can test for ABO incompatibility." 5. "Your husband's being type B puts you at risk for ABO incompatibility."

Answer: 2, 3 Explanation: 2. Rh incompatibility is a possibility when the mother is Rh-negative and the father is Rh-positive. 3. ABO incompatibility is limited to type O mothers with a type A or B fetus and occurs after the baby is born.

Under which circumstances would the nurse remove prostaglandin from the client's cervix? Note: Credit will be given only if all correct choices and no incorrect choices are selected. Select all that apply. 1. Contractions every 5 minutes 2. Nausea and vomiting 3. Uterine tachysystole 4. Cardiac tachysystole 5. Baseline fetal heart rate of 140-148

Answer: 2, 3, 4 Explanation: 2. A reason to remove prostaglandin from a client's cervix is the presence of nausea and vomiting. 3. A reason to remove prostaglandin from a client's cervix is uterine tachysystole. 4. A reason to remove prostaglandin from a client's cervix is cardiac tachysystole.

The nurse is preparing an in-service presentation for a group of middle-school nurses. Which statements by the nurse would indicate that the middle-school nurse understood the role of culture in adolescent pregnancy? Note: Credit will be given only if all correct choices and no incorrect choices are selected. Select all that apply. 1. "Studies show that 85% of teen mothers are middle class, and give birth to gain adult status." 2. "Teenage pregnancy is the leading reason why adolescent women drop out of school." 3. "Teens of color are more likely to become pregnant." 4. "Intelligence and academic ability are positively associated with delayed sexual activity, greater use of contraception, and lower rates of pregnancy." 5. "Over 50% of adolescents who have had an abortion or recent birth become pregnant again within two years."

Answer: 2, 3, 4 Explanation: 2. An estimated 30% to 40% of female teenage dropouts are mothers. 3. In the United States, the adolescent birth rate is higher among African American and Hispanic teens than among white teens. To some degree, the higher teenage pregnancy rate in these groups reflects the impact of poverty because a disproportionately higher number of African American and Hispanic youths live in poverty. 4. Teens with future goals (i.e., college or job) tend to use birth control more consistently compared with other teens; if they become pregnant, they are also more likely to have abortions.

The nurse recognizes that subjective pregnancy changes such as amenorrhea can be caused by which conditions? Note: Credit will be given only if all correct and no incorrect choices are selected. Select all that apply. 1. Goodell sign 2. Anemia 3. Pseudocyesis 4. Thyroid dysfunction 5. Fetal heartbeat

Answer: 2, 3, 4 Explanation: 2. Anemia can cause amenorrhea, and is a subjective sign of pregnancy. 3. Pseudocyesis (intense desire for pregnancy) can cause amenorrhea. 4. Thyroid dysfunction can cause amenorrhea, and is a subjective sign of pregnancy.

Which instructions should the nurse include when teaching parents of a newborn about caring for the umbilical cord? Note: Credit will be given only if all correct choices and no incorrect choices are selected. Select all that apply. 1. Use triple-dye to cleanse the umbilical cord at home. 2. Fold the diaper down to prevent covering the cord stump. 3. Keep the umbilical stump clean and dry to avoid infection. 4. Observe for signs of infection such as foul smell, redness, and drainage. 5. Begin tub baths to help cleanse the cord stump at home.

Answer: 2, 3, 4 Explanation: 2. Folding the diaper down to prevent coverage of the cord stump can prevent contamination of the area and promote drying. 3. Keeping the umbilical stump clean and dry can reduce the risk of infection. 4. It is the nurse's responsibility to instruct parents in caring for the cord and observing for signs and symptoms of infection after discharge, such as foul smell, redness and greenish yellow drainage, localized heat and tenderness, or bright red bleeding or if the area remains unhealed 2 to 3 days after the cord has sloughed off.

When caring for a new mother after cesarean birth, what complications would the nurse anticipate? Note: Credit will be given only if all correct choices and no incorrect choices are selected. Select all that apply. 1. Back pain 2. Pulmonary infection 3. Deep vein thrombosis 4. Pulmonary embolism 5. Perineal edema

Answer: 2, 3, 4 Explanation: 2. Immobility after delivery increases the risk of pulmonary infection. 3. Immobility after delivery increases the risk of deep vein thrombosis. 4. Immobility after delivery increases the risk of pulmonary embolism.

The nurse is caring for a client who delivered by cesarean birth. The client received a general anesthetic. To prevent or minimize abdominal distention, which of the following would the nurse encourage? Note: Credit will be given if all correct choices and no incorrect choices are selected. Select all that apply. 1. Increased intake of cold beverages 2. Leg exercises every 2 hours 3. Abdominal tightening 4. Ambulation 5. Using a straw when drinking fluids

Answer: 2, 3, 4 Explanation: 2. Immobility increases the risk of abdominal distention and discomfort. Leg exercises serve to prevent or minimize abdominal distention in a surgical client who received a general anesthetic. 3. Abdominal tightening serves to prevent or minimize abdominal distention in a surgical client who received a general anesthetic. 4. Early ambulation prevents abdominal distention that can occur with excess accumulation of gas in the intestines.

Prior to conducting the initial assessment of a newborn, the nurse reviews the mother's prenatal record and the delivery record to obtain information concerning possible risk factors for the infant and to anticipate the impact of these factors on the infant's ability to successfully transition to the extrauterine environment. Which information is pertinent to this assessment? Note: Credit will be given only if all correct choices and no incorrect choices are selected. Select all that apply. 1. Drug or alcohol use by the father 2. Infectious disease screening results 3. Maternal history of gestational diabetes 4. Prolonged rupture of the membranes 5. Maternal use of prenatal vitamins

Answer: 2, 3, 4 Explanation: 2. Infectious disease screening results help to determine if the infant is also at risk of obtaining any infectious diseases. 3. Gestational diabetes is a risk factor for the newborn. 4. Prolonged rupture of the membranes is a possible risk factor for the infant.

The nurse is seeing a client who asks about the accuracy of Nagele's rule. The nurse explains that accuracy can be compromised under which conditions? Note: Credit will be given only if all correct and no incorrect choices are selected. Select all that apply. 1. There is a history of regular menses every 28 days. 2. Amenorrhea is present and ovulation occurs with breastfeeding. 3. Oral contraception was discontinued, but no regular menstruation was established. 4. There has been 1 or more months of amenorrhea. 5. There is an accurate date for the last menstrual period.

Answer: 2, 3, 4 Explanation: 2. Nagele's rule is not always accurate for women who have amenorrhea but are ovulating and conceive while breastfeeding. 3. Nagele's rule is not always accurate for women who conceive before regular menstruation is established following discontinuation of oral contraceptives or termination of a pregnancy. 4. Nagele's rule is not always accurate for women with markedly irregular periods that include 1 or more months of amenorrhea.

The nurse knows that which of the following are advantages of spinal block? Note: Credit will be given only if all correct answers and no incorrect answers are selected. Select all that apply. 1. Intense blockade of sympathetic fibers 2. Relative ease of administration 3. Maternal compartmentalization of the drug 4. Immediate onset of anesthesia 5. Larger drug volume

Answer: 2, 3, 4 Explanation: 2. One of advantages of spinal block is the relative ease of administration. 3. One of the advantages of spinal block is the maternal compartmentalization of the drug. 4. One of the advantages of spinal block is the immediate onset of anesthesia.

Absolute contraindications to exercise while pregnant include which of the following? Note: Credit will be given only if all correct and no incorrect choices are selected. Select all that apply. 1. Abruptio placentae 2. Placenta previa after 26 weeks' gestation 3. Preeclampsia-eclampsia 4. Cervical insufficiency (cerclage) 5. Intrauterine growth restriction (IUGR)

Answer: 2, 3, 4 Explanation: 2. Placenta previa after 26 weeks' gestation is an absolute contraindication to exercise. 3. Preeclampsia-eclampsia is an absolute contraindication to exercise. 4. Cervical insufficiency (cerclage) is an absolute contraindication to exercise.

The nurse is preparing a brochure for couples considering pregnancy after the age of 35. Which statements should be included? Note: Credit will be given only if all correct choices and no incorrect choices are selected. Select all that apply. 1. There is a decreased risk of Down syndrome. 2. Preexisting medical conditions can complicate pregnancy. 3. Preterm births are more common. 4. Amniocentesis can be performed to detect genetic anomalies. 5. The increased fertility of women over age 35 makes conception easier.

Answer: 2, 3, 4 Explanation: 2. Preexisting medical conditions, such as hypertension or diabetes, probably play a more significant role than age in maternal well-being and the outcome of pregnancy. 3. The incidence of low-birth-weight infants, preterm births, miscarriage, stillbirth, and perinatal morbidity and mortality is higher among women age 35 or older. 4. Amniocentesis is offered to all women over age 35 to permit the early detection of several chromosomal abnormalities, including Down syndrome; noninvasive analysis of fetal nucleic acid is now commonly recommended to women of advanced maternal age.

A pregnant client complains to the clinic nurse that her varicose veins are causing more discomfort than before. Which recommendations does the nurse make to the client? Note: Credit will be given only if all correct and no incorrect choices are selected. Select all that apply. 1. Increase the time she stands. 2. Walk on a daily basis. 3. Not cross her legs at the knees. 4. Wear support hose. 5. Hyperextend her knee with her feet up.

Answer: 2, 3, 4 Explanation: 2. Regular exercise, such as swimming, cycling, or walking, promotes venous return, which helps prevent varicosities. 3. The client should avoid crossing her legs at the knees because of the pressure it puts on her veins. 4. Supportive hose or elastic stockings may be extremely helpful.

Nonreassuring fetal status often occurs with a tachysystole contraction pattern. Intrauterine resuscitation measures may become warranted and can include which of the following measures? Note: Credit will be given only if all correct choices and no incorrect choices are selected. Select all that apply. 1. Position the woman on her right side. 2. Apply oxygen via face mask. 3. Call for anesthesia provider for support. 4. Increase intravenous fluids by at least 700 mL bolus. 5. Call the physician/CNM to the bedside.

Answer: 2, 3, 4 Explanation: 2. The nurse would apply oxygen via face mask. 3. The nurse would call for anesthesia provider for support. 4. The nurse would increase intravenous fluids by at least 500 mL bolus.

The nurse knows that the maternal risks associated with postterm pregnancy include which of the following? Note: Credit will be given only if all correct choices and no incorrect choices are selected. Select all that apply. 1. Polyhydramnios 2. Maternal hemorrhage 3. Maternal anxiety 4. Forceps-assisted delivery 5. Perineal damage

Answer: 2, 3, 4, 5 Explanation: 2. Maternal symptoms and complications in postterm pregnancy may include maternal hemorrhage. 3. Maternal symptoms and complications in postterm pregnancy may include maternal anxiety. 4. Maternal symptoms and complications in postterm pregnancy may include an operative vaginal birth with forceps or vacuum extractor. 5. Maternal symptoms and complications in postterm pregnancy may include perineal trauma and damage.

The nurse is caring for a client who plans to relinquish her baby for adoption. The nurse would implement which approach to care? Note: Credit will be given only if all correct choices and no incorrect choices are selected. Select all that apply. 1. Encourage the client to see and hold her infant. 2. Encourage the client to express her emotions. 3. Respect any special requests for the birth. 4. Acknowledge the grieving process in the client. 5. Allow access to the infant, if the client requests it.

Answer: 2, 3, 4, 5 Explanation: 2. The mother who decides to relinquish her baby needs emotional support and validation of her loss. 3. The woman should decide whether to see and hold her baby and should have any special requests regarding the birth honored. 4. Perinatal nurses should be aware that relinquishing mothers are at risk for disenfranchised grief, in which they are unable to proceed through the grieving process and come to resolution with the loss. The nurse should acknowledge the woman's loss and support her decision. 5. The amount of contact she chooses to have with her newborn should be respected.

The nurse is teaching new parents how to dress their newborn. Which statements indicate that teaching has been effective? Note: Credit will be given only if all correct choices and no incorrect choices are selected. Select all that apply. 1. "We should keep our home air-conditioned so the baby doesn't overheat." 2. "It is important that we dry the baby off as soon as we give him a bath or shampoo his hair." 3. "When we change the baby's diaper, we should change any wet clothing or blankets, too." 4. "If the baby's body temperature gets too low, he will warm himself up without any shivering." 5. "Our baby will have a much faster rate of breathing if he is not dressed warmly enough."

Answer: 2, 3, 4, 5 Explanation: 2. The newborn is particularly prone to heat loss by evaporation immediately after birth and during baths; thus drying the newborn is critical. 3. Changing wet clothing or blankets immediately prevents evaporation, one mechanism of heat loss. 4. Nonshivering thermogenesis (NST), an important mechanism of heat production unique to the newborn, is the major mechanism through which heat is produced. 5. A decrease in the environmental temperature of 2°C is a drop sufficient to double the oxygen consumption of a term newborn and can cause the newborn to show signs of respiratory distress.

Which of the following potential problems would the nurse consider when planning care for a client with a persistent occiput posterior position of the fetus? Note: Credit will be given only if all correct choices and no incorrect choices are selected. Select all that apply. 1. Increased fetal mortality 2. Severe perineal lacerations 3. Ceasing of labor progress 4. Fetus born in posterior position 5. Intense back pain during labor

Answer: 2, 3, 4, 5 Explanation: 2. The woman can have third- or fourth-degree perineal laceration or extension of a midline episiotomy. 3. Sometimes labor progress ceases if the fetus fails to rotate to an occiput anterior position. 4. Occiput posterior positions are associated with a higher incidence of vacuum-assisted births. 5. The woman usually experiences intense back pain in the small of her back throughout labor.

Which nursing interventions are appropriate when caring for the newborn undergoing phototherapy? Note: Credit will be given only if all correct choices and no incorrect choices are selected. Select all that apply. 1. Cover the newborn's eyes at all times, even when not under the lights. 2. Close the newborn's eyelids before applying eye patches. 3. Inspect the eyes each shift for conjunctivitis. 4. Keep the baby swaddled in a blanket to prevent heat loss. 5. Reposition the baby every 2 hours.

Answer: 2, 3, 5 Explanation: 2. Apply eye patches over the newborn's closed eyes during exposure to banks of phototherapy. 3. Discontinue conventional phototherapy and remove the eye patches at least once per shift to assess the eyes for the presence of conjunctivitis. 5. Repositioning allows equal exposure of all skin areas and prevents pressure areas.

When assessing a full-term newborn, the nurse notes tremorlike movements. The nurse is aware that further evaluation is indicated to rule out which of the following? Note: Credit will be given only if all correct choices and no incorrect choices are selected. Select all that apply. 1. Hyperglycemia 2. Hypoglycemia 3. Hypocalcemia 4. Substance withdrawal 5. Neurologic damage

Answer: 2, 3, 4, 5 Explanation: 2. Tremors or jitteriness (tremorlike movements) in the full-term newborn must be evaluated to differentiate the tremors from convulsions. Tremors may be related to hypoglycemia, hypocalcemia, or substance withdrawal. 3. Tremors or jitteriness (tremorlike movements) in the full-term newborn must be evaluated to differentiate the tremors from convulsions. Tremors may be related to hypoglycemia, hypocalcemia, or substance withdrawal. 4. Tremors or jitteriness (tremorlike movements) in the full-term newborn must be evaluated to differentiate the tremors from convulsions. Tremors may be related to hypoglycemia, hypocalcemia, or substance withdrawal. 5. Neurologic damage should be considered if the newborn is experiencing tremors.

The nurse is aware of the different breathing techniques that are used during labor. Why are breathing techniques used during labor? Note: Credit will be given only if all correct choices and no incorrect choices are selected. Select all that apply. 1. They are a form of anesthesia. 2. They are a source of relaxation. 3. They increase the ability to cope with contractions. 4. They are a source of distraction. 5. They increase a woman's pain threshold.

Answer: 2, 3, 4, 5 Explanation: 2. When used correctly, breathing techniques can encourage relaxation. 3. When used correctly, breathing techniques can enhance the ability to cope with uterine contractions. 4. When used correctly, breathing techniques provide some distraction from the pain. 5. When used correctly, breathing techniques increase a woman's pain threshold.

What signs would indicate that a pregnant client's urinalysis culture was abnormal? Note: Credit will be given only if all correct and no incorrect choices are selected. Select all that apply. 1. pH 4.6-8 2. Alkaline urine 3. Cloudy appearance 4. Negative for protein and red blood cells 5. Hemoglobinuria

Answer: 2, 3, 5 Explanation: 2. Alkaline urine could indicate metabolic alkalemia, Proteus infection, or an old specimen. 3. A cloudy appearance could indicate an infection. 5. Hemoglobinuria would be indicated by an abnormal urine color.

Which of the following symptoms would be an indication of postpartum blues? Note: Credit will be given only if all correct choices and no incorrect choices are selected. Select all that apply. 1. Overeating 2. Anger 3. Mood swings 4. Constant sleepiness 5. Crying

Answer: 2, 3, 5 Explanation: 2. Anger would be a symptom of postpartum blues. 3. Mood swings would be a symptom of postpartum blues. 5. Weepiness and crying would be a symptom of postpartum blues.

Which of the following would be considered normal newborn urinalysis values? Note: Credit will be given only if all correct choices and no incorrect choices are selected. Select all that apply. 1. Color bright yellow 2. Bacteria 0 3. Red blood cells (RBC) 0 4. White blood cells (WBC) more than 4-5/hpf 5. Protein less than 5-10 mg/dL

Answer: 2, 3, 5 Explanation: 2. Bacteria value should be 0. 3. Red blood cells (RBC) should be 0. 5. Protein less than 5-10 mg/dL would be considered normal.

The clinic nurse is teaching a pregnant client about her iron supplement. Which information is included in the teaching? Note: Credit will be given only if all correct and no incorrect choices are selected. Select all that apply. 1. Iron does not affect the gastrointestinal tract. 2. A stool softener might be needed. 3. Start a low dose, and increase it gradually. 4. Expect the stools to be black and bloody. 5. Iron absorption is poor if taken with meals.

Answer: 2, 3, 5 Explanation: 2. Constipation can be a problem when taking iron, so a stool softener might be needed. 3. To prevent anemia, experts recommend that all pregnant women start on 30 mg/day of iron supplements daily. If anemia is diagnosed, the dosage should be increased to 60 to 120 mg per day of iron. 5. Iron absorption is reduced by 40% to 50% if taken with meals.

The nurse is working with a community group to reduce teen pregnancy rates. Alternatives for the group to consider include which of the following? Note: Credit will be given only if all correct and no incorrect choices are selected. Select all that apply. 1. Avoidance of sex education in schools 2. High-quality reproductive health services 3. Developmentally appropriate sex education 4. Sexuality curriculum based on community beliefs 5. Provision of strategies to enhance life skills

Answer: 2, 3, 5 Explanation: 2. Having access to high-quality reproductive health services is important in reducing adolescent pregnancy rates. 3. Providing developmentally appropriate sex education is an approach that reduces adolescent pregnancy. 5. Programs that provide strategies to enhance life skills are more successful in reducing adolescent pregnancy rates.

During the initial prenatal visit, the nurse assesses the history of the father of the child for which of the following? Note: Credit will be given only if all correct and no incorrect choices are selected. Select all that apply. 1. Stability of living conditions 2. Blood type and Rh type 3. Significant health problems 4. Nutritional history 5. Current use of tobacco

Answer: 2, 3, 5 Explanation: 2. The father of the fetus should be assessed for blood type and Rh factor. 3. The father of the fetus should be assessed for significant health problems. 5. The father of the fetus should be assessed for current alcohol intake, drug use, and tobacco use.

A NICU nurse plans care for a preterm newborn that will provide opportunities for development. Which interventions support development in a preterm newborn in a NICU? Note: Credit will be given only if all correct choices and no incorrect choices are selected. Select all that apply. 1. Schedule care throughout the day. 2. Silence alarms quickly. 3. Place a blanket over the top portion of the incubator. 4. Do not offer a pacifier. 5. Dim the lights.

Answer: 2, 3, 5 Explanation: 2. Noise levels can be lowered by replacing alarms with lights or silencing alarms quickly. 3. Dimmer switches should be used to shield the baby's eyes from bright lights with blankets over the top portion of the incubator. 5. Dimming the lights may encourage infants to open their eyes and be more responsive to their parents.

The nurse is evaluating the goal "Client will remain free of opportunistic infections" for an HIV-positive pregnant client. The nurse determines the goal was met when the client has which of the following? Note: Credit will be given only if all correct and no incorrect choices are selected. Select all that apply. 1. An absolute CD4+ T-lymphocyte count below 200 2. No complaint of chills or fever during the pregnancy 3. Weight gain of 30 lbs during the pregnancy 4. ESR above 20 mm/hr 5. Normal erythrocyte sedimentation rate maintained during the pregnancy

Answer: 2, 3, 5 Explanation: 2. Not having chills, fever, or a sore throat throughout the pregnancy is an indication the client did not have an infection. 3. Weight gain of 25 to 35 pounds is normal for a pregnancy. This client met the goal for nutrition and remaining infection-free. 5. Having a normal erythrocyte sedimentation rate during the pregnancy is an expected outcome.

Maternal risks of occiput posterior (OP) malposition include which of the following? Note: Credit will be given only if all correct choices and no incorrect choices are selected. Select all that apply. 1. Blood loss greater than 1000 mL 2. Postpartum infection 3. Anal sphincter injury 4. Higher rates of vaginal birth 5. Instrument delivery

Answer: 2, 3, 5 Explanation: 2. Postpartum infection is a maternal risk of OP. 3. Anal sphincter injury is a maternal risk of OP. 5. Instrument delivery is a maternal risk of OP.

Nursing interventions that foster the process of becoming a mother include which of the following? 1. Encouraging detachment from the nurse-patient relationship 2. Promoting maternal-infant attachment 3. Building awareness of and responsiveness to infant interactive capabilities 4. Instruct about promoting newborn independence 5. Preparing the woman for the maternal social role

Answer: 2, 3, 5 Explanation: 2. Promoting maternal-infant attachment is a nursing intervention that fosters the process of becoming a mother. 3. Building awareness of and responsiveness to infant interactive capabilities is a nursing intervention that fosters the process of becoming a mother. 5. Preparing the woman for the maternal social role is a nursing intervention that fosters the process of becoming a mother.

When working with pregnant adolescents, the nurse plans on assessing which of the following? Note: Credit will be given only if all correct and no incorrect choices are selected. Select all that apply. 1. High-birth-weight infants 2. Pelvic measurements 3. For sexually transmitted infections 4. Low blood pressure readings 5. Hemoglobin and hematocrit

Answer: 2, 3, 5 Explanation: 2. Teenagers are more likely to have cephalopelvic disproportion. 3. Teenagers are more likely to have sexually transmitted infections. 5. Teenagers are more likely to be anemic, so the hemoglobin and hematocrit must be assessed.

The nurse in the prenatal clinic will tell the client at 38-weeks' gestation to lie on her left side when the client complains of which of the following? Note: Credit will be given only if all correct and no incorrect choices are selected. Select all that apply. 1. Nausea 2. Pallor 3. Clamminess 4. Constipation 5. Dizziness

Answer: 2, 3, 5 Explanation: 2. Vena caval syndrome can cause pallor, which is relieved when the client turns to lie on her left side. 3. Vena caval syndrome can cause clamminess, which is relieved when the client turns to lie on her left side. 5. Vena caval syndrome can cause dizziness, which is relieved when the client turns to lie on her left side.

Which physical assessment findings would the nurse consider normal for the postpartum client following a vaginal delivery? Note: Credit will be given only if all correct choices and no incorrect choices are selected. Select all that apply. 1. Elevated blood pressure 2. Fundus firm and midline 3. Moderate amount of lochia serosa 4. Edema and bruising of perineum 5. Inflamed hemorrhoids

Answer: 2, 4 Explanation: 2. A firm fundus that is midline indicates the normal progression of uterine involution. 4. During the early postpartum period, the soft tissue in and around the perineum may appear edematous with some bruising.

A male client visits the infertility clinic for the results of his comprehensive exam. The exam indicated oligospermia. The client asks the nurse which procedure would assist him and his wife to conceive. The nurse's best response would include which of the following? Note: Credit will be given only if all correct choices and no incorrect choices are selected. Select all that apply. 1. "You might want to consider adoption." 2. "An option you might consider is in vitro fertilization." 3. "Surrogacy might be your best option." 4. "Many couples utilize therapeutic husband insemination." 5. "The GIFT procedure has had much success."

Answer: 2, 4 Explanation: 2. The in vitro fertilization procedure is used in cases in which infertility has resulted from male infertility. 4. Therapeutic husband insemination is generally indicated for such seminal deficiencies as oligospermia.

In which clinical situations would it be appropriate for an obstetrician to order a labor nurse to perform amnioinfusion? Note: Credit will be given only if all correct choices and no incorrect choices are selected. Select all that apply. 1. Placental abruption 2. Meconium-stained fluid 3. Polyhydramnios 4. Variable decelerations 5. Early decelerations

Answer: 2, 4 Explanation: 2. The physician may order amnioinfusion for meconium-stained fluid. 4. Amnioinfusion is sometimes done to prevent the possibility of variable decelerations.

A couple is at the clinic for preconceptual counseling. Both parents are 40 years old. The nurse knows that the education session has been successful when the wife makes which statement? Note: Credit will be given only if all correct choices and no incorrect choices are selected. Select all that apply. 1. "We are at low risk for having a baby with Down syndrome." 2. "Our children are more likely to have genetic defects." 3. "Children born to parents this age have sex-linked disorders." 4. "The tests for genetic defects can be done early in pregnancy." 5. "It will be almost impossible for us to conceive a child."

Answer: 2, 4 Explanation: 2. Women 35 or older are at greater risk for having children with chromosome abnormalities. 4. Genetic testing such as amniocentesis and chorionic villus sampling are done in the first trimester.

The nurse determines that a client is carrying her fetus in the vertical (longitudinal) lie. The nurse's judgment should be questioned if the fetal presenting part is which of the following? Note: Credit will be given only for all correct choices and no incorrect choices. Select all that apply. 1. Sacrum 2. Left arm 3. Mentum 4. Left scapula 5. Right scapula

Answer: 2, 4, 5 Explanation: 2. A fetus with an arm presenting is likely in a horizontal lie. 4. A fetus with a left scapula presenting is in a horizontal lie. 5. A fetus with a right scapula presenting is in a horizontal lie.

The nurse is monitoring a client who is receiving an amnioinfusion. Which assessments must the nurse perform to prevent a serious complication? Note: Credit will be given only if all correct choices and no incorrect choices are selected. Select all that apply. 1. Color of amniotic fluid 2. Maternal blood pressure 3. Cervical effacement 4. Uterine resting tone 5. Fluid leaking from the vagina

Answer: 2, 4, 5 Explanation: 2. Blood pressure should be monitored along with other vital signs. 4. The nurse should monitor contraction status (frequency, duration, intensity, resting tone, and associated maternal discomfort). 5. The nurse should continually check to make sure the infused fluid is being expelled from the vagina.

Benefits of skin-to-skin care as a developmental intervention include which of the following? Note: Credit will be given only if all correct choices and no incorrect choices are selected. Select all that apply. 1. Routine discharge 2. Stabilization of vital signs 3. Increased periods of awake-alert state 4. Decline in the episodes of apnea and bradycardia 5. Increased growth parameters

Answer: 2, 4, 5 Explanation: 2. Stabilization of vital signs is a benefit of skin-to-skin care as a developmental intervention. 4. Decline in the episodes of apnea and bradycardia is a benefit of skin-to-skin care as a developmental intervention. 5. Increased growth parameters are a benefit of skin-to-skin care as a developmental intervention.

During the first several postpartum weeks, the new mother must accomplish certain physical and developmental tasks, including which of the following? Note: Credit will be given only if all correct choices and no incorrect choices are selected. Select all that apply. 1. Establish a therapeutic relationship with her physician 2. Adapt to altered lifestyles and family structure resulting from the addition of a new member 3. Restore her intellectual abilities 4. Restore physical condition 5. Develop competence in caring for and meeting the needs of her infant

Answer: 2, 4, 5 Explanation: 2. During the first several postpartum weeks, the new mother must adapt to altered lifestyles and family structure resulting from the addition of a new member. 4. During the first several postpartum weeks, the new mother must restore her physical condition. 5. During the first several postpartum weeks, the new mother must develop competence in caring for and meeting the needs of her infant.

) Risk factors for labor dystocia include which of the following? Note: Credit will be given only if all correct choices and no incorrect choices are selected. Select all that apply. 1. Tall maternal height 2. Labor induction 3. Small-for-gestational-age (SGA) fetus 4. Malpresentation 5. Prolonged latent phase

Answer: 2, 4, 5 Explanation: 2. Labor induction is a risk factor of dystocia. 4. Malpresentation is a risk factor of dystocia. 5. Prolonged latent phase is a risk factor of dystocia.

A nurse is discussing the serving sizes in the grains food group with a new prenatal patient in the clinic. Which food equals one serving size from the grains food group? Note: Credit will be given only if all correct choices and no incorrect choices are selected. Select all that apply. 1. One hamburger roll 2. One slice of bread 3. One cup of pasta 4. One tortilla 5. One ounce of dry cereal

Answer: 2, 4, 5 Explanation: 2. One slice of bread is one serving. 4. One tortilla is one serving. 5. One ounce of dry cereal is one serving.

The nurse is caring for a newborn 30 minutes after birth. After assessing respiratory function, the nurse would report which findings as abnormal? Note: Credit will be given only if all correct and no incorrect choices are selected. Select all that apply. 1. Respiratory rate of 66 breaths per minute 2. Periodic breathing with pauses of 25 seconds 3. Synchronous chest and abdomen movements 4. Grunting on expiration 5. Nasal flaring

Answer: 2, 4, 5 Explanation: 2. Periodic breathing with pauses longer than 20 seconds (apnea) is an abnormal finding that should be reported to the physician. 4. Grunting on expiration is an abnormal finding that should be reported to the physician. 5. Nasal flaring is an abnormal finding that should be reported to the physician.

Which of the following are considered risk factors for development of severe hyperbilirubinemia? Note: Credit will be given only if all correct choices and no incorrect choices are selected. Select all that apply. 1. Northern European descent 2. Previous sibling received phototherapy 3. Gestational age 27 to 30 weeks 4. Exclusive breastfeeding 5. Infection

Answer: 2, 4, 5 Explanation: 2. Previous sibling received phototherapy is considered a risk factor for development of severe hyperbilirubinemia. 4. Exclusive breastfeeding, particularly if nursing is not going well and excessive weight loss is experienced, is considered a risk factor for development of severe hyperbilirubinemia. 5. Infection is considered a risk factor for development of severe hyperbilirubinemia.

What possible approaches should the nurse use to provide sensitive, holistic nursing care for the mother who is relinquishing her newborn? Note: Credit will be given only if all correct choices and no incorrect choices are selected. Select all that apply. 1. Allow the mother minimal control over the infant. 2. Use active listening strategies to determine the client's needs. 3. Provide only physical care in the early postpartum period. 4. Demonstrate empathy, concern, and compassion. 5. Provide nonjudgmental support and personalized care.

Answer: 2, 4, 5 Explanation: 2. The nurse can support the mother by encouraging her to share her feelings, by listening actively, and by being present for her. 4. The mother who decides to relinquish her baby needs emotional support and validation of her loss. The nurse should demonstrate empathy, concern, and compassion. 5. The nurse needs to acknowledge the significance of the birth mother's experience. The nurse should acknowledge the woman's loss and support her decision.

Amniotomy as a method of labor induction has which of the following advantages? Note: Credit will be given only if all correct choices and no incorrect choices are selected. Select all that apply. 1. The danger of a prolapsed cord is decreased. 2. There is usually no risk of hypertonus or rupture of the uterus. 3. The intervention can cause a decrease in pain. 4. The color and composition of amniotic fluid can be evaluated. 5. The contractions elicited are similar to those of spontaneous labor.

Answer: 2, 4, 5 Explanation: 2. There is usually no risk of hypertonus or rupture of the uterus and this is an advantage of amniotomy. 4. The color and composition of amniotic fluid can be evaluated and this is an advantage of amniotomy. 5. The contractions elicited are similar to those of spontaneous labor and this is an advantage of amniotomy.

A client at 40 weeks' gestation is to undergo stripping of the membranes. The nurse provides the client with information about the procedure. Which information is accurate? Note: Credit will be given only if all correct choices and no incorrect choices are selected. Select all that apply. 1. Intravenous administration of oxytocin will be used to initiate contractions. 2. The physician/CNM will insert a gloved finger into the cervical os and rotate the finger 360 degrees. 3. Stripping of the membranes will not cause discomfort, and is usually effective. 4. Labor should begin within 24-48 hours after the procedure. 5. Uterine contractions, cramping, and a bloody discharge can occur after the procedure.

Answer: 2, 4, 5 Explanation: 2. This motion separates the amniotic membranes that are lying against the lower uterine segment and internal os from the distal part of the lower uterine segment. 4. If labor is initiated, it typically begins within 24-48 hours. 5. Uterine contractions, cramping, scant bleeding, and bloody discharge can occur after stripping of the membranes.

In early-pregnancy class, the nurse emphasizes the importance of 8-10 glasses of fluid per day. How many of these should be water? 1. 1 to 2 2. 2 to 4 3. 4 to 6 4. 3 to 5

Answer: 3 Explanation: 3. A pregnant woman should consume at least 8 to 12 (8 oz) glasses of fluid each day, of which 4 to 6 glasses should be water.

An anesthesiologist informs the nurse that a client scheduled for a caesarean section will be having general anesthesia with postoperative self-controlled analgesia. For which clients would a general anesthesia be recommended? Note: Credit will be given only if all correct choices and no incorrect choices are selected. Select all that apply. 1. The client with a history of hypertension 2. The client who has had a lower back fusion 3. The client who is 13 years old 4. The client who is allergic to morphine sulfate 5. The client who has had surgery for scoliosis

Answer: 2, 5 Explanation: 2. Contraindications for epidural block include patients with previous back surgery. 5. Contraindications for epidural block include patients with previous back surgery.

The primary physician orders a narcotic analgesic for a client in labor. Which situations would lead the nurse to hold the medication? Note: Credit will be given only if all correct and no incorrect choices are selected. Select all that apply. 1. Contraction pattern every 3 minutes for 60 seconds 2. Fetal monitor tracing showing late decelerations 3. Client sleeping between contractions 4. Blood pressure 150/90 5. Blood pressure 80/42

Answer: 2, 5 Explanation: 2. Maternal hypotension results in uteroplacental insufficiency in the fetus, which is manifested as late decelerations on the fetal monitoring strip. 5. This would be a contraindication, as a narcotic can lower the blood pressure even more.

A client is being admitted to the labor area with the diagnosis of eclampsia. Which actions by the nurse are appropriate at this time? Note: Credit will be given only if all correct choices and no incorrect choices are selected. Select all that apply. 1. Tape a tongue blade to the head of the bed. 2. Pad the side rails. 3. Have the woman sit up. 4. Provide the client with grief counseling. 5. The airway should be maintained and oxygen administered.

Answer: 2, 5 Explanation: 2. Side rails should be up and padded. 5. Suctioning may be necessary to keep the airway clear.

The nurse determines the gestational age of an infant to be 40 weeks. Which characteristics are most likely to be observed? Note: Credit will be given only if all correct choices and no incorrect choices are selected. Select all that apply. 1. Lanugo abundant over shoulders and back 2. Plantar creases over entire sole 3. Pinna of ear springs back slowly when folded. 4. Vernix well distributed over entire body 5. Testes are pendulous, and the scrotum has deep rugae

Answer: 2, 5 Explanation: 2. Sole (plantar) creases are reliable indicators of gestational age in the first 12 hours of life. 5. By term, the testes are generally in the lower scrotum, which is pendulous and covered with rugae.

A pregnant client has been admitted with a diagnosis of hyperemesis. Which orders written by the primary healthcare provider are the highest priorities for the nurse to implement? Note: Credit will be given only if all correct choices and no incorrect choices are selected. Select all that apply. 1. Obtain complete blood count. 2. Start intravenous fluid with multivitamins. 3. Check admission weight. 4. Obtain urine for urinalysis. 5. Give a medication to stop the nausea and vomiting.

Answer: 2, 5 Explanation: 2. Starting intravenous fluid with multivitamins is a priority if the client has been vomiting. 5. Giving a medication to stop the nausea and vomiting is a priority.

A client tells the nurse that she does not like citrus fruits, and would like suggestions for alternate vitamin C sources. What should the nurse suggest as good sources of vitamin C? Note: Credit will be given only if all correct choices and no incorrect choices are selected. Select all that apply. 1. Barley and brown rice 2. Strawberries and potatoes 3. Buckwheat and lentils 4. Wheat flour and figs 5. Blueberries and broccoli

Answer: 2, 5 Explanation: 2. Strawberries and potatoes are very good sources of vitamin C.

A pregnant client who is a lacto-vegetarian asks the nurse for assistance with her diet. What instruction should the nurse give? Note: Credit will be given only if all correct choices and no incorrect choices are selected. Select all that apply. 1. "Protein is important; therefore, the addition of one serving of meat a day is necessary." 2. "A daily supplement of vitamin B12 is important." 3. "The high fiber in a vegetarian diet is dangerous for pregnant women." 4. "Eggs are important to add to your diet. Eat six eggs per week." 5. "Milk products contain protein, but they are very low in iron."

Answer: 2, 5 Explanation: 2. Supplementation may be recommended for vegans who have difficulty meeting the recommended amounts of vitamin B12 through food sources.

The primary care provider is performing a fetal scalp stimulation test. What result would the nurse hope to observe? Note: Credit will be given only if all correct choices and no incorrect choices are selected. Select all that apply. 1. Spontaneous fetal movement 2. Fetal heart acceleration 3. Increase in fetal heart variability 4. Resolution of late decelerations 5. Reactivity associated with the stimulation

Answer: 2, 5 Explanation: 2. The fetal heart rate should accelerate with stimulation. 5. There will be some reaction with the stimulation.

When counseling a newly pregnant client at 8 weeks' gestation of twins, the nurse teaches the woman about the need for increased caloric intake. What would the nurse tell the woman that the minimum recommended intake should be? 1. 2500 kcal and 120 grams protein 2. 3000 kcal and 150 grams protein 3. 4000 kcal and 135 grams protein 4. 5000 kcal and 190 grams protein

Answer: 3 Explanation: 3. 4000 kcal and 135 grams protein is the recommended caloric and protein intake in a twin-gestation pregnancy.

Whether sensitization is the result of a blood transfusion or maternal-fetal hemorrhage for any reason, what test can be performed to determine the amount of Rh(D) positive blood present in the maternal circulation and to calculate the amount of Rh immune globulin needed? 1. Indirect Coombs' test 2. Nonstress test 3. Kleihauer-Betke or rosette test 4. Direct Coombs' test

Answer: 3 Explanation: 3. A Kleihauer-Betke or rosette test can be performed to determine the amount of Rh(D) positive blood present in the maternal circulation and to calculate the amount of Rh immune globulin needed.

The client demonstrates understanding of the implications for future pregnancies secondary to her classic uterine incision when she states which of the following? 1. "The next time I have a baby, I can try to deliver vaginally." 2. "The risk of rupturing my uterus is too high for me to have any more babies." 3. "Every time I have a baby, I will have to have a cesarean delivery." 4. "I can only have one more baby."

Answer: 3 Explanation: 3. A classic uterine incision is made in the upper uterine segment and is associated with an increased risk of rupture in subsequent pregnancy, labor, and birth. Therefore, subsequent deliveries will be done by cesarean.

The nurse is aware that a fetus that is not in any stress would respond to a fetal scalp stimulation test by showing which change on the monitor strip? 1. Late decelerations 2. Early decelerations 3. Accelerations 4. Fetal dysrhythmia

Answer: 3 Explanation: 3. A fetus that is not experiencing stress responds to scalp stimulation with an acceleration of the FHR.

The client requires vacuum extraction assistance. To provide easier access to the fetal head, the physician cuts a mediolateral episiotomy. After delivery, the client asks the nurse to describe the episiotomy. What does the nurse respond? 1. "The episiotomy goes straight back toward your rectum." 2. "The episiotomy is from your vagina toward the urethra." 3. "The episiotomy is cut diagonally away from your vagina." 4. "The episiotomy extends from your vagina into your rectum."

Answer: 3 Explanation: 3. A mediolateral episiotomy is angled from the vaginal opening toward the buttock. It begins in the midline of the posterior fourchette and extends at a 45-degree angle downward to the right or left.

The nurse is caring for a 15-year-old client who gave birth to her first child yesterday. What action is the best indicator that the nurse understands the parenting adolescent? 1. The client's mother is included in all discussions and demonstrations. 2. The father of the baby is encouraged to change a diaper and give a bottle. 3. The nurse explains the characteristics and cues of the baby when assessing him. 4. A discussion on contraceptive methods is the first topic of teaching.

Answer: 3 Explanation: 3. A newborn physical examination performed at the bedside gives the parent(s) immediate feedback about the newborn's health and demonstrates methods of handling an infant. This action helps the client learn about her baby as an individual and facilitates maternal-infant attachment. This is the highest priority.

The postpartum client has chosen to bottle-feed her infant. Nursing actions that aid in lactation suppression include which of the following? 1. Warm showers 2. Pumping milk 3. Ice packs to each breast 4. Avoiding wearing a bra for 5 to 7 days

Answer: 3 Explanation: 3. A nonbreastfeeding mother should use cooling packs for comfort and to decrease the flow of breast milk.

A client calls the urologist's office to receive instructions about semen analysis. What does the nurse instruct the client to do? 1. Avoid sexual intercourse 24 hours prior to obtaining a specimen. 2. Use a latex condom to collect the specimen. 3. Expect that a repeat test might be required. 4. Expect a small sample.

Answer: 3 Explanation: 3. A repeat semen analysis might be required to adequately assess the man's fertility potential.

A nurse explains to new parents that their newborn has developed respiratory distress syndrome (RDS). Which of the following signs and symptoms would not be characteristic of RDS? 1. Grunting respirations 2. Nasal flaring 3. Respiratory rate of 40 during sleep 4. Chest retractions

Answer: 3 Explanation: 3. A respiratory rate of 40 during sleep is normal.

A 58-year-old father and a 45-year-old mother gave birth to a baby boy 2 days ago. The nurse assesses a single palmar crease and low-set ears on the newborn. The nurse plans to counsel the couple about which chromosomal abnormality? 1. Trisomy 13 2. Trisomy 18 3. Trisomy 21 4. Trisomy 26

Answer: 3 Explanation: 3. A single palmar crease and low-set ears are characteristics of trisomy 21 (Down syndrome).

The client is at 6 weeks' gestation, and is spotting. The client had an ectopic pregnancy 1 year ago, so the nurse anticipates that the physician will order which intervention? 1. A urine pregnancy test 2. The client to be seen next week for a full examination 3. An antiserumpregnancy test 4. An ultrasound to be done

Answer: 3 Explanation: 3. A β-Subunit radioimmunoassay (RIA) uses an antiserum with specificity for the β-subunit of hCG in blood plasma. This test may not only detect pregnancy but also detect an ectopic pregnancy or trophoblastic disease.

A client is admitted to the labor and delivery unit with a history of ruptured membranes for 2 hours. This is her sixth delivery; she is 40 years old, and smells of alcohol and cigarettes. What is this client at risk for? 1. Gestational diabetes 2. Placenta previa 3. Abruptio placentae 4. Placenta accreta

Answer: 3 Explanation: 3. Abruptio placentae is more frequent in pregnancies complicated by smoking, premature rupture of membranes, multiple gestation, advanced maternal age, cocaine use, chorioamnionitis, and hypertension.

The nurse assessing a pregnant African American woman in the first trimester understands that a cultural practice is which of the following? 1. Use of herbs like dandelion during pregnancy to increase lactation 2. Drinking ginseng tea for faintness 3. Eating clay to supply dietary minerals 4. Consulting a spiritual advisor to ensure a healthy pregnancy and birth

Answer: 3 Explanation: 3. African American pregnant women may be guided by their extended family into common practices such as geophagia, the ingestion of dirt or clay, which is believed to alleviate mineral deficiencies.

What would the nurse include as part of a routine physical assessment for a second-trimester primiparous patient whose prenatal care began in the first trimester and is ongoing? 1. Pap smear 2. Hepatitis B screening (HBsAg) 3. Fundal height measurement 4. Complete blood count

Answer: 3 Explanation: 3. At each prenatal visit, the blood pressure, pulse, and weight are assessed, and the size of the fundus is measured. Fundal height should be increasing with each prenatal visit.

A 2-day-old newborn is asleep, and the nurse assesses the apical pulse to be 88 beats/min. What would be the most appropriate nursing action based on this assessment finding? 1. Call the physician. 2. Administer oxygen. 3. Document the finding. 4. Place the newborn under the radiant warmer.

Answer: 3 Explanation: 3. An apical pulse rate of 88 beats/min is within the normal range of a sleeping full-term newborn. The average resting heart rate in the first week of life is 110 to 160 beats/min in a healthy full-term newborn but may vary significantly during deep sleep or active awake states. In full-term newborns, the heart rate may drop to a low of 80 to 100 beats/min during deep sleep.

The nurse is assessing a new client in the clinic. The nurse knows that the subjective (presumptive) signs and symptoms of pregnancy include which of the following? 1. Positive urine pregnancy test, enlarged abdomen, and Braxton Hicks contractions 2. Positive urine pregnancy test, amenorrhea, changes in pigmentation of the skin, and softening of the cervix 3. Increase in urination, amenorrhea, fatigue, breast tenderness, and quickening 4. Enlarged abdomen and fetal heartbeat

Answer: 3 Explanation: 3. An increase in urination, amenorrhea, fatigue, breast tenderness, and quickening are all subjective (presumptive) changes of pregnancy.

On the 3rd day postpartum, a client who is not breastfeeding experiences engorgement. To relieve her discomfort, the nurse should encourage the client to do which of the following? 1. Remove her bra 2. Apply heat to the breasts 3. Apply cold packs to the breasts 4. Use a breast pump to release the milk

Answer: 3 Explanation: 3. Applying cold packs to the breasts relieves discomfort and helps suppress lactation.

Approximately what percentage of the newborn's body weight is water? 1. 5% to 10% 2. 90% to 95% 3. 70% to 75% 4. 50% to 60%

Answer: 3 Explanation: 3. Approximately 70% to 75% of the newborn's body weight is water.

The nurse is presenting a class of important "dos and don'ts" during pregnancy, including travel considerations. What method of travel does the nurse recommend as most appropriate for a client in her 25th week of pregnancy? 1. Automobile 2. Airplane 3. Train 4. None; this client should not travel

Answer: 3 Explanation: 3. As pregnancy progresses, travel by train is generally recommended for long distances.

Intercourse is contraindicated if the pregnancy is vulnerable because of which diagnosis? 1. Gestational diabetes 2. Cervical insufficiency (cerclage) 3. Abruptio placentae 4. Placenta previa

Answer: 3 Explanation: 3. As the uterus enlarges, the couple will have to experiment with different positions.

The parents of a newborn male ask the nurse whether they should circumcise their son. What is the best response by the nurse? 1. "Circumcision should be undertaken to prevent problems in the future." 2. "Circumcision might decrease the child's risk of developing a urinary tract infection." 3. "Circumcision can sometimes cause complications. What questions do you have?" 4. "Circumcision is painful, and should be avoided unless you are Jewish."

Answer: 3 Explanation: 3. Asking this question allows the nurse to determine what the parents' concerns are, then address them specifically.

The nurse is performing a postpartum homecare visit. Which teaching has the highest priority? 1. Teaching or reviewing how to bathe the baby 2. Teaching how to thoroughly childproof the house 3. How many wet diapers the baby should have daily 4. Prevention of plagiocephaly

Answer: 3 Explanation: 3. Assessment of intake, output, weight, and hydration status is imperative. The baby should have at least six diapers that are saturated with clear urine each day by 1 week of age. Wet diapers are an indication of hydration of the newborn. This is the highest priority.

A woman is experiencing preterm labor. The client asks why she is on betamethasone. Which is the nurse's best response? 1. "This medication will halt the labor process until the baby is more mature." 2. "This medication will relax the smooth muscles in the infant's lungs so the baby can breathe." 3. "This medication is effective in stimulating lung development in the preterm infant." 4. "This medication is an antibiotic that will treat your urinary tract infection, which caused preterm labor."

Answer: 3 Explanation: 3. Betamethasone or dexamethasone is often administered to the woman whose fetus has an immature lung profile to promote fetal lung maturation.

The nurse is supervising care in the emergency department. Which situation most requires an intervention? 1. Moderate vaginal bleeding at 36 weeks' gestation; client has an IV of lactated Ringer's solution running at 125 mL/hour 2. Spotting of pinkish-brown discharge at 6 weeks' gestation and abdominal cramping; ultrasound scheduled in 1 hour 3. Bright red bleeding with clots at 32 weeks' gestation; pulse = 110, blood pressure 90/50, respirations = 20 4. Dark red bleeding at 30 weeks' gestation with normal vital signs; client reports an absence of fetal movement

Answer: 3 Explanation: 3. Bleeding in the third trimester is usually a placenta previa or placental abruption. Observe the woman for indications of shock, such as pallor, clammy skin, perspiration, dyspnea, or restlessness. Monitor vital signs, particularly blood pressure and pulse, for evidence of developing shock.

The client at 30 weeks' gestation is admitted with painless late vaginal bleeding. The nurse understands that expectant management includes which of the following? 1. Limiting vaginal exams to only one per 24-hour period. 2. Evaluating the fetal heart rate with an internal monitor. 3. Monitoring for blood loss, pain, and uterine contractibility. 4. Assessing blood pressure every 2 hours.

Answer: 3 Explanation: 3. Blood loss, pain, and uterine contractibility need to be assessed for client comfort and safety.

When is breastfeeding contraindicated? 1. Infant has hypertension 2. Mother has a history of treated tuberculosis 3. Mother is HIV positive or has AIDS 4. Mother has a history of treated herpes

Answer: 3 Explanation: 3. Breastfeeding is contraindicated when the mother is HIV positive or has AIDS and is counseled against breastfeeding.

A woman has been in labor for 16 hours. Her cervix is dilated to 3 cm and is 80% effaced. The fetal presenting part is not engaged. The nurse would suspect which of the following? 1. Breech malpresentation 2. Fetal demise 3. Cephalopelvic disproportion (CPD) 4. Abruptio placentae

Answer: 3 Explanation: 3. Cephalopelvic disproportion (CPD) prevents the presenting part from becoming engaged.

The nurse is preparing a client education handout on the differences between false labor and true labor. What information is most important for the nurse to include? 1. True labor contractions begin in the back and sweep toward the front. 2. False labor often feels like abdominal tightening, or "balling up." 3. True labor can be diagnosed only if cervical change occurs. 4. False labor contractions do not increase in intensity or duration.

Answer: 3 Explanation: 3. Cervical change is the only factor that actually distinguishes false from true labor. The contractions of true labor produce progressive dilatation and effacement of the cervix. The contractions of false labor do not produce progressive cervical effacement and dilatation.

When assisting with a transabdominal sampling, which of the following would the nurse do? 1. Obtain preliminary urinary samples. 2. Have the woman empty her bladder before the test begins. 3. Assist the woman into a supine position on the examining table. 4. Instruct the woman to eat a fat-free meal 2 hours before the scheduled test time.

Answer: 3 Explanation: 3. Clients are placed in a supine position on the table.

The client delivered her second child 1 day ago. The client's temperature is 101.4° F, her pulse is 100, and her blood pressure is 110/70. Her lochia is moderate, serosanguinous, and malodorous. She is started on IV antibiotics. The nurse provides education for the client and her partner. Which statement indicates that teaching has been effective? 1. "This condition is called parametritis." 2. "Gonorrhea is the most common organism that causes this type of infection." 3. "My Beta-strep culture's being positive might have contributed to this problem." 4. "If I had walked more yesterday, this probably wouldn't have happened."

Answer: 3 Explanation: 3. Clinical findings of metritis in the initial 24 to 36 hours postpartum tend to be related to group B streptococcus (GBS).

The nurse who is counseling a group of middle-school girls on pregnancy avoidance should include which statement? 1. "Although sexuality is common in the media, peer pressure to have sex is not an important factor." 2. "It has become far less acceptable to give birth during your teenage years than it used to be." 3. "Although condom use is growing, there is still an increasing rate of STIs among teens." 4. "You have learned enough from your friends and families to understand how pregnancy occurs."

Answer: 3 Explanation: 3. Condom use is increasing, but the rate of STI infections, including HIV, is also rising.

Specific cellular immunity is mediated by T lymphocytes, which enhance the efficiency of the phagocytic response. What do cytotoxic activated T cells do? 1. Enable T or B cells to respond to antigens 2. Repress responses to specific B or T lymphocytes to antigens 3. Kill foreign or virus-infected cells 4. Remove pathogens and cell debris

Answer: 3 Explanation: 3. Cytotoxic activated T cells kill foreign or virus-infected cells.

The postpartum client is about to go home. The nurse includes which subject in the teaching plan? 1. Replacement of fluids 2. Striae 3. Diastasis of the recti muscles 4. REEDA scale

Answer: 3 Explanation: 3. Diastasis recti abdominis can be improved with exercise and abdominal muscle tone can improve significantly best taught when the mother is receptive to instruction during the postpartum assessment.

Which maternal-child client should the nurse see first? 1. Blood type O, Rh-negative 2. Indirect Coombs' test negative 3. Direct Coombs' test positive 4. Blood type B, Rh-positive

Answer: 3 Explanation: 3. Direct Coombs' test is done on the infant's blood to detect antibody-coated Rh-positive RBCs. If the mother's indirect Coombs' test is positive and her Rh-positive infant has a positive direct Coombs' test, Rh immune globulin is not given; in this case, the infant is carefully monitored for hemolytic disease.

The nurse is making an initial assessment of the newborn. The findings include a chest circumference of 32.5 cm and a head circumference of 33.5 cm. Based on these findings, which action should the nurse take first? 1. Notify the physician. 2. Elevate the newborn's head. 3. Document the findings in the chart. 4. Assess for hypothermia immediately.

Answer: 3 Explanation: 3. Documentation is the appropriate first step. The average circumference of the head at birth is 32 to 37 cm, and average chest circumference ranges from 30 to 35 cm.

The nurse is listening to the fetal heart tones of a client at 37 weeks' gestation while the client is in a supine position. The client states, "I'm getting lightheaded and dizzy." What is the nurse's best action? 1. Assist the client to sit up. 2. Remind the client that she needs to lie still to hear the baby. 3. Help the client turn onto her left side. 4. Check the client's blood pressure.

Answer: 3 Explanation: 3. During pregnancy the enlarging uterus may put pressure on the vena cava when the woman is supine, resulting in supine hypotensive syndrome. This pressure interferes with returning blood flow and produces a marked decrease in blood pressure with accompanying dizziness, pallor, and clamminess, which can be corrected by having the woman lie on her left side.

The client in early labor asks the nurse what the contractions are like as labor progresses. What would the nurse respond? 1. "In normal labor, as the uterine contractions become stronger, they usually also become less frequent." 2. "In normal labor, as the uterine contractions become stronger, they usually also become less painful." 3. "In normal labor, as the uterine contractions become stronger, they usually also become longer in duration." 4. "In normal labor, as the uterine contractions become stronger, they usually also become shorter in duration."

Answer: 3 Explanation: 3. During the active and transition phases, contractions become more frequent, are longer in duration, and increase in intensity.

During her first months of pregnancy, a client tells the nurse, "It seems like I have to go to the bathroom every 5 minutes." The nurse explains to the client that this is because of which of the following? 1. The client probably has a urinary tract infection. 2. Bladder capacity increases throughout pregnancy. 3. The growing uterus puts pressure on the bladder. 4. Some women are very sensitive to body function changes.

Answer: 3 Explanation: 3. During the first trimester, the growing uterus puts pressure on the bladder, producing urinary frequency until the second trimester, when the uterus becomes an abdominal organ. Near term, when the presenting part engages in the pelvis, pressure is again exerted on the bladder.

The nurse is using the New Ballard Score to assess the gestational age of a newborn delivered 4 hours ago. The infant's gestational age is 33 weeks based on early ultrasound and last menstrual period. The nurse expects the infant to exhibit which of the following? 1. Full sole creases, nails extending beyond the fingertips, scarf sign showing the elbow beyond the midline 2. Testes located in the upper scrotum, rugae covering the scrotum, vernix covering the entire body 3. Ear cartilage folded over, lanugo present over much of the body, slow recoil time 4. 1 cm breast bud, peeling skin and veins not visible, rapid recoil of legs and arms to extension

Answer: 3 Explanation: 3. Ear cartilage folded over, lanugo present over much of the body, and slow recoil time are all characteristics of a preterm infant.

Persistent early decelerations are noted. What would the nurse's first action be? 1. Turn the mother on her left side and give oxygen. 2. Check for prolapsed cord. 3. Do nothing. This is a benign pattern. 4. Prepare for immediate forceps or cesarean delivery.

Answer: 3 Explanation: 3. Early decelerations are considered benign, and do not require any intervention.

The client has delivered her first child at 37 weeks. The nurse would describe this to the client as what type of delivery? 1. Preterm 2. Postterm 3. Early term 4. Near term

Answer: 3 Explanation: 3. Early term births extend from 37 to 38 weeks' gestation.

The laboring client with meconium-stained amniotic fluid asks the nurse why the fetal monitor is necessary, as she finds the belt uncomfortable. Which response by the nurse is most important? 1. "The monitor is necessary so we can see how your labor is progressing." 2. "The monitor will prevent complications from the meconium in your fluid." 3. "The monitor helps us to see how the baby is tolerating labor." 4. "The monitor can be removed, and oxygen given instead."

Answer: 3 Explanation: 3. Electronic fetal monitoring (EFM) provides a continuous tracing of the fetal heart rate (FHR), allowing characteristics of the FHR to be observed and evaluated.

Which statement, if made by a pregnant client, would indicate that she understands health promotion during pregnancy? 1. "I lie down after eating to relieve heartburn." 2. "I try to limit my fluid intake to 3 or 4 glasses each day." 3. "I elevate my legs while sitting at my desk." 4. "I am avoiding exercise to stay well rested."

Answer: 3 Explanation: 3. Elevating the legs can help decrease lower leg edema.

A client dilated to 5 cm has just received an epidural for pain. She complains of feeling lightheaded and dizzy within 10 minutes after the procedure. Her blood pressure was 120/80 before the procedure and is now 80/52. In addition to the bolus of fluids she has been given, which medication is preferred to increase her BP? 1. Epinephrine 2. Terbutaline 3. Ephedrine 4. Epifoam

Answer: 3 Explanation: 3. Ephedrine is the medication of choice to increase maternal blood pressure.

If a woman has the pre-existing condition of diabetes, the nurse knows that she would be prone to what high-risk factor when pregnant? 1. Vasospasm 2. Postpartum hemorrhage 3. Episodes of hypoglycemia and hyperglycemia 4. Cerebrovascular accident (CVA)

Answer: 3 Explanation: 3. Episodes of hypoglycemia and hyperglycemia would be a high-risk factor for a client with pre-existing diabetes.

The client with a normal pregnancy had an emergency cesarean birth under general anesthesia 2 hours ago. The client now has a respiratory rate of 30, pale blue nail beds, a pulse rate of 110, and a temperature of 102.6°F, and is complaining of chest pain. The nurse understands that the client most likely is experiencing which of the following? 1. Pulmonary embolus 2. Pneumococcal pneumonia 3. Pneumonitis 4. Gastroesophageal reflux disease

Answer: 3 Explanation: 3. Even when food and fluids have been withheld, the gastric juice produced during fasting is highly acidic and can produce chemical pneumonitis if aspirated. This pneumonitis is known as Mendelson syndrome. The signs and symptoms are chest pain, respiratory distress, cyanosis, fever, and tachycardia. Women undergoing emergency cesarean births appear to be at considerable risk for adverse events.

Which of the following is a major side effect of butorphanol tartrate (Stadol)? 1. Blurred vision 2. Agitation 3. Feelings of dysphoria 4. Drowsiness

Answer: 3 Explanation: 3. Feelings of dysphoria are a major side effect of Stardol.

The client is being admitted to the birthing unit. As the nurse begins the assessment, the client's partner asks why the fetus's heart rate will be monitored. After the nurse explains, which statement by the partner indicates a need for further teaching? 1. "The fetus's heart rate will vary between 110 and 160." 2. "The heart rate is monitored to see whether the fetus is tolerating labor." 3. "By listening to the heart, we can tell the gender of the fetus." 4. "After listening to the heart rate, you will contact the midwife."

Answer: 3 Explanation: 3. Fetal heart rate is not a predictor of gender.

As the couple and their families begin to confront the pain of their loss, many normal manifestations of grief may be present. Which of the following would indicate an emotional response to the loss? 1. Lack of meaning or direction 2. Preoccupation 3. Flat affect 4. Dreams of the deceased

Answer: 3 Explanation: 3. Flat affect would be an emotional response to loss.

The nurse is reviewing preconception questionnaires in charts. Which couple are the most likely candidates for preconceptual genetic counseling? 1. Wife is 30 years old, husband is 31 years old 2. Wife and husband are both 29 years old, first baby for husband, wife has a normal 4-year-old 3. Wife's family has a history of hemophilia 4. Single 32-year-old woman is using donor sperm

Answer: 3 Explanation: 3. For families in which the woman is a known or possible carrier of an X-linked disorder, such as hemophilia, the risk of having an affected male fetus is 25%.

The client is recovering from a delivery that included a midline episiotomy. Her perineum is swollen and sore. Ten minutes after an ice pack is applied, the client asks for another. What is the best response from the nurse? 1. "I'll get you one right away." 2. "You only need to use one ice pack." 3. "You need to leave it off for at least 20 minutes and then reapply." 4. "I'll bring you an extra so that you can change it when you are ready."

Answer: 3 Explanation: 3. For optimal effect, the ice pack should be applied for 20 to 30 minutes and removed for at least 20 minutes before being reapplied.

The postpartum nurse is performing a homecare visit to a first-time mother on her third day after delivery. She reports that her nipples are becoming sore. What statement indicates that further teaching is needed? 1. "I can apply lanolin cream to help with the nipple pain." 2. "Watching how much areola is visible will help me see whether my baby has a good mouthful of breast or not." 3. "My nipples will heal if I switch to bottle feeding for about 3 days while I pump my breasts." 4. "Rotating breastfeeding positions will allow the sore areas of my nipples to have less friction."

Answer: 3 Explanation: 3. For severe cases, in which the mother is unable to tolerate breastfeeding, the mother will need to pump or hand-express the breast milk until the nipple condition improves. Bottle feeding may not be necessary.

New parents decide not to have their newborn circumcised. What should the nurse teach regarding care for the uncircumcised infant? 1. The foreskin will be retractable at 2 months. 2. Retract the foreskin and clean thoroughly. 3. Avoid retracting the foreskin. 4. Use soap and Betadine to cleanse the penis daily.

Answer: 3 Explanation: 3. Foreskin will retract normally over time and may take 3 to 5 years.

The nurse is planning care for a preterm newborn. Which nursing diagnosis has the highest priority? 1. Tissue Integrity, Impaired 2. Infection, Risk for 3. Gas Exchange, Impaired 4. Family Processes, Dysfunctional

Answer: 3 Explanation: 3. Gas Exchange, Impaired is related to immature pulmonary vasculature and inadequate surfactant production and has the highest priority.

The nurse is providing preoperative teaching to a client for whom a cesarean birth under general anesthesia is scheduled for the next day. Which statement by the client indicates that she requires additional information? 1. "General anesthesia can be accomplished with inhaled gases." 2. "General anesthesia usually involves administering medication into my IV." 3. "General anesthesia will provide good pain relief after the birth." 4. "General anesthesia takes effect faster than an epidural

Answer: 3 Explanation: 3. General anesthesia provides no pain relief after birth, as regional anesthesia does.

The nurse has returned from working as a maternal-child nurse volunteer for a nongovernmental organization. After completing a community presentation about this experience, the nurse knows that learning has occurred when a participant states which of the following? 1. "Malaria is a chronic disease, and rarely causes fetal loss." 2. "Escherichia coli bacteria can cause diarrhea but not stillbirth." 3. "Group B streptococci can cause infection and the death of the fetus." 4. "Viral infections don't cause fetal death in developing nations."

Answer: 3 Explanation: 3. Group B streptococci can cause ascending infections prior to or after rupture of membranes.

The nurse is caring for an infant who was delivered in a car on the way to the hospital and who has developed cold stress. Which finding requires immediate intervention? 1. Increased skin temperature and respirations 2. Blood glucose level of 45 3. Room-temperature IV running 4. Positioned under radiant warmer

Answer: 3 Explanation: 3. IV fluids should be warmed prior to administration and the newborn can be wrapped in a chemically activated warming mattress immediately following birth to decrease the postnatal fall in temperature that normally occurs.

A client's labor has progressed so rapidly that a precipitous birth is occurring. What should the nurse do? 1. Go to the nurse's station and immediately call the physician. 2. Run to the delivery room for an emergency birth pack. 3. Stay with the client and ask auxiliary personnel for assistance. 4. Hold back the infant's head forcibly until the physician arrives for the delivery.

Answer: 3 Explanation: 3. If birth is imminent, the nurse must not leave the client alone.

The nurse teaches the parents of an infant who recently was circumcised to observe for bleeding. What should the parents be taught to do if bleeding does occur? 1. Wrap the diaper tightly. 2. Clean with warm water with each diaper change. 3. Apply gentle pressure to the site with gauze. 4. Apply a new petroleum ointment gauze dressing.

Answer: 3 Explanation: 3. If bleeding does occur, apply light pressure with a sterile gauze pad to stop the bleeding within a short time. If this is not effective, contact the physician immediately or take the newborn to the healthcare provider.

If the woman is Rh negative and not sensitized, she is given Rh immune globulin to prevent what? 1. The potential for hemorrhage 2. Hyperhomocysteinemia 3. Antibody formation 4. Tubal pregnancy

Answer: 3 Explanation: 3. If the woman is Rh negative and not sensitized, she is given Rh immune globulin to prevent antibody formation.

A nurse is caring for several postpartum clients. Which client is demonstrating a problem attaching to her newborn? 1. The client who is discussing how the baby looks like her father 2. The client who is singing softly to her baby 3. The client who continues to touch her baby with only her fingertips 4. The client who picks her baby up when the baby cries

Answer: 3 Explanation: 3. In a progression of touching activities, the mother proceeds from fingertip exploration of the newborn's extremities toward palmar contact with larger body areas and finally to enfolding the infant with the whole hand and arms. If the client continues to touch with only her fingertips, she might not be developing adequate early attachment.

A 14-year-old girl is brought to the clinic by her mother. The nurse determines that the teen is about 28 weeks pregnant. The mother states, "We knew she was gaining weight, but we can't tell anyone she is pregnant." The nurse understands that the client's mother's behavior exemplifies which of the following? 1. Low self-esteem 2. Anger 3. Shame 4. Ignorance

Answer: 3 Explanation: 3. In families who foster educational and career goals for their children, adolescent pregnancy is often a shock. Anger, shame, and sorrow are common reactions.

Which statement made by the pregnant adolescent would indicate to the nurse that she understood her increased risk of physiologic complications during pregnancy? 1. "It's no big deal that I started prenatal care in my seventh month." 2. "My anemia and eating mostly fast food are not important." 3. "I need to take good care of myself so my baby doesn't come early." 4. "Smoking and using crack cocaine won't harm my baby."

Answer: 3 Explanation: 3. In the adolescent age group, prenatal care is the critical factor that most influences pregnancy outcome.

What is the increased vascularization causing the softening of the cervix known as? 1. Hegar sign 2. Chadwick sign 3. Goodell sign 4. McDonald sign

Answer: 3 Explanation: 3. Increased vascularization causes the softening of the cervix known as Goodell sign.

The nurse is teaching an early pregnancy class for clients in the first trimester of pregnancy. Which statement by a client requires immediate intervention by the nurse? 1. "When my nausea is bad, I will drink some ginger tea." 2. "The fatigue I am experiencing will improve in the second trimester." 3. "It is normal for my vaginal discharge to be green." 4. "I will urinate less often during the middle of my pregnancy."

Answer: 3 Explanation: 3. Increased whitish vaginal discharge, called leukorrhea, is common in pregnancy. Green discharge is not a normal finding, and indicates a vaginal infection.

A client in her second trimester is complaining of spotting. Causes for spotting in the second trimester are diagnosed primarily through the use of which of the following? 1. A nonstress test 2. A vibroacoustic stimulation test 3. An ultrasound 4. A contraction stress test

Answer: 3 Explanation: 3. Indirect diagnosis is made by localizing the placenta via tests that require no vaginal examination. The most commonly employed diagnostic test is the transabdominal ultrasound scan.

During discharge planning for a drug-dependent newborn, the nurse explains to the mother how to do which of the following? 1. Place the newborn in a prone position. 2. Limit feedings to three a day to decrease diarrhea. 3. Place the infant supine and operate a home apnea-monitoring system. 4. Wean the newborn off the pacifier.

Answer: 3 Explanation: 3. Infants with neonatal abstinence syndrome are at a significantly higher risk for sudden infant death syndrome (SIDS) when the mother used heroin, cocaine, or opiates. The infant should sleep in a supine position, and home apnea monitoring should be implemented.

A pregnant client confides to the nurse that she is eating laundry starch daily. The nurse should assess the client for which of the following? 1. Alopecia 2. Weight loss 3. Iron deficiency anemia 4. Fecal impaction

Answer: 3 Explanation: 3. Iron deficiency anemia is the most common concern with pica. The ingestion of laundry starch or certain types of clay may contribute to iron deficiency by replacing iron-containing foods from the diet or by interfering with iron absorption.

Which of the following drugs and drug categories can cause multiple fetal central nervous system (CNS), facial, and cardiovascular anomalies? 1. Category C: Zidovudine 2. Category B: Penicillin 3. Category X: Isotretinoin 4. Category A: Vitamin C

Answer: 3 Explanation: 3. Isotretinoin (Accutane), the acne medication, can cause multiple central nervous system (CNS), facial, and cardiovascular anomalies.

The labor and delivery nurse is preparing a prenatal class about facilitating the progress of labor. Which of the following frequent responses to pain should the nurse indicate is most likely to impede progress in labor? 1. Increased pulse 2. Elevated blood pressure 3. Muscle tension 4. Increased respirations

Answer: 3 Explanation: 3. It is important for the woman to relax each part of her body. Be alert for signs of muscle tension and tightening. Dissociative relaxation, controlled muscle relaxation, and specified breathing patterns are used to promote birth as a normal process.

Two hours after an epidural infusion has begun, a client complains of itching on her face and neck. What should the nurse do? 1. Remove the epidural catheter and apply a Band-Aid to the injection site. 2. Offer the client a cool cloth and let her know the itching is temporary. 3. Recognize that this is a common side effect, and follow protocol for administration of Benadryl. 4. Call the anesthesia care provider to re-dose the epidural catheter.

Answer: 3 Explanation: 3. Itching is a side effect of the medication used for an epidural infusion. Benadryl, an antihistamine, can be administered to manage pruritus.

A telephone triage nurse gets a call from a postpartum client who is concerned about jaundice. The client's newborn is 37 hours old. What data point should the nurse gather first? 1. Stool characteristics 2. Fluid intake 3. Skin color 4. Bilirubin level

Answer: 3 Explanation: 3. Jaundice (icterus) is the yellowish coloration of the skin and sclera caused by the presence of bilirubin in elevated concentrations. Inspection of the skin would be the first step in assessing for jaundice.

The nurse is assessing a drug-dependent newborn. Which symptom would require further assessment by the nurse? 1. Occasional watery stools 2. Spitting up after feeding 3. Jitteriness and irritability 4. Nasal stuffiness

Answer: 3 Explanation: 3. Jitteriness and irritability can be an indicator of drug withdrawal.

The nurse has been asked by a community organization to give a presentation on prevention of teen pregnancy. Which statement indicates appropriate steps toward reduction of the local teen pregnancy rate? 1. Abstinence-only education will be offered in the school and clinics. 2. Classes on how to parent will be mandatory in high school. 3. A low-cost reproductive health clinic will be planned. 4. Parents will be encouraged to avoid discussing sexual activity.

Answer: 3 Explanation: 3. Key strategies for prevention of unintended teen pregnancy and sexual health promotion include the provision of services that ensure accessible and high-quality reproductive health care; sex education programs that provide developmentally appropriate, evidence-based curricula; and youth development strategies to enhance life skills. A low-cost reproductive health clinic would be able to provide these services.

The nurse is preparing a teaching brochure for Spanish-speaking postpartum clients. Which topics are critical for this population? 1. Baby baths and birth certificates 2. Hygiene practices 3. When and how to contact their healthcare provider 4. Pain-relief options in labor and after birth

Answer: 3 Explanation: 3. Knowing how to contact their healthcare provider at all times is critical so that clients receive appropriate advice and care in case of a problem or emergency. Knowing what to watch for and when to call the healthcare provider also facilitates safety. These are the highest priorities.

Which of the following is the primary carbohydrate in the breastfeeding newborn? 1. Glucose 2. Fructose 3. Lactose 4. Maltose

Answer: 3 Explanation: 3. Lactose is the primary carbohydrate in the breastfeeding newborn and is generally easily digested and well absorbed.

The nurse is analyzing several fetal heart rate patterns. The pattern that would be of most concern to the nurse would be which of the following? 1. Moderate variability 2. Early decelerations 3. Late decelerations 4. Accelerations

Answer: 3 Explanation: 3. Late decelerations are caused by uteroplacental insufficiency. The late deceleration pattern is considered a nonreassuring sign.

The labor and delivery nurse is assigned to four clients in early labor. Which electronic fetal monitoring finding would require immediate intervention? 1. Early decelerations with each contraction 2. Variable decelerations that recover to the baseline 3. Late decelerations with minimal variability 4. Accelerations

Answer: 3 Explanation: 3. Late decelerations are considered a nonreassuring fetal heart rate (FHR) pattern, and therefore require immediate intervention.

The nurse is providing discharge teaching to a woman who delivered her first child 2 days ago. The nurse understands that additional information is needed if the client makes which statement? 1. "I should expect a lighter flow next week." 2. "The flow will increase if I am too active." 3. "My bleeding will remain red for about a month." 4. "I will be able to use a pantiliner in a day or two."

Answer: 3 Explanation: 3. Lochia rubra is dark red in color. It is present for the first 2 to 3 days postpartum. Lochia serosa is a pinkish color and it follows from about the 3rd to the 10th day.

The client vaginally delivers an infant that weighs 4750 g. Moderate shoulder dystocia occurred during the birth. During the initial assessment of this infant, what should the nurse look for? 1. Bell's palsy 2. Bradycardia 3. Erb palsy 4. Petechiae

Answer: 3 Explanation: 3. Macrosomic newborns should be inspected for cephalhematoma, Erb palsy, and fractured clavicles.

Major perineal trauma (extension to or through the anal sphincter) is more likely to occur if what type of episiotomy is performed? 1. Mediolateral 2. Episiorrhaphy 3. Midline 4. Medical

Answer: 3 Explanation: 3. Major perineal trauma is more likely to occur if a midline episiotomy is performed. The major disadvantage is that a tear of the midline incision may extend through the anal sphincter and rectum.

The client presents to the labor and delivery unit stating that her water broke 2 hours ago. Barring any abnormalities, how often would the nurse expect to take the client's temperature? 1. Every hour 2. Every 2 hours 3. Every 4 hours 4. Every shift

Answer: 3 Explanation: 3. Maternal temperature is taken every 4 hours unless it is above 37.5°C. If elevated, it is taken every hour.

After noting meconium-stained amniotic fluid and fetal heart rate decelerations, the physician diagnoses a depressed fetus. The appropriate nursing action at this time would be to do what? 1. Increase the mother's oxygen rate. 2. Turn the mother to the left lateral position. 3. Prepare the mother for a higher-risk delivery. 4. Increase the intravenous infusion rate.

Answer: 3 Explanation: 3. Meconium-stained fluid and heart rate decelerations are indications that delivery is considered higher-risk.

The pregnant 16-year-old is seeing the nurse during a prenatal visit. Based on the client's developmental level, which statement would the nurse expect the client to make? 1. "My friends and I all wear totally different styles of clothing." 2. "Having a baby will change my college plans." 3. "I drink alcohol at parties most weekends." 4. "My mom is my best friend."

Answer: 3 Explanation: 3. Middle adolescence (15-17) is a time of experimentation, including drinking alcohol, using other drugs, and sex.

A laboring mother has recurrent late decelerations. At birth, the infant has a heart rate of 100, is not breathing, and is limp and bluish in color. What nursing action is best? 1. Begin chest compressions. 2. Begin direct tracheal suctioning. 3. Begin bag-and-mask ventilation. 4. Obtain a blood pressure reading.

Answer: 3 Explanation: 3. Most newborns can be effectively resuscitated by bag-and-mask ventilation.

After nalbuphine hydrochloride (Nubain) is administered, labor progresses rapidly, and the baby is born less than 1 hour later. The baby shows signs of respiratory depression. Which medication should the nurse be prepared to administer to the newborn? 1. Fentanyl (Sublimaze) 2. Butorphanol tartrate (Stadol) 3. Naloxone (Narcan) 4. Pentobarbital (Nembutal)

Answer: 3 Explanation: 3. Narcan is useful for respiratory depression caused by nalbuphine (Nubain). Respiratory depression in the mother or fetus/newborn can be improved by the administration of naloxone (Narcan), which is a specific antagonist for this agent.

The nurse is working with a student nurse during assessment of a 2-hour-old newborn. Which action indicates that the student nurse understands neonatal assessment? 1. The student nurse listens to bowel sounds then assesses the head for skull consistency and size and tension of fontanelles. 2. The student nurse checks for Ortolani's sign, then palpates the femoral pulse, then assesses respiratory rate. 3. The student nurse determines skin color, then describes the shape of the chest and looks at structures and flexion of the feet. 4. The student nurse counts the number of cord vessels, then assesses genitals, then sclera color and eyelids.

Answer: 3 Explanation: 3. Neonatal assessment proceeds in a head-to-toe fashion.

Before a newborn and mother are discharged from the hospital, the nurse informs the parents about routine screening tests for newborns. What is a good reason for having the screening tests done? 1. The tests prevent infants from developing phenylketonuria. 2. The tests detect such disorders as hypertension and diabetes. 3. The tests detect disorders that cause physical, intellectual, and developmental complications or death if left undiscovered. 4. The tests prevent sickle-cell anemia, galactosemia, and homocystinuria.

Answer: 3 Explanation: 3. Newborn screening tests use a few drops of the newborn's blood to detect disorders that are often asymptomatic at birth but cause irreversible harm if not detected early. Profound physical, intellectual, and developmental complications and even death can result from many of the conditions for which newborns are screened prior to discharge.

The community nurse is working with a client at 32 weeks' gestation who has been diagnosed with preeclampsia. Which statement by the client would indicate that additional information is needed? 1. "I should call the doctor if I develop a headache or blurred vision." 2. "Lying on my left side as much as possible is good for the baby." 3. "My urine could become darker and smaller in amount each day." 4. "Pain in the top of my abdomen is a sign my condition is worsening."

Answer: 3 Explanation: 3. Oliguria is a complication of preeclampsia. Specific gravity of urine readings over 1.040 correlate with oliguria and proteinuria and should be reported to the physician.

The client at 39 weeks' gestation calls the clinic and reports increased bladder pressure but easier breathing and irregular, mild contractions. She also states that she just cleaned the entire house. Which statement should the nurse make? 1. "You shouldn't work so much at this point in pregnancy." 2. "What you are describing is not commonly experienced in the last weeks." 3. "Your body may be telling you it is going into labor soon." 4. "If the bladder pressure continues, come in to the clinic tomorrow."

Answer: 3 Explanation: 3. One of the premonitory signs of labor is lightening: The fetus begins to settle into the pelvic inlet (engagement). With fetal descent, the uterus moves downward, and the fundus no longer presses on the diaphragm, which eases breathing.

A client has experienced a stillbirth. Which statement by the nurse would be appropriate? 1. "You are young. You can try again." 2. "At least you have your other children." 3. "I'm sure you had many dreams and hopes for the future." 4. "It's a blessing in disguise."

Answer: 3 Explanation: 3. Perinatal loss is unique in that the parents have not had experiences with the child that was to be, and attachment is based mostly upon hopes and dreams for the future relationship.

The nurse is examining a pregnant woman in the third trimester. What skin changes should the nurse highlight as an alteration for the woman's healthcare provider? 1. Linea nigra 2. Melasma gravidarum 3. Petechiae 4. Vascular spider nevi

Answer: 3 Explanation: 3. Petechiae are pinpoint red or purple spots on the skin. They are seen in hemorrhagic conditions.

The community nurse is working with a client whose only child is 8 months old. Which statement does the nurse expect the mother to make? 1. "I have a lot more time to myself than I thought I would have." 2. "My confidence level in my parenting is higher than I anticipated." 3. "I am constantly tired. I feel like I could sleep for a week." 4. "My baby likes everyone, and never fusses when she's held by a stranger."

Answer: 3 Explanation: 3. Physical fatigue often affects adjustments and functions of the new mother. The nurse can also provide information about the fatigue that a new mother experiences, strategies to promote rest and sleep at home, and the impact fatigue can have on a woman's emotions and sense of control.

Induction of labor is planned for a 31-year-old client at 39 weeks due to insulin-dependent diabetes. Which nursing action is most important? 1. Administer 100 mcg of misoprostol (Cytotec) vaginally every 2 hours. 2. Place dinoprostone (Prepidil) vaginal gel and ambulate client for 1 hour. 3. Begin Pitocin (oxytocin) 4 hours after 50 mcg misoprostol (Cytotec). 4. Prepare to induce labor after administering a tap water enema.

Answer: 3 Explanation: 3. Pitocin should not administered less than 4 hours after the last Cytotec dose.

The nurse assesses the gestational age of a newborn and informs the parents that the newborn is premature. Which of the following assessment findings is not congruent with prematurity? 1. Cry is weak and feeble 2. Clitoris and labia minora are prominent 3. Strong sucking reflex 4. Lanugo is plentiful

Answer: 3 Explanation: 3. Poor suck, gag, and swallow reflexes are characteristic of a preterm newborn.

The postpartum client states that she doesn't understand why she can't enjoy being with her baby. What would the nurse be concerned about? 1. Postpartum psychosis 2. Postpartum infection 3. Postpartum depression 4. Postpartum blues

Answer: 3 Explanation: 3. Postpartum depression can impair maternal-infant bonding and can cause developmental and cognitive delays in the child.

The postpartum client who is being discharged from the hospital experienced severe postpartum depression after her last birth. What should the nurse include in the plan of follow-up care for this client? 1. One visit from a homecare nurse, to take place in 2 days 2. Two visits from a public health nurse over the next month 3. An appointment with a mental health counselor 4. Follow-up with the obstetrician in 6 weeks

Answer: 3 Explanation: 3. Postpartum depression has a high recurrence rate. Women with a history of postpartum psychosis or depression or other risk factors may benefit from a referral to a mental health professional for counseling during pregnancy or postpartum.

A young adolescent is transferred to the labor and delivery unit from the emergency department. The client is in active labor, but did not know she was pregnant. What is the most important nursing action? 1. Determine who might be the father of the baby for paternity testing. 2. Ask the client what kind of birthing experience she would like to have. 3. Assess blood pressure and check for proteinuria. 4. Obtain a Social Services referral to discuss adoption.

Answer: 3 Explanation: 3. Preeclampsia is more common among adolescents than in young adults, and is potentially life-threatening to both mother and fetus. This assessment is the highest priority.

The prenatal client in her third trimester tells the clinic nurse that she works 8 hours a day as a cashier and stands when at work. What response by the nurse is best? 1. "No problem. Your baby will be fine." 2. "Do you get regular breaks for eating?" 3. "Your risk of preterm labor is higher." 4. "Standing might increase ankle swelling."

Answer: 3 Explanation: 3. Pregnant women who are employed in jobs that require prolonged standing (more than 3 hours) do have a higher incidence of preterm birth.

A client with a normal prepregnancy weight asks why she has been told to gain 25-35 pounds during her pregnancy while her underweight friend was told to gain more weight. What should the nurse tell the client the recommended weight gain is during pregnancy? 1. 25-35 pounds, regardless of a client's prepregnant weight 2. More than 25-35 pounds for an overweight woman 3. Up to 40 pounds for an underweight woman 4. The same for a normal weight woman as for an overweight woman

Answer: 3 Explanation: 3. Prepregnant weight determines the recommended weight gain during pregnancy. Underweight women are advised to gain 28-40 pounds.

Which of the following may lead to the development of disseminated intravascular coagulation (DIC), also called consumption coagulopathy, in the mother? 1. Hypertensive disorders 2. Abruptio placentae 3. Prolonged retention of the dead fetus 4. Heritable thrombophilias

Answer: 3 Explanation: 3. Prolonged retention of the dead fetus may lead to the development of disseminated intravascular coagulation (DIC), also called consumption coagulopathy, in the mother.

The nurse has received a phone call from a multigravida who is 21 weeks pregnant and has not felt fetal movement yet. What is the best action for the nurse to take? 1. Reassure the client that this is a normal finding in multigravidas. 2. Suggest that she should feel for movement with her fingertips. 3. Schedule an appointment for her with her physician for that same day. 4. Tell her gently that her fetus is probably dead.

Answer: 3 Explanation: 3. Quickening, or the mother's perception of fetal movement, occurs about 18 to 20 weeks after the LMP in a primigravida (a woman who is pregnant for the first time) but may occur as early as 16 weeks in a multigravida (a woman who has been pregnant more than once).

The nurse is teaching a class on infant care to new parents. Which statement by a parent indicates that additional teaching is needed? 1. "The white spots on my baby's nose are called milia, and are harmless." 2. "The whitish cheeselike substance in the creases is vernix, and will be absorbed." 3. "The red spots with a white center on my baby are abnormal acne." 4. "Jaundice is a yellowish discoloration of skin that if noticed on the 1st day of life should be reported to the physician."

Answer: 3 Explanation: 3. Red spots with white or yellow centers are erythema toxicum. The peak incidence is at 24 to 48 hours of life. The condition rarely presents at birth or after 5 days of life. The cause is unknown, and no treatment is necessary.

Screening for gestational diabetes mellitus (GDM) is typically completed between which of the following weeks of gestation? 1. 36 and 40 weeks 2. Before 20 weeks 3. 24 and 28 weeks 4. 30 and 34 weeks

Answer: 3 Explanation: 3. Screening for gestational diabetes mellitus (GDM) is typically completed between 24 and 28 weeks' gestation.

The physician has prescribed the medication clomiphene citrate (Clomid) for a client with infertility. What should the nurse's instructions to the woman include? 1. "Have intercourse every day of 1 week, starting 5 days after completion of medication." 2. "This medication is administered intravenously." 3. "Contact the doctor if visual disturbances occur." 4. "A contraindication is kidney disease."

Answer: 3 Explanation: 3. Side effects of clomiphene citrate include visual symptoms such as spots and flashes.

A postpartum client reports sharp, shooting pains in her nipple during breastfeeding and flaky, itchy skin on her breasts. Which of the following does the nurse suspect? 1. Nipple soreness 2. Engorgement 3. Mastitis 4. Letdown reflex

Answer: 3 Explanation: 3. Signs of mastitis include late-onset nipple pain, followed by shooting pain between feedings, often radiating to the chest wall. Eventually, the skin of the affected breast may become pink, flaking, and pruritic.

A client from Mexico has just delivered a son, and the nurse offers to assist in putting the baby to breast. Although the client indicated before the birth that she wanted to breastfeed, she is very hesitant, and says she would like to bottle-feed for the first few days. After talking to her, the nurse understands that her primary reason for wanting to delay breastfeeding is based on what cultural belief? 1. Breast milk causes skin rashes. 2. It is harmful to breastfeed immediately. 3. Colostrum is bad for the baby. 4. Thin milk causes diarrhea.

Answer: 3 Explanation: 3. Some Hispanics may delay breastfeeding because they believe colostrum is "bad."

The nurse is supervising a student nurse who is working with a 14-year-old client who delivered her first child yesterday. Which statement indicates that the nursing student understands the particular needs of an adolescent client? 1. "This client will need less teaching, because she will have gotten the right information in school." 2. "Because of her age, this client will require less frequent fundal checks to assess for postpartal hemorrhage." 3. "Because of her age, this client will probably need extra teaching about the terminology for her anatomy." 4. "This client will need to have her grandmother provide day care and help raise the baby."

Answer: 3 Explanation: 3. Some adolescents may not have a working knowledge of their own anatomy and physiology or the related terminology, and they may require special assistance with postpartum hygiene and care.

A client has delivered a stillborn infant at 28 weeks' gestation. Which nursing action is appropriate? 1. Discuss funeral options for the baby. 2. Encourage the couple to try to get pregnant again soon. 3. Ask the couple whether or not they would like to hold the baby. 4. Advise the couple that the baby's death was probably for the best.

Answer: 3 Explanation: 3. Some parents will hold their infant for a short time before returning him or her to the nurse, whereas others will wish to spend a great deal of time with their infant. Allow the infant to remain with the parents for as long as they desire.

A mother states that her breasts leak between feedings. Which of the following can contribute to the letdown reflex in breastfeeding mothers? 1. Pain with breastfeeding 2. Number of hours passed since last feeding 3. The newborn's cry 4. Maternal fluid intake

Answer: 3 Explanation: 3. Some women will leak milk when their breasts are full and it is nearly time to breastfeed again or whenever they experience letdown, which can be triggered by hearing, seeing, or even thinking of their baby.

On the first postpartum day, the nurse teaches the client about breastfeeding. Two hours later, the mother seems to remember very little of the teaching. The nurse understands this memory lapse to be related to which of the following? 1. The taking-hold phase 2. Postpartum hemorrhage 3. The taking-in period 4. Epidural anesthesia

Answer: 3 Explanation: 3. Soon after birth during the taking-in period, the woman tends to be passive and somewhat dependent. She follows suggestions, hesitates about making decisions, and is still rather preoccupied with her needs.

A client who wishes to have an unmedicated birth is in the transition stage. She is very uncomfortable and turns frequently in the bed. Her partner has stepped out momentarily. How can the nurse be most helpful? 1. Talk to the client the entire time. 2. Turn on the television to distract the client. 3. Stand next to the bed with hands on the railing next to the client. 4. Sit silently in the room away from the bed.

Answer: 3 Explanation: 3. Standing next to the bed is supportive without being irritating. The laboring woman fears being alone during labor. The woman's anxiety may be decreased when the nurse remains with her.

An expectant father has been at the bedside of his laboring partner for more than 12 hours. An appropriate nursing intervention would be to do which of the following? 1. Insist that he leave the room for at least the next hour. 2. Tell him he is not being as effective as he was, and that he needs to let someone else take over. 3. Offer to remain with his partner while he takes a break. 4. Suggest that the client's mother might be of more help.

Answer: 3 Explanation: 3. Support persons frequently are reluctant to leave the laboring woman to take care of their own needs. The laboring woman often fears being alone during labor. Even though there is a support person available, the woman's anxiety may be decreased when the nurse remains with her while he takes a break.

The nurse has instructed a new mother on quieting activities for her newborn. The nurse knows that the mother understands when she overhears the mother telling the father to do what? 1. Hold the newborn in an upright position. 2. Massage the hands and feet. 3. Swaddle the newborn in a blanket. 4. Make eye contact while talking to the newborn.

Answer: 3 Explanation: 3. Swaddling or bundling the baby increases a sense of security and is a quieting activity.

A new father asks the nurse to describe what his baby will experience while sleeping and awake. What is the best response? 1. "Babies have several sleep and alert states. Keep watching and you'll notice them." 2. "You might have noticed that your child was in an alert awake state for an hour after birth." 3. "Newborns have two stages of sleep: deep or quiet sleep and rapid eye movement sleep." 4. "Birth is hard work for babies. It takes them a week or two to recover and become more awake."

Answer: 3 Explanation: 3. Teaching the parents how to recognize the two sleep stages helps them tune in to their infant's behavioral states.

During an assessment of a 12-hour-old newborn, the nurse notices pale pink spots on the nape of the neck. The nurse documents this finding as which of the following? 1. Nevus vasculosus 2. Nevus flammeus 3. Telangiectatic nevi 4. A Mongolian spot

Answer: 3 Explanation: 3. Telangiectatic nevi (stork bites) appear as pale pink or red spots and are frequently found on the eyelids, nose, lower occipital bone, and nape of the neck.

The nurse suspects that a client has developed a perineal hematoma. What assessment findings would the nurse have detected to lead to this conclusion? 1. Facial petechiae 2. Large, soft hemorrhoids 3. Tense tissues with severe pain 4. Elevated temperature

Answer: 3 Explanation: 3. Tenseness of tissues that overlie the hematoma is characteristic of perineal hematomas.

During a non-stress test, the nurse notes that the fetal heart rate decelerates about 15 beats during a period of fetal movement. The decelerations occur twice during the test, and last 20 seconds each. The nurse realizes these results will be interpreted as which of the following? 1. A negative test 2. A reactive test 3. A nonreactive test 4. An equivocal test

Answer: 3 Explanation: 3. The FHR acceleration must be at least 15 beats per minute above baseline for at least 15 seconds from baseline to baseline. A nonreactive NST is one that lacks sufficient FHR accelerations over a 40-minute period.

A laboring client asks the nurse, "Why does the physician want to use an intrauterine pressure catheter (IUPC) during my labor?" The nurse would accurately explain that the best rationale for using an IUPC is which of the following? 1. The IUPC can be used throughout the birth process. 2. A tocodynamometer is invasive. 3. The IUPC provides more accurate data than does the tocodynamometer. 4. The tocodynamometer can be used only after the cervix is dilated 2 cm.

Answer: 3 Explanation: 3. The IUPC has several benefits over an external tocotransducer or palpation. Because the IUPC is inserted directly into the uterus, it provides near-exact pressure measurements for contraction intensity and uterine resting tone. The increased sensitivity of the IUPC allows for very accurate timing of uterine contractions (UCs).

If the physician indicates a shoulder dystocia during the delivery of a macrosomic fetus, how would the nurse assist? 1. Call a second physician to assist. 2. Prepare for an immediate cesarean delivery. 3. Assist the woman into McRoberts maneuver. 4. Utilize fundal pressure to push the fetus out.

Answer: 3 Explanation: 3. The McRoberts maneuver is thought to change the maternal pelvic angle and therefore reduce the force needed to extract the shoulders, thereby decreasing the incidence of brachial plexus stretching and clavicular fracture.

The nurse is instructing a new mother on circumcision care with a Plastibell. The nurse knows the mother understands when she states that the Plastibell should fall off within how long? 1. 2 days 2. 10 days 3. 8 days 4. 14 days

Answer: 3 Explanation: 3. The Plastibell should fall off within 8 days. If it remains on after 8 days, the parents should consult with the newborn's physician.

The pregnant client cannot tolerate milk or meat. What would the nurse recommend to the client to assist in meeting protein needs? 1. Wheat bread and pasta 2. Ice cream and peanut butter 3. Eggs and tofu 4. Beans and potatoes

Answer: 3 Explanation: 3. The best food choices that are nondairy and complete proteins alone are eggs and tofu.

The student nurse is to perform Leopold maneuvers on a laboring client. Which assessment requires intervention by the staff nurse? 1. The client is assisted into supine position, and the position of the fetus is assessed. 2. The upper portion of the uterus is palpated, then the middle section. 3. After determining where the back is located, the cervix is assessed. 4. Following voiding, the client's abdomen is palpated from top to bottom.

Answer: 3 Explanation: 3. The cervical exam is not part of Leopold maneuvers. Abdominal palpation is the only technique used for Leopold maneuvers.

A woman asks her nurse what she can do before she begins trying to get pregnant to help her baby, as she is prone to anemia. What would the nurse correctly advise her to do? 1. Get pregnant, then start iron supplementation. 2. Add more carbohydrates to her diet. 3. Begin taking folic acid supplements daily. 4. Have a hemoglobin baseline done now so her progress can be followed.

Answer: 3 Explanation: 3. The common anemias of pregnancy are due either to insufficient hemoglobin production related to nutritional deficiency in iron or folic acid during pregnancy. Folic acid deficiency during pregnancy is prevented by a daily supplement of 0.4 mg (400 micrograms) of folate.

The nurse knows that a lecithin/sphingomyelin (L/S) ratio finding of 2:1 on amniotic fluid means which of the following? 1. Fetal lungs are still immature. 2. The fetus has a congenital anomaly. 3. Fetal lungs are mature. 4. The fetus is small for gestational age.

Answer: 3 Explanation: 3. The concentration of lecithin begins to exceed that of sphingomyelin, and at 35 weeks the L/S ratio is 2:1. When at least two times as much lecithin as sphingomyelin is found in the amniotic fluid, RDS is very unlikely.

The nurse has just palpated contractions and compares the consistency to that of the forehead to estimate the firmness of the fundus. What would the intensity of these contractions be identified as? 1. Mild 2. Moderate 3. Strong 4. Weak

Answer: 3 Explanation: 3. The consistency of strong contractions is similar to that of the forehead.

The nurse is caring for an infant born at 37 weeks that weighs 1750 g (3 pounds 10 ounces). The head circumference and length are in the 25th percentile. What statement would the nurse expect to find in the chart? 1. Preterm appropriate for gestational age, symmetrical IUGR 2. Term small for gestational age, symmetrical IUGR 3. Preterm small for gestational age, asymmetrical IUGR 4. Preterm appropriate for gestational age, asymmetrical IUGR

Answer: 3 Explanation: 3. The infant is preterm at 37 weeks. Because the weight is below the 10th percentile, the infant is small for gestational age. Head circumference and length between the 10th and 90th percentiles indicate asymmetrical IUGR.

The nurse is teaching a class on perinatal loss to student nurses. What would the nurse explain about the relationship between attachment and the grief response? 1. The mother has no attachment to the fetus before it is born. 2. The severity of the grieving has nothing to do with attachment to the fetus. 3. The intensity of the grief response can be assessed by determining the level of attachment to the anticipated infant. 4. The mother would feel grief only if it were a planned pregnancy.

Answer: 3 Explanation: 3. The intensity to which the grief will be experienced is best understood from the aspect of the level of attachment the grieving person had to the deceased and usually entails finding personal meaning in the loss for successful integration into the grieving person's life.

The postpartum client has developed thrombophlebitis in her right leg. Which finding requires immediate intervention? 1. The client reports she had this condition after her last pregnancy. 2. The client develops pain and swelling in her left lower leg. 3. The client appears anxious, and describes pressure in her chest. 4. The client becomes upset that she cannot go home yet.

Answer: 3 Explanation: 3. The most common clinical findings of a pulmonary embolism include dyspnea, pleuritic chest pain, cough with or without hemoptysis, cyanosis, tachypnea and tachycardia, panic, syncope, or sudden hypotension and require immediate intervention.

The nurse is aware that labor and birth will most likely proceed normally when the fetus is in what position? 1. Right-acromion-dorsal-anterior 2. Right-sacrum-transverse 3. Occiput anterior 4. Posterior position

Answer: 3 Explanation: 3. The most common fetal position is occiput anterior. When this position occurs, labor and birth are likely to proceed normally.

The nurse is planning care for a newborn. Which nursing intervention would best protect the newborn from the most common form of heat loss? 1. Placing the newborn away from air currents 2. Pre-warming the examination table 3. Drying the newborn thoroughly 4. Removing wet linens from the isolette

Answer: 3 Explanation: 3. The most common form of heat loss is evaporation. The newborn is particularly prone to heat loss by evaporation immediately after birth (when the baby is wet with amniotic fluid) and during baths; thus drying the newborn is critical.

What is the most significant cause of neonatal morbidity and mortality? 1. Amenorrhea 2. Posttraumatic stress disorder 3. Prematurity 4. Endometriosis

Answer: 3 Explanation: 3. The most significant cause of neonatal morbidity and mortality is prematurity and its associated complications such as respiratory distress syndrome, necrotizing enterocolitis, and intraventricular hemorrhage.

When a breastfeeding mother complains that her breasts are leaking milk, the nurse can offer which effective intervention? 1. Decrease the number of minutes the newborn is at the breast per feeding. 2. Decrease the mother's fluid intake. 3. Place absorbent pads in the bra. 4. Administer oxytocin.

Answer: 3 Explanation: 3. The mother can wear nursing pads inside her bra with instructions to change wet pads frequently.

A new grandfather is marveling over his 12-hour-old newborn grandson. Which statement indicates that the grandfather needs additional education? 1. "I can't believe he can already digest fats, carbohydrates, and proteins." 2. "It is amazing that his whole digestive tract can move things along at birth." 3. "Incredibly, his stomach capacity was already a cupful when he was born." 4. "He will lose some weight but then miraculously regain it by about 10 days

Answer: 3 Explanation: 3. The newborn's stomach has a capacity of 22 mL to 27 mL by day 3 of life.

Which of the following tests has become a widely accepted method of evaluating fetal status? 1. Contraction stress test (CST) 2. MSAFP test 3. Non-stress test (NST) 4. Nuchal translucency test

Answer: 3 Explanation: 3. The non-stress test (NST) has become a widely accepted method of evaluating fetal status. This test involves using an external electronic fetal monitor to obtain a tracing of the fetal heart rate (FHR) and observation of acceleration of the FHR with fetal movement.

The parents of a preterm newborn wish to visit their baby in the NICU. A statement by the nurse that would not support the parents as they visit their newborn is which of the following? 1. "Your newborn likes to be touched." 2. "Stroking the newborn will help with stimulation." 3. "Visits must be scheduled between feedings." 4. "Your baby loves her pink blanket."

Answer: 3 Explanation: 3. The nurse always should encourage parents to visit and get to know their newborn, even in the NICU. Nurses foster the development of a safe, trusting environment by viewing the parents as essential caregivers, not as visitors or nuisances in the unit.

The nurse should explain to new parents that their infant's position should be changed periodically during the early months of life to prevent which of the following? 1. Muscle contractures 2. Respiratory distress 3. Permanently flattened areas of the skull 4. Esophageal reflux

Answer: 3 Explanation: 3. The nurse can describe plagiocephaly as a flattened area on the head and can recommend that parents alternate their infant's head position between the right and the left side when placing the infant supine for sleep. Placing the infant's head at alternate ends of the crib every few days is helpful as well.

The nurse is providing guidance for a woman in her second trimester of pregnancy and telling her about some of the signs and symptoms that she might experience. Which statement by the client indicates that further teaching is necessary? 1. "During the third trimester, I might have frequent urination." 2. "During the third trimester, I might have heartburn." 3. "During the third trimester, I might have back pain." 4. "During the third trimester, I might have a persistent headache."

Answer: 3 Explanation: 3. The nurse responds to this most critical statement because of the danger of fetal alcohol syndrome.

The breastfeeding client asks the nurse about appropriate contraception. What is the nurse's best response? 1. "Breastfeeding has many effects on sexual intercourse." 2. "IUDs are easy to use and easy to insert prior to sexual intercourse." 3. "It's possible to get pregnant before your menstrual period returns. Let's talk about some different options for contraception." 4. "Breastfeeding hampers ovulation, so no contraception is needed."

Answer: 3 Explanation: 3. The nurse should discuss the importance of contraception during the early postpartum period and provide information on the advantages and disadvantages of different methods, including special considerations for breastfeeding mothers. The woman's body needs adequate time to heal and recover from the stress of pregnancy and childbirth.

The pregnant client in her second trimester states, "I didn't know my breasts would become so large. How do I find a good bra?" The best answer for the nurse to make would be which of the following? 1. "Avoid cotton fabrics and get an underwire bra; they fit everyone best." 2. "Just buy a bra one cup size bigger than usual, and it will fit." 3. "Look for wide straps and cups big enough for all of your breast tissue." 4. "There isn't much you can do for comfort. Try not wearing a bra at all."

Answer: 3 Explanation: 3. The nurse should instruct the client to get a bra that fits with straps that are wide and do not stretch and a cup that holds all breast tissue comfortably.

The nurse is planning a homecare visit to a mother who just recently delivered. The neighborhood is known to have a significant crime rate. What should the nurse do when planning this visit to facilitate personal safety? 1. Be friendly to all pets encountered on the visit to build client rapport. 2. Wait to find the exact location until arrival in the neighborhood. 3. Put personal possessions in the trunk when leaving the office. 4. Wear flashy jewelry to garner respect.

Answer: 3 Explanation: 3. The nurse should lock personal belongings in the trunk of the car, out of sight, before starting out or before arriving at the home.

The nurse in a prenatal clinic finds that four clients have called with complaints related to their pregnancies. Which call should the nurse return first? 1. Pregnant woman at 7 weeks' gestation reporting nasal stuffiness 2. Pregnant woman at 38 weeks' gestation experiencing rectal itching and hemorrhoids 3. Pregnant woman at 15 weeks' gestation with nausea and vomiting and a 15-pound weight loss 4. Pregnant woman at 32 weeks' gestation treating constipation with prune juice

Answer: 3 Explanation: 3. The nurse should return this call first because this patient is the highest priority. A 15-pound weight loss is not an expected finding. Although some nausea is common, the woman who suffers from extreme nausea coupled with vomiting requires further assessment.

A client comes into the prenatal clinic accompanied by her boyfriend. When asked by the nurse why she is there, the client looks down, and the boyfriend states, "She says she is pregnant. She constantly complains of feeling tired, and her vomiting is disgusting." What is a priority for the nurse to do at this point? 1. Ask the client what time of the day her fatigue is more common. 2. Recommend that the woman have a pregnancy test done as soon as possible. 3. Continue the interview of the client in private. 4. Give the woman suggestions on ways to decrease the vomiting.

Answer: 3 Explanation: 3. The nurse should suspect that the client is in an abusive relationship. The priority is for the nurse to get the client away from the boyfriend and continue the interview.

The postpartum nurse is caring for a client who gave birth to full-term twins earlier today. The nurse will know to assess for symptoms of which of the following? 1. Increased blood pressure 2. Hypoglycemia 3. Postpartum hemorrhage 4. Postpartum infection

Answer: 3 Explanation: 3. The nurse will assess for postpartum hemorrhage. Overstretching of uterine muscles with conditions such as multiple gestation, polyhydramnios, or a very large baby may set the stage for slower uterine involution.

A couple is seeking advice regarding what they can do to increase the chances of becoming pregnant. What recommendation can the nurse give to the couple? 1. The couple could use vaginal lubricants during intercourse. 2. The couple should delay having intercourse until the day of ovulation. 3. The woman should refrain from douching. 4. The woman should be on top during intercourse.

Answer: 3 Explanation: 3. This is the correct answer, as douching can alter sperm mobility.

The nurse determines the fundus of a postpartum client to be boggy. Initially, what should the nurse do? 1. Document the findings. 2. Catheterize the client. 3. Massage the uterine fundus until it is firm. 4. Call the physician immediately.

Answer: 3 Explanation: 3. The nurse would massage the uterine fundus until it is firm by keeping one hand in position and stabilizing the lower portion of the uterus. With one hand used to massage the fundus, the nurse would put steady pressure on the top of the now-firm fundus and to see if she was able to express any clots.

The client has been pushing for 2 hours and is exhausted. The physician is performing a vacuum extraction to assist the birth. Which finding is expected and normal? 1. The head is delivered after eight "pop-offs" during contractions. 2. A cephalohematoma is present on the fetal scalp. 3. The location of the vacuum is apparent on the fetal scalp after birth. 4. Positive pressure is applied by the vacuum extraction during contractions.

Answer: 3 Explanation: 3. The parents need to be informed that the caput (chignon) on the baby's head will disappear within 2 to 3 days.

The nurse is observing a new graduate perform a postpartum assessment. Which action requires intervention by the nurse? 1. Asking the client to void and donning clean gloves 2. Listening to bowel sounds and then asking when her last bowel movement occurred 3. Offering the patient pre-medication 2 hours before the assessment 4. Completing the assessment and explaining the results to the client

Answer: 3 Explanation: 3. The patient should be offered premedication 30-45 minutes before assessing the fundus, especially if the patient has had a cesareansection.

A client is admitted to the labor and delivery unit with contractions that are regular, are 2 minutes apart, and last 60 seconds. She reports that her labor began about 6 hours ago, and she had bloody show earlier that morning. A vaginal exam reveals a vertex presenting, with the cervix 100% effaced and 8 cm dilated. The client asks what part of labor she is in. The nurse should inform the client that she is in what phase of labor? 1. Latent phase 2. Active phase 3. Transition phase 4. Fourth stage

Answer: 3 Explanation: 3. The transition phase begins with 8 cm of dilatation, and is characterized by contractions that are closer and more intense.

A client is admitted to the labor and delivery unit with contractions that are 2 minutes apart, lasting 60 seconds. She reports that she had bloody show earlier that morning. A vaginal exam reveals that her cervix is 100 percent effaced and 8 cm dilated. The nurse knows that the client is in which phase of labor? 1. Active 2. Latent 3. Transition 4. Fourth

Answer: 3 Explanation: 3. The transition phase begins with 8 cm to 10 cm of dilatation, and contractions become more frequent, are longer in duration, and increase in intensity.

A client who is 11 weeks pregnant presents to the emergency department with complaints of dizziness, lower abdominal pain, and right shoulder pain. Laboratory tests reveal a beta-hCG at a lower-than-expected level for this gestational age. An adnexal mass is palpable. Ultrasound confirms no intrauterine gestation. The client is crying and asks what is happening. The nurse knows that the most likely diagnosis is an ectopic pregnancy. Which statement should the nurse include? 1. "You're feeling dizzy because the pregnancy is compressing your vena cava." 2. "The pain is due to the baby putting pressure on nerves internally." 3. "The baby is in the fallopian tube; the tube has ruptured and is causing bleeding." 4. "This is a minor problem. The doctor will be right back to explain it to you."

Answer: 3 Explanation: 3. The woman who experiences one-sided lower abdominal pain or diffused lower abdominal pain, vasomotor disturbances such as fainting or dizziness, and referred right shoulder pain from blood irritating the subdiaphragmatic phrenic nerve is experiencing an ectopic pregnancy.

The nurse educator is describing the different kinds of abruptio placentae to a group of students, explaining that in a complete abruptio placentae, which of the following occurs? 1. Separation begins at the periphery of the placenta. 2. The placenta separates centrally and blood is trapped between the placenta and the uterine wall. 3. There is massive vaginal bleeding in the presence of almost total separation. 4. Blood passes between the fetal membranes and the uterine wall, and escapes vaginally.

Answer: 3 Explanation: 3. There is massive vaginal bleeding in the presence of almost total separation describes a complete separation of the placenta.

The nurse is caring for a client at 30 weeks' gestation who is experiencing preterm premature rupture of membranes (PPROM). Which statement indicates that the client needs additional teaching? 1. "If I were having a singleton pregnancy instead of twins, my membranes would probably not have ruptured." 2. "If I develop a urinary tract infection in my next pregnancy, I might rupture membranes early again." 3. "If I want to become pregnant again, I will have to plan on being on bed rest for the whole pregnancy." 4. "If I have aminocentesis, I might rupture the membranes again."

Answer: 3 Explanation: 3. There is no evidence that bed rest in a subsequent pregnancy decreases the risk for PPROM.

The nurse is admitting a client to the labor and delivery unit. Which aspect of the client's history requires notifying the physician? 1. Blood pressure 120/88 2. Father a carrier of sickle-cell trait 3. Dark red vaginal bleeding 4. History of domestic abuse

Answer: 3 Explanation: 3. Third-trimester bleeding is caused by either placenta previa or abruptio placentae. Dark red bleeding usually indicates abruptio placentae, which is life-threatening to both mother and fetus.

Several adolescent female students are waiting to be seen by the school sexual health clinic nurse. Which student should the nurse see first? 1. 14-year-old whose 17-year-old sister is pregnant 2. 15-year-old who reports using condoms regularly 3. 16-year-old who had chlamydia treated 2 weeks ago 4. 17-year-old with a history of child abuse

Answer: 3 Explanation: 3. This client is the top priority. Teenagers 15 to 19 years old have a high incidence of STIs. The incidence of chlamydial infection is increased in this age group. The presence of such infections during a pregnancy greatly increases the risk to the fetus.

The nurse is presenting a prenatal class to a group of women pregnant for the first time who are all over 35 years of age. The nurse knows that the advantage of waiting until later to start a family is which of the following? 1. That the woman will have an easier labor and delivery. 2. That the baby will be at less of a risk for congenital anomalies. 3. That the woman is more likely to be financially secure. 4. That the woman will be more fertile than a younger woman would.

Answer: 3 Explanation: 3. This delay in starting a family allows women to pursue advanced educational degrees and prepare financially for the impact children will have on their lives.

The client in the prenatal clinic tells the nurse that she is sure she is pregnant because she has not had a menstrual cycle for 3 months, and her breasts are getting bigger. What response by the nurse is best? 1. "Lack of menses and breast enlargement are presumptive signs of pregnancy." 2. "The changes you are describing are definitely indicators that you are pregnant." 3. "Lack of menses can be caused by many things. We need to do a pregnancy test." 4. "You're probably not pregnant, but we can check it out if you like."

Answer: 3 Explanation: 3. This is a true statement, and addresses that these changes could be caused by conditions other than pregnancy.

The nurse is providing education to the new family. Which question by the nurse is best? 1. "Do you know how to give the baby a bath?" 2. "You have diapers and supplies at home, right?" 3. "How have your breastfeedings been going?" 4. "How much formal education do you have?"

Answer: 3 Explanation: 3. This is an open-ended question about an important physiologic issue. A discussion that includes both partners can facilitate an open dialog between them and can provide an opportunity for questions and answers.

The nurse is completing a newborn care class. The nurse knows that teaching has been effective if a new parent states which of the following? 1. "My baby might open her arms wide and pull her legs up to her tummy if she is passing gas." 2. "When I hold my baby upright with one of his feet on the floor, his feet will automatically remain still. 3. "When I put my finger in the palm of my daughter's hand, she will curl her fingers and hold on." 4. "I can get my baby to turn his head toward the right if I lift his right arm over his head."

Answer: 3 Explanation: 3. This is the Palmar grasp reflex and is elicited by stimulating the newborn's palm with a finger or object.

Which statement by a new mother 1 week postpartum indicates maternal role attainment? 1. "I don't think I'll ever know what I'm doing." 2. "This baby feels like a real stranger to me." 3. "It works better for me to undress the baby and to nurse in the chair rather than the bed." 4. "My sister took to mothering in no time. Why can't I?"

Answer: 3 Explanation: 3. This statement indicates a stage of maternal role attainment in which the new mother feels comfortable enough to make her own decisions about parenting.

The nurse is talking with the parents of a pregnant 15-year-old. Which statement by one of the parents is expected? 1. "We're very happy for her. It will be easier to focus on education with a new baby." 2. "I'm not going to get involved. She understands how her health insurance works." 3. "Her father told her to stop dating that boy. Now look at the trouble she's gotten into." 4. "An abortion is the best choice for her. She can deal with our Catholic priest later."

Answer: 3 Explanation: 3. This statement indicates anger, which is to be expected when a parent finds out about a teen daughter's pregnancy.

The nurse is preparing a client in her second trimester for a three-dimensional ultrasound examination. Which statement indicates that teaching has been effective? 1. "If the ultrasound is normal, it means my baby has no abnormalities." 2. "The nuchal translucency measurement will diagnose Down syndrome." 3. "I might be able to see who the baby looks like with the ultrasound." 4. "Measuring the length of my cervix will determine whether I will deliver early."

Answer: 3 Explanation: 3. Three-dimensional ultrasound uses algorithms to vary opacity, transparency, and depth to project an image. This allows curved structures such as the fetal face to be viewed.

During the initial prenatal visit, the pregnant teenager states that she does not know how she got pregnant. The nurse can help to educate her regarding anatomy by doing what? 1. Allowing her to witness a pelvic exam on another teenager 2. Encouraging her to ask her mother about the physiology of pregnancy 3. Including anatomic models and drawings in the teaching session 4. Discussing the process of fetal development with the client

Answer: 3 Explanation: 3. To teach the client about anatomy and physiology, use simple explanations and lots of audiovisuals.

The nurse is planning an educational presentation on hyperbilirubinemia for nursery nurses. Which statement is most important to include in the presentation? 1. Conjugated bilirubin is eliminated in the conjugated state. 2. Unconjugated bilirubin is neurotoxic, and cannot cross the placenta. 3. Total bilirubin is the sum of the direct and indirect levels. 4. Hyperbilirubinemia is a decreased total serum bilirubin level.

Answer: 3 Explanation: 3. Total serum bilirubin is the sum of conjugated (direct) and unconjugated (indirect) bilirubin.

Which relief measure would be most appropriate for a postpartum client with superficial thrombophlebitis? 1. Urge ambulation 2. Apply ice to the leg 3. Elevate the affected limb 4. Massage her calf

Answer: 3 Explanation: 3. Treatment for superficial thrombophlebitis involves application of local heat, elevation of the affected limb, and analgesic agents.

The postpartum client is concerned about mastitis because she experienced it with her last baby. Preventive measures the nurse can teach include which of the following? 1. Wearing a tight-fitting bra 2. Limiting breastfeedings 3. Frequent breastfeedings 4. Restricting fluid intake

Answer: 3 Explanation: 3. Treatment of mastitis includes frequent and complete emptying of the breasts.

At 1 minute after birth, the infant has a heart rate of 100 beats per minute, and is crying vigorously. The limbs are flexed, the trunk is pink, and the feet and hands are cyanotic. The infant cries easily when the soles of the feet are stimulated. How would the nurse document this infant's Apgar score? 1. 7 2. 8 3. 9 4. 10

Answer: 3 Explanation: 3. Two points each are scored in each of the categories of heart rate, respiratory effort, muscle tone, and reflex irritability. One point is scored in the category of skin color. The total Apgar would be 9.

The client with an abnormal quadruple screen is scheduled for an ultrasound. Which statement indicates that the client understands the need for this additional antepartal fetal surveillance? 1. "After the ultrasound, my partner and I will decide how to decorate the nursery." 2. "During the ultrasound we will see which of us the baby looks like most." 3. "The ultrasound will show whether there are abnormalities with the baby's spine." 4. "The blood test wasn't run correctly, and now we need to have the sonogram."

Answer: 3 Explanation: 3. Ultrasound is used to detect neural tube defects. An abnormal serum quadruple screen is not the result of a lab error, and can indicate either an open neural tube defect or trisomy 18 or 21.

What is the advantage of a client using a patient-controlled analgesia (PCA) following a cesarean birth? 1. The client receives a bolus of the analgesia when pressing the button. 2. The client experiences pain relief within 30 minutes. 3. The client feels a greater sense of control, and is less dependent on the nursing staff. 4. The client can deliver as many doses of the medication as needed.

Answer: 3 Explanation: 3. Using a special intravenous (IV) pump system, the woman presses a button to self-administer small doses of the medication as needed. For safety, the pump is preset with a time lockout so that the pump cannot deliver another dose until a specified time has elapsed. Women using PCA feel less anxious and have a greater sense of control with less dependence on the nursing staff.

The charge nurse is assessing several postpartum clients. Which client has the greatest risk for postpartum hemorrhage? 1. The client who was overdue and delivered vaginally 2. The client who delivered by scheduled cesarean delivery 3. The client who had oxytocin augmentation of labor 4. The client who delivered vaginally at 36 weeks

Answer: 3 Explanation: 3. Uterine atony is a cause of postpartal hemorrhage. A contributing factor to uterine atony is oxytocin augmentation of labor.

The nurse attempts to elicit the Moro reflex on a newborn, and assesses movement of the right arm only. Based on this finding, the nurse immediately assesses for which of the following? 1. Ortolani maneuver 2. Palmar grasping reflex 3. Clavicle 4. Tonic neck reflex

Answer: 3 Explanation: 3. When the Moro reflex is elicited, the newborn straightens arms and hands outward while the knees flex. Slowly the arms return to the chest, as in an embrace. If this response is not elicited, the nurse assesses the clavicle for a possible fracture.

A client is consulting a certified nurse-midwife because she is hoping for a vaginal birth after cesarean (VBAC) with this pregnancy. Which statement indicates that the client requires more information about VBAC? 1. "I can try a vaginal birth because my uterine incision is a low segment transverse incision." 2. "The vertical scar on my skin doesn't mean that the scar on my uterus goes in the same direction." 3. "There is about a 90% chance of giving birth vaginally after a cesarean." 4. "Because my hospital has a surgery staff on call 24 hours a day, I can try a VBAC there."

Answer: 3 Explanation: 3. Women whose previous cesarean was performed because of nonrecurring indications have been reported to have approximately a 60% to 80% chance of success with VBAC.

The nurse knows that in some cases, breastfeeding is not advisable. Which mother should be counseled against breastfeeding? 1. A mother with a poorly balanced diet 2. A mother who is overweight 3. A mother who is HIV positive 4. A mother who has twins

Answer: 3 Explanation: 3. Women with HIV or AIDS are counseled against breastfeeding.

The nurse is preparing a class for mothers and their partners who have just recently delivered. One topic of the class is infant attachment. Which statement by a participant indicates an understanding of this concept? Note: Credit will be given only if all correct choices and no incorrect choices are selected. Select all that apply. 1. "We should avoid holding the baby too much." 2. "Looking directly into the baby's eyes might frighten him." 3. "Talking to the baby is good because he'll recognize our voices." 4. "Holding the baby so we have direct face-to-face contact is good." 5. "We should only touch the baby with our fingertips for the first month."

Answer: 3, 4 Explanation: 3. Attachment behaviors include cuddling, soothing, and calling the baby by name. 4. Attachment behaviors include holding the baby in the en face position.

Lacerations of the cervix or vagina may be present when bright red vaginal bleeding persists in the presence of a well-contracted uterus. The incidence of lacerations is higher among which of the following childbearing women? Note: Credit will be given only if all correct choices and no incorrect choices are selected. Select all that apply. 1. Over the age of 35 2. Have not had epidural block 3. Have had an episiotomy 4. Have had a forceps-assisted or vacuum-assisted birth 5. Nulliparous

Answer: 3, 4 Explanation: 3. The incidence of lacerations is higher among childbearing women who undergo an episiotomy. 4. The incidence of lacerations is higher among childbearing women who undergo forceps-assisted or vacuum-assisted birth.

The labor and birth nurse is admitting a client. The nurse's assessment includes asking the client whom she would like to have present for the labor and birth, and what the client would prefer to wear. The client's partner asks the nurse the reason for these questions. What would the nurse's best response be? Note: Credit will be given only if all correct choices and no incorrect choices are selected. Select all that apply. 1. "These questions are asked of all women. It's no big deal." 2. "I'd prefer that your partner ask me all the questions, not you." 3. "A client's preferences for her birth are important for me to understand." 4. "Many women have beliefs about childbearing that affect these choices." 5. "I'm gathering information that the nurses will use after the birth."

Answer: 3, 4 Explanation: 3. The nurse incorporates the family's expectations into the plan of care to be culturally appropriate and to facilitate the birth. 4. The nurse incorporates the family's expectations into the plan of care to be culturally appropriate and to facilitate the birth.

A diabetic client goes into labor at 36 weeks' gestation. Provided that tests for fetal lung maturity are successful, the nurse will anticipate which of the following interventions? Note: Credit will be given only if all correct and no incorrect choices are selected. Select all that apply. 1. Administration of tocolytic therapy 2. Beta-sympathomimetic administration 3. Allowance of labor to progress 4. Hourly blood glucose monitoring 5. Cesarean birth may be indicated if evidence of reassuring fetal status exists

Answer: 3, 4 Explanation: 3. There will be no attempt to stop the labor, as this can compromise the mother and fetus. 4. To reduce incidence of congenital anomalies and other problems in the newborn, the woman should be euglycemic (have normal blood glucose) throughout the pregnancy.

Clinical risk factors for severe hyperbilirubinemia include which of the following? Note: Credit will be given only if all correct and no incorrect choices are selected. Select all that apply. 1. African American ethnicity 2. Female gender 3. Cephalohematoma 4. Bruising 5. Assisted delivery with vacuum or forceps

Answer: 3, 4, 5 Explanation: 3. A clinical risk factor for severe hyperbilirubinemia includes cephalohematoma. 4. A clinical risk factor for severe hyperbilirubinemia includes bruising. 5. A clinical risk factor for severe hyperbilirubinemia includes assisted delivery with vacuum or forceps.

The nurse is caring for laboring clients. Which women are experiencing problems related to a critical factor of labor? Note: Credit will be given only for all correct choices and no incorrect choices. Select all that apply. 1. Woman at 7 cm, fetus in general flexion 2. Woman at 3 cm, fetus in longitudinal lie 3. Woman at 4 cm, fetus with transverse lie 4. Woman at 6 cm, fetus at -2 station, mild contractions 5. Woman at 5 cm, fetal presenting part is right shoulder

Answer: 3, 4, 5 Explanation: 3. A transverse lie occurs when the cephalocaudal axis of the fetal spine is at a right angle to the woman's spine and is associated with a shoulder presentation and can lead to complications in the later stages of labor. 4. Station refers to the relationship of the presenting part to an imaginary line drawn between the ischial spines of the maternal pelvis. If the presenting part is higher than the ischial spines, a negative number is assigned, noting centimeters above zero station. A -2 station is high in the pelvis. Contractions should be strong to cause fetal descent. Mild contractions will not move the baby down or open the cervix. This client is experiencing a problem between the maternal pelvis and the presenting part. 5. When the fetal shoulder is the presenting part, the fetus is in a transverse lie and the acromion process of the scapula is the landmark. This type of presentation occurs less than 1% of the time. This client is experiencing a problem between the maternal pelvis and the presenting part.

A woman is scheduled to have an external version for a breech presentation. The nurse carefully reviews the client's chart for contraindications to this procedure, including which of the following? Note: Credit will be given only if all correct choices and no incorrect choices are selected. Select all that apply. 1. Station -2 2. 38 weeks' gestation 3. Abnormal fetal heart rate and tracing 4. Previous cesarean section 5. Rupture of membranes

Answer: 3, 4, 5 Explanation: 3. An abnormal fetal heart rate or tracing would be a contraindication to performing a version. A nonreassuring FHR pattern might indicate that the fetus is already stressed and other action needs to be taken. 4. A previous cesarean is a contraindication for version. 5. Rupture of membranes is a contraindication for version because of insufficient amniotic fluid.

A pregnant client has a hemoglobin of 10 g/dL and a Hct of 30%. The clinic nurse recognizes the fetus is at risk for which of the following? Note: Credit will be given only if all correct and no incorrect choices are selected. Select all that apply. 1. Macrosomia 2. Respiratory distress syndrome 3. Low birth weight 4. Prematurity 5. Fetal death

Answer: 3, 4, 5 Explanation: 3. Anemia places the fetus at risk for a low birth weight. 4. Anemia places the fetus at risk for premature birth. 5. Anemia places the fetus at risk for fetal death.

The OB-GYN nurse is assessing a pregnant client, and recognizes genetic amniocentesis will be indicated. The nurse makes this conclusion because the indications for genetic amniocentesis include which of the following? Note: Credit will be given only if all correct and no incorrect choices are selected. Select all that apply. 1. Maternal age under 35 2. Fetus with no abnormalities on ultrasound 3. One child with a chromosome abnormality 4. A family history of neural tube defects 5. Both parents with an abnormal chromosome

Answer: 3, 4, 5 Explanation: 3. Couples who have had a child with trisomy 21, 18, or 13 have approximately a 1% risk or their age-related risk, whichever is higher, of a future child having a chromosome abnormality. 4. Family history of neural tube defects is an indication for genetic amniocentesis. 5. If both parents carry an autosomal recessive disease, they have a 25% chance with each pregnancy that the fetus will be affected.

Which couples may benefit from prenatal diagnosis? Note: Credit will be given only if all correct and no incorrect choices are selected. Select all that apply. 1. Couples including women under the age of 35 2. Couples with an unbalanced translocation 3. Couples with a family history of known or suspected single-gene disorder 4. Couples including women with a teratogenic risk secondary to an exposure or maternal health condition 5. Family history of birth defects and/or intellectual disability

Answer: 3, 4, 5 Explanation: 3. Couples with a family history of known or suspected single-gene disorder (e.g., cystic fibrosis, hemophilia A or B, Duchenne muscular dystrophy) may benefit from prenatal diagnosis. 4. Women with a teratogenic risk secondary to an exposure or maternal health condition (e.g., diabetes, seizure disorder) may benefit from prenatal diagnosis. 5. Family history of birth defects and/or intellectual disability (mental retardation) (e.g., neural tube defects, congenital heart disease, cleft lip and/or palate) may benefit from prenatal diagnosis.

An abbreviated systematic physical assessment of the newborn is performed by the nurse in the birthing area to detect any abnormalities. Normal findings would include which of the following? Note: Credit will be given only if all correct choices and no incorrect choices are selected. Select all that apply. 1. Skin color: Body blue with pinkish extremities 2. Umbilical cord: two veins and one artery 3. Respiration rate of 30-60 irregular 4. Temperature of above 36.5°C (97.8°F) 5. Sole creases that involve the heel

Answer: 3, 4, 5 Explanation: 3. Normal findings would include a respiration rate of 30-60 irregular. 4. Normal findings would include temperature of above 36.5°C (97.8°F). 5. Normal findings would include sole creases that involve the heel.

A 7 pound 14 ounce girl was born to an insulin-dependent type II diabetic mother 2 hours ago. The infant's blood sugar is 47 mg/dL. What is the best nursing action? 1. To recheck the blood sugar in 6 hours 2. To begin an IV of 10% dextrose 3. To feed the baby 1 ounce of formula 4. To document the findings in the chart

Answer: 4 Explanation: 4. A blood sugar level of 47 mg/dL is a normal finding; documentation is an appropriate action.

The nurse is working with parents who have just experienced the birth of their first child at 34 weeks. Which statements by the parents indicate that additional teaching is needed? Note: Credit will be given only if all correct choices and no incorrect choices are selected. Select all that apply. 1. "Our baby will be in an incubator to keep him warm." 2. "Breathing might be harder for our baby because he is early." 3. "The growth of our baby will be faster than if he were term." 4. "Tube feedings will be required because his stomach is small." 5. "Because he came early, he will not produce urine for 2 days."

Answer: 3, 4, 5 Explanation: 3. Preterm infants grow more slowly than do term infants because of difficulty in meeting high caloric and fluid needs for growth due to small gastric capacity. 4. Although tube feedings might be required, it would be because preterm babies have a marked danger of aspiration and its associated complications due to the infant's poorly developed gag reflex, incompetent esophageal cardiac sphincter, and inadequate suck/swallow/breathe reflex. 5. Although preterm babies have diminished kidney function due to incomplete development of the glomeruli, they can produce urine. Preterm infants usually have some urine output during the first 24 hours of life.

True postterm pregnancies are frequently associated with placental changes that cause a decrease in the uterine-placental-fetal circulation. Complications related to alternations in placenta functioning include which of the following? Note: Credit will be given only if all correct choices and no incorrect choices are selected. Select all that apply. 1. Increased fetal oxygenation 2. Increased placental blood supply 3. Reduced nutritional supply 4. Macrosomia 5. Risk of shoulder dystocia

Answer: 3, 4, 5 Explanation: 3. Reduced nutritional supply is a complication related to alternations in placenta functioning. 4. Macrosomia is a complication related to alternations in placenta functioning. 5. Risk of shoulder dystocia is a complication

The nurse is meeting with a new mother for the first time during a home visit. The client delivered her first child 3 days ago. She had a normal pregnancy and a vaginal delivery. The infant is breastfeeding. Which statements by the mother indicate that she needs more information about the home visit? Note: Credit will be given only if all correct choices and no incorrect choices are selected. Select all that apply. 1. "You are going to check my baby's weight." 2. "You are going to watch me nurse the baby and give me tips." 3. "You are going to teach my mother about the baby." 4. "You are checking for safety issues when my son starts crawling." 5. "You are going to take blood samples from me and my son."

Answer: 3, 4, 5 Explanation: 3. Teaching of family members might occur, but the main purpose of the visit is to assess the infant's physiologic stability. 4. Safety when the infant crawls should be assessed later. 5. Not all home visits require blood samples. If there were no pregnancy or birth complications, there may not be the need to draw blood from either the mother or the child.

The nurse has given a community presentation on adolescent pregnancy. Which statements indicate that further teaching is necessary? Note: Credit will be given only if all correct choices and no incorrect choices are selected. Select all that apply. 1. "Low educational achievement is major risk factor for adolescent pregnancy." 2. "Teens who are on a low economic trajectory are more likely to become pregnant." 3. "When a teen plans to attend college, abortion is less likely." 4. "In the United States, the adolescent birth rate is higher among white teens." 5. "Teens from low-income households are less likely to become pregnant."

Answer: 3, 4, 5 Explanation: 3. Teens with future goals (i.e., college or job) tend to use birth control more consistently compared with other teens; if they become pregnant, they are also more likely to have abortions. 4. In the United States, the adolescent birth rate is higher among African American and Hispanic teens than among white teens. 5. Poverty is a major risk factor for adolescent pregnancy. As many as 85% of births to unmarried teens occur to those from poor or low-income families.

The nurse is planning discharge teaching for a postpartum woman. What information recommendations should the woman receive before being discharged? Note: Credit will be given only if all correct choices and no incorrect choices are selected. Select all that apply. 1. To abstain from sexual intercourse for 6 months 2. To avoid showers for 4 weeks 3. To avoid overexertion 4. To practice postpartum exercises 5. To obtain adequate rest

Answer: 3, 4, 5 Explanation: 3. The client should avoid overexertion. 4. The client should receive information and instruction on postpartum exercises. 5. The client should receive information on the need for adequate rest.

What should the healthcare provider consider when prescribing a medication to a woman who is breastfeeding? Note: Credit will be given only if all correct choices and no incorrect choices are selected. Select all that apply. 1. Drug's potential effect on hormone production 2. Amount of drug excreted into the mother's blood 3. Drug's potential adverse effects to the infant 4. Infant's age and health 5. Mother's need for the medication

Answer: 3, 4, 5 Explanation: 3. The healthcare provider should consider the drug's potential adverse effects to the infant. 4. The healthcare provider should consider the infant's age and health. 5. The healthcare provider should consider the mother's need for the medication

A nurse working with couples undergoing genetic testing recognizes which of the following as nursing responsibilities? Note: Credit will be given only if all correct and no incorrect choices are selected. Select all that apply. 1. Allowing the family to interact with the genetic counselor without interference 2. Giving information about support groups when asked 3. Identifying families at risk for genetic problems 4. Aiding families in coping with the crisis 5. Ensuring continuity of nursing care to the family

Answer: 3, 4, 5 Explanation: 3. The nurse has a responsibility to identify families at risk for genetic problems. 4. The nurse should aid families in coping. 5. The nurse needs to ensure continuity of care to the family.

Five clients are in active labor in the labor unit. Which women should the nurse monitor carefully for the potential of uterine rupture? Note: Credit will be given only if all correct choices and no incorrect choices are selected. Select all that apply. 1. Age 15, in active labor 2. Age 22, with eclampsia 3. Age 25, last delivery by cesarean section 4. Age 32, first baby died during labor 5. Age 27, last delivery 11 months ago

Answer: 3, 5 Explanation: 3. A woman who has had a previous cesarean section is at risk for uterine rupture. 5. A woman who does not have at least 18 months between deliveries is at greater risk for uterine rupture.

The midwife performs a vaginal exam and determines that the fetal head is at a -2 station. What does this indicate to the nurse about the birth? 1. The birth is imminent. 2. The birth is likely to occur in 1-2 hours. 3. The birth will occur later in the shift. 4. The birth is difficult to predict.

Answer: 4 Explanation: 4. A -2 station means that the fetus is 2 cm above the ischial spines. The ischial spines as a landmark have been designated as zero station. If the presenting part is higher than the ischial spines, a negative number is assigned, noting centimeters above zero station. With the fetus's head that high in the pelvis, it is difficult to predict when birth will occur.

A breastfeeding mother calls the pediatric clinic concerned about her 4-day-old baby's failure to gain weight. She states that the infant has lost several ounces since birth. The most appropriate response by the nurse would be which of the following? Note: Credit will be given only if all correct choices and no incorrect choices are selected. Select all that apply. 1. "Newborns tend to lose about 5% to 10% of their birth weight because of failure to give adequate supplements when breastfeeding." 2. "Newborns grow approximately 1 inch a month in the first 6 months. You will need to increase feedings to compensate for the growth spurt." 3. "Newborns have an initial weight loss in the first 3 to 4 days. Your baby's weight loss is normal." 4. "Newborns lose a lot of heat, so make sure you keep the baby's formula warm when you supplement the breast milk." 5. "Keep the baby from getting chilled or too warm because that can contribute to weight loss."

Answer: 3, 5 Explanation: 3. Newborns have a physiological weight loss of 5% to 10% in the first 3 or 4 days. 5. Weight loss in the newborn can be caused by temperature elevation or consistent chilling.

The nurse is teaching experienced postpartum nurses about homecare visits. Which statements indicate that teaching was effective? Note: Credit will be given only if all correct choices and no incorrect choices are selected. Select all that apply. 1. "I should tell the family to put any guns or knives away." 2. "It is best to blend in with the community and not bring attention to myself on visits." 3. "If I encounter a crime in progress, I should leave the area." 4. "Wearing jewelry is a good way to demonstrate my professionalism." 5. "Ignoring my 'gut' feelings might lead to an unsafe situation."

Answer: 3, 5 Explanation: 3. Nurses should avoid entering areas where violence is in progress. In such cases, they should return to the car and contact the appropriate authorities by calling 911. 5. The nurse should terminate the visit if a situation arises that feels unsafe or if the previous requests are not honored.

The nurse is assessing the newborn for symptoms of anemia. If the blood loss is acute, the baby may exhibit which of the following signs of shock? Note: Credit will be given only if all correct choices and no incorrect choices are selected. Select all that apply. 1. Increased pulse 2. High blood pressure 3. Tachycardia 4. Bradycardia 5. Capillary filling time greater than 3 seconds

Answer: 3, 5 Explanation: 3. Tachycardia would be a sign of shock. 5. Capillary filling time greater than 3 seconds would be a sign of shock.

The nurse admits into the labor area a client who is in preterm labor. What assessment finding would constitute a diagnosis of preterm labor? 1. Cervical effacement of 30% or more 2. Cervical change of 0.5 cm per hour 3. 2 contractions in 30 minutes 4. 8 contractions in 1 hour

Answer: 4 Explanation: 4. 8 contractions in a 60 minute period does define a diagnosis of preterm labor.

A postpartum client with endometritis is being discharged home on antibiotic therapy. The new mother plans to breastfeed her baby. What should the nurse's discharge instruction include? 1. The client can douche every other day. 2. Sexual intercourse can be resumed when the client feels up to it. 3. Light housework will provide needed exercise. 4. The baby's mouth should be examined for thrush.

Answer: 4 Explanation: 4. A breastfeeding mother on antibiotics should check her baby's mouth for signs of thrush, which should be reported to the physician.

The nurse is explaining induction of labor to a client. The client asks what the indications for labor induction are. Which of the following should the nurse include when answering the client? 1. Suspected placenta previa 2. Breech presentation 3. Prolapsed umbilical cord 4. Hypertension

Answer: 4 Explanation: 4. A client with hypertension is appropriate for labor induction.

The nurse is performing a postpartum assessment on a newly delivered client. When checking the fundus, there is a gush of blood. The client asks why that is happening. What is the nurse's best response? 1. "We see this from time to time. It's not a big deal." 2. "The gush is an indication that your fundus isn't contracting." 3. "Don't worry. I'll make sure everything is fine." 4. "Blood pooled in the vagina while you were in bed."

Answer: 4 Explanation: 4. A gush of blood when a fundal massage is undertaken may occur because of normal pooling of blood in vagina when the woman lies down to rest or sleep.

The nurse is caring for a pregnant client. The client's husband has come to the prenatal visit. Which question is the best for the nurse to use to assess the father's adaptation to the pregnancy? 1. "What kind of work do you do?" 2. "What furniture have you gotten for the baby?" 3. "How moody has your wife been lately?" 4. "How are you feeling about becoming a father?"

Answer: 4 Explanation: 4. A husband's adaptation to pregnancy includes his feelings about impending fatherhood.

The nurse is discharging a 15-year-old first-time mother. Which statement should the nurse include in the discharge teaching? 1. "Call your pediatrician if the baby's temperature is below 98.6°F axillary." 2. "Your baby's stools will change to a greenish color when your milk comes in." 3. "You can wipe away any eye drainage that might form." 4. "Your infant should wet a diaper at least 6 times per day."

Answer: 4 Explanation: 4. A minimum of 6 to 10 wet diapers per day indicates adequate fluid intake.

The nurse is returning phone calls from clients. Which client does the nurse anticipate would not require a serum beta hCG? 1. A client with a risk of ectopic pregnancy 2. A client with spotting during pregnancy 3. A client with previous pelvic inflammatory disease 4. A client with a previous history of twins

Answer: 4 Explanation: 4. A previous history of twins is not a risk factor for ectopic pregnancy. Beta hCG testing is not indicated for this client.

The client has delivered a 4200 g fetus. The physician performed a midline episiotomy, which extended into a third-degree laceration. The client asks the nurse where she tore. Which response is best? 1. "The episiotomy extended and tore through your rectal mucosa." 2. "The episiotomy extended and tore up near your vaginal mucous membrane." 3. "The episiotomy extended and tore into the muscle layer." 4. "The episiotomy extended and tore through your anal sphincter."

Answer: 4 Explanation: 4. A third degree laceration includes the anal sphincter.

The couple at 12 weeks' gestation has been told that their fetus has sickle cell disease. Which statement by the couple indicates that they are adequately coping? 1. "We knew we were both carriers of sickle cell. We shouldn't have tried to have a baby." 2. "If we had been healthier when we conceived, our baby wouldn't have this disease now." 3. "Taking vitamins before we got pregnant would have prevented this from happening." 4. "The doctor told us there was a 25% chance that our baby would have sickle disease."

Answer: 4 Explanation: 4. A true statement indicates coping. When both parents are carriers of an autosomal recessive disease, there is a 25% risk for each pregnancy that the fetus will be affected.

The parents of a newborn comment to the nurse that their infant seems to enjoy being held, and that holding the baby helps him calm down after crying. They ask the nurse why this happens. After explaining newborn behavior, the nurse assesses the parents' learning. Which statement indicates that teaching was effective? 1. "Some babies are easier to deal with than others." 2. "We are lucky to have a baby with a calm disposition." 3. "Our baby spends more time in the active alert phase." 4. "Cuddliness is a social behavior that some babies have."

Answer: 4 Explanation: 4. According to Brazelton Neonatal Behavioral Assessment Scale, cuddliness can be an indicator of personality.

A 15-year-old client has delivered a 22-week stillborn fetus. What does the nurse understand? 1. Grieving a fetal loss manifests with very similar behaviors regardless of the age of the client. 2. Teens tend to withhold emotions and need older adults with the same type of loss to help process the experience. 3. Most teens have had a great deal of contact with death and loss and have an established method of coping. 4. Assisting the client might be difficult because of her mistrust of authority figures.

Answer: 4 Explanation: 4. Adolescents rely heavily on peer support and have a natural mistrust of authority figures, which can make assisting them more difficult.

In the operating room, a client is being prepped for a cesarean delivery. The doctor is present. What is the last assessment the nurse should make just before the client is draped for surgery? 1. Maternal temperature 2. Maternal urine output 3. Vaginal exam 4. Fetal heart tones

Answer: 4 Explanation: 4. Ascertain fetal heart rate (FHR) before surgery and during preparation because fetal hypoxia can result from aortocaval compression.

After inserting prostaglandin gel for cervical ripening, what should the nurse do? 1. Apply an internal fetal monitor. 2. Insert an indwelling catheter. 3. Withhold oral intake and start intravenous fluids. 4. Place the client in a supine position with a right hip wedge.

Answer: 4 Explanation: 4. After the gel, intravaginal insert, or tablet is inserted, the woman is instructed to remain lying down with a rolled blanket or hip wedge under her right hip to tip the uterus slightly to the left for the first 30 to 60 minutes to maintain the cervical ripening agent in place.

Each of the following pregnant women is scheduled for a 14-week antepartal visit. In planning care, the nurse would give priority teaching on amniotic fluid alpha-fetoprotein (AFP) screening to which client? 1. 28-year-old with history of rheumatic heart disease 2. 18-year-old with exposure to HIV 3. 20-year-old with a history of preterm labor 4. 35-year-old with a child with spina bifida

Answer: 4 Explanation: 4. Alpha-fetoprotein (AFP) is a fetal protein that is excreted from the fetal yolk sac during the first 6 weeks of pregnancy. AFP levels can be high or low, with each having different implications for the fetus. If the fetus has a neural tube defect (NTD), the AFP levels will be elevated. NTDs can range from anencephaly to spina bifida. With a past history of a child with spina bifida, this client would be strongly encouraged to have the AFP screening.

The nurse is assessing a newly pregnant client. Which finding does the nurse note as a normal psychosocial adjustment in this client's first trimester? 1. An unlisted telephone number 2. Reluctance to tell the partner of the pregnancy 3. Parental disapproval of the woman's partner 4. Ambivalence about the pregnancy

Answer: 4 Explanation: 4. Ambivalence toward a pregnancy is a common psychosocial adjustment in early pregnancy.

The nurse is analyzing assessment findings on four newborns. Which finding might suggest a congenital heart defect? 1. Apical heart rate of 140 beats per minute 2. Respiratory rate of 40 3. Temperature of 36.5°C 4. Visible, blue discoloration of the skin

Answer: 4 Explanation: 4. Central cyanosis is defined as a visible, blue discoloration of the skin caused by decreased oxygen saturation levels and is a common manifestation of a cardiac defect.

The nurse is caring for a new breastfeeding mother who is from Pakistan. The nurse plans her care so that the newborn is offered the breast on which of the following? 1. Day of birth 2. First day after birth 3. Second day after birth 4. Third to fourth day after birth

Answer: 4 Explanation: 4. Among some traditional cultures around the world, it is believed that colostrum is "unclean" or even harmful to a newborn. Because of this ancient belief, mothers living the Middle East and parts of Asia even today discard their colostrum or wait 2 to 4 days to begin breastfeeding, when their "true milk" arrives. This mother would begin breastfeeding the third or fourth day after the birth.

A newborn has been diagnosed with a disorder that occurs through an autosomal recessive inheritance pattern. The parents ask the nurse, "Which of us passed on the gene that caused the disorder?" Which answer should the nurse tell them? 1. The female 2. The male 3. Neither 4. Both

Answer: 4 Explanation: 4. An affected individual can have clinically normal parents, but both parents are generally carriers of the abnormal gene.

When preparing nutritional instruction, which pregnant client would the nurse consider the highest priority? 1. 40-year-old gravida 2 2. 22-year-old primigravida 3. 35-year-old gravida 4 4. 15-year-old nulligravida

Answer: 4 Explanation: 4. An expectant adolescent must meet the nutritional needs for her own growth in addition to the nutritional needs of pregnancy.

The nurse is preparing to assess a laboring client who has just arrived in the labor and birth unit. Which statement by the client indicates that additional education is needed? 1. "You are going to do a vaginal exam to see how dilated my cervix is." 2. "The reason for a pelvic exam is to determine how low in the pelvis my baby is." 3. "When you check my cervix, you will find out how thinned out it is." 4. "After you assess my pelvis, you will be able to tell when I will deliver."

Answer: 4 Explanation: 4. An experienced labor and birth nurse can estimate the time of delivery based on the cervix, fetal position, station, and contraction pattern. However, during a pelvic exam, no information is obtained about contractions. The nurse will not have enough information following the cervical exam to estimate time of birth.

The client at 20 weeks' gestation has had an ultrasound that revealed a neural tube defect in her fetus. The client's hemoglobin level is 8.5. The nurse should include which statement when discussing these findings with the client? 1. "Your low iron intake has caused anemia, which leads to the neural tube defect." 2. "You should increase your vitamin C intake to improve your anemia." 3. "You are too picky about food. Your poor diet caused your baby's defect." 4. "You haven't had enough folic acid in your diet. You should take a supplement."

Answer: 4 Explanation: 4. An inadequate intake of folic acid has been associated with neural tube defects (NTDs) (e.g., spina bifida, anencephaly, meningomyelocele) in the fetus or newborn.

The nurse is discussing parent-infant attachment with a prenatal class. Which statement indicates that teaching was successful? 1. "I should avoid looking directly into the baby's eyes to prevent frightening the baby." 2. "My baby will be very sleepy immediately after birth and should go to the nursery." 3. "Newborns cannot focus their eyes, so it doesn't matter how I hold my new baby." 4. "Giving the baby his first bath can really give me a chance to get to know him."

Answer: 4 Explanation: 4. Another situation that can facilitate attachment is the interactive bath. While bathing their newborn for the first time, parents attend closely to their baby's behavior and the nurse can observe and point out behaviors.

The nurse is preparing to receive a newly delivered client. The client is a young single mother who is relinquishing custody of her newborn through an open adoption. What action is most important? 1. Assign the client a room on the GYN surgical floor instead of on the postpartum floor. 2. Prepare to complete teaching in time for discharge at 24 hours post-delivery. 3. Make an effort not to bring up the topic of the baby, and discuss the mother's health instead. 4. Ask the client how much contact she would like with the baby, and whether she wants to feed it.

Answer: 4 Explanation: 4. Assessing the birth mother's preferences by respectfully asking questions and making no assumptions facilitates a more positive experience.

A client scheduled a laparoscopy. After the procedure, what does the nurse instruct the client to do? 1. Stay on bed rest for 48 hours. 2. Expect to have shoulder and arm pain. 3. Purchase a rectal tube to relieve the gas. 4. Lie on her back to relieve the gas pain after the procedure.

Answer: 4 Explanation: 4. Assuming a supine position may help relieve residual shoulder and chest discomfort caused by any remaining gas.

The nurse is preparing to assess a newborn's neurological status. Which finding would require an immediate intervention? 1. At rest, the infant has partially flexed arms and the legs drawn up to the abdomen. 2. When the corner of the mouth is touched, the infant turns the head that direction. 3. The infant blinks when the exam light is turned on over the face and body. 4. The right arm is flaccid while the infant brings the left arm and fist upward to the head.

Answer: 4 Explanation: 4. Asymmetrical movement is not an expected finding, and could indicate neurological abnormality. Muscle tone should be symmetric and diminished muscle tone and flaccidity requires further evaluation.

Nurses should educate parents about which of the following AAP recommendations to promote a safe sleep environment and decrease the risk of SIDS and SUID in infants less than 12 months of age? 1. Babies should not be offered a pacifier while falling asleep. 2. Babies should be bottlefed unless contraindicated. 3. Babies should be under many covers when sleeping to keep them warm. 4. Babies should have "tummy time" when they are awake.

Answer: 4 Explanation: 4. Babies should have "tummy time" when they are awake and observed by an adult to prevent positional plagiocephaly and to promote motor development.

Toward the end of the first stage of labor, a pudendal block is administered transvaginally. What will the nurse anticipate the client's care will include? 1. Monitoring for hypotension every 15 minutes 2. Monitoring FHR every 15 minutes 3. Monitoring for bladder distention 4. No additional assessments

Answer: 4 Explanation: 4. Because a pudendal block is done using a local anesthetic, there is no need for additional monitoring of the mother or the fetus.

As compared with admission considerations for an adult woman in labor, the nurse's priority for an adolescent in labor would be which of the following? 1. Cultural background 2. Plans for keeping the infant 3. Support persons 4. Developmental level

Answer: 4 Explanation: 4. Because her cognitive development is incomplete, the younger adolescent may have fewer problem-solving capabilities. The very young woman needs someone to rely on at all times during labor. She may be more childlike and dependent than older teens.

The nurse is teaching a pregnant 14-year-old client who is at 10 weeks' gestation about the expected body changes that will occur during pregnancy. Which client statement indicates that additional information is needed? 1. "My breasts are going to get even bigger than they've gotten over the past couple of years." 2. "My belly will gradually get more round, especially from the middle of pregnancy on." 3. "I will feel my baby move in about 16 weeks." 4. "My diet can stay the same even though I'm pregnant."

Answer: 4 Explanation: 4. Because the adolescent is at risk for anemia, she will need education regarding the importance of iron in her diet.

The nurse is caring for a laboring client. A cervical exam indicates 8 cm dilation. The client is restless, frequently changing position in an attempt to get comfortable. Which nursing action is most important? 1. Leave the client alone so she can rest. 2. Ask the family to take a coffee-and-snack break. 3. Encourage the client to have an epidural for pain. 4. Reassure the client that she will not be left alone.

Answer: 4 Explanation: 4. Because the client is in the transitional phase of the first stage of labor, she will not want to be left alone; staying with the client and reassuring her that she will not be alone are the highest priorities at this time.

The nurse caring for a postterm newborn would not perform what intervention? 1. Providing warmth 2. Frequently monitoring blood glucose 3. Observing respiratory status 4. Restricting breastfeeding

Answer: 4 Explanation: 4. Breastfeeding is an appropriate means of feeding for the postterm newborn.

Which third-trimester client would the nurse suspect might be having difficulty with psychological adjustments to her pregnancy? 1. A woman who says, "Either a boy or a girl will be fine with me" 2. A woman who puts her feet up and listens to some music for 15 minutes when she is feeling too stressed 3. A woman who was a smoker but who has quit at least for the duration of her pregnancy 4. A woman who has not investigated the kind of clothing or feeding methods the baby will need

Answer: 4 Explanation: 4. By the third trimester, the client should be planning and preparing for the baby (for example, living arrangements, clothing, feeding methods).

The nurse is caring for a client in active labor. The membranes spontaneously rupture, with a large amount of clear amniotic fluid. Which nursing action is most important to undertake at this time? 1. Assess the odor of the amniotic fluid. 2. Perform Leopold maneuvers. 3. Obtain an order for pain medication. 4. Complete a sterile vaginal exam.

Answer: 4 Explanation: 4. Checking the cervix will determine whether the cord prolapsed when the membranes ruptured. The nurse would assess for prolapsed cord via vaginal examination.

The hospital is developing a new maternity unit. What aspects should be included in the planning of the new unit to best promote family wellness? 1. Normal newborn nursery centrally located to all client rooms 2. A kitchen with a refrigerator stocked with juice and sandwiches 3. Small, cozy rooms with a client bed and rocking chair 4. A nursing care model based on providing couplet care

Answer: 4 Explanation: 4. Couplet care, which is care of both the mother and her baby, is an important part of the family-centered care approach, in which the infant remains at the mother's bedside and both are cared for by the same nurse.

Which of the following is a sign of dehydration in the newborn? 1. Slow, weak pulse 2. Soft, loose stools 3. Light colored, concentrated urine 4. Depressed fontanelles

Answer: 4 Explanation: 4. Depressed fontanelles are a sign of dehydration in the newborn.

To actively involve the postpartal client during discharge teaching, the postpartum nurse applies which learning principle? 1. Reprints of magazine articles 2. Classroom lectures 3. Audiotapes 4. Interactive nurse-patient relationships

Answer: 4 Explanation: 4. Effective parent learning requires precise timing of teaching, as well as choice of a teaching method that is effective for the family, such as DVDs and return demonstration. Content on self-care, infant care, and anticipatory guidance is important.

The client delivered her first child vaginally 7 hours ago. She has not voided since delivery. She has an IV of lactated Ringer's solution running at 100 mL/hr. Her fundus is firm and to the right of midline. What is the best nursing action? 1. To massage the fundus vigorously 2. To assess the client's pain level 3. To increase the rate of the IV 4. To assist the client to the bathroom

Answer: 4 Explanation: 4. Emptying the bladder is the top priority.

The nurse is speaking to a community group about the controversy regarding the length of the hospital stay for postpartum clients. Which statement indicates that a participant needs additional information? 1. "As of 1998, there's a law that requires insurance to pay for a 48-hour stay after an uncomplicated birth." 2. "The length of stay was shortened by insurance companies to decrease healthcare costs." 3. "Early discharge became more popular in the 1980s as an alternative to having a home birth." 4. "With current length-of-stay laws, newborns have no problems at home, and get recommended follow-up care."

Answer: 4 Explanation: 4. Even with the current length-of-stay laws, many newborns do not always receive the recommended follow-up care when they go home early. The health and stability of the mother and baby, the mother's ability and confidence regarding self and newborn care, support systems available, and access to follow-up care should form the basis of the decision.

The client with thalassemia intermedia has a hemoglobin level of 9.0. The nurse is preparing an education session for the client. Which statement should the nurse include? 1. "You need to increase your intake of meat and other iron-rich foods." 2. "Your low hemoglobin could put you into preterm labor." 3. "Increasing your vitamin C intake will help your hemoglobin level." 4. "You should not take iron supplements."

Answer: 4 Explanation: 4. Folic acid supplements are indicated for women with thalassemia, but iron supplements are not given.

A client is having contractions that last 20-30 seconds and that are occurring every 8-20 minutes. The client is requesting something to help relieve the discomfort of contractions. What should the nurse suggest? 1. That a mild analgesic be administered 2. An epidural 3. A local anesthetic block 4. Nonpharmacologic methods of pain relief

Answer: 4 Explanation: 4. For this pattern of labor, nonpharmacologic methods of pain relief should be suggested. These can include back rubs, providing encouragement, and clean linens.

A 26-year-old client is 26 weeks pregnant. Her previous births include two large-for-gestational-age babies and one unexplained stillbirth. Which tests would the nurse anticipate as being most definitive in diagnosing gestational diabetes? 1. A 50g, 1-hour glucose screening test 2. A single fasting glucose level 3. A 100g, 1-hour glucose tolerance test 4. A 100g, 3-hour glucose tolerance test

Answer: 4 Explanation: 4. Gestational diabetes is diagnosed if two or more of the following values are met or exceeded after taking the 100 g, 3-hour OGTT: Fasting: 95 mg/dL; 1 hour: 180 mg/dL; 2 hours: 155 mg/dL; 3 hours: 140 mg/dL.

A mother notices that her newborn is able to sleep without waking even when in the nursery with other newborns crying. The mother asks whether her baby might have a hearing problem because her father wears hearing aids. What should the nurse explain? 1. Newborn risk factors associated with potential hearing loss do not include a family history of hearing loss. 2. Newborns cannot hear, due to mucus accumulated in the middle ear, which takes several days to drain. 3. Newborns who are asleep do not respond to loud noises that are not accompanied by vibrations. 4. Newborns in a noisy nursery are able to habituate to the sounds, and might not react unless a sound is sudden or much louder.

Answer: 4 Explanation: 4. Habituation is the newborn's ability to diminish or shut down innate responses to specific repeated stimuli.

The nurse is beginning the postpartum teaching of a mother who has given birth to her first child. What aspect of teaching is most important? 1. Describe the likely reaction of siblings to the new baby. 2. Discuss adaptation to grandparenthood by her parents. 3. Determine whether father-infant attachment is taking place. 4. Assist the mother in identifying the baby's behavior cues.

Answer: 4 Explanation: 4. Helping the mother to identify her baby's behavior cues facilitates the acquaintance phase of maternal-infant attachment.

While doing a prenatal assessment on a woman who has hepatitis B and intends to become pregnant, the nurse explains the impact of the hepatitis B on pregnancy and birth. Which statement does the nurse include in the teaching? 1. "Your baby contracted hepatitis B from you when she was conceived." 2. "Don't worry about your baby during the birth. You're more likely to be affected then by the hepatitis B." 3. "Your baby will be immune to your hepatitis B." 4. "Hepatitis B does not usually affect the course of pregnancy."

Answer: 4 Explanation: 4. Hepatitis B does not usually affect the course of pregnancy.

A pregnant woman is having a nipple-stimulated contraction stress test. Which result indicates hyperstimulation? 1. The fetal heart rate decelerates when three contractions occur within a 10-minute period. 2. The fetal heart rate accelerates when contractions last up to 60 seconds. 3. There are more than five fetal movements in a 10-minute period. 4. There are more than three uterine contractions in a 6-minute period.

Answer: 4 Explanation: 4. Hyperstimulation is characterized by contractions that occur more frequently than every 2 minutes or last longer than 90 seconds.

A 38-week newborn is found to be small for gestational age (SGA). Which nursing intervention should be included in the care of this newborn? 1. Monitor for feeding difficulties. 2. Assess for facial paralysis. 3. Monitor for signs of hyperglycemia. 4. Maintain a warm environment.

Answer: 4 Explanation: 4. Hypothermia is a common complication in the SGA newborn; therefore, the newborn's environment must remain warm, to decrease heat loss.

) The nurse has assessed four newborns' respiratory rates immediately following birth. Which respiratory rate would require further assessment by the nurse? 1. 60 breaths per minute 2. 70 breaths per minute 3. 64 breaths per minute 4. 20 breaths per minute

Answer: 4 Explanation: 4. If respirations drop below 20 when the baby is at rest the primary care provider should be notified.

The postpartum homecare nurse is assessing a new mother, and finds her temperature to be 101.6°F. What is the most important nursing action? 1. Ask the mother how often and how well the baby is nursing. 2. Determine the frequency of the mother's voiding and stooling. 3. Verify how many hours of sleep she is getting per day. 4. Assess the odor and color of the lochia and perineum.

Answer: 4 Explanation: 4. If the lochia is malodorous, or if the perineum is reddened or malodorous, an infection is present that could be causing the fever.

A nurse is caring for a newborn on a ventilator who has respiratory distress syndrome (RDS). The nurse informs the parents that the newborn is improving. Which data support the nurse's assessment? 1. Decreased urine output 2. Pulmonary vascular resistance increases 3. Increased PCO2 4. Increased urination

Answer: 4 Explanation: 4. In babies with respiratory distress syndrome (RDS) who are on ventilators, increased urination/diuresis may be an early clue that the baby's condition is improving.

The nurse examines the client's placenta and finds that the umbilical cord is inserted at the placental margin. The client comments that the placenta and cord look different than they did for her first two births. The nurse should explain that this variation in placenta and cord is called what? 1. Placenta accreta 2. Circumvallate placenta 3. Succenturiate placenta 4. Battledore placenta

Answer: 4 Explanation: 4. In battledore placenta, the umbilical cord is inserted at or near the placental margin.

A newborn is receiving phototherapy. Which intervention by the nurse would be most important? 1. Measurement of head circumference 2. Encouraging the mother to stop breastfeeding 3. Stool blood testing 4. Assessment of hydration status

Answer: 4 Explanation: 4. Infants undergoing phototherapy treatment have increased water loss and loose stools as a result of bilirubin excretion. This increases their risk of dehydration.

The partner of a client at 16 weeks' gestation accompanies her to the clinic. The partner tells the nurse that the baby just doesn't seem real to him, and he is having a hard time relating to his partner's fatigue and food aversions. Which statement would be best for the nurse to make? 1. "If you would concentrate harder, you'd be aware of the reality of this pregnancy." 2. "My husband had no problem with this. What was your childhood like?" 3. "You might need professional psychological counseling. Ask your physician." 4. "Many men feel this way. Feeling the baby move in a few weeks will help make it real to you."

Answer: 4 Explanation: 4. Initially, expectant fathers may have ambivalent feelings.The extent of ambivalence depends on many factors, including the father's relationship with his partner, his previous experience with pregnancy, his age, his economic stability, and whether the pregnancy was planned. The expectant father must first deal with the reality of the pregnancy and then struggle to gain recognition as a parent from his partner, family, friends, coworkers, society-and from his baby as well.

The nurse working with mothers over 35 having their first baby knows there are some disadvantages. For what disadvantage would the nurse carefully assess in each client? 1. What kind of insurance the client has for maternity care 2. Whether the client is married 3. Whether the client will continue working after the baby arrives 4. Whether the client has any chronic disease that will have to be addressed

Answer: 4 Explanation: 4. It is important for the nurse to question and assess for any chronic illnesses. The risk of pregnancy complications is higher in women over age 35 who have a chronic condition such as hypertension or diabetes, or who are in poor general health.

A woman calls the clinic and tells a nurse that she thinks she might be pregnant. She wants to use a home pregnancy test before going to the clinic, and asks the nurse how to use it correctly. What information should the nurse give? 1. The false-positive rate of these tests is quite high. 2. If the results are negative, the woman should repeat the test in 2 weeks if she has not started her menstrual period. 3. A negative result merely indicates growing trophoblastic tissue and not necessarily a uterine pregnancy. 4. The client should follow up with a healthcare provider after taking the home pregnancy test.

Answer: 4 Explanation: 4. It is important that clients remember that the tests are not always accurate and they should follow up with a healthcare provider.

A woman at 28 weeks' gestation is asked to keep a fetal activity record and to bring the results with her to her next clinic visit. One week later, she calls the clinic and anxiously tells the nurse that she has not felt the baby move for more than 30 minutes. Which of the following would be the nurse's most appropriate initial comment? 1. "You need to come to the clinic right away for further evaluation." 2. "Have you been smoking?" 3. "When did you eat last?" 4. "Your baby might be asleep."

Answer: 4 Explanation: 4. Lack of fetal activity for 30 minutes typically is insignificant. Movement varies considerably, but most women feel fetal movement at least 10 times in 3 hours.

The nurse has presented a session on pain relief options to a prenatal class. Which statement indicates that additional teaching is needed? 1. "An epidural can be continuous or can be given in one dose." 2. "A spinal is usually used for a cesarean birth." 3. "Pudendal blocks are effective when a vacuum is needed." 4. "Local anesthetics provide good labor pain relief."

Answer: 4 Explanation: 4. Local anesthetics are not used for labor pain relief. They are used prior to episiotomy and for laceration repair.

Which of the following would be a newborn care procedure that will decrease the probability of high bilirubin levels? 1. Monitor urine for amount and characteristics. 2. Encourage late feedings to promote intestinal elimination. 3. All infants should be routinely monitored for iron intake. 4. Maintain the newborn's skin temperature at 36.5°C (97.8°F) or above.

Answer: 4 Explanation: 4. Maintain the newborn's skin temperature at 36.5°C (97.8°F) or above; cold stress results in acidosis.

The client is at 42 weeks' gestation. Which order should the nurse question? 1. Obtain biophysical profile today. 2. Begin nonstress test now. 3. Schedule labor induction for tomorrow. 4. Have the client return to the clinic in 1 week.

Answer: 4 Explanation: 4. Many practitioners use twice-weekly testing providing the amniotic fluid level is normal. One week is too long a period between assessments.

The client has stated that she wants to avoid an epidural and would like an unmedicated birth. Which nursing action is most important for this client? 1. Encourage the client to vocalize during contractions. 2. Perform vaginal exams only between contractions. 3. Provide a CD of soft music with sounds of nature. 4. Offer to teach the partner how to massage tense muscles.

Answer: 4 Explanation: 4. Massage is helpful for many clients, especially during latent and active labor. Massage can increase relaxation and therefore decrease tension and pain.

A woman is in labor. The fetus is in vertex position. When the client's membranes rupture, the nurse sees that the amniotic fluid is meconium-stained. What should the nurse do immediately? 1. Change the client's position in bed. 2. Notify the physician that birth is imminent. 3. Administer oxygen at 2 liters per minute. 4. Begin continuous fetal heart rate monitoring.

Answer: 4 Explanation: 4. Meconium-stained amniotic fluid is an abnormal fetal finding, and is an indication for continuous fetal monitoring.

The client is carrying monochorionic-monoamniotic twins. The nurse teaches the client what this is, and the implications of this finding. The nurse knows that teaching is successful when the client states which of the following? 1. "My babies came from two eggs." 2. "About two thirds of twins have this amniotic sac formation." 3. "My use of a fertility drug led to this issue." 4. "My babies have a lower chance of surviving to term than fraternal twins do."

Answer: 4 Explanation: 4. Monochorionic-monoamniotic twins are both in one amniotic sac. There is an increased risk of umbilical cords becoming tangled or knotted and a higher incidence of fetal demise.

The nurse is caring for a newborn in the special care nursery. The infant has hydrocephalus, and is positioned in a prone position. The nurse is especially careful to cleanse all stool after bowel movements. This care is most appropriate for an infant born with which of the following? 1. Omphalocele 2. Gastroschisis 3. Diaphragmatic hernia 4. Myelomeningocele

Answer: 4 Explanation: 4. Myelomeningocele is a saclike cyst containing meninges, spinal cord, and nerve roots in thoracic and/or lumbar area. Meticulous cleaning of the buttocks and genitals helps prevent infection. The infant is positioned on abdomen or on side and restrain (to prevent pressure and trauma to sac). Hydrocephalus often is present.

The nurse assesses the newborn and notes the following behaviors: nasal flaring, facial grimacing, and excessive mucus. What is the nurse most concerned about? 1. Neonatal jaundice 2. Neonatal hypothermia 3. Neonatal hyperthermia 4. Respiratory distress

Answer: 4 Explanation: 4. Nasal flaring and facial grimacing are signs of respiratory distress.

Before the newborn and mother are discharged from the birthing unit, the nurse teaches the parents about newborn screening tests that includes which of the following? 1. Preeclampsia screening 2. Congenital kidney disease screening 3. Visual screening 4. Hearing screening

Answer: 4 Explanation: 4. Newborn screening tests include hearing screening tests.

The nurse is instructing the parents of a newborn about car seat safety. Which statement indicates that the parents need additional information? 1. "The baby should be in the back seat." 2. "Newborns must be in rear-facing car seats." 3. "We need instruction on how to use the car seat before installing it." 4. "We can bring the baby home from the hospital without a car seat as it is only a short drive home."

Answer: 4 Explanation: 4. Newborns must go home from the birthing unit in a car seat adapted to fit newborns.

The nurse assesses the postpartum client who has not had a bowel movement by the third postpartum day. Which nursing intervention would be appropriate? 1. Encourage the new mother, saying, "It will happen soon." 2. Instruct the client to eat a low-fiber diet. 3. Decrease fluid intake. 4. Obtain an order for a stool softener.

Answer: 4 Explanation: 4. Obtaining an order for a stool softener is the correct intervention by the third day. In resisting or delaying the bowel movement, the woman may cause increased constipation and more pain when elimination finally occurs.

The nurse is analyzing various teaching strategies for teaching new mothers about newborn care. To enhance learning, which teaching method should the nurse implement? 1. Select videos on various topics of newborn care. 2. Organize a class that includes first-time mothers only. 3. Have mothers return in 1 week, when they feel more rested. 4. Schedule time for one-to-one teaching in the mother's room.

Answer: 4 Explanation: 4. One-to-one teaching while the nurse is in the mother's room is shown to be the most effective educational model. Individual instruction is helpful to answer specific questions.

The nurse is planning home visits to the homes of new parents and their newborns. Which client should the nurse see first? 1. 3-day-old male who received hepatitis B vaccine prior to discharge 2. 4-day-old female whose parents are both hearing-impaired 3. 5-day-old male with light, sticky, yellow drainage on the circumcision site 4. 6-day-old female with greenish discharge from the umbilical cord site

Answer: 4 Explanation: 4. Oozing of greenish yellow material, or reddened areas around the cord is not an expected finding. This family should be seen first because the child is experiencing a complication.

The nurse is teaching a group of new parents about newborn behavior. Which statement made by a parent would indicate a need for additional information? 1. "Sleep and alert states cycle throughout the day." 2. "We can best bond with our child during an alert state." 3. "About half of the baby's sleep time is in active sleep." 4. "Babies sleep during the night right from birth."

Answer: 4 Explanation: 4. Over time, the newborn's sleep-wake patterns become diurnal, that is, the newborn sleeps at night and stays awake during the day. Page Ref: 666

The nurse is assessing a newborn at 1 hour of age. Which finding requires an immediate intervention? 1. Respiratory rate 60 and irregular in depth and rhythm 2. Pulse rate 145, cardiac murmur heard 3. Mean blood pressure 55 mm Hg 4. Pauses in respiration lasting 30 seconds

Answer: 4 Explanation: 4. Pauses in respirations greater than 20 seconds are considered episodes of apnea, and require further intervention.

The nurse is working at a clinic for pregnant teens. What issues related to development will the nurse expect to encounter in most of the pregnant clients? 1. Peer pressure to stop using alcohol once pregnancy is diagnosed 2. Contraception failure that resulted in this pregnancy 3. The father of the baby's being emotionally supportive to the client 4. Feelings of not living up to parents' expectations

Answer: 4 Explanation: 4. Pregnant teens face risk factors based on the developmental tasks of adolescence. One of these is developing an identity. If the adolescent feels she has not lived up to parental expectations by becoming pregnant, she could adopt a negative identity.

The nurse is working with a pregnant 14-year-old. Which statement indicates that additional education is required? 1. "Because I am still growing, I need more calories than a pregnant adult." 2. "I need to eat fruit and vegetables every day to get enough vitamins." 3. "My favorite food is pizza, and I eat it once a week." 4. "Because I don't eat breakfast, I'll have to eat more at supper."

Answer: 4 Explanation: 4. Pregnant young adolescents should eat breakfast to ensure the adequate calorie and protein intake needed. In assessing the diet of the pregnant adolescent, the nurse should consider the eating pattern over time, not simply a single day's intake.

The neonatal special care unit nurse is overseeing the care provided by a nurse new to the unit. Which action requires immediate intervention? 1. The new nurse holds the infant after giving a gavage feeding. 2. The new nurse provides skin-to-skin care. 3. The new nurse provides care when the baby is awake. 4. The new nurse gives the feeding with room-temperature formula.

Answer: 4 Explanation: 4. Preterm babies have little subcutaneous fat, and do not maintain their body temperature well. Formula should be warmed prior to feedings to help the baby maintain its temperature.

The nurse is caring for a premature infant in the NICU, and is going to attempt a bottle feeding with thawed breast milk. How long can thawed breast milk be stored in the refrigerator before the nurse must discard it? 1. 4 hours 2. 8 hours 3. 12 hours 4. 24 hours

Answer: 4 Explanation: 4. Previously frozen thawed breast milk is good in the refrigerator for 24 hours only.

The clinic nurse is compiling data for a yearly report. Which client would be classified as a primigravida? 1. A client at 18 weeks' gestation who had a spontaneous loss at 12 weeks 2. A client at 13 weeks' gestation who had an ectopic pregnancy at 8 weeks 3. A client at 14 weeks' gestation who has a 3-year-old daughter at home 4. A client at 15 weeks' gestation who has never been pregnant before

Answer: 4 Explanation: 4. Primigravida means a woman who is pregnant for the first time.

) The nurse is caring for a client who recently emigrated from a Southeast Asian country. The mother has been resting since the birth, while her sister has changed the diapers and fed the infant. What is the most likely explanation for this behavior? 1. The client is not attaching to her infant appropriately. 2. The client is not going to be a good mother, and the baby is at risk. 3. The client has no mother present to role-model behaviors. 4. The client is exhibiting normal behavior for her culture.

Answer: 4 Explanation: 4. Rest, seclusion, and dietary restraint practices in many traditional non-Western cultures (South Asian groups) are designed to assist the woman and her baby during postpartum vulnerable periods.

Intervention to reduce preterm birth can be divided into primary prevention and secondary prevention. What does secondary prevention include? 1. Diagnosis and treatment of infections 2. Cervical cerclage 3. Progesterone administration 4. Antibiotic treatment and tocolysis

Answer: 4 Explanation: 4. Secondary prevention strategies are antibiotic treatment and tocolysis.

A client at 20 weeks' gestation has not decided on a feeding method for her infant. She asks the nurse for advice. The nurse presents information about the advantages and disadvantages of formula-feeding and breastfeeding. Which statements by the client indicate that the teaching was successful? 1. "Formula-feeding gives the baby protection from infections." 2. "Breast milk cannot be stored; it has to be thrown away after pumping." 3. "Breastfeeding is more expensive than formula-feeding." 4. "My baby will have a lower risk of food allergies if I breastfeed."

Answer: 4 Explanation: 4. Secretory IgA, an immunoglobulin present in colostrum and mature breast milk, has antiviral, antibacterial, and antigenic-inhibiting properties and plays a role in decreasing the permeability of the small intestine to help prevent large protein molecules from triggering an allergic response.

A pregnant woman is married to an intravenous drug user. She had a negative HIV screening test just after missing her first menstrual period. What would indicate that the client needs to be retested for HIV? 1. Hemoglobin of 11 g/dL and a rapid weight gain 2. Elevated blood pressure and ankle edema 3. Shortness of breath and frequent urination 4. Persistent candidiasis

Answer: 4 Explanation: 4. Signs and symptoms of infections include fever, weight loss, fatigue, persistent candidiasis, diarrhea, cough, and skin lesions (Kaposi's sarcoma and hairy leukoplakia in the mouth).

The nurse is providing health teaching to a group of women of childbearing age. One woman states that she is a smoker, and asks about the effect of smoking on her fetus. The nurse tells her that which fetal complication can occur when the mother smokes? 1. Genetic changes in the fetal reproductive system 2. Extensive central nervous system damage 3. Addiction to the nicotine inhaled from the cigarette 4. Low birth rate

Answer: 4 Explanation: 4. Smoking can cause low birth rate.

Which of the following is common in many non-Western cultures and is on the increase in the United States? 1. Ceremonial rituals and rites 2. Cultural assessment 3. Cultural values 4. Cosleeping

Answer: 4 Explanation: 4. Some parents advocate cosleeping or bed sharing (one or both parents sleeping with their baby or young child). Cosleeping, which is common in many non-Western cultures, is on the increase in the United States.

The nurse is assessing a client who is at 35 weeks' gestation. What does the nurse expect the client to report at this phase of pregnancy? 1. Nausea and vomiting 2. Maternal ambivalence 3. Emotional shifts from highs to lows 4. Stretch marks on the abdomen

Answer: 4 Explanation: 4. Striae are purplish stretch marks that may develop as the pregnancy progresses.

The nurse has just palpated a laboring woman's contractions. The uterus cannot be indented during a contraction. What would the intensity of these contractions best be characterized as? 1. Weak 2. Mild 3. Moderate 4. Strong

Answer: 4 Explanation: 4. Strong intensity exists when the uterine wall cannot be indented.

The nurse developing a care plan for a pregnant teen knows that what plan is best suited to pregnant teens? 1. Assess menstrual history and the presence of any food allergies. 2. Inquire about family relationships and location of the home. 3. Ask what her body image is, then correct her misconceptions. 4. Determine whether there are substance abuse issues.

Answer: 4 Explanation: 4. Substance abuse issues are important to assess during pregnancy of teens.

The nurse working in an adolescent prenatal clinic knows which of the following about the clients who are 15-17 years old? 1. They are more at ease with their individuality. 2. They see authority as resting with parents. 3. They are able to solve problems and make decisions. 4. They seek independence and identify with their peer group.

Answer: 4 Explanation: 4. Teens who are in middle adolescence (15-17 years old) seek independence and identify with their peer group.

Parents have been told their child has fetal alcohol syndrome (FAS). Which statement by a parent indicates that additional teaching is required? 1. "Our baby's heart murmur is from this syndrome." 2. "He might be a fussy baby because of this." 3. "His face looks like it does due to this problem." 4. "Cuddling and rocking will help him stay calm."

Answer: 4 Explanation: 4. The FASD baby is most comfortable in a quiet, minimally stimulating environment.

When comparing the anterior and posterior fontanelles of a newborn, the nurse knows that both are what? 1. Both are approximately the same size 2. Both close within 12 months of birth 3. Both are used in labor to identify station 4. Both allow for assessing the status of the newborn after birth

Answer: 4 Explanation: 4. The anterior and posterior fontanelles are clinically useful in identifying the position of the fetal head in the pelvis and in assessing the status of the newborn after birth.

The client has just been diagnosed as diabetic. The nurse knows teaching was effective when the client makes which statement? 1. "Ketones in my urine mean that my body is using the glucose appropriately." 2. "I should be urinating frequently and in large amounts to get rid of the extra sugar." 3. "My pancreas is making enough insulin, but my body isn't using it correctly." 4. "I might be hungry frequently because the sugar isn't getting into the tissues the way it should."

Answer: 4 Explanation: 4. The client who understands the disease process is aware that if the body is not getting the glucose it needs, the message of hunger will be sent to the brain.

During the fourth stage of labor, the client's assessment includes a BP of 110/60, pulse 90, and the fundus is firm midline and halfway between the symphysis pubis and the umbilicus. What is the priority action of the nurse? 1. Turn the client onto her left side. 2. Place the bed in Trendelenburg position. 3. Massage the fundus. 4. Continue to monitor.

Answer: 4 Explanation: 4. The client's assessment data are normal for the fourth stage of labor, so monitoring is the only action necessary. During the fourth stage of labor, the mother experiences a moderate drop in both systolic and diastolic blood pressure, increased pulse pressure, and moderate tachycardia.

A new parent reports to the nurse that the baby looks cross-eyed several times a day. The nurse teaches the parents that this finding should resolve in how long? 1. 2 months 2. 2 weeks 3. 1 year 4. 4 months

Answer: 4 Explanation: 4. The newborn might be demonstrating transient strabismus that is caused by poor neuromuscular control of the eye muscles. This will gradually regress in 3 to 4 months.

During a postpartum examination of a client who delivered an 8-pound newborn 6 hours ago, the following assessment findings are noted: fundus firm and at the umbilicus, and moderate lochia rubra with a steady trickle of blood from the vagina. What is the assessment finding that would necessitate follow-up? 1. Firm fundus 2. Fundus at the umbilical level 3. Moderate lochia rubra 4. Steady trickle of blood

Answer: 4 Explanation: 4. The continuous seepage of blood is more consistent with cervical or vaginal lacerations. Lacerations should be suspected if the uterus is firm and of expected size and if no clots can be expressed. This finding would indicate a follow-up.

During a maternal assessment, the nurse determines the fetus to be in a left occiput anterior (LOA) position. Auscultation of the fetal heart rate should begin in what quadrant? 1. Right upper quadrant 2. Left upper quadrant 3. Right lower quadrant 4. Left lower quadrant

Answer: 4 Explanation: 4. The fetal heart rate (FHR) is heard most clearly at the fetal back. Thus, in a cephalic presentation, the FHR is best heard in the lower quadrant of the maternal abdomen.

While caring for a client in labor, the nurse notices during a vaginal exam that the fetus's head has rotated internally. What would the nurse expect the next set of cardinal movements for a fetus in a vertex presentation to be? 1. Flexion, extension, restitution, external rotation, and expulsion 2. Expulsion, external rotation, and restitution 3. Restitution, flexion, external rotation, and expulsion 4. Extension, restitution, external rotation, and expulsion

Answer: 4 Explanation: 4. The fetus changes position in the following order: descent, flexion, internal rotation, extension, restitution, external rotation, and expulsion.

The primigravida at 22 weeks' gestation has a fundal height palpated slightly below the umbilicus. Which of the following statements would best describe to the client why she needs to be seen by a physician today? 1. "Your baby is growing too much and getting too big." 2. "Your uterus might have an abnormal shape." 3. "The position of your baby can't be felt." 4. "Your baby might not be growing enough."

Answer: 4 Explanation: 4. The fundal height at 20-22 weeks should be about even with the umbilicus. At 22 weeks' gestation, a fundal height below the umbilicus and the size of the uterus that is inconsistent with length of gestation could indicate fetal demise.

A 21-year-old at 12 weeks' gestation with her first baby has known cardiac disease, class III, as a result of childhood rheumatic fever. During a prenatal visit, the nurse reviews the signs of cardiac decompensation with her. The nurse will know that the client understands these signs and symptoms if she states that she would notify her doctor if she had which symptom? 1. "A pulse rate increase of 10 beats per minute" 2. "Breast tenderness" 3. "Mild ankle edema" 4. "A frequent cough"

Answer: 4 Explanation: 4. The heart's signal of its decreased ability to meet the demands of pregnancy includes frequent cough (with or without hemoptysis).

A postpartum client calls the nursery to report that her 3-day-old newborn has passed a green stool. What is the nurse's best response? 1. "Take your newborn to the pediatrician." 2. "There might be a possible food allergy." 3. "Your newborn has diarrhea." 4. "This is a normal occurrence."

Answer: 4 Explanation: 4. The newborn's stools change from meconium (thick, tarry, black) to transitional stools (thinner, brown to green).

A client has just been admitted for labor and delivery. She is having mild contractions lasting 30 seconds every 15 minutes. The client wants to have a medication-free birth. When discussing medication alternatives, the nurse should be sure the client understands which of the following? 1. In order to respect her wishes, no medication will be given. 2. Pain relief will allow a more enjoyable birth experience. 3. The use of medications allows the client to rest and be less fatigued. 4. Maternal pain and stress can have a more adverse effect on the fetus than would a small amount of analgesia.

Answer: 4 Explanation: 4. The nurse can explain to the couple that, although pharmacologic agents do affect the fetus, so do the pain and stress experienced by the laboring mother. If the woman's pain and anxiety are more than she can cope with, the adverse physiologic effects on the fetus may be as great as would occur with the administration of a small amount of an analgesic agent. Once the effects are explained, however, it is still the client's choice whether to receive medication.

The nurse is working with new parents who have recently immigrated to the United States. The nurse is not familiar with the family's cultural background. Which approach is most appropriate when discussing the newborn? 1. "You appear to be Muslim. Do you want your son circumcised?" 2. "Let me explain newborn care here in the United States." 3. "Your baby is a United States citizen. You must be very happy about that." 4. "Could you explain your preferences regarding childrearing?"

Answer: 4 Explanation: 4. The nurse must be sensitive to the cultural beliefs and values of the family and be aware of cultural variations in newborn care.

A client has just delivered her third child, who was stillborn and had obvious severe defects. Which statement by the nurse is most helpful? 1. "Thank goodness you have other children." 2. "I am so happy that your other children are healthy." 3. "These things happen. They are the will of God." 4. "It is all right for you to cry. I will stay here with you."

Answer: 4 Explanation: 4. The nurse needs to let the client know that crying is a normal reaction to the loss event, and that the nurse will stay with her to offer support and understanding.

A client in her third trimester of pregnancy reports frequent leg cramps. What strategy would be most appropriate for the nurse to suggest? 1. Point the toes of the affected leg 2. Increase intake of protein-rich foods 3. Limit activity for several days 4. Flex the foot to stretch the calf

Answer: 4 Explanation: 4. The nurse should advise the client to practice dorsiflexion of feet to stretch affected muscle.

The nurse is caring for a couple who are in the labor/delivery room immediately after the delivery of a stillborn baby with visible defects. Which of the following actions by the nurse is appropriate? 1. Discourage the parents from naming the baby. 2. Advise the parents that the baby's defects would be too upsetting for them to see. 3. Transport the baby to the morgue as soon as possible. 4. Offer the parents the choice to see and hold the baby.

Answer: 4 Explanation: 4. The nurse should offer the couple the opportunity to see and hold the infant, and reassure the couple that any decision they make for themselves is the right one.

The nurse is working in an outpatient clinic. Which client's indications most warrant fetal monitoring in the third trimester? 1. Gravida 4, para 3, 39 weeks, with a history of one spontaneous abortion at 8 weeks 2. Gravida 1, para 0, 40 weeks, with a history of endometriosis and a prior appendectomy 3. Gravida 3, para 2, with a history of gestational diabetes controlled by diet 4. Gravida 2, para 1, 36 weeks, with a history of history of preterm labor or cervical insufficiency

Answer: 4 Explanation: 4. The preterm client with a history of preterm labor or cervical insufficiency needs close monitoring for preterm labor onset.

After delivery, it is determined that there is a placenta accreta. Which intervention should the nurse anticipate? 1. 2 L oxygen by mask 2. Intravenous antibiotics 3. Intravenous oxytocin 4. Hysterectomy

Answer: 4 Explanation: 4. The primary complication of placenta accreta is maternal hemorrhage and failure of the placenta to separate following birth of the infant. An abdominal hysterectomy may be the necessary treatment, depending on the amount and depth of involvement.

During the initial visit with the nurse at the fertility clinic, the client asks what effect cigarette smoking has on the ability to conceive. What is the nurse's best response? 1. "Smoking has no effect." 2. "Only if you smoke more than one pack a day will you experience difficulty." 3. "After your first semen analysis, we will determine whether there will be any difficulty." 4. "Smoking can affect the quantity of sperm."

Answer: 4 Explanation: 4. The quantity and quality of male sperm are affected by cigarette smoking.

The nurse is explaining the nutritional differences between breast milk and formula to an expectant couple. The mother-to-be asks whether breast milk is nutritionally superior to formula. What should the nurse reply? 1. The vitamins and minerals in formula are more bioavailable to the infant. 2. There is no cholesterol in breast milk. 3. The only carbohydrate in breast milk is lactose. 4. The ratio of whey to casein proteins in breast milk changes to meet the nutritional needs of the growing infant.

Answer: 4 Explanation: 4. The ratio of whey to casein proteins in breast milk, unlike that in formula, is not static. It changes to meet the nutritional needs of the growing infant.

To assess the healing of the uterus at the placental site, what does the nurse assess? 1. Lab values 2. Blood pressure 3. Uterine size 4. Type, amount, and consistency of lochia

Answer: 4 Explanation: 4. The type, amount, and consistency of lochia determine the state of healing of the placental site, and a progressive color change from bright red at birth to dark red to pink to white or clear should be observed.

Prior to receiving lumbar epidural anesthesia, the nurse would anticipate placing the laboring client in which position? 1. On her right side in the center of the bed with her back curved 2. Lying prone with a pillow under her chest 3. On her left side with the bottom leg straight and the top leg slightly flexed 4. Sitting on the edge of the bed

Answer: 4 Explanation: 4. The woman is positioned on her left or right side, at the edge of the bed with the assistance of the nurse, with her legs slightly flexed, or she is asked to sit on the edge of the bed.

Why is it important for the nurse to assess the bladder regularly and encourage the laboring client to void frequently? 1. A full bladder impedes oxygen flow to the fetus. 2. Frequent voiding prevents bruising of the bladder. 3. Frequent voiding encourages sphincter control. 4. A full bladder can impede fetal descent.

Answer: 4 Explanation: 4. The woman should be encouraged to void because a full bladder can interfere with fetal descent. If the woman is unable to void, catheterization may be necessary.

A nurse counsels a couple on sex-linked disorders. Both the man and the woman are carriers of the disorder. They ask the nurse how this disorder will affect any children they might have. What is the nurse's best response? 1. "If you have a daughter, she will not be affected." 2. "Your son will be affected because the father has the disorder." 3. "There is a 25% chance that your son will have the disorder because the mother has the disorder." 4. "There is a 50% chance that your son will be a carrier only."

Answer: 4 Explanation: 4. There is a 50% chance that a carrier mother will pass the normal gene to each of her sons, who will be unaffected.

A client had a cesarean birth 3 days ago. She has tenderness, localized heat, and redness of the left leg. She is afebrile. As a result of these symptoms, what would the nurse anticipate would be the next course of action? 1. That the client would be encouraged to ambulate freely 2. That the client would be given aspirin 650 mg by mouth 3. That the client would be given Methergine IM 4. That the client would be placed on bed rest

Answer: 4 Explanation: 4. These symptoms indicate the presence of superficial thrombophlebitis. The treatment involves bed rest, elevation of the affected limb, analgesics, and use of elastic support hose.

The nurse will be bringing the parents of a neonate with sepsis to the neonatal intensive care nursery for the first time. Which statement is best? 1. "I'll bring you to your baby and then leave so you can have some privacy." 2. "Your baby is on a ventilator with 50% oxygen, and has an umbilical line." 3. "I am so sorry this has all happened. I know how stressful this can be." 4. "Your baby is working hard to breathe and lying quite still, and has an IV."

Answer: 4 Explanation: 4. This answer is best because it explains what the parents will see in terminology that they will understand. A trusting relationship is essential for collaborative efforts in caring for the infant. The nurse should respond therapeutically to relate to the parents on a one-to-one basis.

The charge nurse has received the shift change report. Which client requires immediate intervention? 1. Woman at 6 cm undergoing induction of labor, strong contractions every 3 minutes 2. Woman at 4 cm whose fetus is in a longitudinal lie with a cephalic presentation 3. Woman at 10 cm and fetus at +2 station experiencing a strong expulsion urge 4. Woman at 3 cm screaming in fear because her mother died during childbirth

Answer: 4 Explanation: 4. This client is most likely fearful that she will die during labor because her mother died during childbirth. This client requires education and a great deal of support, and is therefore the top priority.

The client at 30 weeks' gestation with her first child is upset. She tells the prenatal clinic nurse that she is excited to become a mother, and has been thinking about what kind of parent she will be. But her mother has told her that she doesn't want to be a grandmother because she doesn't feel old enough, while her husband has said that the pregnancy doesn't feel real to him yet, and he will become excited when the baby is actually here. What is the most likely explanation for what is happening within this family? 1. Her husband will not attach with this child and will not be a good father. 2. Her mother is rejecting the role of grandparent, and will not help out. 3. The client is not progressing through the developmental tasks of pregnancy. 4. The family members are adjusting to the role change at their own paces.

Answer: 4 Explanation: 4. This is a true statement. With each pregnancy, routines and family dynamics are altered, requiring readjustment and realignment.

A nurse working in an infertility clinic should include which information in her discussions with the clients? 1. It is important to know the statistics surrounding couples who never learn why they are infertile. 2. Couples should understand the legal controversy concerning therapeutic insemination. 3. Couples should seek marriage counseling before undergoing fertility treatments. 4. Couples should discuss therapeutic insemination and in vitro fertilization as alternatives.

Answer: 4 Explanation: 4. This is the correct answer. This information should be presented to clients so that they are aware of all the alternatives and can make an informed decision.

The multiparous client at term has arrived to the labor and delivery unit in active labor with intact membranes. Leopold maneuvers indicate the fetus is in a transverse lie with a shoulder presentation. Which physician order is most important? 1. Artificially rupture membranes. 2. Apply internal fetal scalp electrode. 3. Monitor maternal blood pressure every 15 minutes. 4. Alert surgical team of urgent cesarean.

Answer: 4 Explanation: 4. This is the highest priority because vaginal birth is impossible with a transverse lie. Labor should not be allowed to continue, and a cesarean birth is done quickly.

Recommendations for parents to help their teens avoid pregnancy include which of the following? 1. Parents need to encourage frequent and steady dating. 2. Parents need to let their children set their own goals for the future. 3. Parents need to have their children taught about sex by school programs and community resources. 4. Parents should be clear about their own sexual attitudes and values.

Answer: 4 Explanation: 4. This is true. Parents should be clear about their own sexual attitudes and values in order to communicate clearly with children.

A client has been diagnosed with fallopian tube obstruction and told that her best option for becoming pregnant is with in vitro fertilization. The client asks the nurse about the procedure. What is the nurse's best explanation of this procedure? 1. "In vitro fertilization (IVF) occurs over a full menstrual cycle." 2. "In IVF, a woman's ovaries are stimulated by a combination of egg and sperm donations." 3. "After ovarian stimulation, you will be inseminated with your partner's sperm." 4. "The oocytes are aspirated from the ovaries and fertilized in the laboratory."

Answer: 4 Explanation: 4. This is true. The oocytes are aspirated from the client's ovaries and fertilized in the laboratory.

The nurse is working with a client from Southeast Asia. The client tells the nurse that she should not put the baby to breast until her milk comes in and her breasts are warm, because "cold milk" (colostrum) is bad for the baby. After the nurse explains the benefits of colostrum, the client still insists that "cold milk" is bad. Which response by the nurse is best? 1. "What kind of formula would you like to use?" 2. "That idea is folklore. Colostrum is good for the baby." 3. "Now that you are here, you need to feed your baby the right way." 4. "Let's give the baby formula after you breastfeed."

Answer: 4 Explanation: 4. This response attempts to provide a compromise between acknowledging the client's desire to give formula and getting the baby to breast to get colostrum. Nurses should be aware that some immigrant mothers may have this misconception about their colostrum.

The nurse is working with an adolescent parent. The adolescent tells the nurse, "I'm really scared that I won't take care of my baby correctly. My mother says I'll probably hurt the baby because I'm too young to be a mother." What is the best response by the nurse? 1. "You are very young, and parenting will be a challenge for you." 2. "Your mother was probably right. Be very careful with your baby." 3. "Mothers have instincts that kick in when they get their babies home." 4. "We can give the baby's bath together. I'll help you learn how to do it."

Answer: 4 Explanation: 4. This response is best because bathing the newborn offers an excellent opportunity for teaching and welcoming parent involvement in the care of their baby.

The nurse is completing discharge teaching for a client who delivered 2 days ago. Which statement by the client indicates that further information is required? 1. "Because I have a midline episiotomy, I should keep my perineum clean." 2. "I can use an ice pack to relieve some the pain from the episiotomy." 3. "I can take ibuprofen (Motrin) when my perineum starts to hurt." 4. "The tear I have through my rectum is unrelated to my episiotomy."

Answer: 4 Explanation: 4. This statement is incorrect. The major disadvantage is that a tear of the midline incision may extend through the anal sphincter and rectum.

How does the nurse assess for Homans' sign? 1. Extending the foot and inquiring about calf pain. 2. Extending the leg and inquiring about foot pain. 3. Flexing the knee and inquiring about thigh pain. 4. Dorsiflexing the foot and inquiring about calf pain.

Answer: 4 Explanation: 4. To assess for thrombophlebitis, the nurse should have the woman stretch her legs out, with the knees slightly flexed and the legs relaxed. The nurse then grasps the foot and dorsiflexes it sharply. If pain is elicited, the nurse notifies the physician/CNM that the woman has a positive Homans' sign. The pain is caused by inflammation of a vessel.

A new parent reports to the nurse that the baby looks cross-eyed several times a day. The nurse teaches the parents that this finding should resolve in how long? 1. 2 months 2. 2 weeks 3. 1 year 4. 4 months

Answer: 4 Explanation: 4. Transient strabismus is caused by poor neuromuscular control of the eye muscles and gradually regresses in 3 to 4 months.

The nurse is presenting a class to newly pregnant families. What form of trauma will the nurse describe as the leading cause of fetal and maternal death? 1. Falls 2. Domestic violence 3. Gun accidents 4. Motor vehicle accidents

Answer: 4 Explanation: 4. Trauma from motor vehicle accidents is the leading cause of fetal and maternal death.

The nurse is orienting a new graduate nurse to the labor and birth unit. Which statement indicates that teaching has been effective? 1. "When a client arrives in labor, a urine specimen is obtained by catheter to check for protein and ketones." 2. "When a client arrives in labor, she will be positioned supine to facilitate a normal blood pressure." 3. "When a client arrives in labor, her prenatal record is reviewed for indications of domestic abuse." 4. "When a client arrives in labor, a vaginal exam is performed unless birth appears to be imminent."

Answer: 4 Explanation: 4. Unless delivery seems imminent because the client is bearing down or contractions are very close and strong, the vaginal exam is performed after the vital signs are obtained.

After several hours of labor, the electronic fetal monitor (EFM) shows repetitive variable decelerations in the fetal heart rate. The nurse would interpret the decelerations to be consistent with which of the following? 1. Breech presentation 2. Uteroplacental insufficiency 3. Compression of the fetal head 4. Umbilical cord compression

Answer: 4 Explanation: 4. Variable decelerations occur when there is umbilical cord compression.

The client delivered her second child yesterday, and is preparing to be discharged. She expresses concern to the nurse because she developed an upper urinary tract infection (UTI) after the birth of her first child. Which statement indicates that the client needs additional teaching about this issue? 1. "If I start to have burning with urination, I need to call the doctor." 2. "Drinking 8 glasses of water each day will help prevent another UTI." 3. "I will remember to wipe from front to back after I move my bowels." 4. "Voiding 2 or 3 times per day will help prevent a recurrence."

Answer: 4 Explanation: 4. Voiding only 2 or 3 times per day is not sufficient to prevent recurrence of a urinary tract infection (UTI). The woman needs to empty her bladder whenever she feels the urge to void at least every 2 to 4 hours while awake.

The client is having fetal heart rate decelerations. An amnioinfusion has been ordered for the client to alleviate the decelerations. The nurse understands that the type of decelerations that will be alleviated by amnioinfusion is which of the following? 1. Early decelerations 2. Moderate decelerations 3. Late decelerations 4. Variable decelerations

Answer: 4 Explanation: 4. When cord compression is suspected, amnioinfusion (AI) may be considered. AI helps to prevent the possibility of variable decelerations by increasing the volume of amniotic fluid.

A clinic nurse is planning when to administer Rh immune globulin (RhoGAM) to an Rh-negative pregnant client. When should the first dose of RhoGAM be administered? 1. After the birth of the infant 2. 1 month postpartum 3. During labor 4. At 28 weeks' gestation

Answer: 4 Explanation: 4. When the woman is Rh negative and not sensitized and the father is Rh positive or unknown, Rh immune globulin is given prophylactically at 28 weeks' gestation.

A 21-year-old woman is at 12 weeks' gestation with her first baby. She has cardiac disease, class III, as a result of having had childhood rheumatic fever. Which planned activity would indicate to the nurse that the client needs further teaching? 1. "I will be sure to take a rest period every afternoon." 2. "I would like to take childbirth education classes in my last trimester." 3. "I will have to cancel our trip to Disney World." 4. "I am going to start my classes in water aerobics next week."

Answer: 4 Explanation: 4. With the slightest exertion, the client's heart rate will rise, and she will become symptomatic. Therefore, she should not establish a new exercise program.

Which of the following is a risk factor for urinary retention after childbirth? 1. Multiparity 2. Precipitous labor 3. Unassisted childbirth 4. Not sufficiently recovering from the effects of anesthesia

Answer: 4 Explanation: 4. Women who have not sufficiently recovered from the effects of anesthesia and cannot void spontaneously are at risk for urinary retention after childbirth.

A 20-year-old woman is at 28 weeks' gestation. Her prenatal history reveals past drug abuse, and urine screening indicates that she has recently used heroin. The nurse should recognize that the woman is at increased risk for which condition? 1. Erythroblastosis fetalis 2. Diabetes mellitus 3. Abruptio placentae 4. Pregnancy-induced hypertension

Answer: 4 Explanation: 4. Women who use heroin are at risk for poor nutrition, anemia, and pregnancy-induced hypertension (or preeclampsia-eclampsia).

The client at 24 weeks' gestation is experiencing painless vaginal bleeding after intercourse. The physician has ordered a transvaginal ultrasound examination. Which statements by the client indicate an understanding of why this exam has been requested? Note: Credit will be given only if all correct choices and no incorrect choices are selected. Select all that apply. 1. "This ultrasound will show the baby's gender." 2. "This ultrasound might cause the miscarriage of my baby." 3. "This ultrasound carries a risk of creating a uterine infection." 4. "This ultrasound can determine the location of my placenta." 5. "This ultrasound might detect whether the placenta is detaching prematurely."

Answer: 4, 5 Explanation: 4. Painless bleeding in the second and third trimesters can be a symptom of placenta previa. Transvaginal ultrasound will determine the placental location. 5. Painless bleeding in the second and third trimesters can be a symptom of placenta previa. Transvaginal ultrasound will determine the placental location.

The nurse is caring for a client who was just admitted to rule out ectopic pregnancy. Which orders are the most important for the nurse to perform? Note: Credit will be given only if all correct choices and no incorrect choices are selected. Select all that apply. 1. Assess the client's temperature. 2. Document the time of the client's last meal. 3. Obtain urine for urinalysis and culture. 4. Report complaints of dizziness or weakness. 5. Have the lab draw blood for B-hCG level every 48 hours.

Answer: 4, 5 Explanation: 4. Reporting complaints of dizziness and weakness is important, as it can indicate hypovolemia from internal bleeding. 5. Having the lab draw blood for B-hCG levels every 48 hours is important, as the level rises much more slowly in ectopic pregnancy than in normal pregnancy.

The mother of a client who has experienced a term stillbirth arrives at the hospital and goes to the nurses' desk. The mother asks what she should say to her daughter in this difficult time. What is the nurse's best response? Note: Credit will be given only if all correct choices and no incorrect choices are selected. Select all that apply. 1. "Use clichés; your daughter will find the repetition comforting." 2. "Remind her that she is young and can have more children." 3. "Keep talking about other things to keep her mind off the loss." 4. "Express your sadness, and sit silently with her if she doesn't respond." 5. "Encourage her to talk about the baby whenever she wants to."

Answer: 4, 5 Explanation: 4. Silence is commonly what is needed most, and simply saying "I'm sorry for your loss" might help to facilitate communication. 5. Talking is a way for the client experiencing grief and begin to come to terms with what has happened, and is important for resolution of grief. Intuitive grievers will need to talk about the event.

Which nursing intervention is appropriate in the management of the preterm infant with hypothermia? Note: Credit will be given only if all correct choices and no incorrect choices are selected. Select all that apply. 1. Warm the baby rapidly to reverse the hypothermia. 2. Monitor skin temperature every 2 hours to determine whether the infant's temperature is increasing. 3. Keep IV fluids at room temperature. 4. Initiate efforts to maintain the newborn in a neutral thermal environment. 5. Warm the baby slowly to reverse hypothermia and reach a neutral thermal environment

Answer: 4, 5 Explanation: 4. The nurse should initiate efforts to block heat loss by evaporation, radiation, convection, and conduction. 5. The infant should be warmed slowly to prevent hypotension and apnea.

During the initial intrapartal assessment of a client in early labor, the nurse performs a vaginal examination. The client's partner asks why this pelvic exam needs to be done. The nurse should explain that the purpose of the vaginal exam is to obtain information about which of the following? Note: Credit will be given only if all correct choices and no incorrect choices are selected. Select all that apply. 1. Uterine contraction pattern 2. Fetal position 3. Presence of the mucous plug 4. Cervical dilation and effacement 5. Presenting part

Answer: 4, 5 Explanation: 4. The vaginal examination of a laboring client obtains information about the station of the presenting part and the dilation and effacement of the cervix. 5. The vaginal examination of a laboring client obtains information about the fetal presenting part.

A woman who is 40 weeks pregnant calls the labor suite to ask whether she should be evaluated. Which statements by the client indicate she is likely in labor? Note: Credit will be given only for all correct choices and no incorrect choices. Select all that apply. 1. "The contractions are 5-20 minutes apart." 2. "I had pink discharge on the toilet paper." 3. "I have had cramping for the past 3-4 hours." 4. "The contractions start in my back and then go to my abdomen and are very intense." 5. "The contractions hurt more when I walk."

Answer: 4, 5 Explanation: 4. This is a sign of true labor. The contractions increase in duration and intensity and begin in the back and radiate around to the abdomen. 5. It is a sign of true labor when the client is unable to walk during the contraction.

The nurse has received the shift change report on infants born within the previous 4 hours. Which newborn should the nurse see first? 1. 37-week male, respiratory rate 45 2. 8 pound 1 ounce female, pulse 150 3. Term male, nasal flaring 4. 4-hour-old female who has not voided

Explanation: 3. Nasal flaring is an indication of respiratory distress. The nurse must be immediately available to provide appropriate interventions for a newborn in distress.

On assessment, a labor client is noted to have cardiovascular and respiratory collapse and is unresponsive. What should the nurse suspect? 1. An amniotic fluid embolus 2. Placental abruption 3. Placenta accreta 4. Retained placenta

answer 1 Explanation: 1. Cardiovascular and respiratory collapse are symptoms of an amniotic fluid embolus and cor pulmonale.


संबंधित स्टडी सेट्स

Neuroscience Chapter 14: Cranial Nerves

View Set

Microbiology (EXAM #1) CH. 2 Homework & Reading Questions

View Set

Final Prep Multiple Choice CIST1220-Structured Query Language-SQL

View Set